Sunteți pe pagina 1din 391

McGILL UNIVERSITY

FACULTY OF SCIENCE

DEPARTMENT OF
MATHEMATICS AND STATISTICS

MATH 329 2009 01


THEORY OF INTEREST

Information for Students


(Winter Term, 2008/2009)
Pages 1 - 9 of these notes may be considered the
Course Outline for this course.
W. G. Brown
April 14, 2009
Information for Students in MATH 329 2009 01

Contents 8 Solutions, Third Problem Assign-


ment 25
1 General Information 1
1.1 Instructor and Times . . . . . 1 9 Fourth Problem Assignment 32
1.2 Course Description . . . . . . 1
1.2.1 Calendar Description . 1 10 Class Tests 34
1.2.2 Syllabus (in terms of sec- 10.1 Class Test, Version 1 . . . . . 34
tions of the text-book) 1 10.2 Class Test, Version 2 . . . . . 41
1.2.3 “Verbal” arguments . 3 10.3 Class Test, Version 3 . . . . . 47
1.3 Evaluation of Your Progress . 4 10.4 Class Test, Version 4 . . . . . 53
1.3.1 Term Mark . . . . . . 4
1.3.2 Assignments. . . . . . 4 11 Draft Solutions to Problems on
1.3.3 Class Test . . . . . . . 4 the Class Tests 59
1.3.4 Final Examination . . 4 11.1 Version 1 . . . . . . . . . . . 59
1.3.5 Supplemental Assessments 5 11.2 Version 2 . . . . . . . . . . . 62
1.3.6 Machine Scoring . . . 5 11.3 Version 3 . . . . . . . . . . . 64
1.3.7 Plagiarism . . . . . . . 5 11.4 Version 4 . . . . . . . . . . . 67
1.4 Published Materials . . . . . 5
12 Fifth Problem Assignment 71
1.4.1 Required Text-Book . 5
1.4.2 Website . . . . . . . . 6 13 Draft Solutions, Fourth Problem
1.4.3 Reference Books . . . 6 Assignment 73
1.5 Other information . . . . . . 6
1.5.1 Prerequisites . . . . . 6 14 Draft Solutions, Fifth Problem As-
1.5.2 Calculators . . . . . . 6 signment 80
1.5.3 Self-Supervision . . . . 7
1.5.4 Escape Routes . . . . 7 15 References 901
1.5.5 Showing your work; good
mathematical form; sim- A Supplementary Lecture Notes 2001
plifying answers . . . . 7 A.1 Supplementary Notes for the Lec-
ture of January 05th, 2009 . . 2001
2 Timetable 8 A.1.1 These notes . . . . . . 2001
A.1.2 §1.1 INTRODUCTION 2001
3 First Problem Assignment 10 A.1.3 §1.2 THE ACCUMULA-
TION AND AMOUNT
4 Second Problem Assignment 12 FUNCTIONS . . . . . 2002
A.2 Supplementary Notes for the Lec-
5 Solutions, First Problem Assign-
ture of January 07th, 2009 . . 2003
ment 14
A.3 Supplementary Notes for the Lec-
6 Third Problem Assignment 18 ture of January 09th, 2009 . . 2009
A.3.1 §1.3 THE EFFECTIVE
7 Solutions, Second Problem As- RATE OF INTEREST 2009
signment 21 A.3.2 §1.4 SIMPLE INTEREST 2011
Information for Students in MATH 329 2009 01

A.3.3 §1.5 COMPOUND INTER- A.9.1 §2.3 EQUATIONS OF VALUE


EST . . . . . . . . . . 2012 (conclusion) . . . . . . 2038
A.4 Supplementary Notes for the Lec- A.9.2 §2.4 UNKNOWN TIME 2039
ture of January 12th, 2009 . . 2016 A.10 Supplementary Notes for the Lec-
A.4.1 §1.5 COMPOUND INTER- ture of January 26th, 2009 . . 2043
EST (conclusion) . . . 2016 A.10.1 §2.5 UNKNOWN RATE
A.4.2 §1.6 PRESENT VALUE 2016 OF INTEREST . . . . 2043
A.4.3 §1.7 THE EFFECTIVE A.10.2 §2.6 DETERMINING TIME
RATE OF DISCOUNT 2018 PERIODS . . . . . . . 2044
A.5 Supplementary Notes for the Lec- A.10.3 §2.7 PRACTICAL EX-
ture of January 14th, 2009 . . 2021 AMPLES . . . . . . . 2046
A.5.1 §1.7 THE EFFECTIVE A.11 Supplementary Notes for the Lec-
RATE OF DISCOUNT ture of January 28th, 2009 . . 2050
(conclusion) . . . . . . 2021 A.11.1 §2.8 MISCELLANEOUS
A.5.2 §1.8 NOMINAL RATES PROBLEMS . . . . . 2050
OF INTEREST AND DIS- A.11.2 §3.1 INTRODUCTION 2052
COUNT . . . . . . . . 2022 A.11.3 §3.2 ANNUITY-IMMEDIATE 2052
A.6 Supplementary Notes for the Lec- A.12 Supplementary Notes for the Lec-
ture of January 16th, 2009 . . 2025 ture of January 30th, 2009 . . 2054
A.6.1 §1.8 NOMINAL RATES A.12.1 §3.2 ANNUITY-IMMEDIATE
OF INTEREST AND DIS- (conclusion) . . . . . . 2054
COUNT (conclusion) 2025 A.13 Supplementary Notes for the Lec-
A.6.2 §1.9 FORCES OF INTER- ture of February 02nd, 2009 . 2056
EST AND DISCOUNT 2027 A.13.1 §3.3 ANNUITY-DUE 2056
A.7 Supplementary Notes for the Lec- A.14 Supplementary Notes for the Lec-
ture of January 19th, 2009 . . 2029 ture of February 04th, 2009 . 2058
A.7.1 §1.9 FORCES OF INTER- A.14.1 §3.4 ANNUITY VALUES
EST AND DISCOUNT ON ANY DATE . . . 2058
(conclusion) . . . . . . 2029 A.15 Supplementary Notes for the Lec-
A.7.2 §1.10 VARYING INTER- ture of February 06th, 2009 . 2064
EST . . . . . . . . . . 2032 A.15.1 §3.5 PERPETUITIES 2064
A.7.3 §1.11 SUMMARY OF RE- A.16 Supplementary Notes for the Lec-
SULTS . . . . . . . . 2033 ture of February 09th, 2009 . 2067
A.8 Supplementary Notes for the Lec- A.16.1 §3.6 UNKNOWN TIME 2067
ture of January 21st, 2009 . . 2034 A.17 Supplementary Notes for the Lec-
A.8.1 §2.1 INTRODUCTION 2034 ture of February 11th, 2009 . 2071
A.8.2 §2.2 THE BASIC PROB- A.17.1 §3.6 UNKNOWN TIME
LEM . . . . . . . . . . 2034 (conclusion) . . . . . . 2071
A.8.3 §2.3 EQUATIONS OF VALUE 2036 A.17.2 §3.7 UNKNOWN RATE
A.9 Supplementary Notes for the Lec- OF INTEREST . . . . 2072
ture of January 23rd, 2009 . . 2038 A.18 Supplementary Notes for the Lec-
ture of February 13th, 2009 . 2076
Information for Students in MATH 329 2009 01

A.18.1 §3.7 UNKNOWN RATE A.23.1 §4.6 PAYMENTS VARY-


OF INTEREST (conclu- ING IN ARITHMETIC
sion) . . . . . . . . . . 2076 PROGRESSION (contin-
A.18.2 §3.8 VARYING INTER- ued) . . . . . . . . . . 2096
EST . . . . . . . . . . 2077 A.24 Supplementary Notes for the Lec-
A.18.3 §3.9 ANNUITIES NOT ture of March 06th, 2009 . . . 2098
INVOLVING COMPOUND A.24.1 §4.6 PAYMENTS VARY-
INTEREST . . . . . . 2077 ING IN ARITHMETIC
A.18.4 §3.10 MISCELLANEOUS PROGRESSION (conclu-
PROBLEMS . . . . . 2078 sion) . . . . . . . . . . 2098
A.18.5 APPENDIX 3 . . . . 2079 A.24.2 §4.7 PAYMENTS VARY-
A.19 Supplementary Notes for the Lec- ING IN GEOMETRIC PRO-
ture of February 16th, 2009 . 2080 GRESSION . . . . . . 2102
A.19.1 §4.1 INTRODUCTION 2080 A.24.3 §4.8 MORE GENERAL
A.19.2 §4.2 ANNUITIES PAYABLE VARYING ANNUITIES 2105
AT A DIFFERENT FRE- A.24.4 §4.10 MISCELLANEOUS
QUENCY THAN INTER- PROBLEMS . . . . . 2105
EST IS CONVERTIBLE 2080 A.25 Supplementary Notes for the Lec-
A.20 Supplementary Notes for the Lec- ture of March 09th, 2009 . . . 2108
ture of February 18th, 2009 . 2084 A.25.1 §5.1 INTRODUCTION 2108
A.20.1 §4.3 ANNUITIES PAYABLE A.25.2 §5.2 FINDING THE OUT-
LESS FREQUENTLY THAN STANDING LOAN BAL-
INTEREST IS CONVERT- ANCE . . . . . . . . . 2108
IBLE . . . . . . . . . 2084 A.26 Supplementary Notes for the Lec-
A.21 Supplementary Notes for the Lec- ture of March 11th, 2009 . . . 2113
ture of February 20th, 2009 . 2089 A.26.1 §5.3 AMORTIZATION SCHED-
A.21.1 §4.4 FURTHER ANAL- ULES . . . . . . . . . 2113
YSIS OF ANNUITIES PAYABLE A.27 Supplementary Notes for the Lec-
MORE FREQUENTLY ture of March 16th, 2009 . . . 2120
THAN INTEREST IS CON- A.27.1 §5.4 SINKING FUNDS 2120
VERTIBLE . . . . . . 2089 A.28 Supplementary Notes for the Lec-
A.22 Supplementary Notes for the Lec- ture of March 18th, 2009 . . . 2122
ture of March 02nd, 2009 . . 2093 A.28.1 §5.4 SINKING FUNDS
A.22.1 §4.5 CONTINUOUS AN- (conclusion) . . . . . . 2122
NUITIES . . . . . . . 2093 A.28.2 §5.5 DIFFERING PAY-
A.22.2 §4.6 PAYMENTS VARY- MENT PERIODS AND
ING IN ARITHMETIC INTEREST CONVERSION
PROGRESSION . . . 2095 PERIODS . . . . . . . 2126
A.23 Supplementary Notes for the Lec- A.28.3 §5.6 VARYING SERIES
ture of March 04th, 2009 . . . 2096 OF PAYMENTS . . . 2127
A.28.4 §5.7 AMORTIZATION WITH
CONTINUOUS PAYMENTS 2127
Information for Students in MATH 329 2009 01

A.28.5 §5.8 STEP-RATE AMOUNTS A.34.3 §7.6 TIME-WEIGHTED


OF PRINCIPAL . . . 2127 RATES OF INTEREST
A.29 Supplementary Notes for the Lec- (Omit) . . . . . . . . . 2146
ture of March 20th, 2009 . . . 2128 A.34.4 §7.7 PORTFOLIO METH-
A.29.1 §6.1 INTRODUCTION 2128 ODS AND INVESTMENT
A.29.2 §6.2 TYPES OF SECU- YEAR METHODS (Omit) 2146
RITIES . . . . . . . . 2128 A.34.5 §7.8 SHORT SALES (Omit) 2146
A.30 Supplementary Notes for the Lec- A.34.6 §7.9 CAPITAL BUDGET-
ture of March 23rd, 2009 . . . 2131 ING - BASIC TECHNIQUES
A.30.1 §6.3 PRICE OF A BOND 2131 (Omit) . . . . . . . . . 2146
A.31 Supplementary Notes for the Lec- A.34.7 §7.10 CAPITAL BUDGET-
ture of March 25th, 2009 . . . 2134 ING - OTHER TECH-
A.31.1 §6.4 PREMIUM AND DIS- NIQUES (Omit) . . . 2146
COUNT . . . . . . . . 2134 A.34.8 Appendix 7 (Omit) . . 2146
A.32 Supplementary Notes for the Lec- A.35 Supplementary Notes for the Lec-
ture of March 27th, 2009 . . . 2136 ture of April 03rd, 2009 . . . 2147
A.32.1 §6.4 PREMIUM AND DIS- A.36 Supplementary Notes for the Lec-
COUNT (conclusion) 2136 ture of April 06th, 2009 . . . 2147
A.32.2 §6.5 VALUATION BE- A.37 Supplementary Notes for the Lec-
TWEEN COUPON PAY- ture of April 08th, 2009 . . . 2147
MENT DATES (Omit) 2137 A.38 Supplementary Notes for the Lec-
A.32.3 §6.6 DETERMINATION ture of April 14th, 2009 . . . 2147
OF YIELD RATES (omit) 2137
A.32.4 §6.7 CALLABLE AND B Problem Assignments, Tests, and
PUTABLE BONDS . 2137 Examinations from Previous Years 3001
A.33 Supplementary Notes for the Lec- B.1 2002/2003 . . . . . . . . . . . 3001
ture of March 30th, 2009 . . . 2139 B.1.1 First 2002/2003 Problem
A.33.1 §6.7 CALLABLE AND Assignment, with Solu-
PUTABLE BONDS (con- tions . . . . . . . . . . 3001
tinued) . . . . . . . . 2139 B.1.2 Second 2002/2003 Prob-
A.33.2 §7.1 INTRODUCTION (omit) 2142 lem Assignment, with So-
A.33.3 §7.2 DISCOUNTED CASH lutions . . . . . . . . . 3006
FLOW ANALYSIS (omit) 2142 B.1.3 Third 2002/2003 Prob-
A.33.4 §7.3 UNIQUENESS OF lem Assignment, with So-
THE YIELD RATE (part) 2142 lutions . . . . . . . . . 3011
A.34 Supplementary Notes for the Lec- B.1.4 Fourth 2002/2003 Prob-
ture of April 01st, 2009 . . . 2143 lem Assignment, with So-
A.34.1 §7.4 REINVESTMENT lutions . . . . . . . . . 3018
RATES . . . . . . . . 2143 B.1.5 Fifth 2002/2003 Problem
A.34.2 §7.5 INTEREST MEA- Assignment, with Solu-
SUREMENT OF A FUND tions . . . . . . . . . . 3027
(Omit) . . . . . . . . . 2146
Information for Students in MATH 329 2009 01

B.1.6 2002/2003 Class Tests, with B.3.5 Fifth Problem Assignment,


Solutions . . . . . . . 3035 with Solutions . . . . 3126
B.1.7 Final Examination, 2002/2003 3040 B.3.6 Solutions to Problems on
B.2 2003/2004 . . . . . . . . . . . 3043 the Class Test . . . . . 3134
B.2.1 First 2003/2004 Problem B.3.7 Final Examination 2005/2005 3145
Assignment, with Solu- B.3.8 Supplemental/Deferred Ex-
tions . . . . . . . . . . 3043 amination 2004/2005 . 3149
B.2.2 Second 2003/2004 Prob-
lem Assignment, with So-
lutions . . . . . . . . . 3046
B.2.3 Third 2003/2004 Prob-
lem Assignment, with So-
lutions . . . . . . . . . 3055
B.2.4 Fourth 2003/2004 Prob-
lem Assignment, with So-
lutions . . . . . . . . . 3065
B.2.5 Fifth 2003/2004 Problem
Assignment, with Solu-
tions . . . . . . . . . . 3070
B.2.6 2003/2004 Class Test, Ver-
sion 1 . . . . . . . . . 3075
B.2.7 2003/2004 Class Test, Ver-
sion 2 . . . . . . . . . 3077
B.2.8 2003/2004 Class Test, Ver-
sion 3 . . . . . . . . . 3078
B.2.9 2003/2004 Class Test, Ver-
sion 4 . . . . . . . . . 3079
B.2.10 Solutions to Problems on
the 2003/2004 Class Tests 3081
B.2.11 Final Examination, 2003/2004 3090
B.2.12 Supplemental/Deferred Ex-
amination, 2003/2004 3092
B.3 2004/2005 . . . . . . . . . . . 3095
B.3.1 First Problem Assignment,
with Solutions . . . . 3095
B.3.2 Second Problem Assign-
ment, with Solutions . 3101
B.3.3 Third Problem Assignment,
with Solutions . . . . 3109
B.3.4 Fourth Problem Assign-
ment, with Solutions . 3118
Information for Students in MATH 329 2009 01 1

1 General Information
Distribution Date: Friday, January 05th, 2009
subject to correction

(All information is subject to change, either by announcements at lectures,


on WebCT, or in print.)
An updated version will be accessible via a link from WebCT.)
The Course Outline for MATH 329 2009 01 can be considered to be pages
1 through 9 of these notes.

1.1 Instructor and Times

CRN: 610
INSTRUCTOR: Prof. W. G. Brown
OFFICE: BURN 1224
OFFICE HRS. W 13:15→14:15 h.;
(subject to F 10:30→11:30 h.;
change) and by appointment
TELEPHONE: 398–3836
E-MAIL: BROWN@MATH.MCGILL.CA
CLASSROOM: BURN 1B23
CLASS HOURS: MWF 14:35–15:25 h.

Table 1: Instructor and Times

1.2 Course Description


1.2.1 Calendar Description
THEORY OF INTEREST. (3 credits) (Prerequisite: MATH 141.) Simple and com-
pound interest, annuities certain, amortization schedules, bonds, depreciation.

1.2.2 Syllabus (in terms of sections of the text-book)


The central part of the course consists of many of the topics in the first eight chapters
of the textbook [4]1 ; section numbers, where shown, refer to that book. In the list below
I show the chapters and appendices of the textbook. Following each is a description as
of the date of this revision, of the sections to be excluded. This list could be updated
1
[n] refers to item n in the bibliography, page 901.
Information for Students in MATH 329 2009 01 2

during the semester, as it becomes apparent that certain sections are not appropriate to
the level of the course or the lecture time available.

2
Chapter 1. The Measurement of Interest §§1.1–1.11. Appendix 1. For the
present, please omit the discussion of force of discount on [4, §1.9, p. 30].

Chapter 2. Solution of Problems in Interest §§2.1–2.7. Omit method of equated


time, p. 55 and Appendix 2. Students should peruse §2.6, but will not be expected to
memorize the definitions contained therein.

Chapter 3. Basic Annuities §§3.1–3.6. In §3.7 students should know how to de-
rive and use the approximation formulæ from Appendix 3, but need not memorize the
formulæ. Omit §3.8, with the exception of the definitions of the Portfolio and Yield
Methods. Only one example from §3.9 will be considered. Appendix 3 is included in
connection with §3.7.

Chapter 4. More General Annuities §§4.1-4.7. Appendix 4. Omit §§4.8, 4.9.

Chapter 5. Amortization schedules and sinking funds §§5.1–5.4. Omit §§5.5,


5.6, 5.7, 5.8.

Chapter 6. Bonds and other securities §§6.1–6.4, and 6.7. Omit §§6.5, 6.6, 6.8–
6.11 and Appendix 6.

Chapter 7. Yield rates §7.3 (part), §7.4. Omit §§7.1–7.2, 7.5–7.10, Appendix 7.

Chapter 8. Practical applications §8.6. Omit the remainder of this chapter.

Chapter 9. More advanced financial analysis Omit this chapter.

Chapter 10. The term structure of interest rates Omit this chapter.

Chapter 11. Duration, convexity and immunization Omit this chapter.

Chapter 12. Stochastic approaches to interest Omit this chapter.


2
Added on 16 January, 2009
Information for Students in MATH 329 2009 01 3

Chapter 13. Options and other derivatives Omit this chapter.

Appendix A. Table numbering the days of the year Omit.

Appendix B. Illustrative mortgage loan amortization schedule This is an ex-


ample of a schedule, but much longer than any that you might be expected to be able
to generate manually. The type of schedule that you could be asked to generate, in
connection with chapters 5 and 6 could show more columns than this example.

Appendix C. Basic mathematical review Mostly definitions and a few formulæ


that should be known to all mathematics undergraduates. Sections E, F are from Cal-
culus 3, and may be discussed in the course.

Appendix D. Statistical background Omit.

Appendix E. Iteration methods The only method which will be discussed exten-
sively will be successive bisection [4, §B].

1.2.3 “Verbal” arguments


An essential feature of investment and insurance mathematics is the need to be able to
understand and to formulate “verbal” arguments; that is, explanations of the truth of an
identity presented verbally i.e., in words, rather then as an algebraic proof. In a verbal
argument we seek more than mathematically correctness: we wish to see an explanation
that could be presented to a layman who is not competent in the mathematical bases
of this subject, but is still possessed of reason, and needs to be assured that he is
not being exploited. When the skill has been mastered it can be used to verify the
correctness of statements proved mathematically. Verbal arguments require some care
with the underlying language; students who have difficulty with expression in English
are reminded that McGill students have the right to submit any written materials in
either English or French.3
3
For a lexicon of actuarial terms in English/French, see The Canadian Institute of Actuaries English-
French lexicon [10], at

http://www.actuaries.ca/publications/lexicon/
Information for Students in MATH 329 2009 01 4

1.3 Evaluation of Your Progress


1.3.1 Term Mark
The Term Mark will be computed one-third from the assignment grades, and two-thirds
from the class test. The Term Mark will count for 30 of the 100 marks in the final
grade, but only if it exceeds 30% of the final examination percentage; otherwise the final
examination will be used exclusively in the computation of the final grade.

1.3.2 Assignments.
A total of about 5 assignments will together be worth 10 of the 30 marks assigned to
Term Work.

1.3.3 Class Test


A class test will be held on Friday, March 13th, 20094 , at the regular class time, counting
for 20 of the 30 marks in the Term Mark. Before the date was revised, students had been
advised that ‘This date may be changed, after discussion with the class at any scheduled
lecture date. Students who don’t come to class should ensure that they are aware of any
changes in the date of the test. There will be no “make-up” test for persons who miss
the test.’

1.3.4 Final Examination


Written examinations form an important part of the tradition of actuarial mathematics.
The final examination in MATH 329 2009 01 will count for either 70% or 100% of the
numerical grade from which the submitted final letter grade will be computed. Where
a student’s Final Examination percentage is superior to her Term Mark percentage, the
Final Examination grade will replace the Term Mark grade in the calculations.
A 3-hour-long final examination will be scheduled during the regular examination
period for the winter term (April 15th, 2009 through April 30th, 2009). You are advised
not to make any travel arrangements that would prevent you from being present on cam-
pus at any time during this period. Students who have religious or other constraints that
could affect their ability to write examinations at particular times should watch for the
Preliminary Examination Timetable, as their rights to apply for special consideration at
their faculty may have expired by the time the Final Examination Timetable is published.
4
Date revised in consultation with the students present at the class of 02 March, 2009

UPDATED TO April 14, 2009


Information for Students in MATH 329 2009 01 5

1.3.5 Supplemental Assessments


Supplemental Examination. For eligible students who obtain a Final Grade of F
or D in the course there will be a supplemental examination. (For information about
Supplemental Examinations, see the McGill Calendar, [7].)

There is No Additional Work Option. “Will students with marks of D, F, or J


have the option of doing additional work to upgrade their mark?” No. (“Additional
Work” refers to an option available in certain Arts and Science courses, but not available
in this course.)

1.3.6 Machine Scoring


“Will the final examination be machine scored?” While there could be Multiple Choice
questions on the Class Test and/or the Final Examination, such questions will not be
machine scored.

1.3.7 Plagiarism
While students are not discouraged from discussing assignment problems with their col-
leagues, the work that you submit — whether through homework, the class test, or on
tutorial quizzes or the final examination should be your own. The Handbook on Student
Rights and Responsibilities states in ¶15(a)5 that

“No student shall, with intent to deceive, represent the work of another person
as his or her own in any academic writing, essay, thesis, research report,
project or assignment submitted in a course or program of study or represent
as his or her own an entire essay or work of another, whether the material so
represented constitutes a part or the entirety of the work submitted.”

You are also referred to the following URL:

http://www.mcgill.ca/integrity/studentguide/

1.4 Published Materials


1.4.1 Required Text-Book
The textbook for the course this semester is [4] Stephen G. Kellison, The Theory of
Interest, Third Edition. McGraw-Hill/Irwin, Boston, etc. (2009), ISBN 978-0-07-338244-
9.
5
http://upload.mcgill.ca/secretariat/greenbookenglish.pdf
Information for Students in MATH 329 2009 01 6

1.4.2 Website
These notes, and other materials distributed to students in this course, will be accessible
via myCourses (=WebCT) at the following URL:

http://www.math.mcgill.ca/brown/math329b.html

The notes will be in “pdf” (.pdf) form, and can be read using the Adobe Acrobat reader,
which many users have on their computers. This free software may be downloaded from
the following URL:

http://www.adobe.com/prodindex/acrobat/readstep.html6

Where revisions are made to distributed printed materials — for example these informa-
tion sheets — it is expected that the last version will be posted on the Web.
The notes will also be available via a link from the WebCT URL:

http://mycourses.mcgill.ca

but not all features of WebCT will be implemented.

1.4.3 Reference Books


The textbooks used in the course in previous years may be used as references.

1.5 Other information


1.5.1 Prerequisites
It is your responsibility as a student to verify that you have the prescribed calculus
prerequisites. It would be foolish to attempt to take the course without them.

1.5.2 Calculators
It is intended that the use of non-programmable, non-graphing calculators only will be
permitted in homework, tests, or the final examination in this course. Students may
be required to convince examiners and invigilators that all memories have been cleared.
The use of calculators that are either graphing, programmable will not be permitted
during test or examinations, in order to “level the playing field”. It is not intended that
students should be permitted to use financial calculators.
6
At the time of this writing the current version is Version 9.
Information for Students in MATH 329 2009 01 7

1.5.3 Self-Supervision
This is not a high-school course, and McGill is not a high school. The monitoring of
your progress before the final examination is largely your own responsibility. While the
instructor is available to help you, he cannot do so unless and until you identify the
need for help. While the significance of the homework assignments and class test in the
computation of your grade is minimal, these are important learning experiences, and
can assist you in gauging your progress in the course. This is not a course that can
be crammed for: you must work steadily through the term if you wish to develop the
facilities needed for a strong performance on the final examination.

Working Problems on Your Own. You are advised to work large numbers of prob-
lems from your textbook. The skills you acquire in solving textbook problems could have
much more influence on your final grade than either the homework or the class test.

1.5.4 Escape Routes


At any time, even after the last date for dropping the course, students who are experi-
encing medical or personal difficulties should not hesitate to consult their advisors or the
Student Affairs office of their faculty. Don’t allow yourself to be overwhelmed by such
problems; the University has resource persons who may be able to help you.

1.5.5 Showing your work; good mathematical form; simplifying answers


When, in a test or examination problem, you are explicitly instructed to show all your
work, failure to do so could result in a substantial loss of marks — possibly even all
of the marks; this is the default. The guiding principle should be that you want to be
able to communicate your precise reasoning to others and to yourself. You are always
expected to “simplify” any algebraic or numerical expressions that arise in your solutions
or calculations. Verbal proofs are expected to be “convincing”: it will not be sufficient
to simply describe mathematical expressions verbally.
Information for Students in MATH 329 2009 01 8

2 Timetable
Distribution Date: (original version) Monday, January 5th, 2009
this revision as of April 14, 2009
(Subject to correction and change.)
Section numbers refer to the text-book.7

MONDAY WEDNESDAY FRIDAY


JANUARY
05 §§1.1-1.2 07 §§1.1-1.2 09 §§1.3-1.5
12 §§1.5-1.6 14 §1.7-1.8 16 §§1.8-1.9
Course changes must be completed on MINERVA by Tuesday, Jan. 20, 2009
19 §§1.9-1.10 21 §§2.1-2.2 23 §§2.3-2.4
Deadline for withdrawal with fee refund = Jan. 25, 2009
Verification Period: January 26 – 30, 2009
26 §§2.5-2.7 °1 28 §2.8, SS3.1-3.2 30 §§3.1-3.2
FEBRUARY
02 §3.3 04 §3.4 06 §3.5
09 §3.6 °
2 11 §3.7 13 §§3.8-3.9
Deadline for web withdrawal (with W) from course via MINERVA = Feb. 15, 2009
16 §§4.1-4.2 18 §§4.3 20 §4.4
Study Break: February 22 – 28, 2009
No lectures, no regular office hours!
23 NO LECTURE 25 NO LECTURE 27 NO LECTURE

Notation: °
n = Assignment #n due today
°
R = Read Only
X = reserved for eXpansion or review
Information for Students in MATH 329 2009 01 9

Section numbers refer to the text-book.


MONDAY WEDNESDAY FRIDAY
MARCH
02 §§4.5, 4.6 °
3 04 §4.6 06 §§4.6, 4.7, 4.8
09 §5.1, §5.2 11 §5.3 13 CLASS TEST(revised
date)
16 §5.4 18 §5.4 20 §§6.1, 6.2
23 §6.3 °
4 25 §6.4 27 §6.7
30 §6.7 °
5

APRIL
01 §§7.3,7.4 03 §7.4
06 X 08 X 10 NO LECTURE
13 NO LECTURE 14 (Tuesday, to replace
13 April) X
Information for Students in MATH 329 2009 01 10

3 First Problem Assignment


Distribution Date: Monday, January 05th, 2009
Solutions are to be submitted by Monday, January 26th, 2009
These problems are to be solved with full solutions, modelled either on the
solutions to problems in the textbook, presented in class, or in the notes on
the Web for this or previous years. The essence is that the reader should
be able to reconstruct every step of the proof from what you have written:
getting the right answer is never enough. You are not being graded for
elegance, but simply for the proof being logical, without serious gaps. While
the data given may sometimes not justify a large number of decimal places,
you should show your intermediate calculations in sufficient detail that the
grader can determine that your calculations are correct.
1. Find the accumulated value of 1000 at the end of 4 years
(a) if the nominal annual rate of discount is 8%, convertible quarterly;
(b) if the nominal annual rate of interest is 8%, convertible quarterly;
(c) if the nominal annual rate of interest is 100%, convertible annually;
(d) if the nominal annual rate of discount is 50%, convertible annually.
2. Find the nominal rate of discount convertible every 3 months which is equivalent
to a nominal rate of interest of 8% converted every 2 years.
3. A treasury bill which costs 99.00 matures after 6 months at 100.00
(a) What is the effective annual rate of discount?
(b) What is the effective annual rate of interest?
(c) What is the nominal annual rate of discount compounded half-yearly?
(d) What is the nominal annual rate of interest compounded every 4 months?
(e) If interest is interpreted as being simple, what is the annual rate of simple
interest?
4. (a) Find the length of time for 10,000 to accumulate at compound interest to
25,000, if invested at 6.00% per annum, accumulated monthly.
(b) Find the length of time for 10,000 to accumulate to 25,000, if interest is
compounded monthly for full months, but is simple 6% annual interest for
proper fractions of a month. (In this part of the problem it is important that
you show all your work carefully, as the grader may be unable to reconstruct
the reasoning from the computations alone.)
Information for Students in MATH 329 2009 01 11

5. (cf. [4, Example 1.16, p. 35]) Suppose that money accumulates at an increasing
force of interest δt = 0.04 + 0.005t for 0 ≤ t ≤ 20.

(a) Find the value at time t = 2 of three payments of 50 to be paid at times


t = 0, 1, 2.
(b) Find the constant rate of interest compounded annually which would produce
the same accumulation at time t = 2.

(Note: Since the force of interest is not constant, this is not compound interest, and
you must not use results like δ = ln(1 + i), which was proved on the assumption of
compound interest.)
Information for Students in MATH 329 2009 01 12

4 Second Problem Assignment


Distribution Date: Thursday, January 22nd, 2009
Solutions are to be submitted by Monday, February 09th, 2009

These problems are to be solved with full solutions, modelled either on the
solutions to problems in the textbook, presented in class, or in the notes on
the Web for this or previous years. The essence is that the reader should
be able to reconstruct every step of the proof from what you have written:
getting the right answer is never enough. You are not being graded for
elegance, but simply for the proof being logical, without serious gaps. While
the data given may sometimes not justify a large number of decimal places,
you should show your intermediate calculations in sufficient detail that the
grader can determine that your calculations are correct.

1. K has 3,000 in a bank account that pays interest monthly at the nominal annual
rate i(12) = 1.8%. She needs to withdraw 5,000 a year from now to pay her tuition,
but plans to make equal deposits 3 months from now and 9 months from now.

(a) Showing all your work, and using an equation of value, determine what the
size of these equal deposits should be in order that the balance in the account
13 months from now will be 1,000.
(b) Repeat the calculations if the interest rate for the account is now an effective
annual rate of 1.8%; there is no change in the timing of the payments or the
compounding of interest.

(Since the interest rate is very low here, it is important that you not round your
computations too early.)

2. Funds A and B have not been receiving contributions for the past 10 years, and
will receive no future contributions. Fund A has been accumulating, and will
accumulate at an effective annual rate of 7%; Fund B has been accumulating and
will accumulate at a nominal annual rate of 8% compounded semi-annually. At the
end of 30 years the total of the two funds will be 50,000. At the end of 12 years
Fund A will be 90% the size of Fund B.

(a) Determine the values of each of the funds as of 4 years ago.


(b) Determine when the two funds will be equal.

3. Dr. X’s daughter has given birth to his first granddaughter today, and the doctor
wishes to establish a fund to pay the new baby’s tuition expenses in medical school
Information for Students in MATH 329 2009 01 13

20 years from now. If he assumes that the fund will need to contain 1,000,000 in
20 years, how much should he invest semi-annually now? He plans to deposit the
same fixed amount at the end of every half-year for 7 years, and wishes to base his
computations on an effective annual interest rate of 6%; the first deposit will be 6
months from now.

4. Mary purchases a high definition flat-screen TV worth 500 by paying an immediate


“down” payment and monthly payments of 25 for 15 months.

(a) If the monthly payments begin one month after the purchase, and the nominal
annual interest rate compounded monthly is 20%, determine the amount of
the down payment.
(b) The store offers customers the option of delaying the first payment until 1 year
from the date of purchase. If Mary elects to accept this option, and wishes to
pay the same down payment now as previously, what should be the amount
of each of her 15 equal regular payments? (Hint: One way to approach this
problem — not the only way — is to express the [deferred] payment annuity
as the difference of two annuities, both with first payment 1 month from now.)
Information for Students in MATH 329 2009 01 14

5 Solutions, First Problem Assignment


Distribution Date: Thursday, January 29th, 2009
Solutions were to be submitted by Monday, January 26th, 2009

These problems were to be solved with full solutions, modelled either on the solutions
to problems in the textbook, presented in class, or in the notes on the Web for this
or previous years. The essence is that the reader should be able to reconstruct every
step of the proof from what you have written: getting the right answer is never enough.
Students were advised that they were “not being graded for elegance, but simply for
the proof being logical, without serious gaps. While the data given may sometimes not
justify a large number of decimal places, you should show your intermediate calculations
in sufficient detail that the grader can determine that your calculations are correct.”

1. Find the accumulated value of 1000 at the end of 4 years

(a) if the nominal annual rate of discount is 8%, convertible quarterly;


(b) if the nominal annual rate of interest is 8%, convertible quarterly;
(c) if the nominal annual rate of interest is 100%, convertible annually;
(d) if the nominal annual rate of discount is 50%, convertible annually.

Solution:

(a) The accumulated value is 1000(1 − 0.02)−4×4 = 13861.60248 or 1,381.60.


(b) The accumulated value is 1000(1.02)4×4 = 1372.7857 or 1,372.79.
(c) The accumulated value is 1000(1 + 1)4 = 16, 000.
(d) The accumulated value is 1000(0.5)−4 = 16, 000.

2. Find the nominal rate of discount convertible every 3 months which is equivalent
to a nominal rate of interest of 8% converted every 2 years.
Solution: An equation of value is (cf. [4, equation (1.23a)])

µ ¶ Ã ! 12
d(4) −4
i( 1
2 )
1− = 1+ 1
4 2

which implies that


µ ¶− 18
d(4) 0.08 1
1− = 1+ 1 = 1.16− 8 ,
4 2
Information for Students in MATH 329 2009 01 15

implying that ³ ´
1
d(4) = 4 1 − 1.16− 8 = 0.0735258496
or 7.35%.

3. A treasury bill which costs 99.00 matures after 6 months at 100.00

(a) What is the effective annual rate of discount?


(b) What is the effective annual rate of interest?
(c) What is the nominal annual rate of discount compounded half-yearly?
(d) What is the nominal annual rate of interest compounded every 4 months?
(e) If interest is interpreted as being simple, what is the annual rate of simple
interest?

Solution:

(a) If d is the effective annual rate of discount, then an equation of value is


1
99 = (1 − d) 2 100 ,

hence d = 1 − (0.99)2 = 1.99%.


(b) If i is the effective annual rate of interest, then an equation of value is
1
100 = (1 + i) 2 99 ,
µ¶2
100
hence i = − 1 = 0.020304050 or 2.03%.
99
(c) An equation of value is
µ ¶
d(2)
99 = 1 − 100 ,
2

hence d(2) = 2.00%.


(d) An equation of value is
µ ¶6
i(3) 4
100 = 1 + 99 ,
3
õ ¶ 23 !
(3) 100
hence i =3 −1 = 0.020168163 or 2.017%.
99
Information for Students in MATH 329 2009 01 16

µ ¶
i 100 100
(e) If i is the rate of simple interest, then 1 + = ⇒i=2 −1 =
2 99 99
0.020202020 or 2.02%.

4. (a) Find the length of time for 10,000 to accumulate at compound interest to
25,000, if invested at 6.00% per annum, accumulated monthly.
(b) Find the length of time for 10,000 to accumulate to 25,000, if interest is
compounded monthly for full months, but is simple 6% annual interest for
proper fractions of a month. (In this part of the problem it is important that
you show all your work carefully, as the grader may be unable to reconstruct
the reasoning from the computations alone.)
Solution:

(a) If n denotes the number of years, then an equation of value is


µ ¶12n
0.06
1+ 10000 = 25000 ,
12
ln 2.5
so (1.005)12n = 2.5000, and n = ln 1.005
= 15.30965924 or 15.31 years.
(b) We have proved in the preceding part that the number of months required is
approximately 15.30965924 × 12 = 183.7, so the time will be 183 full months
plus a fraction of a month. In 183 full months 10000 accumulates at compound
interest to µ ¶183
0.06
1+ 10000 = 24910.89 .
12
Let t denote the fraction of a month needed to this amount to accumulate at
simple interest to 25000. Then an equation of value is
µ ¶183 µ ¶
0.06 t
1+ 1 + 0.06 · 10000 ,
12 12

yielding
25000
10000(1.005)183
−1
t= = 0.7154036000
0.005
or 0.72 months. Thus the total time required for the accumulation is 183.72
3.72
months, or 15 + = 15.31 years. The condition that interest be simple
12
has not been strong enough to affect the final answer (because of the low rate
of interest, and the short time involved).
Information for Students in MATH 329 2009 01 17

5. (cf. [4, Example 1.16, p. 35]) Suppose that money accumulates at an increasing
force of interest δt = 0.04 + 0.005t for 0 ≤ t ≤ 20.

(a) Find the value at time t = 2 of three payments of 50 to be paid at times


t = 0, 1, 2.
(b) Find the constant rate of interest compounded annually which would produce
the same accumulation at time t = 2.

(Note: Since the force of interest is not constant, this is not compound interest, and
you must not use results like δ = ln(1 + i), which was proved on the assumption of
compound interest.)
Solution:
Rt
δr dr
(a) Using formula [4, (1.27), p. 29]), which states that a(t) = e0 , we find the
accumulation at time t = 2 to be
à R2 R2 R2
!
δr dr δr dr δr dr
50 e0 + e1 + e2
à R2 R2 R2
!
(0.04+0.005r) dr (0.04+0.005r) dr (0.04+0.005r) dr
= 50 e0 + e1 + e2
µ 2 2 2

= 50 e[0.04r+0.0025r ]0 + e[0.04r+0.0025r ]1 + e[0.04r+0.0025r ]2
2 2 2

¡ ¢
= 50 e0.09 + e0.0475 + e0
= 50(1.094174284 + 1.048646201 + 1) = 157.1410242 .

(b) Suppose the annual rate of compound interest is i. Then an equation of value
is ¡ ¢
50 (1 + i)2 + (1 + i)1 + 1 = 156.8795170 .
This is a quadratic equation, equivalent to (1 + i)2 + (1 + i) √
− 2.13759034 =
3
0 . Solving by completion of the square yields i + 2 = ± 2.38759034 =
±1.545182947, implying that 0 = 0.045182947 or i = −3.045182947. The
second root is absurd, and extraneous. Hence the equivalent constant rate of
compound interest for this particular configuration of payments is 4.52%.
Information for Students in MATH 329 2009 01 18

6 Third Problem Assignment


Distribution Date: Saturday, February 07th, 2009
Solutions are to be submitted by Monday, March 02nd, 2009

1. X borrows 20,000 and starts repaying it with semi-annual payments of 1,200, at an


effective annual rate of 6%. After he makes 10 payments he is laid off from his job,
and asks the lenders to permit him to delay repayment. The lenders permit him to
skip just one payment, but tell him that interest rates have risen, and that they will
agree to the delay only if X agrees to accept the new rates as of the last payment
made. X counters that he can’t pay more than 1,200 per half-year, and agrees that,
when he resumes payments after the one missed payment, the payments will be at
a constant level of not more than 1,200 per half year at a nominal annual rate of
8%, compounded semi-annually.

(a) Determine, as of the date of the last payment made at the effective annual
rate of 6%, the amount of loan still outstanding.
(b) Determine, as of the date of the missed payment, the amount L of loan still
outstanding.
(c) By solving an inequality for the smallest value of n such that 1200 · an ≥ L,
determine the minimum number n of payments that X will still have to make.
(d) For the minimum number of payments n that you have determined above,
find the exact amount of each of the equal payments.
(e) Suppose that X had had the foresight to know, at the time he was borrow-
ing 20,000, that he would not be able to make the projected 11th payment.
Determine, as of the original date of the loan, the number m of payments he
would have to make after the missed payment, if all except the last of the
10 + m payments are to be equal to 1,200, and the last “drop” payment could
be smaller.
(f) Continuing the discussion in the previous item, suppose that, knowing that
he was going to have to make 10 + 0 + m payments not to exceed 1200, X
contracted at the time of the loan to make all of these payments equal (except
for the payment of 0 five and one-half years into the loan). What would be
the amount of these equal payments? (Hint: There are different ways of
approaching this problem. One method would be to proceed as above:
• Assume the first 10 payments are at the full amount of 1,200, and deter-
mine the amount L outstanding at the time of the missed payment.
• Determine the smallest m such that 1200 · am ≥ L.
Information for Students in MATH 329 2009 01 19

• Then determine the exact level amount of the 10 + 0 + m payments (re-


membering that there is a hiatus between the 10th payment and the
next).)

2. Suppose that you know that, for some n and for some interest rate i, än i = X, and
sn+1 i = Y . Use your (algebraic) knowledge of the values of the annuity functions
to determine a formula which expresses i in terms of X, Y . Verify your work by
selecting values of i and n and computing X and Y , and checking that the formula
is correct in that special case.

3. George has just begun working at the type of job he has dreamed of, where he would
work for 20 years and then retire for life. He plans to invest 1,000 at the beginning
of every month that he works. When he retires he will spend 150,000 to purchase a
chalet in the mountains, and will begin to draw fixed monthly payments from the
balance. The first payment he receives will be one month after he retires — i.e., 1
month after he draws the capital for his chalet. The fixed payments he receives will
begin a perpetuity, which will continue to his estate, forever (unless his executors
negotiate a termination with the trust company that holds the account). Suppose
that the interest rate during the years when he is working is 6%, compounded
monthly, and that the interest rate for the perpetuity-immediate is 7% annual,
effective. Determine, to the nearest unit, the constant amount of his perpetuity
payments.

4. Juliette was born on February 29th, 1980, and has always felt sorry for people
born on that day, because of the paucity of their birthdays. After making her
first billion, she decided to endow the Leap Year Foundation, with the purpose of
making awards to persons with 29 February as their birthday. Awards totalling
20,000 will be made on New Year’s Day of every leap year — assume that a leap
year is one divisible by 4, excluding years divisible by 100 except those divisible
by 400, which are leap years. If the Foundation makes its first awards in 2012,
determine how much money Juliette needs to contribute on April 1st, 2009 to fund
the Foundation, if the effective annual rate of interest is taken to be i = 5%.

5. The owner of a perpetuity-immediate which pays 10,000 per year wishes to adjust
the payments so that he receives 12 equal monthly payments through the year. If
the monthly payments work out to 825 each, what is the effective annual interest
rate? (It is intended that you determine this rate by first “trapping” the correct
rate in an interval between two interest rates, and then repeatedly cut the interval
in half by testing the midpoint. Finding the ends of the first interval you work
with is easy, particularly if you don’t demand that it be small. For example, see
what happens if you take an interest rate of 0%. Apply this “bisection” method
Information for Students in MATH 329 2009 01 20

10 times, and use the result to obtain an approximation to the effective annual
interest rate the question requests. The validity of this procedure derives from the
Intermediate Value Theorem, based on the continuity of the inverse function of sn i ,
i.e., of i as a function of sn i for fixed n.)
Information for Students in MATH 329 2009 01 21

7 Solutions, Second Problem Assignment


Distribution Date: Sunday, February 15th, 2009
Solutions were to be submitted by Monday, February 09th, 2009.

These problems are to be solved with full solutions, modelled either on the
solutions to problems in the textbook, presented in class, or in the notes on
the Web for this or previous years. The essence is that the reader should
be able to reconstruct every step of the proof from what you have written:
getting the right answer is never enough. You are not being graded for
elegance, but simply for the proof being logical, without serious gaps. While
the data given may sometimes not justify a large number of decimal places,
you should show your intermediate calculations in sufficient detail that the
grader can determine that your calculations are correct.

1. K has 3,000 in a bank account that pays interest monthly at the nominal annual
rate i(12) = 1.8%. She needs to withdraw 5,000 a year from now to pay her tuition,
but plans to make equal deposits 3 months from now and 9 months from now.
(a) Showing all your work, and using an equation of value, determine what the
size of these equal deposits should be in order that the balance in the account
13 months from now will be 1,000.
(b) Repeat the calculations if the interest rate for the account is now an effective
annual rate of 1.8%; there is no change in the timing of the payments or the
compounding of interest.
(Since the interest rate is very low here, it is important that you not round your
computations too early.)
Solution:
(a) Denote the payment that must be made 3 months from now, and again 9
i(12)
months from now by X. The effective interest rate per month is = 0.15%.
12
An equation of value as of a comparison date 13 months from now is
à !13 à !10 à !4 à !1
i(12) i(12) i(12) i(12)
3000 1 + +X 1+ +X 1+ −5000 1 + = 1000
12 12 12 12

⇔ 3000(1.0015)13 + X(1.0015)10 + X(1.0015)4 − 5000(1.0015) = 1000


1000 − 3000(1.0015)13 + 5000(1.0015)
⇔ X= = 1458.833537
(1.0015)10 + (1.0015)4
Information for Students in MATH 329 2009 01 22

so the amount to be deposited at each of the two given times is 1,458.83.


³ (12)
´12
(b) Since 1 + i 12 = 1 + i = 1.018,

i(12) 1
1+ = 1.018 12 = 1.001487765 ,
12
so the amount of each of the equal payments will be
1000 − 3000(1.001487765)13 + 5000(1.001487765)
X= = 1459.168645
1.00148776510 + 1.0014877654
or 1,459.17.

2. Funds A and B have not been receiving contributions for the past 10 years, and
will receive no future contributions. Fund A has been accumulating, and will
accumulate at an effective annual rate of 7%; Fund B has been accumulating and
will accumulate at a nominal annual rate of 8% compounded semi-annually. At the
end of 30 years the total of the two funds will be 50,000. At the end of 12 years
Fund A will be 90% the size of Fund B.

(a) Determine the values of each of the funds as of 4 years ago.


(b) Determine when the two funds will be equal.

Solution: Denote the respective values of Funds A and B now by A and B. Then
the two hypotheses give rise to the following equations:

A(1.07)30 + B(1.04)60 = 50000


A(1.07)12 = 0.9B(1.04)24

We can solve these two linear equations in two unknowns by eliminating one vari-
able between them and thereby determine first one variable, then the other. The
solutions thereby obtained by these (pre-college) operations are
0.9(50000)
A = = 2796.095323
0.9(1.07)30+ (1.04)36 · (1.07)12

50000
B = = 2729.698321
0.9(1.04)24 · (1.07)18 + (1.04)60
so we may take A = 2, 796.10, B = 2, 729.70.

(a) As of 4 years ago Fund A had value 2, 796.10(1.07)−4 = 2, 133.13, while Fund
B had value 2, 729.70(1.04)−8 = 1994.57.
Information for Students in MATH 329 2009 01 23

(b) The funds will be equal at time t, when A · (1.07)t = B · (1.04)2t , equivalently,
when µ ¶t
1.07 B
2
= ,
(1.04) A
which we can solve by taking logarithms:
ln B − ln A
t= = 2.228815096
ln 1.07 − 2 ln 1.04
i.e., 2.229 years from now (i.e., approximately 2 years and 84 days from now).

Some students read the phrase “At the end of 30 years” differently from what was
intended, and counted the 30 years from 10 years ago. While this reading could
be justified, some of these students appear to have read “as of 4 years ago” as
meaning 6 years in the future, and that reading is perhaps not so obvious. In any
case, the grader accepted these readings, and advises that “the answers (were) A
=4370.33andB =4196.18.”

3. Dr. X’s daughter has given birth to his first granddaughter today, and the doctor
wishes to establish a fund to pay the new baby’s tuition expenses in medical school
20 years from now. If he assumes that the fund will need to contain 1,000,000 in
20 years, how much should he invest semi-annually now? He plans to deposit the
same fixed amount at the end of every half-year for 7 years, and wishes to base his
computations on an effective annual interest rate of 6%; the first deposit will be 6
months from now.
Solution: Let i denote the effective semi-annual interest rate, and X denote
√ the
amount of Dr. X’s semi-annual contribution. Then (1 + i)2 = 1.06, so i = 1.06 −
1 = 0.029563014. An equation of value as of today is X · a20 i = 1000000(1.06)−20 ,
implying that

1000000(1.06)−20 (0.029563014) 1000000(1.06)−20 (0.029563014)


X= = = 27520.77408 ,
1 − (1.029563014)14 1 − (1.06)7

so Dr. X’s semi-annual contribution for seven years should be 27,520.77.

4. Mary purchases a high definition flat-screen TV worth 500 by paying an immediate


“down” payment and monthly payments of 25 for 15 months.

(a) If the monthly payments begin one month after the purchase, and the nominal
annual interest rate compounded monthly is 20%, determine the amount of
the down payment.
Information for Students in MATH 329 2009 01 24

(b) The store offers customers the option of delaying the first payment until 1 year
from the date of purchase. If Mary elects to accept this option, and wishes to
pay the same down payment now as previously, what should be the amount
of each of her 15 equal regular payments? (Hint: One way to approach this
problem — not the only way — is to express the [deferred] payment annuity
as the difference of two annuities, both with first payment 1 month from now.)

Solution:

(a) Let D denote the down payment. Then an equation of value is

1 − (1.0166666667)−15
D + 25a15 20 % = 500 ⇒ D = 500 − 25 ·
12 0.0166666667
⇒ D = 500 − 25(13.17557219) = 170.6106952 ,

so the down payment will be 170.61.


(b) Let the amount of each of the regular payments be X. We can think of the
payment annuity as part of an annuity beginning 1 month from now, but
extending for 26 months; we will have to subtract the present value of the
first 11 payments. An equation of value now is
³ ´
170.61 + X · a26 20 % − a11 20 % = 500
12 12

329.39
⇒ X=³ −26
´ ³ ´
1−(1.066666667) 1−(1.066666667)−11
0.0166666667
− 0.0166666667

329.39(0.0166666667)
⇒ X= = 29.98509009 ,
(1.066666667)−11 − (1.066666667)−26

so the regular monthly payments should be in the amount 29.99. We can also
think of each of the previous payments now being delayed by 11 months; thus
the payments will each be

25(1.0166666667)11 = 29.98502678

or 29.99.
Information for Students in MATH 329 2009 01 25

8 Solutions, Third Problem Assignment


Distribution Date: Wednesday, March 04th, 2009
Solutions were to be submitted by Monday, March 02nd, 2009

1. X borrows 20,000 and starts repaying it with semi-annual payments of 1,200, at an


effective annual rate of 6%. After he makes 10 payments he is laid off from his job,
and asks the lenders to permit him to delay repayment. The lenders permit him to
skip just one payment, but tell him that interest rates have risen, and that they will
agree to the delay only if X agrees to accept the new rates as of the last payment
made. X counters that he can’t pay more than 1,200 per half-year, and agrees that,
when he resumes payments after the one missed payment, the payments will be at
a constant level of not more than 1,200 per half year at a nominal annual rate of
8%, compounded semi-annually.

(a) Determine, as of the date of the last payment made at the effective annual
rate of 6%, the amount of loan still outstanding.
(b) Determine, as of the date of the missed payment, the amount L of loan still
outstanding.
(c) By solving an inequality for the smallest value of n such that 1200 · an ≥ L,
determine the minimum number n of payments that X will still have to make.
(d) For the minimum number of payments n that you have determined above,
find the exact amount of each of the equal payments.
(e) Suppose that X had had the foresight to know, at the time he was borrow-
ing 20,000, that he would not be able to make the projected 11th payment.
Determine, as of the original date of the loan, the number m of payments he
would have to make after the missed payment, if all except the last of the
10 + m payments are to be equal 1,200, and the last “drop” payment could
be smaller.
(f) Continuing the discussion in the previous item, suppose that, knowing that
he was going to have to make 10 + 0 + m payments not to exceed 1200, X
contracted at the time of the loan to make all of these payments equal (except
for the payment of 0 five and one-half years into the loan). What would be
the amount of these equal payments? (Hint: There are different ways of
approaching this problem. One method would be to proceed as above:
• Assume the first 10 payments are at the full amount of 1,200, and deter-
mine the amount L outstanding at the time of the missed payment.
• Determine the smallest m such that 1200 · am ≥ L.
Information for Students in MATH 329 2009 01 26

• Then determine the exact level amount of the 10 + 0 + m payments (re-


membering that there is a hiatus between the 10th payment and the
next).)

Solution:

(a) The effective annual interest rate is 6%. The corresponding nominal annual
µ ¶2
(2) i(2)
interest rate, compounded semi-annually, is i , given by 1 + = 1.06;
2
(2) 1
the effective semi-annual interest rate is i 2 = (1.06) 2 − 1, which I will denote
by j: computations of the value of the annuity of payments must be based on
this interest rate; (1 + j)2 = 1.06. Immediately after the 10th payment the
value of the payments already made is
(1 + j)10 − 1
1200 · s10 j = 1200 ·
j
(1.06)5 − 1
= 1200 · 1
(1.06) 2 − 1
= 13729.00252

The amount of the loan outstanding just after the 10th payment is

20000(1.06)5 − 13729.00252 = 13035.50904

or 13,035.51. This amount will be repaid by an annuity-immediate of level


payments of at most 1,200, one payment deferred, at an effective interest rate
per period of 8%
2
= 4% per period.
(b) At the time of the missed payment the outstanding loan has grown to
13, 035.51(1.04) = 13556.9304 or L = 13, 556.93.
(c)

1 − (1.04)−n
1200 · an 4% ≥ 13, 556.93 ⇔ 1200 · ≥ 13, 556.93
0.04
13556.93 · 0.04
⇔ (1.04)−n ≤ 1 − = .5481023333
1200
ln .5481023333
⇔ −n ≤ = −15.33101317
ln 1.04
⇔ n ≥ 15.33101317

Thus it is going to take at least 16 more payments to repay the loan at the
new interest rate.
Information for Students in MATH 329 2009 01 27

13, 556.93
(d) The remainder of the loan can be repaid in 16 equal payments of =
a16 4%
13, 556.93 · (0.04)
= 1163.455722, i.e., a new level payment of 1,163.46.
1 − (1.04)−16

(e) Under the original effective semi-annual interest rate of j = 1.06 − 1,
the outstanding
√ loan at the time of the missed payment would be L =
13, 035.51 1.06 = 13420.87896 or 13, 420.88.
m
1 − (1.06)− 2
1200 · am j ≥ 13, 420.88 ⇔ 1200 · √ ≥ 13, 420.88
1.06 − 1

−m 13420.88 · ( 1.06 − 1)
⇔ (1.06) 2 ≤ 1 − = .6693652806
1200
−2 ln .6693652806
⇔ m≥ = 13.77837241 ,
ln 1.06
so 14 payments are needed after the missed payment.
(f) Suppose the amount of each payment will be Y . We can think of a 25-payment
annuity-immediate with the projected 11th payment missing.
³ ´
− 11
Y a25j − 1.06 2 = 20000
so
20000
Y = − 25
= 1192.432517 ,
1−(1.06) 2 11

1.06−1
− (1.06)− 2
and the level payment will be 1,192.43.
2. Suppose that you know that, for some n and for some interest rate i, än i = X, and
sn+1 i = Y . Use your (algebraic) knowledge of the values of the annuity functions
to determine a formula which expresses i in terms of X, Y . Verify your work by
selecting values of i and n and computing X and Y , and checking that the formula
is correct in that special case.
Solution: There will be many ways of solving this problem, but the solutions must
be equivalent. To begin with, it would be useful to have the annuity information
all for the same numbers of payments. This can be done by, for example, recalling
that sn+1 i includes a last payment of 1 which has not had time to accumulate.
Hence s̈n i = sn+1 i − 1 = Y − 1. Since
X = än i = v n · s̈n i = v n (Y − 1) ,
we now know that
Y −1
(1 + i)n = v −n = .
X
Information for Students in MATH 329 2009 01 28

Hence
än X X(Y − 1)
d= = X
= .
1 − vn 1 − Y −1 Y −1−X
X(Y −1)
Since (1 − d)(1 + i) = 1, it follows that 1 + i = Y (X−1)+1
and

Y −X −1
i= .
Y (X − 1) + 1

To verify, I am selecting the value i = 4%, and the number of years as n = 10. 8
1+i 1.04 ¡ ¢
Then X = än i = · (1 − v n ) = · 1 − 1.04−10 = 8.435331611. Y =
i 0.04
(1 + i)n+1 − 1 (1.04)11 − 1 Y −X −1
sn+1 i = = = 13.48635140. Then =
i 0.04 Y (X − 1) + 1
13.48635140 − 8.435331611 − 1
= 0.03999999993, showing a small rounding error
13.48635140 × 7.435331611 + 1
from 4%.

3. George has just begun working at the type of job he has dreamed of, where he would
work for 20 years and then retire for life. He plans to invest 1,000 at the beginning
of every month that he works. When he retires he will spend 150,000 to purchase a
chalet in the mountains, and will begin to draw fixed monthly payments from the
balance. The first payment he receives will be one month after he retires — i.e., 1
month after he draws the capital for his chalet. The fixed payments he receives will
begin a perpetuity, which will continue to his estate, forever (unless his executors
negotiate a termination with the trust company that holds the account). Suppose
that the interest rate during the years when he is working is 6%, compounded
monthly, and that the interest rate for the perpetuity-immediate is 7% annual,
effective. Determine, to the nearest unit, the constant amount of his perpetuity
payments.
Solution: At the time of his retirement, George’s monthly investments have ac-
cumulated to 1000 · s̈240 1 % , so the amount available to purchase his perpetuity is
2
1000 · s̈240 1 % − 150000 . Denote the amount of the level monthly payments of his
2
1
perpetuity by X, and observe that the effective monthly interest rate is (1.07) 12 −1.
An equation of value just after the removal of the funds for the chalet is
X
1000 · s̈240 1 % − 150000 = 1
2
(1.07) 12 − 1
8
It might be tempting to verify with a trivial case, e.g., n = 0, where X = 0 and Y = 1; but that
leads to an indeterminate ratio — clearly the possibility that Y − X = 1 should have been excluded
somewhere in the problem.
Information for Students in MATH 329 2009 01 29

hence
³ 1
´ µ (1.005)240 − 1

X = (1.07) 12 − 1 · 1000(1.005) · − 150000 = 1777.39 ,
0.005
so the monthly payments under the perpetuity are in the amount of 1764.

4. Juliette was born on February 29th, 1980, and has always felt sorry for people
born on that day, because of the paucity of their birthdays. After making her
first billion, she decided to endow the Leap Year Foundation, with the purpose of
making awards to persons with 29 February as their birthday. Awards totalling
20,000 will be made on New Year’s Day of every leap year — assume that a leap
year is one divisible by 4, excluding years divisible by 100 except those divisible
by 400, which are leap years. If the Foundation makes its first awards in 2012,
determine how much money Juliette needs to contribute on April 1st, 2009 to fund
the Foundation, if the effective annual rate of interest is taken to be i = 5%.
Solution:

(a) As of the year 2012, the cost of awards in every year divisible by 4 would be
20000 · ä∞ j , where j1 is the effective rate of interest for a 4-year period, i.e.,
it is 4i( 4 ) = (1.05)4 − 1 = 0.21550625 or 21.550625%; as of April 1st, 2009,
1

this cost is
µ ¶
−2 34 1
(1.05) · 20000 · 1 + = 98, 640.84 .
0.21550625

(b) Funding not required for the years divisible by 100 (including those divisible
by 400) would be worth 20000·ä∞ j2 as of the year 2100, where j2 is the effective
rate of interest for a 100-year period, i.e., j2 = (1.05)100 − 1 = 130.5012578;
as of April 1st, 2009, this cost is
µ ¶
−90 43 1
(1.05) · 20000 · 1 + = 240.67 .
130.5012578

(c) Fund that is required for the years divisible by 400 would be worth 20000·ä∞ j3
as of the year 2400, where j3 is the effective rate of interest for a 400-year
period, i.e., j3 = (1.05)400 − 1 = 299, 033, 350.2488392; as of April 1st, 2009,
this cost is
µ ¶
−390 43 1
(1.05) · 20000 · 1 + = 0.0001050 ,
299, 033, 350.2488392
which is negligible.
Information for Students in MATH 329 2009 01 30

Thus the cost to Juliette on April 1st, 2009, is 98, 640.84 − 240.67 + 0 = 98, 400.17.

5. The owner of a perpetuity-immediate which pays 10,000 per year wishes to adjust
the payments so that he receives 12 equal monthly payments through the year. If
the monthly payments work out to 825 each, what is the effective annual interest
rate? (It is intended that you determine this rate by first “trapping” the correct
rate in an interval between two interest rates, and then repeatedly cut the interval
in half by testing the midpoint. Finding the ends of the first interval you work
with is easy, particularly if you don’t demand that it be small. For example, see
what happens if you take an interest rate of 0%. Apply this “bisection” method
10 times, and use the result to obtain an approximation to the effective annual
interest rate the question requests. The validity of this procedure derives from the
Intermediate Value Theorem, based on the continuity of the inverse function of sn i ,
i.e., of i as a function of sn i for fixed n.)
Solution: Let j denote the effective monthly interest rate. Then 825 · s12 j =
10000, so s12 j = 12.12121212... Clearly j > 0, since s12 0 = 12 exactly. s12 0.01 =
1.0112 − 1
= 12.6825, so we know the correct interest rate will lie between 0% and
0.01
1%.
1.00512 − 1
(a) Next we test the midpoint of the interval [0, 0.01]: s12 0.005 = =
0.005
12.33556; since this exceeds 12.1212, the correct value will be in the interval
[0, 0.005].
(b) We test the midpoint of this half-interval, i.e., i2 = 0.0025, and find s12 0.0025 =
1.002512 − 1
= 12.1664, which is still larger than 12.1212.... The correct value
0.0025
will lie in the interval [0, 0.0025].
(c) We test the midpoint of this interval, i.e., i3 = 0.00125, and find s12 0.00125 =
1.0012512 − 1
= 12.0828 < 12.121212..., implying that the solution lies in the
0.00125
interval [0.00125, 0.0025].
1.00187512 − 1
(d) We test the midpoint 0.001875 of [0.00125, 0.0025]: s12 0.001875 =
0.001875
= 12.12453 > 12.121212, showing that the correct value is in the interval
[0.00125, 0.001875].
1.001562512 − 1
(e) We test the mid-point 0.0015625: s12 0.0015625 = = 12.10366 <
0.0015625
12.121212, showing that the correct value is in the interval [0.0015625, 0.001875].
Information for Students in MATH 329 2009 01 31

1.0017187512 − 1
(f) We test the mid-point 0.00171875: s12 0.00171875 = = 12.11409 <
0.00171875
12.121212, showing that the correct value is in the interval [0.00171875, 0.001875].
1.00179687512 − 1
(g) We test the mid-point 0.001796875: s12 0.001796875 = =
0.001796875
12.11931. The correct interest rate is in the interval [0.001796875, 0.001875].
1.00183593712 − 1
(h) We test the mid-point 0.001835937: s12 0.001835937 = =
0.001835937
12.12192 > 12.121212, so the correct value is in the interval [0.001796875, 0.001835937].
1.0018164112 − 1
(i) We test the mid-point 0.00181641: s12 0.00181641 = = 12.12058 <
0.00181641
12.121212 so the solution will be in the interval [0.00181641, 0.001835937],
with mid-point 0.00182617.
1.0018261712 − 1
(j) We test 0.00182617: s12 0.00182617 = = 12.12126.
0.00182617
The effective annual interest rate is approximately (1.00182617)12 −1 = 0.022135489,
i.e., approximately 2.2%.
Information for Students in MATH 329 2009 01 32

9 Fourth Problem Assignment


Distribution Date: Friday, March 06th, 2009
Solutions are to be submitted by Monday, March 23rd, 2009

1. It is New Years Day, and Jacques and Jacqueline have just taken out a mortgage
on their new condo, in the amount of 100,000. They have agreed that, in any
calendar year except possibly the last year of the mortgage, Jacques’ contribution
will be exactly twice that of Jacqueline’s. The annual interest rate, compounded
semi-annually, is 6%,

(a) Suppose that Jacques will always pay 2X on the last day of April; and Jacque-
line will pay X at the end of December. Determine the value of X if the
mortgage is to be paid off in 20 years.
(b) Suppose that Jacques will always pay 6000 on the last day of April; and
Jacqueline will pay 3000 on the last day of December, until a final year wherein
the payments of Jacques and Jacqueline — still in the ratio of 2:1 — would
not be as large as in previous years. Determine when that final year occurs,
and what the respective payments of Jacques and Jacqueline will be that year.
(c) Suppose that Jacques will always pay 6000 on the last day of April; and
Jacqueline will pay 3000 on the last day of December, until a time when the
payment which is due exceeds the amount owing on the mortgage, in which
case the payment is reduced to simply pay off the mortgage. Determine when
this final year occurs, which of Jacques and Jacqueline has a reduced payment,
and what that reduced payment is in that final year.

2. Mary is contributing to an RSSP for her retirement in 3 years by purchasing semi-


annually a Certificate of Deposit which expires on the date of her retirement. She
invests 10,000 every half-year, with the last investment just one-half year before
retirement. Suppose that the interest rate on the certificate she is purchasing now
is 3%, and that the rates fall by 12 % each half-year, until the last half-year before
retirement, when the rate reaches just 12 %; these rates, each applying to a half-year
period, are all nominal annual rates, compounded semi-annually. Determine the
magnitude of Mary’s account on the day she retires.

(a) Compute the value of the account at retirement, using the Yield Curve Method
[4, p. 96].
(b) Compute the value of the account at retirement, using the Portfolio Rate
Method [4, p. 96].

UPDATED TO April 14, 2009


Information for Students in MATH 329 2009 01 33

(Of course, Mary doesn’t have the choice of which method will be used: in either the
fine print of her contract or in the established practices of the banking institution
the method will be made more explicit.)

3. Jack’s mortgage, in the amount of 80,000, requires regular payments of 600 every
month, at an interest rate of 8%, compounded semiannually.

(a) Suppose that Jack has agreed that the last payment will be a “balloon” pay-
ment of at least 600. Determine when that payment is due, and its amount.
(b) Suppose that Jack has agreed that the last payment will be a ”drop” payment
of at most 600. Determine when that payment is due, and its amount.

4. Under a nominal interest rate of 6% compounded continuously an annuity pays


10,000 per year for 5 years. Determine

(a) The value of the annuity 2 years before the payment begins.
(b) The total annual payment under a continuous perpetuity at the same rate of
interest, whose value at time 0 is the same as this annuity.
(c) The time t such that the value at time 0 of the portion of the annuity up to
time t is equal to the value at time 0 of the portion of the annuity from time
t to time 5.

5. Gasoline costs 1.00 per litre now, and the price will increase by 0.05 per week for
the next year. If my tank holds 65 litres, and money is worth 4.5% per annum
effective, what will be the value 52 weeks from now, just after the tank has been
filled, of the gasoline that I have purchased, if I fill the tank completely every 2
weeks?
Information for Students in MATH 329 2009 01 34

10 Class Tests
10.1 Class Test, Version 1
McGILL UNIVERSITY, FACULTY OF SCIENCE
CLASS TEST in MATH 329, THEORY OF INTEREST

EXAMINER: Professor W. G. Brown DATE: Wednesday, 13 March, 2009.


TIME: 45 minutes, 14:35→15:20
FAMILY NAME:
GIVEN NAMES:
STUDENT NUMBER:

Instructions

• The time available for writing this test is about 45 minutes.

• This test booklet consists of this cover, Pages 36 through 39 containing questions together
worth 66 marks; and Page 40, which is blank.

• Show all your work. All solutions are to be written in the space provided on the page where
the question is printed. When that space is exhausted, you may write on the facing page, on
the blank page, or on the back cover of the booklet, but you must indicate any continuation
clearly on the page where the question is printed! (Please inform the instructor if you find
that your booklet is defective.)

• All your writing — even rough work — must be handed in.

• Calculators. While you are permitted to use a calculator to perform arithmetic and/or ex-
ponential calculations, you must not use the calculator to calculate such actuarial functions
as an i , sn i , etc. without first stating a formula for the value of the function in terms of
exponentials and/or polynomials involving n and the interest rate. You must not use your
calculator in any programmed calculations. If your calculator has memories, you are expected
to have cleared them before the test. Your solutions should include sufficient detail that the
examiner can conclude that you have not used built-in annuity functions from a financial
calculator.

• In your solutions to problems on this test you are expected to show all your work. You are
expected to simplify algebraic and numerical answers as much as you can.

• Your neighbours may be writing a version of this test which is different from yours.
Information for Students in MATH 329 2009 01 35

PLEASE DO NOT WRITE INSIDE THIS BOX


1(a) 1(b) 1(c) 2(a) 2(b) 2(c) 3(a) 3(b) 4 Total

/4 /4 /4 /6 /6 /6 /6 /12 /18 /66


Information for Students in MATH 329 2009 01 36

1. Showing your work in detail, determine each of the following; the rates you de-
termine should be accurate to 4 decimal places, or as a percentage accurate to 2
decimal places:

(a) [4 MARKS] the effective interest rate per 2-year period corresponding to a
nominal discount rate, compounded quarterly, of d = 3.%
(b) [4 MARKS] the nominal annual interest rate, compounded every two months,
corresponding to an effective annual interest rate of i = 6%
(c) [4 MARKS] the effective monthly interest rate corresponding to nominal an-
nual rate of interest of 4%, compounded continuously.
Information for Students in MATH 329 2009 01 37

2. For each of the following sequences of payments, determine, as of the given time,
and for the given interest or discount rate, the value, showing all of your work.
Before determining the numeric value you are expected

• to express the value using standard symbols, and


• to state the formula that you have used to evaluate the standard symbol.

(a) [6 MARKS] the value now of 25 payments of 5 each at the end of every year
— the first payment being exactly 2 years from now — at an interest rate of
3%;
(b) [6 MARKS] the value four years after the last payment of 12 payments of 3 at
the end of every half-year, the first to be paid 2 years from now, at a nominal
interest rate of 7% compounded every 3 months;
(c) [6 MARKS] 200 payments of 1 at the end of every month, as of the date of the
120th payment, which has just been made; the interest rate is 6% compounded
monthly.
Information for Students in MATH 329 2009 01 38

3. (a) [12 MARKS] The accumulated value just after the last payment under a 20-
year annuity of 10,000 per year, paying interest at the rate of 5% per annum
effective, is to be used to purchase a perpetuity, first payment to be made
2 years after the last payment under the annuity. Showing all your work,
determine the size of the payments under the perpetuity, assuming that the
interest rate from the time of the last payment under the 20-year annuity is
4%.
(b) [6 MARKS] An annuity at interest rate i consists of payments of 10 now, 13
at the end of one year, 16 at the end of two years, increasing by a constant
amount until the last payment in the amount of 70, is to be evaluated as of
6 years ago. Express its value in terms of symbols (Ia)n , (Is)n , an , sn , i but
do not evaluate.
Information for Students in MATH 329 2009 01 39

4. [18 MARKS] A loan of 8000 is to be repaid by semi-annual payments of 430 to


commence at the end of the 1st half-year, and to continue thereafter for as long
as necessary. Find the time and amount of the final payment if the final payment
is to be no smaller than the regular payments. Assume an interest rate of i = 5%
per half-year.
Information for Students in MATH 329 2009 01 40

continuation page for problem number

You must refer to this continuation page on the page where the problem is printed!
Information for Students in MATH 329 2009 01 41

10.2 Class Test, Version 2


McGILL UNIVERSITY, FACULTY OF SCIENCE
CLASS TEST in MATH 329, THEORY OF INTEREST
EXAMINER: Professor W. G. Brown DATE: Wednesday, 13 March, 2009.
TIME: 45 minutes, 14:35→15:20
FAMILY NAME:
GIVEN NAMES:
STUDENT NUMBER:
Instructions
• The time available for writing this test is about 45 minutes.
• This test booklet consists of this cover, Pages 42 through 45 containing questions together
worth 66 marks; and Page 46, which is blank.
• Show all your work. All solutions are to be written in the space provided on the page where
the question is printed. When that space is exhausted, you may write on the facing page, on
the blank page, or on the back cover of the booklet, but you must indicate any continuation
clearly on the page where the question is printed! (Please inform the instructor if you find
that your booklet is defective.)
• All your writing — even rough work — must be handed in.
• Calculators. While you are permitted to use a calculator to perform arithmetic and/or ex-
ponential calculations, you must not use the calculator to calculate such actuarial functions
as an i , sn i , etc. without first stating a formula for the value of the function in terms of
exponentials and/or polynomials involving n and the interest rate. You must not use your
calculator in any programmed calculations. If your calculator has memories, you are expected
to have cleared them before the test. Your solutions should include sufficient detail that the
examiner can conclude that you have not used built-in annuity functions from a financial
calculator.
• In your solutions to problems on this test you are expected to show all your work. You are
expected to simplify algebraic and numerical answers as much as you can.
• Your neighbours may be writing a version of this test which is different from yours.
PLEASE DO NOT WRITE INSIDE THIS BOX
1 2(a) 2(b) 2(c) 3(a) 3(b) 3(c) 4(a) 4(b) Total

/18 /4 /4 /4 /6 /6 /6 /6 /12 /66


Information for Students in MATH 329 2009 01 42

1. [18 MARKS] A loan of 12,000 is to be repaid by semi-annual payments of 800


to commence immediately, and to continue at the beginning of each half-year for
as long as necessary. Find the time and amount of the final payment if the final
payment is to be no larger than the regular payments. Assume an interest rate of
14% compounded semi-annually.
Information for Students in MATH 329 2009 01 43

2. Showing your work in detail, determine each of the following; the rates you de-
termine should be accurate to 4 decimal places, or as a percentage accurate to 2
decimal places:

(a) [4 MARKS] the effective annual discount rate corresponding to a nominal


interest rate, compounded quarterly, of i = 2.4%
(b) [4 MARKS] the nominal annual discount rate, compounded quarterly, equiv-
alent to an effective semi-annual interest rate of i = 4%
(c) [4 MARKS] the effective semi-annual interest rate corresponding to a force of
interest of δ = 0.04.
Information for Students in MATH 329 2009 01 44

3. For each of the following sequences of payments, determine, as of the given time,
and for the given interest or discount rate, the value, showing all of your work.
Before determining the numeric value you are expected

• to express the value using standard symbols, and


• to state the formula that you have used to evaluate the standard symbol.

(a) [6 MARKS] the value of 20 payments of 6 at the end of every year, the last
one made 3 years ago, at an interest rate of 4%;
(b) [6 MARKS] the value 6 months from now of 34 payments of 10 at the end of
every 3 months, the first to be paid 6 years from now, at a nominal interest
rate of 12% compounded 4 times a year;
(c) [6 MARKS] 150 payments of 2 at the end of every 4 months, as of the date
of the 100th payment, which has just been made; the interest rate is 8%
compounded every 4 months.
Information for Students in MATH 329 2009 01 45

4. (a) [6 MARKS] An annuity at interest rate i consists of payments of 260 now, 235
at the end of 1 year, 210 at the end of 2 years, decreases by a constant amount
until the last payment in the amount of 35, is to be evaluated as of just after
the third payment. Express its value then in terms of symbols (Da)n , (Ds)n ,
an , sn , i, but do not evaluate.
(b) [12 MARKS] The accumulated value just after the last payment under a 14-
year annuity of 9,000 per year, paying interest at the rate of 8% per annum
effective, is to be used to purchase a 20-year annuity at an interest rate of 6%,
first payment to be made 3 years after the last payment under the annuity.
Showing all your work, determine the size of the payments under the 20-year
annuity. Assume that the 6% rate is in effect from the time of the last payment
under the 14-year annuity.
Information for Students in MATH 329 2009 01 46

continuation page for problem number

You must refer to this continuation page on the page where the problem is printed!
Information for Students in MATH 329 2009 01 47

10.3 Class Test, Version 3


McGILL UNIVERSITY, FACULTY OF SCIENCE
CLASS TEST in MATH 329, THEORY OF INTEREST
EXAMINER: Professor W. G. Brown DATE: Wednesday, 13 March, 2009.
TIME: 45 minutes, 14:35→15:20
FAMILY NAME:
GIVEN NAMES:
STUDENT NUMBER:
Instructions
• The time available for writing this test is about 45 minutes.
• This test booklet consists of this cover, Pages 48 through 51 containing questions together
worth 66 marks; and Page 52, which is blank.
• Show all your work. All solutions are to be written in the space provided on the page where
the question is printed. When that space is exhausted, you may write on the facing page, on
the blank page, or on the back cover of the booklet, but you must indicate any continuation
clearly on the page where the question is printed! (Please inform the instructor if you find
that your booklet is defective.)
• All your writing — even rough work — must be handed in.
• Calculators. While you are permitted to use a calculator to perform arithmetic and/or ex-
ponential calculations, you must not use the calculator to calculate such actuarial functions
as an i , sn i , etc. without first stating a formula for the value of the function in terms of
exponentials and/or polynomials involving n and the interest rate. You must not use your
calculator in any programmed calculations. If your calculator has memories, you are expected
to have cleared them before the test. Your solutions should include sufficient detail that the
examiner can conclude that you have not used built-in annuity functions from a financial
calculator.
• In your solutions to problems on this test you are expected to show all your work. You are
expected to simplify algebraic and numerical answers as much as you can.
• Your neighbours may be writing a version of this test which is different from yours.
PLEASE DO NOT WRITE INSIDE THIS BOX
1(a) 1(b) 2 3(a) 3(b) 3(c) 4(a) 4(b) 4(c) Total

/6 /12 /18 /4 /4 /4 /6 /6 /6 /66


Information for Students in MATH 329 2009 01 48

1. (a) [6 MARKS] An annuity at interest rate i consists of payments of 24 now, 29


at the end of one year, 34 at the end of two years, increasing by a constant
amount until the last payment in the amount of 224, and is to be evaluated
as of 4 years ago. Express its value then in terms of symbols (Ia)n , (Is)n , an ,
sn , i, but do not evaluate.
(b) [12 MARKS] The accumulated value just after the last payment under a 22-
year annuity of 11,000 per year, paying interest at the rate of 8% per annum
effective, is to be used to purchase a perpetuity at an interest rate of 6%,
first payment to be made at the same time as the last payment under the
annuity. Showing all your work, determine the size of the payments under the
perpetuity.
Information for Students in MATH 329 2009 01 49

2. [18 MARKS] A loan of 9,000 is to be repaid by annual payments of 1,000 to


commence at the end of the 1st year, and to continue thereafter for as long as
necessary. Find the time and amount of the final payment if the final payment is
to be smaller than the regular payments. Assume i = 6%.
Information for Students in MATH 329 2009 01 50

3. Showing your work in detail, determine each of the following; the rates you de-
termine should be accurate to 4 decimal places, or as a percentage accurate to 2
decimal places:

(a) [4 MARKS] the effective annual interest rate, corresponding to an nominal


annual interest rate of i = 6% compounded every 4 months.
(b) [4 MARKS] the effective monthly interest rate corresponding to a force of
interest of δ = 0.30.
(c) [4 MARKS] the effective annual interest rate corresponding to d(4) = 3.20%
Information for Students in MATH 329 2009 01 51

4. For each of the following sequences of payments, determine, as of the given time,
and for the given interest or discount rate, the value, showing all of your work.
Before determining the numeric value you are expected

• to express the value using standard symbols, and


• to state the formula that you have used to evaluate the standard symbol.

(a) [6 MARKS] the value 3 months ago of 24 payments of 5 at the end of every
half-year, the first to be paid 7 years from now, at a nominal interest rate of
9% compounded semi-annually;
(b) [6 MARKS] 240 payments of 10 at the end of every month, as of the date
of the 150th payment, which has just been made; the interest rate is 12%
compounded monthly
(c) [6 MARKS] the value now of 18 payments of 500 at the end of every year, the
first payment 8 years from now, at an interest rate of 5%
Information for Students in MATH 329 2009 01 52

continuation page for problem number

You must refer to this continuation page on the page where the problem is printed!
Information for Students in MATH 329 2009 01 53

10.4 Class Test, Version 4


McGILL UNIVERSITY, FACULTY OF SCIENCE
CLASS TEST in MATH 329, THEORY OF INTEREST
EXAMINER: Professor W. G. Brown DATE: Wednesday, 13 March, 2009.
TIME: 45 minutes, 14:35→15:20
FAMILY NAME:
GIVEN NAMES:
STUDENT NUMBER:
Instructions
• The time available for writing this test is about 45 minutes.
• This test booklet consists of this cover, Pages 54 through 57 containing questions together
worth 66 marks; and Page 58, which is blank.
• Show all your work. All solutions are to be written in the space provided on the page where
the question is printed. When that space is exhausted, you may write on the facing page, on
the blank page, or on the back cover of the booklet, but you must indicate any continuation
clearly on the page where the question is printed! (Please inform the instructor if you find
that your booklet is defective.)
• All your writing — even rough work — must be handed in.
• Calculators. While you are permitted to use a calculator to perform arithmetic and/or ex-
ponential calculations, you must not use the calculator to calculate such actuarial functions
as an i , sn i , etc. without first stating a formula for the value of the function in terms of
exponentials and/or polynomials involving n and the interest rate. You must not use your
calculator in any programmed calculations. If your calculator has memories, you are expected
to have cleared them before the test. Your solutions should include sufficient detail that the
examiner can conclude that you have not used built-in annuity functions from a financial
calculator.
• In your solutions to problems on this test you are expected to show all your work. You are
expected to simplify algebraic and numerical answers as much as you can.
• Your neighbours may be writing a version of this test which is different from yours.
PLEASE DO NOT WRITE INSIDE THIS BOX
1(a) 1(b) 1(c) 2(a) 2(b) 3 4(a) 4(b) 4(c) Total

/6 /6 /6 /6 /12 /18 /4 /4 /4 /66


Information for Students in MATH 329 2009 01 54

1. For each of the following sequences of payments, determine, as of the given time,
and for the given interest or discount rate, the value, showing all of your work.
Before determining the numeric value you are expected

• to express the value using standard symbols, and


• to state the formula that you have used to evaluate the standard symbol.

(a) [6 MARKS] 72 payments of 1 at the end of every 3 months, as of the date of


the 24th payment, which has just been made; interest is at a nominal annual
rate of 12% compounded every 4 months.
(b) [6 MARKS] the value of 24 payments of 7 at the end of every half-year,
the last one having just been made, at a nominal annual interest rate of 6%
compounded quarterly;
(c) [6 MARKS] the value now of 50 payments of 2 at the end of every 8 months,
the first to be paid 2 years from now, at a nominal interest rate of 4.5%
compounded 3 times every 2 years.
Information for Students in MATH 329 2009 01 55

2. (a) [6 MARKS] An annuity at interest rate i consists of payments of 1,118 now,


1,112 at the end of 1 year, 1,106 at the end of 2 years, decreasing until the last
payment in the amount of 1,058, the totality to be evaluated as of the time
of the payment of 1,106. Express its value then in terms of symbols (Da)n ,
(Ds)n , an , sn , i, but do not evaluate.
(b) [12 MARKS] The accumulated value just after the last payment under a 19-
year annuity of 1000 per year, paying interest at the rate of 8% per annum
effective, is to be used to purchase a 10-year annuity-immediate at an interest
rate of 6%, first payment to be made 1 year after the last payment under the
19-year annuity. Showing all your work, determine the size of the payments
under the 10-year annuity. Assume that the 6% rate applies from the time of
the last payment under the 8% annuity.
Information for Students in MATH 329 2009 01 56

3. [18 MARKS] A loan of 18,000 is to be repaid by annual payments of 2,500 to


commence immediately, and to continue at the beginning of each year for as long
as necessary. Find the time and amount of the final payment if the final payment
is to be no smaller than the regular payments. Assume i = 13%.
Information for Students in MATH 329 2009 01 57

4. Showing your work in detail, determine each of the following; the rates you de-
termine should be accurate to 4 decimal places, or as a percentage accurate to 2
decimal places:

(a) [4 MARKS] the nominal annual interest rate, compounded quarterly, equiva-
lent to an effective semi-annual discount rate of d = 4%
(b) [4 MARKS] the effective semi-annual interest rate corresponding to a force of
interest of δ = 0.14.
(c) [4 MARKS] the effective annual discount rate corresponding to a nominal
interest rate, compounded quarterly, of i = 2.5%
Information for Students in MATH 329 2009 01 58

continuation page for problem number

You must refer to this continuation page on the page where the problem is printed!
Information for Students in MATH 329 2009 01 59

11 Draft Solutions to Problems on the Class Tests


The tests were administered on 13 March, 2009.

Distribution Date: Wednesday, 18 March, 2009


corrected on 14 April, 2009, and subject to further correction

11.1 Version 1
1. Showing your work in detail, determine each of the following; the rates you de-
termine should be accurate to 4 decimal places, or as a percentage accurate to 2
decimal places:

(a) [4 MARKS] the effective interest rate per 2-year period corresponding to a
nominal discount rate, compounded quarterly, of d = 3.%
(b) [4 MARKS] the nominal annual interest rate, compounded every two months,
corresponding to an effective annual interest rate of i = 6%
(c) [4 MARKS] the effective monthly interest rate corresponding to nominal an-
nual rate of interest of 4%, compounded continuously.

Solution:

(a) d(4) = 3% ⇒ effective discount rate for 3 months is 43 %. As a 2-year period


contains 21 = 8 disjoint 3-month periods, the accumulation factor for a 2-year
4
period is (1 − 0.0075)−8 = 1.062076688, so the effective interest rate for a
2-year period is 6.21%.
µ ¶6 ³ ´
i(6) 1
(b) 1 + = 1 + i = 1.06 ⇒ i(6) = 6 (1.06) 6 − 1 = 0.058552764, so the
6
nominal annual interest rate, compounded every two months is 5.86%.
(c) When a nominal interest rateµof 4% is ¶ compounded continuously, the annual
m
0.04
accumulation factor is lim 1 + = e0.04 ; hence the effective accu-
m→∞ m
1 0.04
0.04 12
mulation factor for 1 month is (e ) = e 12 = 1.003338895. The effective
monthly interest rate is 0.334%.

2. For each of the following sequences of payments, determine, as of the given time,
and for the given interest or discount rate, the value, showing all of your work.
Before determining the numeric value you are expected

• to express the value using standard symbols, and


Information for Students in MATH 329 2009 01 60

• to state the formula that you have used to evaluate the standard symbol.

(a) [6 MARKS] the value now of 25 payments of 5 each at the end of every year
— the first payment being exactly 2 years from now — at an interest rate of
3%;
(b) [6 MARKS] the value four years after the last payment of 12 payments of 3 at
the end of every half-year, the first to be paid 2 years from now, at a nominal
interest rate of 7% compounded every 3 months;
(c) [6 MARKS] 200 payments of 1 at the end of every month, as of the date of the
120th payment, which has just been made; the interest rate is 6% compounded
monthly.

Solution:
¯ 5 1 − (1.03)−25
(a) 5 ·1 ¯a25 3% = · = 84.52984317 or 84.53.
1.03 0.03
(b) It’s irrelevant when the payments start, since we are evaluating as of 4 years af-
¡ ¢2
ter the last payment. The effective interest rate per half-year is j = 1 + 0.07
4

1 = 0.035306250. The value just after the 12th payment is 3 · s12 j = 3 ·
(1 + j)12 − 1
, and we must accumulate this through 4 × 4 = 8 quarter-years.
j
The value at that time will be
µ ¶16 ¡ ¢24
0.07 1 + 0.07
4
−1
1+ ·3· = 57.92186862 .
4 0.035306250
6
(c) The effective interest rate per month is 12 = 0.5%. The value of the 200
payments 1 month before the first of them is a200 0.5% , so the value just after
the 120th payment is

1 − (1.005)−200
(1.005)120 · a200 0.5% = (1.005)120 · = 229.6816522 .
0.005

3. (a) [12 MARKS] The accumulated value just after the last payment under a 20-
year annuity of 10,000 per year, paying interest at the rate of 5% per annum
effective, is to be used to purchase a perpetuity, first payment to be made
2 years after the last payment under the annuity. Showing all your work,
determine the size of the payments under the perpetuity, assuming that the
interest rate from the time of the last payment under the 20-year annuity is
4%.
Information for Students in MATH 329 2009 01 61

(b) [6 MARKS] An annuity at interest rate i consists of payments of 10 now, 13


at the end of one year, 16 at the end of two years, increasing by a constant
amount until the last payment in the amount of 70, is to be evaluated as of
6 years ago. Express its value in terms of symbols (Ia)n , (Is)n , an , sn , i but
do not evaluate.
Solution:

(a) If we interpret the perpetuity as a perpetuity-immediate, then the amount


available for its purchase is

(1.05)20 − 1
(1.04) × 10000 × s20 5% = 10400 · = 343, 885.92 .
0.05
If X is the payment size under the perpetuity, then an equation of value is
X
343, 885.92 = , so the payment size is 0.04(343, 885.92) = 13, 755.44.
0.04
70 − 10
(b) The total number of payments under this increasing annuity is +1 =
3
21. The value of this increasing annuity one year ago was 3(Ia)21 + 7a21 . To
obtain its value as of 6 years ago we¡ have to discount
¢ back through 5 more
−5
years, obtaining the value (1 + i) 3(Ia)21 + 7a21 .

4. [18 MARKS] A loan of 8000 is to be repaid by annual payments of 430 to commence


at the end of the 1st half-year, and to continue thereafter for as long as necessary.
Find the time and amount of the final payment if the final payment is to be no
smaller than the regular payments. Assume the interest rate per half-year is i = 5%.
Solution: Suppose that the final payment is the nth. Then n is the largest integer
such that9
8000
8000 > 430 · an ⇔ 1 − (1.05)−n < · (0.05)
430
⇔ (1.05)−n > 1 − 0.9302325582
ln(0.06976)
⇔ n<− = 54.57222523.
ln(1.05)

Hence n = 54. The last payment is equal to


µ ¶
54
¡ ¢ 54 (1.05)54 − 1
430+(1.05) 8000 − 430 · a54 = 8000(1.05) +430 1 − = 666.78 .
0.05
9
It’s not enough to state an inequality for n — you need to make it clear which n you will choose
once you have solved the inequality.
Information for Students in MATH 329 2009 01 62

11.2 Version 2
1. [18 MARKS] A loan of 12,000 is to be repaid by semi-annual payments of 800
to commence immediately, and to continue at the beginning of each half-year for
as long as necessary. Find the time and amount of the final payment if the final
payment is to be no larger than the regular payments. Assume an interest rate of
14% compounded semi-annually.
Solution: Let n be the number of the last payment, which is to be a drop payment.
Then an inequality of value as of time 0 is 800än 7% ≥ 12, 000, where n is to be the
smallest10 integer with this property. Solving this inequality we obtain
15(0.07)
800än 7% ≥ 12, 000 ⇔ (1.07)−n ≤ 1 − = 0.0186915888
1.07
ln(0.0186915888)
⇔ n≥− = 58.81999926 .
ln(1.07)
Thus the last payment will be the 59th. Since the 1st payment is at time 0, the
59th payment is at the beginning of the 59th year, i.e.: it is at time 58. The amount
of that payment will be
µ ¶
58 58 (1.07)58 − 1
12, 000(1.07) − 800s̈58 = 12, 000(1.07) − 800(1.07) = 659.96
0.07

2. Showing your work in detail, determine each of the following; the rates you de-
termine should be accurate to 4 decimal places, or as a percentage accurate to 2
decimal places:

(a) [4 MARKS] the effective annual discount rate corresponding to a nominal


interest rate, compounded quarterly, of i = 2.4%
(b) [4 MARKS] the nominal annual discount rate, compounded quarterly, equiv-
alent to an effective semi-annual interest rate of i = 4%
(c) [4 MARKS] the effective semi-annual interest rate corresponding to a force of
interest of δ = 0.04.

Solution:

(a) The effective annual interest rate is (1.006)4 − 1 = 0.024216865. The ef-
fective annual discount rate d is given by (1 − d)(1.024216865) = 1, so
d = 0.0236442748 or 2.36% to the desired accuracy.
10
It’s not enough to state an inequality for n — you need to make it clear which n you will choose
once you have solved the inequality.
Information for Students in MATH 329 2009 01 63

(b) An effective semi-annual interest rate of 4% produces an annual accumulation


factor of (1.04)2 = (1 − d)−1 , so d = 1 − (1.04)−2 = 0.0754 is the effective
³ (4)
´4 2
annual discount rate. Since 1 − d 4 = 1 − d = (1.04)− 4 , d(4) = 0.0776773
or 7.77%.
R 0.5 0.5
(c) The accumulation factor for one-half year is e 0 δ dt = e[0.04t]0 = e0.02 =
1.020201340, so the effective semi-annual interest rate is 2.02%.
3. For each of the following sequences of payments, determine, as of the given time,
and for the given interest or discount rate, the value, showing all of your work.
Before determining the numeric value you are expected
• to express the value using standard symbols, and
• to state the formula that you have used to evaluate the standard symbol.

(a) [6 MARKS] the value of 20 payments of 6 at the end of every year, the last
one made 3 years ago, at an interest rate of 4%;
(b) [6 MARKS] the value 6 months from now of 34 payments of 10 at the end of
every 3 months, the first to be paid 6 years from now, at a nominal interest
rate of 12% compounded 4 times a year;
(c) [6 MARKS] 150 payments of 2 at the end of every 4 months, as of the date
of the 100th payment, which has just been made; the interest rate is 8%
compounded every 4 months.

Solution:
(1.04)23 − (1.04)3
(a) 6(1 + i)3 · s20 4% = 6 · = 200.98.
0.04
(b) The effective interest rate per 3 months is 12 4
= 3%. As of 5 years and 9
months from now, the payments are worth 10 · a34 3% ; as of 6 months from
¯ 10 ¡ ¢
now the value is 1024−1−2 ¯·a34 3% = (1.03)−21 − (1.03)−55 = 113.59.
0.03
(c) When the interest rate is stated as being “compounded every 4 months”, the
clear intention is that the rate is a nominal one: the effective interest rate
per 4 months is .08 %. Four months before the first à payment the annuity
3 ¡ ¢
0.08 −150 µ ¶−150 !
1− 1+ 3 0.08
is worth 2a150 8 % = 2 · 0.08 = 37.5 1 − 1 + =
3
3
3
73.55242200. Just after the 100th payment the annuity is worth
µ ¶100
0.08
1+ (73.55242200) = 1022.22 .
3
Information for Students in MATH 329 2009 01 64

(Alternatively, the value could be viewed as


³ ´
2 s100 8 % + a50 8 %
õ 3 3
¶100 µ ¶−50 !
0.08 0.08
= 75 1+ − 1+ = 1022.217167 .
3 3

4. (a) [6 MARKS] An annuity at interest rate i consists of payments of 260 now, 235
at the end of 1 year, 210 at the end of 2 years, decreases by a constant amount
until the last payment in the amount of 35, is to be evaluated as of just after
the third payment. Express its value then in terms of symbols (Da)n , (Ds)n ,
an , sn , i, but do not evaluate.
(b) [12 MARKS] The accumulated value just after the last payment under a 14-
year annuity of 9,000 per year, paying interest at the rate of 8% per annum
effective, is to be used to purchase a 20-year annuity at an interest rate of 6%,
first payment to be made 3 years after the last payment under the annuity.
Showing all your work, determine the size of the payments under the 20-year
annuity. Assume that the 6% rate is in effect from the time of the last payment
under the 14-year annuity.
Solution:

(a) The number of payments is 260−35


25
+ 1 = 10. The value one year before the
first payment is 25(Da)10¡+ 10a10 . Just after¢ the 3rd payment this decreasing
annuity is worth (1 + i)3 25(Da)10 + 10a10 .
(b) Let X be the amount of each payment under the 20-year annuity. Then an
equation of value just after the last payment under the 14-year annuity is
¯
9000s14 8% = X2 ¯a206%
(1.08)14 −1
0.08
= 9000 · −20
(1.06) · 1−(1.06)
−2
0.06
2 14
9000(0.06)(1.06) ((1.08) − 1)
=
0.08 (1 − (1.06)−20 )
= 21348.97 .

11.3 Version 3
1. (a) [6 MARKS] An annuity at interest rate i consists of payments of 24 now, 29
at the end of one year, 34 at the end of two years, increasing by a constant

UPDATED TO April 14, 2009


Information for Students in MATH 329 2009 01 65

amount until the last payment in the amount of 224, and is to be evaluated
as of 4 years ago. Express its value then in terms of symbols (Ia)n , (Is)n , an ,
sn , i, but do not evaluate.
(b) [12 MARKS] The accumulated value just after the last payment under a 22-
year annuity of 11,000 per year, paying interest at the rate of 8% per annum
effective, is to be used to purchase a perpetuity at an interest rate of 6%,
first payment to be made at the same time as the last payment under the
annuity. Showing all your work, determine the size of the payments under the
perpetuity.
Solution:

(a) The payments are differing by 5, so the number of payments is 224−24


5
+1 = 41.
One year before the first payment the annuity is worth 5(Ia)41 + 19a41 . As of
4 years ago the annuity
¡ must be discounted
¢ an additional 3 years, so its value
−3
then was (1 + i) 5(Ia)41 + 19a41 .
(b) If we denote the amount of each payment under the perpetuity by X, then
an equation of value at the time of the first payment of the perpetuity-due is
11, 000s22 8% = X · ä∞ 6% , which can be solved to yield
³ ´
(1.08)22 −1
11000 0.08
X= 1.06 = 34529.68 .
0.06

2. [18 MARKS] A loan of 9,000 is to be repaid by annual payments of 1,000 to


commence at the end of the 1st year, and to continue thereafter for as long as
necessary. Find the time and amount of the final payment if the final payment is
to be smaller than the regular payments. Assume i = 6%.
Solution: Suppose that the last payment, which is to be a drop payment, is the
nth. Then n is the smallest11 integer such that

9000 < 1000an ⇔ (1.06)−n < 1 − 9(0.06) = 0.46


⇔ n > 13.32664048

Hence the drop payment is the 14th. Just after the 13th payment the amount still
owing is 9000(1.06)13 − 1000s13 , so the amount of the 14th payment is
µ ¶
13 (1.06)13 − 1
1.06 9000(1.06) − 1000 · = 333.07 .
0.06
11
It’s not enough to state an inequality for n — you need to make it clear which n you will choose
once you have solved the inequality.
Information for Students in MATH 329 2009 01 66

3. Showing your work in detail, determine each of the following; the rates you de-
termine should be accurate to 4 decimal places, or as a percentage accurate to 2
decimal places:

(a) [4 MARKS] the effective annual interest rate, corresponding to an nominal


annual interest rate of i = 6% compounded every 4 months.
(b) [4 MARKS] the effective monthly interest rate corresponding to a force of
interest of δ = 0.3.
(c) [4 MARKS] the effective annual interest rate corresponding to d(4) = 3.20%

Solution:
¡ ¢
0.06 3
(a) Since 1 + i = 1 + 3
, the effective annual rate is (1.02)3 − 1 = 0.061208,
or 6.12%.
(b) The accumulation factor for one month is
R 1
12 0.3 dt
e 0 = e0.025 = 1.025315121 ,

so the equivalent effective monthly interest rate is 2.53%.


¡ ¢−4
(c) The effective annual accumulation factor is 1 − 0.032 4
= 1.032650385, so
the equivalent effective annual interest rate is 3.27%.

4. For each of the following sequences of payments, determine, as of the given time,
and for the given interest or discount rate, the value, showing all of your work.
Before determining the numeric value you are expected

• to express the value using standard symbols, and


• to state the formula that you have used to evaluate the standard symbol.

(a) [6 MARKS] the value 3 months ago of 24 payments of 5 at the end of every
half-year, the first to be paid 7 years from now, at a nominal interest rate of
9% compounded semi-annually;
(b) [6 MARKS] 240 payments of 10 at the end of every month, as of the date
of the 150th payment, which has just been made; the interest rate is 12%
compounded monthly
(c) [6 MARKS] the value now of 18 payments of 500 at the end of every year, the
first payment 8 years from now, at an interest rate of 5%

Solution:
Information for Students in MATH 329 2009 01 67

(a) As of one half-year before the first payment, the annuity is worth 5a24 4.5% .
Since the first payment will be in 7 years, the annuity, interpreted as an
annuity-immediate, is being deferred through 13 half-years, and is worth today
5(1.045)−13 a24 4.5% . The value a quarter-year ago, i.e., half of a half-year ago,
was
1 − (1.045)−24
5(1.045)−13.5 a24 4.5% = 5(1.045)−13.5 · = 40.01 .
0.045
(b) The effective monthly interest rate is 1%. The value of the payments which
have been made is 10s150 1% , and the value of the future payments is 10a90 1% ;
10 ¡ ¢
so the present value is 10s150 1% +10a90 1% = (1.01)150 − 1 + 1 − (1.01)−90 =
¡ ¢ 0.01
1000 (1.01)150 − (1.01)−90 = 4, 040.03 .
¯ ¡ ¢ 500 ((1.05)−7 − (1.05)−25 )
(c) 5007 ¯a18 5% = 500 a25 5% − a7 5% = = 4153.79.
0.05

11.4 Version 4
1. For each of the following sequences of payments, determine, as of the given time,
and for the given interest or discount rate, the value, showing all of your work.
Before determining the numeric value you are expected

• to express the value using standard symbols, and


• to state the formula that you have used to evaluate the standard symbol.

(a) [6 MARKS] 72 payments of 1 at the end of every 3 months, as of the date of


the 24th payment, which has just been made; interest is at a nominal annual
rate of 12% compounded every 4 months;
(b) [6 MARKS] the value of 24 payments of 7 at the end of every half-year,
the last one having just been made, at a nominal annual interest rate of 6%
compounded quarterly;
(c) [6 MARKS] the value now of 50 payments of 2 at the end of every 8 months,
the first to be paid 2 years from now, at a nominal interest rate of 4.5%
compounded 3 times every 2 years.

Solution:

(a) The effective interest rate for 4 months is 12


3
% = 4%. But the compound-
ing/payment interval for the annuity is 3 months, for which the accumu-
3
lation factor will be (1.04) 4 = 1.029852445, and the interest rate will be
Information for Students in MATH 329 2009 01 68

j = 2.9852445%. The value of the annuity at the indicated date is


((1.04)18 − 1) + (1 − (1.04)−36 )
s24 j + a48 j = = 59.69855344 .
0.029852445
(b) The effective interest rate per quarter-year is 1.5%; the effective interest rate
per half-year is (1.015)2 −1 = 0.030225. The value of the annuity is, therefore,
(1.030225)24 − 1
7s24 0.030225 = 7 · = 241.67 .
0.030225
(c) Interest is compounded every 2/3 of a year, i.e., 32 times a year. Hence the
µ ¶3
0.045 2
annual accumulation factor is 1 + 3 , and the accumulation factor for
2
8 months is õ ¶ 3 ! 23
0.045 2 0.045
1+ 3 =1+ 3 = 1.03 ,
2 2

i.e., the effective interest rate for an 8-month period is 3%. Since the first
payment is after 2 years — i.e., at the end of the 3rd interest compounding
interval from now, the value of the annuity is
¯ (1.03)−2 − (1.03)−52
2 ·2 ¯a50 3% = 2 · = 48.51.
0.03
2. (a) [6 MARKS] An annuity at interest rate i consists of payments of 1,118 now,
1,112 at the end of 1 year, 1,106 at the end of 2 years, decreasing until the last
payment in the amount of 1,058, the totality to be evaluated as of the time
of the payment of 1,106. Express its value then in terms of symbols (Da)n ,
(Ds)n , an , sn , i, but do not evaluate.
(b) [12 MARKS] The accumulated value just after the last payment under a 19-
year annuity of 1000 per year, paying interest at the rate of 8% per annum
effective, is to be used to purchase a 10-year annuity-immediate at an interest
rate of 6%, first payment to be made 1 year after the last payment under the
19-year annuity. Showing all your work, determine the size of the payments
under the 10-year annuity. Assume that the 6% rate applies from the time of
the last payment under the 8% annuity.
Solution:
(a) The number of payments is 1118−10586
+ 1 = 11. The value of this decreasing
annuity as of the last payment is 6(Ds)11 + 1052s11 . The payment of 1,106
occurs 1106−1058
6
= 8 years
¡ earlier, so the¢ value of the
¡ decreasing annuity
¢ at
−8 3
that time is (1 + i) 6(Ds)11 + 1052s11 = (1 + i) 6(Da)11 + 1052a11 .

UPDATED TO April 14, 2009


Information for Students in MATH 329 2009 01 69

(b) If X denotes the size of payments under the 10-payment annuity, an equation
of value just after the last payment under the first annuity is
s19 8%
1000 · s19 8% = X · a10 6% ⇔ X = 1000 ·
a10 6%
1000 · ((1.08)19 − 1) · (0.06)
⇔ X= = 5631.219160
(1 − (1.06)−10 )) · (0.08)
so the second annuity has 10 equal payments of 5,631.22.

3. [18 MARKS] A loan of 18,000 is to be repaid by annual payments of 2,500 to


commence immediately, and to continue at the beginning of each year for as long
as necessary. Find the time and amount of the final payment if the final payment
is to be no smaller than the regular payments. Assume i = 13%.
Solution: Since the 1st payment is at time t = 0, we may assume that it is the
(n + 1)st payment (made at time t = n) which is the balloon payment. Then n is
the largest12 integer such that

18000 − 2500än+1 > 0 ⇔ 15500 − 2500an > 0


0.13(15500)
⇔ 1 − (1.13)−n < = 0.806
2500
ln(0.194)
⇔ −n > = −13.4
ln(1.13)
⇔ n < 13.4 ,

so n = 13, the balloon payment is the 14th (at time t = 13). The amount outstand-
ing on the loan just after the 13th payment (at time t = 12) is 15500(1.13)12 −
2500s12 3059.66382; so the amount of the balloon payment is
µ ¶
¡ 12
¢ 12 (1.13)12 − 1
1.13 15500(1.13) − 2500s12 = (1.13) 15500(1.13) − 2500 ·
0.13
= 3457.42 .

4. Showing your work in detail, determine each of the following; the rates you de-
termine should be accurate to 4 decimal places, or as a percentage accurate to 2
decimal places:

(a) [4 MARKS] the nominal annual interest rate, compounded quarterly, equiva-
lent to an effective semi-annual discount rate of d = 4%
12
It’s not enough to state an inequality for n — you need to make it clear which n you will choose
once you have solved the inequality.
Information for Students in MATH 329 2009 01 70

(b) [4 MARKS] the effective semi-annual interest rate corresponding to a force of


interest of δ = 0.14.
(c) [4 MARKS] the effective annual discount rate corresponding to a nominal
interest rate, compounded quarterly, of i = 2.5%

Solution:

(a) The effective annual accumulation factor is (1 − 0.04)−2 . If i(4) denotes the
³ (4)
´4
nominal annual interest rate, compounded quarterly, then we have 1 + i 4 =
³ 1
´
(1 − 0.04)−2 , so i(4) = 4 (0.96)− 2 − 1 = 0.082482904 or 8.25%.
(b) The accumulation factor for half a year is
R 0.05 0.5
δt dt
e 0 = e[0.14t]0 = e0.07 = 1.072508181 ,

so the effective semi-annual interest rate is 7.25%.


µ ¶4
0.025
(c) In one year 1 accumulates to 1 + = 1.025235353, whose reciprocal
4
is 0.9753857951, so the effective annual discount rate is 1 − 0.9753857951 =
0.0246142049 or 2.46%.
Information for Students in MATH 329 2009 01 71

12 Fifth Problem Assignment


Distribution Date: Sunday, March 22nd, 2009
Solutions are to be submitted by Monday, March 30th, 2009
Submission of this assignment for grading is optional: the portion of the term mark
based on assignments will be calculated from a student’s 4 best assignment grades,
counting equally.

1. George’s older brother has sold his company for a large cash payment. He wishes to
place his capital in secure investments which will provide him a stream of income
to replace the salary he was previously drawing from the company. As George is
purchasing a condo, and needs immediate capital for a large down payment, they
come to an agreement whereby George will borrow 100,000: he plans to repay the
loan at the end of 10 years, and will pay annual interest monthly at the rate of 5%
per annum effective. In addition to this down payment, George will assume from
the vendor of his condo a mortgage for 400,000 at a nominal annual rate of 4%,
compounded semi-annually, under which he makes equal monthly payments at the
end of each month to amortize the mortgage loan in 20 years. (For the purpose of
this problem assume that a year consists of exactly 12 months.)

(a) After 3 years George realizes that he needs to prepare to repay the loan to
his brother, and decides to accumulate a sinking fund for the purpose, a fund
which will attain a balance of 100,000 when his loan to his brother comes
due. The best interest rate he can now secure is 3% per annum, compounded
monthly. Determine the amount of his equal monthly payments into the
sinking fund.
(b) Set up portions of a table which will show the amortization of George’s mort-
gage with the following columns: Mortgage Payment Number, Interest Com-
ponent, Principal Component, Outstanding Principal After Payment, as of
the following dates: t = 0 (when mortgage is signed), t = 12, t = 24, t = 36,
t = 239.
(c) Determine George’s total monthly cost for these loans during years 0-3, years
3-10, and years 10-20.

2. A borrower is repaying a loan at a quarterly cost of 5,000 for 10 years. One-quarter


of the loan is amortized by equal quarterly payments at 5% effective, included in
each quarterly expenditure of 5,000. The other three-quarters of the loan is repaid
by the sinking fund method, where that portion of the principal is repaid when the
loan matures by the proceeds of a sinking fund which accumulates at a nominal
annual rate of 4%, compounded quarterly: until maturity the lender pays only
Information for Students in MATH 329 2009 01 72

quarterly interest at 5% (annual effective) on the non-amortized part of the loan,


and settles that part of the loan at maturity by transferring to the lender the
balance in the sinking fund. Showing all your work, determine the amount of the
loan.

3. An investor is considering the purchase of a 15-year bond with par value 100.
bearing semi-annual coupons at a nominal annual interest rate of 4%, convertible
semi-annually. The bond is callable at 109.00 on any coupon date from 5 years
to 10 years, inclusive; at 104.50 from on any coupon date from 10.5 years to 14.5
years inclusive; and it matures at 100.00 in 15 years. Showing details of your work,
determine what price an investor should pay in order that he be guaranteed:

(a) an effective annual yield rate of 3%; or


(b) a nominal annual yield rate of 5%, convertible every 6 months.
Information for Students in MATH 329 2009 01 73

13 Draft Solutions, Fourth Problem Assignment


Distribution Date: Monday, March 30th, 2009
Solutions were to be submitted by Monday, March 23rd, 2009.

1. It is New Years Day, and Jacques and Jacqueline have just taken out a mortgage
on their new condo, in the amount of 100,000. They have agreed that, in any
calendar year except possibly the last year of the mortgage, Jacques’ contribution
will be exactly twice that of Jacqueline’s. The annual interest rate, compounded
semi-annually, is 6%,

(a) Suppose that Jacques will always pay 2X on the last day of April; and Jacque-
line will pay X at the end of December. Determine the value of X if the
mortgage is to be paid off in 20 years.
(b) Suppose that Jacques will always pay 6000 on the last day of April; and
Jacqueline will pay 3000 on the last day of December, until a final year wherein
the payments of Jacques and Jacqueline — still in the ratio of 2:1 — would
not be as large as in previous years. Determine when that final year occurs,
and what the respective payments of Jacques and Jacqueline will be that year.
(c) Suppose that Jacques will always pay 6000 on the last day of April; and
Jacqueline will pay 3000 on the last day of December, until a time when the
payment which is due exceeds the amount owing on the mortgage, in which
case the payment is reduced to simply pay off the mortgage. Determine when
this final year occurs, which of Jacques and Jacqueline has a reduced payment,
and what that reduced payment is in that final year.

Solution: The effective annual interest rate is (1.03)2 −1 = 6.09%; the accumulation
2
factor for a 4-month period is, therefore, (1.03) 3 .

(a) Jacques’ contributions may be interpreted as an annuity-immediate which has


been advanced through two-thirds of a year, i.e., through 43 half-years. Ac-
4
cordingly the present value of Jacques’ contributions is (1.03) 3 ·2X ·a20 (1.03)2 −1 ;
Jacqueline’s contributions also may be interpreted as an annuity-immediate
which is today worth X · a20 (1.03)2 −1 . Equating the sum of the present values
of these contributions to the present value of the loan, we obtain the equation
of value
4
(1.03) 3 · 2X · a20 (1.03)2 −1 + X · a20 (1.03)2 −1 = 100, 000
³ 4
´ 1 − (1.03)−40
⇔ X 2(1.03) + 1 ·
3 = 100, 000
(1.03)2 − 1
Information for Students in MATH 329 2009 01 74

100, 000 ((1.03)2 − 1)


⇔ X=³ 4
´ = 2851.016174
2(1.03) 3 + 1 · (1 − (1.03)−40 )

so Jacques’ annual payment on each April 30th is 5702.03, while Jacqueline’s


payment on 31st December of each year is 2851.02. These payments will
continue for 20 years.
(b) Let n denote the number of full years in which Jacques and Jacqueline both
make full payments respectively of 6000 and 3000. Then n is the largest
integer such that
³ 4
´
6000(1.03) 3 + 3000 · an (1.03)2 −1 ≤ 100, 000
100000 ((1.03)2 − 1)
⇔ 1 − (1.03)−2n ≤ ³ 4
´ = 0.6590058875
6000(1.03) + 3000
3

⇔ (1.03)−2n ≥ 0.3409941125
ln 0.3409941125
⇔ n≤− = 18.19914857
ln 1.03
At the end of the 18th year, just after Jacqueline’s payment, the amount
outstanding on the mortgage is
4
(1.03)36 100000 − (1.03) 3 6000 · s18 (1.03)2 −1 − 3000 · s18 (1.03)2 −1
³ 4
´
3000 2 · (1.03) + 1 · ((1.03)36 − 1)
3
36
= (1.03) 100000 − = 1776.0318.
(1.03)2 − 1
If Jacques pays 2Y at the end of April, and Jacqueline Y at the end of 1 year,
an equation of value at the beginning of the 19th year is
2
1776.03 = (1.03)− 3 · 2Y + (1.03)−2 Y ,

implying that Y = 611.67, so Jacques will pay 1223.34 at the end of April,
and Jacqueline will pay 611.67 at the end of the year.
(c) The computations in the previous case show that the mortgage will definitely
be paid off in the 19th year. The balance of 1776.03 which is outstanding at
2
the beginning of the year grows to (1776.03)(1.03) 3 = 1811.38, which becomes
Jacques’ payment: the mortgage is paid off, and Jacqueline will pay nothing
at the end of the 19th year.

2. Mary is contributing to an RSSP for her retirement in 3 years by purchasing semi-


annually a Certificate of Deposit which expires on the date of her retirement. She
Information for Students in MATH 329 2009 01 75

invests 10,000 every half-year, with the last investment just one-half year before
retirement. Suppose that the interest rate on the certificate she is purchasing now
is 3%, and that the rates fall by 12 % each half-year, until the last half-year before
retirement, when the rate reaches just 12 %; these rates, each applying to a half-year
period, are all nominal annual rates, compounded semi-annually. Determine the
magnitude of Mary’s account on the day she retires.

(a) Compute the value of the account at retirement, using the Yield Curve Method
[4, p. 96].
(b) Compute the value of the account at retirement, using the Portfolio Rate
Method [4, p. 96].

(Of course, Mary doesn’t have the choice of which method will be used: in either the
fine print of her contract or in the established practices of the banking institution
the method will be made more explicit.)
Solution:

(a) Using the Yield Curve method, we accumulate each deposit of 10,000 at the
rate in effect when it was deposited; remember that the rate in effect for each
half-year is one-half of the quoted, nominal rate. The value of her account
will be
¡ ¢
10000 (1.0025)1 + (1.0050)2 + (1.0075)3 + (1.0100)4 + (1.0125)5 + (1.0150)6
= 62, 333.24 .

(b) Using the Portfolio Rate Method the first payment of 10,000 accumulates to
10000(1.015) just before the 2nd deposit. That deposit increases the capital
in the account to 10000(1.0150 + 1), which accumulates to 10000((1.0150 +
1)1.0125 + 1) just after the 3rd deposit. Just prior to the 4th deposit the ac-
count has grown to 10000((1.0150+1)1.0125+1)1.0100, becoming 10000(((1.0150+
1)1.0125 + 1)1.0100 + 1) just after the 4th deposit; to 10000((((1.0150 +
1)1.0125 + 1)1.0100 + 1)1.0075 + 1) after the 5th deposit; and to

10000(((((1.0150+1)1.0125+1)1.0100+1)1.0075+1)1.0050+1) = 61, 266.24389

after the last deposit; between this deposit and retirement the account accu-
mulates by an accumulation factor of 1.0025, to a final balance of 61,419.41.

3. Jack’s mortgage, in the amount of 80,000, requires regular payments of 600 every
month, at an interest rate of 8%, compounded semiannually.
Information for Students in MATH 329 2009 01 76

(a) Suppose that Jack has agreed that the last payment will be a “balloon” pay-
ment of at least 600. Determine when that payment is due, and its amount.
(b) Suppose that Jack has agreed that the last payment will be a ”drop” payment
of at most 600. Determine when that payment is due, and its amount.

Solution:

(a) Let n be the time of the balloon payment. Then n will be the largest integer
such that 600·sn < 80, 000(1+i)n , and the last payment will be in the amount
of 600 increased by 80, 000 − 600 · sn , where the interest rate per month is
1
i = (1.04) 6 − 1 = 0.006558197 or 0.6558197%. We have

600 · sn < 80, 000(1 + i)n


(1 + i)n − 1
⇔ 600 · < 80, 000(1 + i)n
i
600 1
⇔ (1 + i)n < =
600 − 80, 000i 1 − 400i
3
400i
¡ 400
¢
− ln(1 − 3 ) −6 ln 1 − 3 (0.006558197)
⇔ n< = = 317.4132252
ln(1 + i) ln(1.04)
The largest integer solution is, therefore n = 317. The balloon payment will
occur in month 317 of the repayment. The payment will be equal to
317 (1.006558197)317 − 1
600 + 80, 000(1.04) 6 − 600 · = 846.78
0.006558197
(b) In the case of a drop payment we need to find the smallest n such that the
reversal of the preceding inequality be satisfied. The preceding calculations
show that this will be n = 318. In that case the excess of 600 of the amount
of the (drop) payment will be

318 (1.006558197)318 − 1
−80, 000(1.04) 6 + 600 · = 351.6041 ,
0.006558197
so the amount of the drop payment will be 248.40.

4. Under a nominal interest rate of 6% compounded continuously an annuity pays


10,000 per year for 5 years. Determine

(a) The value of the annuity 2 years before the payment begins.
(b) The total annual payment under a continuous perpetuity at the same rate of
interest, whose value at time 0 is the same as this annuity.
Information for Students in MATH 329 2009 01 77

(c) The time t such that the value at time 0 of the portion of the annuity up to
time t is equal to the value at time 0 of the portion of the annuity from time
t to time 5.
Solution: The intention was that this annuity should be considered as being contin-
uous, i.e., paying infinitesimal amounts throughout its 5-year term. Some students
interpreted the problem as referring to a discrete annuity; in that case it wasn’t
clear when the payments would be made, but some assumed them to be annual;
one problem with this discrete interpretation is that the second part of the problem
no longer has a clearly defined solution. The interpretation as a continuous annuity
was also indicated by the use of the word “payment”, rather than the plural: rather
than a sequence of discrete payments, there was to be one continuous payment.
Continuous Annuity
(a) The annual accumulation factor corresponding to a nominal annual rate
of 6% compounded instantaneously is 1 + i = e0.06 = 1.061836547; hence
v = e−0.06 , and ln v = −0.06. The value at time t = 0 of all payments
under the annuity is
Z 5 · t ¸5
t v 10, 000 ¡ −0.3 ¢
10, 000a5 = 10, 000 v dt = 10, 000 = e − 1 = 43, 196.96 .
0 ln v 0 −0.06
Two years before payment begins the annuity is worth only v 2 = e−0.12
times this amount, i.e., 38, 312.27.
(b) Let the total annual payment under the continuous perpetuity be X.
Then an equation of value at time t = 0 is
Z 5 Z ∞
t
10, 000a5 = 10, 000 v dt = X v t dt = Xa∞ ,
0 0

implying that
43, 196.96 43, 196.96
X= 1 −0.06b
= 1 = (0.06)(43, 196.96) = 2591.82.
lim −0.06 (e − 1) −0.06
· (−1)
b→∞

(c) The value t we seek satisfies the equation

10, 000at = (0.5)(43, 196.96)


10, 000 ¡ −0.06t ¢
⇔ e − 1 = 21, 598.48
−0.06
⇔ e−0.06t = 1 − (0.000006)(21, 598.48) = 0.87040912
⇔ t = 2.313198750 .
Information for Students in MATH 329 2009 01 78

Discrete Annuity with Annual PaymentsIn this case one can interpret the annuity
as either an annuity-immediate or an annuity-due. I will interpret it as an
annuity-immediate.
µ ¶
0.06
(a) The effective annual accumulation factor is lim 1 + = e0 .06 =
m→∞ m
1.061836547, so the effective annual interest rate is j = 6.183655%. The
value 2 years before the first payment is that of an annuity-immediate
deferred one year, i.e.,
¯ 1 − (1 + j)−5
10, 000 ·1 ¯a5 i = 10, 000 · = 39, 473.13 .
j(1 + j)

(b) The value of this annuity at time t = 0 will depend on whether it’s in-
terpreted as an annuity-immediate or an annuity-due. If it is an annuity-
immediate, its value is 1 + j times the preceding, i.e., 41,914.01; if it is
interpreted as an annuity-due, then the value is increased by multiplica-
tion by another factor 1 + j, i.e., it is 44,505.83.
(c) It’s not clear how to interpret this problem in the discrete case; we can
solve an equation and obtain a fractional number of years; but how do
we interpret that fraction, in view of the fact that payments are only at
the end or beginning of a year?

5. Gasoline costs 1.00 per litre now, and the price will increase by 0.05 per week for
the next year. If my tank holds 65 litres, and money is worth 4.5% per annum
effective, what will be the value 52 weeks from now, just after the tank has been
filled, of the gasoline that I have purchased, if I fill the tank completely every 2
weeks?
Solution: The cost of filling the tank is increasing by the amount of 65×0.10 = 6.50
from one filling to the next. I am assuming that the tank completely every 2
weeks, but that today’s filling is not included in the cost for a full year, and my
computations will start with the value one year from now of the tank filled two
weeks from now. To the amount of 6.50(Is)26 , whose starting payment is 6.50, we
must add (65 × 1.00) − 6.50 + 6.50 = 65.00 times s26 . The effective interest rate
1
for a two-week period is i = (1.045) 26 − 1 = 0.001694391. The terminal value will
be

6.50(Is)26 + 65.00s26
µ ¶ µ ¶
s̈26 − 26 (1.001694391)26 − 1
= 6.50 + 65.00
0.001694391 0.001694391
Information for Students in MATH 329 2009 01 79

à 26 −1
! µ ¶
(1.001694391) · (1.001694391)
0.001694391
− 26 (1.001694391)26 − 1
= 6.50 + 65.00
0.001694391 0.001694391
µ 1.045−1 ¶ µ ¶
(1.001694391) · 0.001694391 − 26 1.045 − 1
= 6.50 + 65.00
0.001694391 0.001694391
µ 0.045 ¶ µ ¶
(1.001694391) · 0.001694391 − 26 0.045
= 6.50 + 65.00 = 4040.328165
0.001694391 0.001694391
or 4,040.33.
Information for Students in MATH 329 2009 01 80

14 Draft Solutions, Fifth Problem Assignment


Distribution Date: Monday, April 06th, 2009
Solutions were to be submitted by Monday, March 30th, 2009
Submission of this assignment for grading is optional: the portion of the term mark
based on assignments will be calculated from a student’s 4 best assignment grades,
counting equally.

1. George’s older brother has sold his company for a large cash payment. He wishes to
place his capital in secure investments which will provide him a stream of income
to replace the salary he was previously drawing from the company. As George is
purchasing a condo, and needs immediate capital for a large down payment, they
come to an agreement whereby George will borrow 100,000: he plans to repay the
loan at the end of 10 years, and will pay annual interest monthly at the rate of 5%
per annum effective. In addition to this down payment, George will assume from
the vendor of his condo a mortgage for 400,000 at a nominal annual rate of 4%,
compounded semi-annually, under which he makes equal monthly payments at the
end of each month to amortize the mortgage loan in 20 years. (For the purpose of
this problem assume that a year consists of exactly 12 months.)
(a) After 3 years George realizes that he needs to prepare to repay the loan to
his brother, and decides to accumulate a sinking fund for the purpose, a fund
which will attain a balance of 100,000 when his loan to his brother comes
due. The best interest rate he can now secure is 3% per annum, compounded
monthly. Determine the amount of his equal monthly payments into the
sinking fund.
(b) Set up portions of a table which will show the amortization of George’s mort-
gage with the following columns: Mortgage Payment Number, Interest Com-
ponent, Principal Component, Outstanding Principal After Payment, as of
the following dates: t = 0 (when mortgage is signed), t = 12, t = 24, t = 36,
t = 239.
(c) Determine George’s total monthly cost for these loans during years 0-3, years
3-10, and years 10-20.
Solution:
(a) The regular monthly payment into the sinking fund, beginning at the end of
the first month of the 4th year, is
100, 000 (100, 000)(0.0025)
= = 1071.33 .
s84 1 % (1.0025)84 − 1
4
Information for Students in MATH 329 2009 01 81

1
(b) The mortgage interest is at an effective monthly rate of (1.02) 6 −1 = 0.003305890;
the regular monthly payment under the mortgage will be

400, 000 (400, 000)(0.003305890)


= = 2416.985777 .
a240 0.330589% 1 − (1.003305890)−240

The principal outstanding just after the 11th payment is

400000(1.003305890)11 − 2416.985777 · s11 0.003305890


(2416.985777) ((1.003305890)11 − 1)
= 400000(1.003305890)11 −
0.003305890
= 387, 758.06 ,

so the interest component of the 12th payment is (0.003305890)·(387, 758.06) =


1, 281.89, and the principal component will be 2, 416.99−1, 281.89 = 1, 135.10;
the outstanding principal after the 12th payment will be 387, 758.06−1, 135.10 =
386, 622.96.
The principal outstanding just after the 23rd payment is

400000(1.003305890)23 − 2416.985777 · s23 0.003305890


(2416.985777) ((1.003305890)23 − 1)
= 400000(1.003305890)23 −
0.003305890
= 373, 886.44 ,

so the interest component of the 24th payment is (0.003305890)·(373, 886.44) =


1, 236.03, and the principal component will be 2, 416.99−1, 236.03 = 1, 180.96;
the outstanding principal after the 24th payment will be 373, 886.44−1, 180.96 =
372, 705.48.
The principal outstanding just after the 35th payment is

400000(1.003305890)35 − 2416.985777 · s35 0.003305890


(2, 416.985777) ((1.003305890)35 − 1)
= 400, 000(1.003305890)35 −
0.003305890
= 359, 454.40 ,

so the interest component of the 36th payment is (0.003305890)·(359, 454.40) =


1, 188.32, and the principal component will be 2, 416.99−1, 188.32 = 1, 228.67;
the outstanding principal after the 36th payment will be 359, 454.40−1, 228.67 =
358, 225.73.
Information for Students in MATH 329 2009 01 82

The principal outstanding just after the 238th payment is

400000(1.003305890)238 − 2416.985777 · s238 0.003305890


(2416.985777) ((1.003305890)238 − 1)
= 400000(1.003305890)238 −
0.003305890
= 4, 810.11 ,

so the interest component of the 239th payment is (0.003305890)·(4, 810.11) =


15.90, and the principal component will be 2, 416.99 − 15.90 = 2, 401.09; the
outstanding principal after the 239th payment will be 4, 810.11 − 2, 401.09 =
2, 409.02.
0.05
(c) The effective monthly rate of interest on his loan from his brother is =
1
12
(1.05) 12 − 1 = 0.004074124, so his monthly interest cost on this loan is 407.41;
this cost is present up to the end of the 10th year.
Month Mortgage Interest to Brother Sinking Fund Total
1–36 2,416.99 407.41 2,824.40
37–120 2,416.99 407.41 1,071.33 3,895.73
21-240 2,416.99 2,416.99

2. A borrower is repaying a loan at a quarterly cost of 5,000 for 10 years. One-quarter


of the loan is amortized by equal quarterly payments at 5% effective, included in
each quarterly expenditure of 5,000. The other three-quarters of the loan is repaid
by the sinking fund method, where that portion of the principal is repaid when the
loan matures by the proceeds of a sinking fund which accumulates at a nominal
annual rate of 4%, compounded quarterly: until maturity the lender pays only
quarterly interest at 5% (annual effective) on the non-amortized part of the loan,
and settles that part of the loan at maturity by transferring to the lender the
balance in the sinking fund. Showing all your work, determine the amount of the
loan.
Solution: For the amortized portion of the loan the effective quarterly interest
1
rate is i = (1.05) 4 − 1 = 0.012272234 = 1.2272234%. Let the total principal
of the loan be denoted by L. For the portion of the loan amortized by quar-
L L(0.012272234)
terly payments, the quarterly cost is = =
4 · a40 1.2272234% 4 (1 − (1.012272234)−40 )
0.007946552452L . The quarterly interest due on the non-amortized portion of
3
the loan is 0.012272234 × × L = 0.00920417550L. For that portion of the
4
3
·L 0.0075L
loan the quarterly contribution to the sinking fund is 4 = =
s40 0.01 (1.01)40 − 1
Information for Students in MATH 329 2009 01 83

0.01534169847L. We sum the three components of the quarterly contribution and


equate to 5,000:

0.007946552452L + 0.00920417550L + 0.01534169847L = 5, 000 ,

which implies that L = 153, 882.01.

3. An investor is considering the purchase of a 15-year bond with par value 100.
bearing semi-annual coupons at a nominal annual interest rate of 4%, convertible
semi-annually. The bond is callable at 109.00 on any coupon date from 5 years to
10 years, inclusive; at 104.50 from on any coupon date from 10.5 years to 14.5 years;
and it matures at 100.00 in 15 years. Showing details of your work, determine what
price an investor should pay in order that he be guaranteed:

(a) an effective annual yield rate of 3%; or


(b) a nominal annual yield rate of 5%, convertible every 6 months.

Solution: Let t denote the time in half-years, so 0 ≤ t ≤ 30.

(a) I am using the “Premium/Discount Formula”, P = C + (F r − Ci)an . The


1
effective semi-annual interest rate is 1.03 2 − 1 = 1.4889156%. As a function
of the half-year t at the end of which the bond is redeemed, the value of the
bond is as follows:
t≥ t≤ Price
10 20 109.00 + (2 − 1.622918)at 1.4889156%
.
21 29 104.50 + (2 − 1.5559168)at 1.4889156%
30 30 100.00 + (2 − 1.4889156)a30 1.4889156%

Because the parenthesized quantity F r − Ci is positive, these prices are min-


imized when t is minimal in each given range. Thus we need to compare

t Formula Price
10 109.00 + (2 − 1.622918)a10 1.4889156% 112.479629
.
21 104.50 + (2 − 1.5559168)a21 1.4889156% 112.4582335
30 100.00 + (2 − 1.4889156)a30 1.4889156% 112.29970329

Thus the lowest price will occur if the bond is not called: the purchaser should
not pay more than 112.30 to guarantee an effective annual yield of 3%.
(b) I am again using the “Premium/Discount Formula”, P = C + (F r − Ci)an .
The effective semi-annual interest rate is 2.5%. As a function of the half-year
Information for Students in MATH 329 2009 01 84

t at the end of which the bond is redeemed, the value of the bond is as follows:

t≥ t≤ Price
10 20 109.00 + (2 − 2.7250)at
.
21 29 104.50 + (2 − 2.6125)at
30 30 100.00 + (2 − 2.5000)a30

As the parenthesized quantity F r − Ci is always negative, the values are


minimized when t is maximal in the given range. Thus we need only consider
the following values:

t Formula Price
20 109.00 + (2 − 2.7250)a20 97.70
.
29 104.50 + (2 − 2.6125)a29 91.97
30 100.00 + (2 − 2.5000)a30 89.53

It follows that the worst price will occur if the bond is not called: the purchaser
should not pay more than 89.53 to guarantee a yield of 5% converted semi-
annually.
Information for Students in MATH 329 2009 01 901

• References to these sources are often given in the notes for completeness. Students
are not expected to look up sources, but may wish to do so out of curiosity.
• The entries in this list may not be in alphabetical order. As the notes are con-
structed, new entries will be added at the end, so as not to upset the earlier
numbering of references.

15 References
[1] J. W. Daniel, L. J. Federer Vaaler, Mathematical Interest Theory, Pearson/Prentice
Hall (2007), ISBN 0-13-147285-2.
[2] L. J. Federer Vaaler, J. W. Daniel Mathematical Interest Theory, Mathematical
Association of America (2009), ISBN-13 978-0-88385-754-0.
[3] H. S. Hall and S. R. Knight, Higher Algebra, Fourth Edition, MacMillan & Co.
(London, 1891).
[4] S. G. Kellison, The Theory of Interest, Third Edition. McGraw Hill/Irwin, Boston,
etc. (2009). ISBN-10 0-07-338244-2, ISBN-13 987-0-07-338244-9.
[5] S. G. Kellison, The Theory of Interest, Second Edition. Irwin/McGraw Hill, Inc.,
Boston, etc. (1991). ISBN 0-256-04051-1.
[6] S. G. Kellison, The Theory of Interest. Richard D. Irwin, Inc., Homewood, Ill. (1970).
ISBN ???-083841.
[7] McGill Undergraduate Programs Calendar 2008/2009. Also accessible at
http://coursecalendar.mcgill.ca/ugcal200809/wwhelp/wwhimpl/js/html/wwhelp.htm
[8] R. Muksian, Mathematics of Interest Rates, Insurance, Social Security, and Pen-
sions. Pearson Education, Inc., Upper Saddle River, NJ. (2003). ISBN 0-13-009425-
0.
[9] M. M. Parmenter, Theory of Interest and Life Contingencies, with Pension Appli-
cations. A Problem-Solving Approach, 3rd Edition. ACTEX Publications, Winsted
CT, (1999). ISBN 1-56698-333-9.
[10] The Canadian Institute of Actuaries English-French lexicon,
http://www.actuaries.ca/members/lexicon
[11] J. Stewart, Single Variable Calculus (Early Transcendentals), 6th Edition.
Brooks/Cole (2008). ISBN-13 978-0-495-01169-9.
Information for Students in MATH 329 2009 01 2001

A Supplementary Lecture Notes


A.1 Supplementary Notes for the Lecture of January 05th,
2009
Distribution Date: Monday, January 05th, 2009
(subject to revision)

A.1.1 These notes


These notes will contain some of the material that I propose to discuss in the lectures, and
also some material that will not make it to the lectures. I will be following the textbook
very closely, sometimes explaining statements that I find require some elaboration. In
the first chapter of the book my notes will be detailed; I do not know yet whether I will
be able to continue this detailed a set of notes for subsequent chapters. Much of the
class time will be spent in discussing problems from the textbook, for which I will be
including sketches of solutions wherever possible.

Textbook Chapter 1. The measurement of interest.

A.1.2 §1.1 INTRODUCTION


While some of the concepts in this course may be applied in non-financial contexts, the
language of the course involves growth of amounts of money under the passage of time.
Most of our discussions will concern one or two specific ways in which amounts of money
are, in practice, assumed to grow; but, initially, we will define very general concepts.

Definition A.1 1. Principal is the initial amount of money borrowed, lent, or in-
vested.
2. Interest is the compensation paid by a borrower of capital to the lender for the use
of the capital.
3. Interest is said to be earned by the lender, or to accrue to the lender.
4. A distinction may be made between when and how interest is earned , and when it
is paid or credited to the lender. When the word interest is used, without further
elaboration, payment is normally at the end of each time period. When interest is
paid at the beginning of a time period it is usually called discount; however, the
compensation itself may still be called interest
Information for Students in MATH 329 2009 01 2002

A.1.3 §1.2 THE ACCUMULATION AND AMOUNT FUNCTIONS


Definition A.2 The growth of a given, specific amount of principal — as a function of
time, t — is often denoted by the amount function, denoted by A(t).

We will always assume that the rate of growth does not depend on the amount of money
that is invested.13 This scalability assumption permits us to “normalize” many of our
discussions, by speaking about the growth of a fund of 1, and then just scaling the
resulting figures by multiplying by the correct initial or terminal value of the fund. We
will take this as our first

Axiom A.1 If A1 (t), A2 (t) are two amount functions under a prescribed “interest envi-
ronment”, then the ratio A1 (t)
A2 (t)
is independent of time.

Under this postulate it is often convenient to study amounts that are “normalized” to
have value 1 at some convenient time. We will sometimes design our notation around
this postulate, by defining pairs of functions with similar names, one for the actual size
of the fund, the other for the “normalized” account. The first example of this is the
normalized amount function, which we call the “accumulation” function.

Definition A.3 Corresponding to a specific amount function, which has value A(0) at
time t = 0, we define a normalized amount function or accumulation function a by
A(t) = A(0) · a(t) or
A(t)
a(t) = . (1)
A(0)
A simple consequence of this equation (1) is

Theorem A.1 a(0) = 1.

(Most mathematicians don’t like to use the word “theorem” for results as trivial as this;
we would be more likely to call it a “proposition” or use some other term that reserves
the word “theorem” for more serious results.)

13
This is an assumption that is not completely realistic, since, in the real financial world, a person
who has, say $1,000,000 to invest can often obtain a higher interest rate on his investment than the
person who has only $100. But this model is used throughout the textbook, and we will follow it in the
course.
Information for Students in MATH 329 2009 01 2003

A.2 Supplementary Notes for the Lecture of January 07th,


2009
Distribution Date: Wednesday, January 07th, 2009
(subject to revision)

Some of the material below was not explicitly discussed in the lecture; it concerns a few
definitions and simple examples from the present or previous versions of the textbook,
illustrating them.
In practice certain types of amount functions would be absurd: for example, we
usually wouldn’t normally want to permit funds to get smaller with increasing time,
although there are situations where such shrinkage could be given some sense. But, for
the present, we will follow the textbook in assuming
Axiom A.2 The function a is non-decreasing; i.e.,

t1 < t2 ⇒ a(t1 ) ≤ a(t2 ) .

(The textbook uses the word increasing, but many mathematicians use that word in a
slightly restrictive way. Here I want to permit the amount to stay unchanged for a time;
I am assuming only that it cannot get smaller as time increases.)
The third axiom of the textbook [4, p. 2], concerning continuity of the function a(t) is
not clear. It appears to say, tautologically, that a(t) is continuous if a(t) is continuous.
I will ignore it until we know what the author means.
Definition A.4 1. The nth period of time is defined to be the period of time between
t = n − 1 and t = n. More precisely, the period normally will consist of the time
interval n − 1 < t ≤ n.
2. Where n is a non-negative integer, the interest earned by the amount A(t) in the
nth period of time is denoted by In , and defined by

In = A(n) − A(n − 1) . (2)

We may relate this notation to a “standard” convention in the “finite difference calculus”,
which would define the increment in A(n) by

4A(n) = A(n + 1) − A(n) .

Thus In = 4A(n − 1).14


14
Note that the name of the function is 2 characters long: 4A. In situations like this, where the
reader might be at risk of not knowing where the name of the function begins or ends, one might use
parentheses, and write (4A)(n − 1).
Information for Students in MATH 329 2009 01 2004

Example A.2 The textbook [4, p. 3] gives graphical examples of 4 kinds of amount
functions.
1. The first has a straight line graph; the graph is sloping upward because the derivative
of the function is positive, in order that the function should be increasing at a
constant rate. In this case the fund is earning a fixed amount which is proportional
to the time that has elapsed, even if the time is not an integer number of time-units;
this is the situation we will later call simple interest.
2. The second example is an exponential curve, where A(t) is a function of the form

A(t) = k · e`t , (3)

` being a real number. If we set t = 0 in (3), we see that k = A(0). Since the
function must be non-decreasing, we know by the calculus that ` ≥ 0. This is the
case that we will be calling compound interest.
3. The third case has a graph which is a horizontal line. This is simply the special
case of the first case where ` = 0, and A(t) is constant; the amount neither grows
nor shrinks.
4. The fourth case is a “step” function, whose graph is horizontal line segments of
constant length, each a fixed distance above the preceding one. This function is
“piecewise continuous”, having points of discontinuity at the integer times. This is
a realistic model, for example, where an amount earns interest only if the amount
remains in the account for a full time period. We can describe this situation by say-
ing that interest is accrued only for completed periods, with no credit for fractional
periods, where the interest earned is credited at the end of the period.
To be more precise, the function is “continuous from the right” everywhere, but it
is “discontinuous from the left” at integer points; this is because the limit from the
left is not equal to the function value there — i.e., because the value you would infer
from the size of the amount preceding an integer time is not equal to the actual size
of the account at integer time.

Summation of Geometric Progressions and Series (These results will be applied


on page 2006 below in the solution of [5, Exercise 3, p. 30].) The reader is reminded of
the easily derived formula [4, Appendix C, A.2, p. 592], [5, Appendix III, A.2, p. 394]
n−1
X µ ¶ µ ¶
k rn − 1 1 − rn
ar = a =a (r 6= 1) . (4)
k=0
r−1 1−r
Information for Students in MATH 329 2009 01 2005

The textbook overlooks the essential restriction that r 6= 1 for the use of this formula;
when r = 1 the sum is
n−1
X
a1k = a · n (r 6= 1) . (5)
k=0
We shall be using these formulæ repeatedly. We shall also be interested in the behavior
of (4) in the limit as n → ∞:
X∞ µ ¶
k 1
ar = a (|r| < 1) . (6)
k=0
1−r

(When |r| ≥ 1, the limit does not exist.)


[4, Exercise 1, p. 42], [5, Exercise 1, p. 29] “Consider the amount function A(t) =
t2 + 2t + 3.
1. “Find the corresponding accumulation function a(t).”
2. (modified) Show that a(0) = 1, that a is a non-decreasing function of time, and
that a is continuous.
3. “Find In .”
Solution:
1.
A(t) t2 + 2t + 3 1 2
a(t) = = 2 = t2 + t + 1 .
A(0) 0 + 2(0) + 3 3 3
2. (a) a(0) = 13 · 02 + 32 · 0 + 1 = 1
(b) If the author intends that we consider this a function of a real variable that
may take any values on the non-negative real axis, then one way to prove
the function is non-decreasing is to show that the derivative is not negative.
(This can be used only for a function that is differentiable, but the present
function is a polynomial, and polynomials have that property.)
1 2 2
a0 (t) = · 2 · t + · 1 = (t + 1) .
3 3 3
We see that a(t) is a non-decreasing function provided t ≥ −1. This could
also have been approached from “first principles”. Suppose that t1 < t2 .
Then
1¡ 2 ¢ 2
a(t2 ) − a(t1 ) = t2 − t21 + (t2 − t1 )
3 µ 3 ¶
t2 t1 2
= (t2 − t1 ) + +
3 3 3
and both factors are positive for non-negative t2 and t1 and t1 < t2 .
Information for Students in MATH 329 2009 01 2006

(c) Polynomial functions are continuous everywhere.


3. In = A(n) − A(n − 1) = (n2 + 2n + 3) − ((n − 1)2 + 2(n − 1) + 3) = 2n + 1.
[4, Exercise 2, p. 42], [5, Exercise 2, p. 30] 1. “Prove that A(n) − A(0) = I1 +
I2 + . . . + In .”
2. “Verify verbally the result obtained” above.
Solution:
1. Summing the equations A(m) − A(m − 1) = Im for 1 ≤ m ≤ n, we obtain
n
X
A(n) − A(0) = Im .
m=1

2. Verbally, the (amount of) interest earned over the concatenation of n periods
is the sum of the interest earned in each of the periods separately.
[5, Exercise 3, p. 30] “Find the amount of interest earned between time t and time
n, where t < n, if
1. Ir = r
2. Ir = 2r .”
Solution: In the preceding problem we showed that the amount earned between
time 0 and time n was the sum of the values of I. The interest earned between
time t and time n will be the total interest from time 0 to time n diminished by
the interest earned from time 0 to time t, so it must be
n
X t
X n
X
Im − Im = Im
m=1 m=1 m=t+1

When Im = m, this is15


n
X t
X n(n + 1) t(t + 1) (n − t)(n + t + 1)
m− m= − = .
m=1 m=1
2 2 2

When Im = 2m , the sum is


n
X µ ¶
m t+1 2n−t − 1
2 =2 = 2n+1 − 2t+1 .
m=t+1
2−1
15
using the familiar formula for the sum of the first natural numbers
Information for Students in MATH 329 2009 01 2007

[4, Exercise 3, p. 42] “For the 5000 investment given in Example 1.1 [4, p. 4], find
the amount of interest earned during the second year of investment, i.e., between
times t = 3 and t = 4.”
Solution: The data given in this example are first a table for a fund of 10,000
invested for 4 years:
t A(t)
0 10, 000.00
1 10, 600.00
2 11, 130.00
3 11, 575.20
4 12, 153.96
In the example 5000 is invested at time t = 2: it appears to be this 5000 to which the
present problem refers. For the full fund the year beginning at t = 3 starts a balance
of 11,575.20 which, at the very end of the period, increases to 12,153.96. The interest
earned is A(4) − A(3) = 12, 153.96 − 11, 575.20 = 578.76. For an amount of 5000
5000
for that year we can scale the interest payment down to × 578.76 = 260.
11130.00
[4, Exercise 3, p. 42] “For the 5,000 investment given in [4, Example 1.1, p. 4], find
the amount of interest earned during the second year of investment, i.e., between
times t = 3 and t = 4.”
Solution: The hypothesis that the investment of 5,000 at time t = 2 is “under the
same interest environment” implicitly requires that the interest be scalable — that
the rate of growth is proportional to the rate of growth in the original fund during
the times 2 ≤ t ≤ 3 (in the example), and 3 ≤ t ≤ 4 in this exercise. In the time
interval 2 ≤ t ≤ 3 the 5,000 grows to become
11, 575.20
× 5, 000 = 5, 200
11, 130.00
at time t = 3; it is this amount whose further growth interests us. In the interval
3 ≤ t ≤ 4 this amount grows to
12, 153.96
× 5, 200 = 5, 460
11, 575.20
at time t = 4. Consequently the interest earned in the interval 3 ≤ t ≤ 4 is
5, 460 − 5, 200 = 260.
[4, Exercise 4, p. 42], [5, Exercise 4, p. 30] “It is known that a(t) is of the form
ct2 +b. If $100 invested at time 0 accumulates to $172 at time 3, find the accumulated
value at time 10 of $100 invested at time 5.”
Information for Students in MATH 329 2009 01 2008

Solution: Let’s begin by normalizing the function: a(0) = 1 ⇔ c·02 +b = 1, so b = 1.


72
The given growth condition implies that 100(c32 + 1) = 172, so c = 900 = 0.08.
Consequently 100 invested at time 5 grows to 100((0.08) · 52 + 1) = 300.
Information for Students in MATH 329 2009 01 2009

A.3 Supplementary Notes for the Lecture of January 09th,


2009
Distribution Date: Friday, January 09th, 2009
(subject to revision)

Summary of some topics from the previous lectures


1. definition of Amount function A(t)
2. definition of Accumulation function a(t): normalized amount function, such that
a(0) = 1.
3. (in notes, but not mentioned in the lectures) nth period = time from t = n − 1 to
t = n (not the usual mathematical convention)
4. Interest earned in nth period : In = A(n) − A(n − 1)

A.3.1 §1.3 THE EFFECTIVE RATE OF INTEREST


When we speak of a rate of interest, we mean the amount of interest per unit investment.
As this rate is normalized, we follow the convention we began with the pair A and a, by
using a lower case letter to represent it.
Definition A.5 The effective rate of interest during the nth period, in , is the ratio of
the amount of interest earned during, and paid at the end of the period to the amount
of principal invested at the beginning of the period. When the effective rate is constant
over all unit time intervals, we may suppress the subscript, and simply write i.
We may use the term effective for periods of lengths other than one unit; but the
most common use will be for terms of one year, where we will speak of the effective
annual rate.
Note that, when we use the unqualified word interest alone, the listener may not know
whether we are referring to the amount of interest, the rate of interest, or the general
concept of payment for the use of capital. There may also be doubts about the accrual
period.
Theorem A.3 In symbols,
In A(n) − A(n − 1) a(n) − a(n − 1)
in = = = (7)
A(n − 1) A(n − 1) a(n − 1)
for integer n ≥ 1. And, when the rate is constant, i, recalling that a(0) = 1, we have

a(n) = (1 + i)a(n − 1) = (1 + i)2 a(n − 2) = . . . = (1 + i)n a(0) = (1 + i)n . (8)


Information for Students in MATH 329 2009 01 2010

In all of this discussion the interest rate is associated with one time interval; this will be
contrasted later with rates — called “nominal” — that are stated for one time interval,
but need to be applied to another.
[4, Exercise 5, p. 42], [5, Exercise 5, p. 30] “Assume that A(t) = 100 + 5t.” Find
i5 and i10 .
Solution:
A(5) − A(4) 5·1 1
i5 = = = .
A(4) 100 + 5 · 4 24
A(10) − A(9) 5·1 1
i10 = = = .
A(9) 100 + 5 · 9 29
[4, Exercise 6, p. 42], [5, Exercise 6, p. 30] “Assume A(t) = 100(1.1)t .” Find i5
and i10 .
Solution:
A(5) − A(4) 100(1.1)4 (0.1)
i5 = = = 0.1 .
A(4) 100(1.1)4
A(10) − A(9) 100(1.1)9 (0.1)
i10 = = = 0.1 .
A(9) 100(1.1)9
[4, Exercise 6, p. 42], [5, Exercise 7, p. 30] “Show that A(n) = (1 + in ) A(n − 1),
where n is a positive integer.” ³ ´
Solution: (1 + in )A(n − 1) = 1 + A(n)−A(n−1)
A(n−1)
A(n)
A(n − 1) = A(n−1) · A(n − 1) =
A(n). ¤
An equation like this is an ideal opportunity to formulate a “verbal” proof. Here is
one:
Since in is defined to be the ratio of interest to the initial amount (at time
t = n − 1) in the account, 1 + in will be the ratio of the terminal amount
A(n) to the initial amount A(n − 1).
[4, Exercise 6, p. 42], [5, Exercise 8, p. 30] “If A(4) = 1000 and in = 0.01n, where
n is a positive integer, find A(7).”
Solution: I(n) = in · A(n − 1) = (0.01)nA(n − 1). It follows that
A(n) = In + A(n − 1) = (0.01(n) + 1)A(n − 1)
A(7) = (0.01(7) + 1)A(6)
= (0.01(7) + 1)(0.01(6) + 1)A(5)
= (0.01(7) + 1)(0.01(6) + 1)(0.01(5) + 1)A(4) = (1.07)(1.06)(1.05)1000
= 1190.91 .
Information for Students in MATH 329 2009 01 2011

A.3.2 §1.4 SIMPLE INTEREST


We saw in the preceding discussion that a(0) = 1 and a(1) = 1 + i1 . Under simple
interest we assume the linear function we saw in the first graphical example above: the
amount of interest earned in a time interval of length t and starting at a fixed time, like
t = 0, is assumed to be proportional to t; we obtain the formula

a(t) = 1 + i1 t

over the 1st time interval, or any longer interval where the so-called simple interest rate
remains equal to i1 . Suppose that the rate of interest is i, and that this rate is to remain
in effect over an interval of n or more successive time periods without adjusting the base
principal from which it is computed. We can determine the rate of interest during the
nth period under this assumption; it is
a(n) − a(n − 1) [1 + i1 n] − [1 + i1 (n − 1)] i1
in = = = (9)
a(n) 1 + i1 (n − 1) 1 + i1 (n − 1)
where we see that, even though the amount of interest earned in any period of time is
a constant multiple of the length of the period, the rate of interest per period is not
constant — it is decreasing from each period to the next.16 Thus a constant rate of
simple interest implies that the effective rates of interest for the successive time intervals
form a decreasing sequence. What is constant for simple interest is the absolute amount
of interest earned in each time interval; under compound interest which we shall meet
in the next section, it is the relative amount of interest that is constant — that is, the
ratio of interest earned to principal.
Note that, when we speak of simple interest, we must specify not only the rate, but
also give enough information so that the reader will know when the principal amount is
adjusted to incorporate the accrued interest, and on what base amount the calculations
are based.
[4, Exercise 9, p. 42], [5, Exercise 9, p. 30] 1. “At what rate of simple interest
will $500 accumulate to $615 in 2 12 years?
2. “In how many years will $500 accumulate to $630 at 7.8% simple interest?”
Solution:
1. Under simple interest at a rate i, 615 = 500a(2.5) = 500(1 + i2.5); solving for
615
−1
i, we obtain i = 5002.5 = 0.092 = 9.2%.
2. Under a rate of 7.8% simple interest for a period of t years, 630 = 500a(t) =
630
−1
500(1 + 0.078t); solving for t, we obtain t = 500
0.078
= 10
3
years.
16
But the derivative of the amount function will be constant.
Information for Students in MATH 329 2009 01 2012

[5, Exercise 10, p. 30] “If ik is the rate of simple interest for period k, where k =
1, 2, . . . , n, show that a(n) − a(0) = i1 + i2 + . . . + in .”
Solution: Note that there is an implicit assumption that the earned interest is not
incorporated into the principle during the period 0 ≤ t ≤ n. What is denoted by
it in this problem is not the effective rate of interest for the tth period, shown in
equation (1.6). Rather, it is the total amount earned during the tth period by the
amount that began as 1 at time t = 0.
Under simple interest an initial investment of a(0) = 1 grows by the prescribed
amounts in each of the following years: a(n) = a(0)+i1 +i2 +. . .+in , so a(n)−a(0) =
i1 + i2 + . . . + in .
[4, Exercise 10, p. 42], [5, Exercise 11, p. 30] “At a certain rate of simple interest
$1000 will accumulate to $1100 after a certain period of time. Find the accumulated
value of $500 at a rate of simple interest three fourths as great over twice as long a
period of time.”
Solution: It is known that, at the rate of interest i, ¡and for a prescribed
¢ ¡ length¢ of
time t, 1000(1 + it) = 1110, so it = 0.11. Then 500 1 + 3i4 · 2t = 500 1 + 23 it =
500(1 + 0.165) = 582.50.
[4, Exercise 11, p. 43], [5, Exercise 12, p. 30] “Simple interest of i = 4% is being
credited to a fund. In which period is this equivalent to an effective rate of 2 12 %?”
Solution: We have to solve for n the equation
0.04
0.025 = in =
1 + (0.04)(n − 1)

and find that n = 16.


[4, Exercise 12, p. 43] “A deposit of 1000 is invested at simple interest at time t = 0.
The rate of simple interest during year t is equal to 0.01t for t = 1, 2, 3, 4, and 5.
Find the total accumulated value of this investment at time t = 5.
Solution: There is no compounding. In year t the investment earns interest in the
amount of (0.01t) ∗ 1000 = 10t. Thus, by time t = 5, the original investment of
1000 has earned interest in the amount of 1000(0.01)(1 + 2 + 3 + 4 + 5) = 150.
The accumulated value of the investment is, therefore, 1000 + 150 = 1150.. (Note
that, if the interest had been compounded, the accumulated value would have been
1000(1.01)(1.02)(1.03)(1.04)(1.05) = 1158.73.)

A.3.3 §1.5 COMPOUND INTEREST


Under compound interest, the earned interest is incorporated into the principal after each
accrual period, following which the interest in the next period is based on the updated
Information for Students in MATH 329 2009 01 2013

principal. Thus the earned interest is reinvested together with the original principal. This
is what we observed in equation (8) above, where the interest rate remained constant
over successive time periods.
The growth of the principal can be analyzed in various ways. Thus, for example, the
value a(2) = (1 + i)2 a(0) = (1 + i)2 may be interpreted
• as (1 + i)a(1) = (1 + i) · (1 + i) as we did in the discussion preceding (8)
• as 1 + 2i + i2 , where the principal of 1 is augmented by 2i, representing the “simple”
interest earned by the principal over two successive time periods, augmented by the
interest at rate i earned by the interest i from the first period and credited at the
end of that period.
Under compound interest at an effective interest rate of i we assume that an amount of 1
grows to (1+i)t during a time interval of length t, where t is not necessarily a non-negative
integer . In most of the sequel it is compound interest that will be assumed to be acting.
Where we assume simple interest, it will often be for fractions of a time period. In some
ways this assumption is obsolete now — a relic of the days when exact computing was
difficult because of the unavailability of reliable and inexpensive calculators. However,
some of the practices of using simple interest have become entrenched in the financial
system, and so we are obliged to continue using them.
[4, Exercise 13, p. 43], [5, Exercise 14, p. 30] “It is known that $600 invested for
two years will earn $264 in interest. Find the accumulated value of $2000 invested
at the same rate of compound interest for three years.”
Solution: What is known is that, at the given interest rate i, 600(1+i)2 = 600+264,
so (1 + i)2 = 1.44 and (1 + i)1 = 1.2 (i.e. i = 20%). At the same rate, 2000(1 + i)3 =
2000(1.2)3 = 2000 × 1.728 = 3456.
[4, Exercise 14, p. 43], [5, Exercise 15, p. 31] “Show that the ratio of the accu-
mulated value of 1 invested at rate i for n periods, to the accumulated value of
1 invested at rate j for n periods, i > j, is equal to the accumulated value of 1
invested for n periods at rate r. Find an expression for r as a function of i and j.”
Solution: µ ¶n
1(1 + i)n i−j
= 1+ .
1(1 + j)n 1+j
i−j
Thus r = .
1+j
[4, Exercise 15, p. 43], [5, Exercise 16, p. 31] “At a certain rate of compound in-
terest, 1 will increase to 2 in a years, 2 will increase to 3 in b years, and 3 will
increase to 14 in c years. If 6 will increase to 10 in n years, express n as a function
of a, b, and c.”
Information for Students in MATH 329 2009 01 2014

Solution: If the common rate is i, the hypotheses are that

1(1 + i)a = 2
2(1 + i)b = 3
3(1 + i)c = 15
6(1 + i)n = 6.

Taking logarithms and solving, we obtain


ln 2
a = (10)
ln(1 + i)
ln 3 − ln 2
b = (11)
ln(1 + i)
ln 5
c = (12)
ln(1 + i)
ln 5 − ln 3
n =
ln(1 + i)
= c−a−b (13)

Note that this answer is “overdetermined”, in the sense that a, b and c are not
independent: by equations (10), (11), (12), a : b : c :: ln 2 : ln 23 : ln 5, that is, a, b,
and c are related by proportionality equations of the form
a b c
= 3 = .
ln 2 ln 2 ln 5

Put another way, we didn’t need all 3 of these equations to solve the problem —
any 2 of them would have been sufficient!
[4, Exercise 16 p. 43], [5, Exercise 17 p. 31] “An amount of money is invested for
one year at a rate of interest of 3% per quarter. Let D(i) be the difference between
the amount of interest earned on a compound interest basis, and on a simple interest
basis for quarter k, where k = 1, 2, 3, 4. Find the ratio of D(4) to D(3).”
Solution: Let the amount of money invested be x. The interest rate is 3% per
quarter . We have
¡ ¢
D(1) = x (1.03)0 ((1.03) − 1) − 0.03
¡ ¢
D(2) = x (1.03)1 ((1.03) − 1) − 0.03
¡ ¢
D(3) = x (1.03)2 ((1.03) − 1) − 0.03
¡ ¢
D(4) = x (1.03)3 ((1.03) − 1) − 0.03
Information for Students in MATH 329 2009 01 2015

from which it follows that


D(4) (1.03)3 ((1.03) − 1) − 0.03 1.033 − 1
= = = 1.522610837 .
D(3) (1.03)2 ((1.03) − 1) − 0.03 1.032 − 1
Information for Students in MATH 329 2009 01 2016

A.4 Supplementary Notes for the Lecture of January 12th,


2009
Distribution Date: Monday, January 12th, 2009
(subject to revision)

A.4.1 §1.5 COMPOUND INTEREST (conclusion)


[4, Exercise 17, p. 43] “The two sets of grandparents for a newborn baby wish to
invest enough money immediately to pay 10,000 per year for four years toward
college costs, starting at age 18. Grandparents A agree to fund the first two pay-
ments, while Grandparents B agree to fund the last two payments. If the effective
rate of interest is 6% per annum, find the difference between the contributions of
Grandparents A and B.”
Solution: The value at time t = 0 of the first two payments is
¡ ¢
10000 (1.06)−18 + (1.06)−19 ,

while the value at time t = 0 of the payments sponsored by Grandparents B is


10000 ((1.06)−20 + (1.06)−21 ). The excess of the value of the A payments over the
B payments is
¡ ¢¡ ¢
10000 (1.06)−18 + (1.06)−19 1 − (1.06)−2
¡ ¢
= 10000(1.06)−21 (1.06 + 1) (1.06)2 − 1
= 10000(1.06)−21 (2.06)(0.1236) = 748.96672... = 748.97.

A.4.2 §1.6 PRESENT VALUE


Definition A.6 1. Under an interest rate of i the sum 1 + i is called the accumulation
factor which, when applied to an amount, yields the value of the amount at the
end of 1 time period.
2. When we view an amount from the past, asking what should be today’s value of an
amount receivable some time in the future, we call the value today the present or
current value or the value discounted (back) to the present.
3. To obtain the amount which, at the beginning of a time period, would yield, at the
interest rate i, an amount of 1 at the end of the period, we may divide by 1 + i or
1
multiply by the discount factor , which is often denoted by v.
1+i
Information for Students in MATH 329 2009 01 2017

1
The textbook denotes the reciprocal by a−1 (t). This is an unfortunate notation,
a(t)
since it appears to involve an inverse function, and that is not the author’s intention.
1
You are advised not to use this notation: write or (a(t))−1 .
a(t)

Definition A.7 Analogous to the accumulation function a(t), we can call (a(t))−1 the
discount function. It represents the amount that needs to be invested today to yield an
amount of 1 at the end of t time periods.

In the following problems there will be a gradual transition from the general formulation
of the preceding definitions to the specific case of compound interest.
[5, Exercise 18 p. 31] “Find an expression for the discount factor during the nth pe-
riod from the date of investment, i.e., (1 + in )−1 , in terms of the amount function.”
1 A(n − 1)
Solution: A(n) = (1 + in ) · A(n − 1) ⇒ = .
1 + in A(n)
[4, Exercise 18 p. 43], [5, Exercise 19 p. 31] “The sum of the present value of 1
paid at the end of n periods and 1 paid at the end of 2n periods is 1. Find (1+i)2n .”
Solution:
¡ ¢2
(1 + i)−n + (1 + i)−2n = 1 ⇔ (1 + i)−n + (1 + i)−n − 1 = 0
µ ¶2
−n 1 5
⇔ (1 + i) + =
2 4
(completing the square)

−n −1 ± 5
⇔ (1 + i) =
2
But the negative sign would be associated with a negative value of (1 + i)n , which
is impossible. Hence

−n −1 + 5
(1 + i) =
2 √
−2n −n 3− 5
(1 + i) = 1 − (1 + i) =
2 √
2n 2 2 3+ 5
(1 + i) = √ = √ · √
3− 5 3− 5 3+ 5
√ √
2(3 + 5) 3+ 5
= =
9−5 2
Information for Students in MATH 329 2009 01 2018

[4, Exercise 19, p. 44], [5, Exercise 21 p. 31] “It is known that an investment of
$500 will increase to $4000 at the end of 30 years. Find the sum of the present
values of three payments of $10,000 each which will occur at the end of 20, 40, and
60 years.”
Solution: At the rate i, 500(1 + i)30 = 4000, so (1 + i)30 = 8, and (1 + i)10 = 2.
Hence the present value of payments of 10,000 to occur at the ends of 20, 40, and
60 years is
µ ¶
¡ 20 40 60
¢ 1 1 1
10000 v + v + v = 10000 + +
4 16 64
210000
= = 3281.25.
64

[5, Exercise 20, p. 31] “Show that the current value of a payment of 1 made n periods
ago and a payment of 1 to be made n periods in the future is greater than 2, if
i > 0.”
Solution: The payment from the past is currently worth (1 + i)n , while the future
payment is currently (=presently) worth v n = (1 + i)−n . The excess of this sum
over 2 is ¡ n n ¢2
(1 + i)n + (1 + i)−n − 2 = (1 + i) 2 − (1 + i)− 2 , (14)
which, being a square, cannot be negative. Hence

(1 + i)n + (1 + i)−n ≥ 2 ,
n n
which equality holding precisely when (1+i) 2 = (1+i)− 2 , i.e., when (1+i)n −1 = 0,
i.e., if either i = 0 or n = 0.

A.4.3 §1.7 THE EFFECTIVE RATE OF DISCOUNT


We will try to develop, in parallel to the interpretation of amount and accumulation
functions from the point of view of the addition of interest, paid at the ends of time
periods, a discount interpretation under which the amount and accumulation functions
are discounted by subtracting an amount of discount. This will lead to several symbols
analogous to those in the interest model. This parallel development will be incomplete,
and most of the time we will be working only with the interest model. For example,
we could interpret In given in equation (2) as an amount of discount. Then we could
parallel equation (7) on 2009 of these notes with

In A(n) − A(n − 1) a(n) − a(n − 1)


= = (15)
A(n) A(n) a(n)
Information for Students in MATH 329 2009 01 2019

Definition A.8 The common amount of the members of equation (15)is denoted by dn ,
and called the effective rate of discount.

Note the differences between the two models:


• Under the interest model, the payment for the use of the money is made at the end
of the period, based on the balance at the beginning of the period.
• Under the discount model, the “payment” is deducted at the beginning of the period
from the final amount which will be present at the end of the period.
We will use the word equivalent to describe schemes of interest and/or discount that
produce the same ultimate accumulated value from an initially invested amount. In the
present application, which is only the first where we will meet this concept, we can, for
example, equate
1
=1−d
1+i
from which we obtain various other relationships, e.g.,
i
d = (16)
1+i
d
i = (17)
1−d
d = 1−v (18)
i − d = id (19)

You should try to explain each of these identities “verbally”, i.e., in words, in a way that
a person lacking your technical knowledge can find the equation plausible.
If a discount rate d is applied t times to a final amount of 1, the current value will be
1
= v t = (1 − d)t .
a(t)

The textbook makes several observations about the use of the discount model, and about
the term discount. For these and other reasons, we shall not be devoting equal time to
the discount model in this course. But you should be aware that most of what we do
could be redeveloped from the discount point of view.

Definition A.9 The accumulation function for simple discount at a discount rate d ≥ 0
1 1
is given by a(t) = , for t < .
1 − dt d
Note that, unlike the situation for simple interest, we must restrict the length of time
over which we propose to apply simple discount.
Information for Students in MATH 329 2009 01 2020

Definition A.10 The accumulation function for compound discount at a discount rate
1
d is given by a(t) = .
(1 − d)t
While we need not restrict t in the compound discount case, we must restrict the effective
discount rate d, requiring that 0 ≤ d < 1.
[4, Exercise 20, p. 44], [5, Exercise 23 p. 31] 1. “Find d5 if the rate of simple in-
terest is 10%.”
2. “Find d5 if the rate of simple discount is 10%.”
Solution:
1. Under simple interest at 10%, In = 0.1 × A(0) for all non-negative integers n.
So
I5 (0.1)A(0) 0.1 1
d5 = = = = .
A(5) (1 + 5(0.1))A(0) 1.5 15
A(0)
2. But, if the rate of simple discount is 10%, then A(n) = .
1 − 0.01n
1 1
A(5) − A(4) − 1
d5 = = 0.05 0.06 = .
A(5) 1 6
0.5
[4, Exercise 21, p. 44] “Find the effective rate of discount at which a payment of 200
immediately and 300 one year from today will accumulate to 600 two years from
today.”
Solution: Denote the effective annual rate of discount by d. The sum of the present
values of the two payments is 200 + 300(1 − d). This must be equal to the present
value of 600 received 2 years from today, i.e., to 600(1 − d)2 . The equation of
value is 200 + 300(1 − d) = 600(1 − d)2 , which has only one positive solution,
1 − d = 0.8791529, from which we see that d = 0.120847, i.e., d = 12.08%.
Information for Students in MATH 329 2009 01 2021

A.5 Supplementary Notes for the Lecture of January 14th,


2009
Distribution Date: Wednesday, January 14th, 2009, updated on January 21st, 2009
(subject to further revision)

A.5.1 §1.7 THE EFFECTIVE RATE OF DISCOUNT (conclusion)


In both the interest and discount points of view we denote the payment for the use
during the nth period of the money in an amount A(t) by In = A(n) − A(n − 1), and
(unfortunately) use the same word “interest” here for In . We define for integers n ≥ 1

In A(n) − A(n − 1) a(n) − a(n − 1)


in = = = equation (7)
A(n − 1) A(n − 1) a(n − 1)
In A(n) − A(n − 1) a(n) − a(n − 1)
dn = = = , (20)
A(n) A(n) a(n)
so
A(n − 1) = (1 + i)−1 A(n) = (1 − d)A(n) . (21)
[4, Exercise 22 p. 44], [5, Exercise 22 p. 31] “The amount of interest earned on A
for one year is $336, while the equivalent amount of discount is $300. Find A.”
Solution: Denote the equivalent rates of interest and discount by i and d respec-
tively. The hypotheses are that

iA = 336
dA = 300

subject to the relationship between i and d, which may be expressed in various


ways. We wish to obtain A, not i or d. So let us solve the preceding equations for
i and d in terms of A,
336 300
i= d=
A A
and substitute into one of the equations relating i and d, e.g., into i − d = id:
336 300 336 × 300
− = ⇔ 36A = 336(300)
A A A2
⇔ A = 2800

[4, Exercise 23, p. 44], [5, Exercise 2, p. 53] “Find the present value of 5000 to be
paid at the end of 25 months at a rate of discount of 8% convertible quarterly:
1. Assuming compound discount throughout.
Information for Students in MATH 329 2009 01 2022

2. Assuming simple discount during the final fractional period.”


Solution:
1. The effective quarterly discount rate is 8%4
= 2%. The present value of 5000 to
25 1 25
be paid at the end of 12 × 4 = 8 3 quarterly periods from now is 5000(1 − d) 3 =
500(0.845053031) = 4225.27.
1
2. The last month is to be interpreted as 12 of a period during which simple interest
8
is charged at 8%, so the discounting factor for that month would be 1 − 12 %=
0.99333333. Further discounting back through 8 quarter-years at a rate of
8
4
% = 2% per quarter year yields a present value of 5000 (0.988 ) (0.99333333) =
4225.46 .
[4, Exercise 24 p. 44], [5, Exercise 27 p. 32] “Show that
d3 (i − d)2
= .”
(1 − d)2 1−v
Solution: I give an algebraic proof. In an algebraic proof of an identity we can, in
the most uninspired proof, prove both members of the alleged equation are equal
to the same quantity. Usually many possible proofs exist, but some may be more
interesting than others, because they may be easier to explain in words, i.e., verbally.
Such proofs are more satisfying if we can start with the quantity on one side of the
equal sign, and, by applying familiar results, transform it into the quantity on the
other side. For example
µ ¶2
d3 1
2
=d = di2 [4, (1.14), p. 17]
(1 − d) 1−d
(id)2 (i − d)2
= =
d 1−v
[4, (1.16), (1.17), p. 18] .
[4, Exercise 25 p. 44], [5, Exercise 26 p. 32] “If i and d are equivalent rates of sim-
ple interest and simple discount over t periods, show that i − d = idt.”
Solution: The discounted value (at simple interest) at time 0 of 1 at time t is 1 − dt.
At simple interest this accumulates to (1 − dt)(1 + it) = 1. Expanding and dividing
by t yields
i − d = idt . ¤

A.5.2 §1.8 NOMINAL RATES OF INTEREST AND DISCOUNT


Definition A.11 [5, p. 14] Two rates of interest or discount are equivalent if an amount
of principal invested for the same length of time under either of the rates accumulates
to the same value.
Information for Students in MATH 329 2009 01 2023

We have already applied this concept in the preceding section, when we related equiv-
alent rates of interest and discount over the same time interval. In this section we will
relate equivalent rates of interest and/or discount over different time intervals, where one
interval is a multiple of the other.
When we speak of a nominal rate of interest, we will normally describe a payment
scheme that is more or less frequent than the usual payment interval; the interest rate
that we will describe will represent the total of all payments made over that usual
interval. Summing the payments does not take into account the interest or discount to
shift all payments to one specific time. We will determine relationships between nominal
rates and effective rates. The definitions are particularly unintuitive, so they must be
memorized.
Definition A.12 Let m be either a positive integer or the reciprocal of a positive integer.
1. A nominal rate of interest i(m) payable m times per period represents m times the
amount of compound interest that will have to be paid at the end of each m1 th of a
period which is equivalent to an effective interest rate of i per period.
2. A nominal rate of discount d(m) charged m times per period represents m times the
amount of compound discount that will have to be charged at the beginning of each
1
m
th of a period which is equivalent to d, the amount charged at the beginning of
the period under an effective discount rate of d per period.
Theorem A.4 1. A nominal rate of interest of i(m) payable and compounded at the
(m)
end of every m1 th of a year is equivalent to17 an effective rate of interest of i m
payable ³every mth ´mof a year. Thus a principal of 1 accumulates at the end of a full
i(m)
year to 1 + m . We have, therefore, the relationship
µ ¶m
i(m)
1+ = 1 + i, (22)
m
or, equivalently, ³ ´
1
i(m) = m (1 + i) m − 1 . (23)

2. A nominal rate of discount of d(m) payable and compounded at the beginning of


(m)
every m1 th of a year is equivalent to an effective rate of discount of d m payable
in advance, every
³ mth(m)of´ma year. Thus an accumulation of 1 at the end of a full
year has value 1 − d m at the beginning of the year. We have, therefore, the
relationship µ ¶m
d(m)
1− = 1 − d, (24)
m
17
Indeed, it has been defined to be
Information for Students in MATH 329 2009 01 2024

or, equivalently,
³ 1
´ ³ 1
´
d(m) = m 1 − (1 − d) m = m 1 − v m . (25)

We can combine equations (22) and (24) into


µ ¶m µ ¶−p
i(m) d(p)
1+ =1+i= 1− ; (26)
m p

when m = p, this yields the equation

i(m) d(m) i(m) d(m)


− = · (27)
m m m m
of which (19) is the special case m = 1.
[4, Exercise 26 p. 44], [5, Exercise 28 p. 32] 1. “Express d(4) as a function of i(3) .”
2. “Express i(6) as a function of d(2) .”
Solution:
1. Since
µ ¶3
i(3)
1+ = 1+i
3
µ ¶4
d(4)
and 1− = 1 − d,
4
µ ¶− 34
d(4) i(3)
(1 + i)(1 − d) = 1 ⇒ 1 − = 1+
4 3
à µ ¶− 34 !
i(3)
⇒ d(4) =4 1− 1+ .
3
µ ¶6 µ ¶−2
i(6) −1 d(2)
2. 1+ = 1 + i = (1 − d) = 1− ⇒
6 2
õ ¶− 13 !
d(2)
i(6) = 6 1− −1 .
2
Information for Students in MATH 329 2009 01 2025

A.6 Supplementary Notes for the Lecture of January 16th,


2009
Distribution Date: Friday, January 16th, 2009
(subject to revision)

A.6.1 §1.8 NOMINAL RATES OF INTEREST AND DISCOUNT (conclu-


sion)
A warning about nominal rates. It is not true that i(m) and d(m) inherit all familiar
relationships that hold between i and d. An example is equation (27) on page 2024 of
these notes, which coincides with the familiar relationship between i and d only when
m = 1. Because of the definitions we have chosen for i(m) and d(m) , the effective rates
(m) (m)
of interest and discount for m1 th of a year are i m and d m , which do have properties
analogous to those that hold between i and d.
1
[4, Exercise 27 p. 44], [5, Exercise 32 p. 32] 1. “Show that i(m) = d(m) · (1 + i) m .
2. “Verbally interpret the result obtained above.”
Solution:
1.
1
³ 1
´ 1
d(m) · (1 + i) m = m 1 − v m · (1 + i) m
³ 1 1 1
´
= m (1 + i) − v · (1 + i)
m m m

³ 1 1
´
= m (1 + i) m − (v(1 + i)) m
³ 1 1
´
= m (1 + i) m − 1 m
³ 1
´ 1
= m (1 + i) m − 1 = i m

2. i(m) = the interest payable on loan of m at the end of m1 period. If this interest
had been prepaid at the beginning of the time period, it would have been d(m) ,
which accumulates for m1 period at a compound interest accumulation factor of
(1 + i)(m) .
[4, Exercise 28 p. 44], [5, Exercise 30 p. 32] “Find the accumulated value of $100
at the end of two years:
1. “if the nominal annual rate of interest is 6%, convertible quarterly; and
2. “if the nominal annual rate of discount is 6%, convertible once every four years.”
Solution:
Information for Students in MATH 329 2009 01 2026

¡ ¢
0.06 2×4
1. 100 1 + 4
= 100(1.015)8
2 1
2. 100 (1 − 4 × 0.06)− 4 = 100(1 − 0.24)− 2
[5, Exercise 29 p. 32] “On occasion, interest is convertible less frequently than once a
year. Define i( m ) and d( m ) to be the nominal annual rates of interest and discount
1 1

convertible once every m years. Find a formula analogous to formula (1.22a) for
this situation.”
Solution:
à ! m1 à ! p1
i( m ) d( p )
1 1

1+ = 1−
m p
[5, Exercise 31 p. 32] “Derive the formula
i(m) d(m) i(m) d(m)
− = · [4, (1.24), p. 25][5, (1.23), p. 20].” (28)
m m m m
Solution: For m1 th of a year, the effective interest and discount rates are, respectively
i(m) (m)
m
and d m . We apply the following relationship between effective rates of interest
and discount:
i − d = id [5, (1.17), p. 15]
for m1 th of a year.
Another derivation of this equation would be from (??) with p = m.
[4, Exercise 29 p. 45], [5, Exercise 33 p. 32] “Given that i(m) = 0.1844144, and
d(m) = 0.1802608, find m.”
Solution: µ ¶µ ¶
i(m) d(m)
1+ 1− =1
m m
i(m) d(m) 0.1844144 × 0.1802608
⇒m= (m) (m)
= = 8.0033434...
−d + i 0.0041536
i(4)
i(n) 1+ 4
[4, Exercise 30 p. 45], [5, Exercise 34 p. 32] “It is known that 1 + = .
n 1+ i(5)
5
Find n.”
Solution:
i(4)
i(n) 1+ 4
1+ =
n 1+ i(5)
5
1 1 1 1
⇒ (1 + i) n = (1 + i) 4 − 5 = (1 + i) 20
⇒ n = 20 .
Information for Students in MATH 329 2009 01 2027

(4)
[4, Exercise 31 p. 45], [5, Exercise 35 p. 32] “If r = di (4) , express v in terms of r.”
Solution: The following is a valid solution, but the form could be improved:
³ 1
´
i(4) = 4 (1 + i) 4 − 1
³ 1
´
d(4) = 4 1 − (1 − d) 4
i(4) 1 − 41
r = = 1 = v
d(4) (1 − d) 4

Hence v = r−4 .
Now, having found the appropriate relationships, let’s rebuild the solution in a more
elegant form, beginning with the function v that we wish to evaluate:
1
v =
1+i
= 1−d
à !4
1
= 1
(1 + i) 4
 4
1
(1 + i) − 1 4
= ³ 1
´ 1

(1 + i) − 1 · (1 + i)
4 4

à 1
!4
1 − (1 + i)− 4
= 1
(1 + i) 4 − 1
à 1
!4
1 − (1 − d) 4
= 1
(1 + i) 4 − 1
 ³ 1
´ 4
4 1 − (1 − d) 4
=  ³ 1
´
4 (1 + i) − 1
4

µ (4) ¶4
d 1
= (4)
= 4.
i r

A.6.2 §1.9 FORCES OF INTEREST AND DISCOUNT


This section of the textbook begins with the definition of the force of interest, δt at time
t:
Information for Students in MATH 329 2009 01 2028

Definition A.13
A0 (t) a0 (t) d d
δt = = = ln A(t) = ln a(t) . (29)
A(t) a(t) dt dt
A definition in this generality is appropriate if we are considering very general interest
schemes. However, in the context of compound interest at a constant rate we can take
the following theorem as a definition:
Theorem A.5 When i is constant, δt = ln(1 + i).
a0 (t)
Proof: When a(t) = (1 + i)t , a0 (t) = (1 + i)t · ln(1 + i), so δt = = ln(1 + i). ¤
a(t)
Theorem A.6 When i is constant, lim i(m) = lim d(m) = δ .
m→∞ m→∞
Proof: When i is constant,
³ 1
´
lim i(m) = lim m (1 + i) m − 1
m→∞ m→∞
1
(1 + i) m − 1
= lim 1
m→∞
m
1
− m12 · ln(1 + i) · (1 + i) m
= lim by l’Hospital’s Rule
m→∞ − m12
= ln(1 + i) = δ
³ 1
´
lim d(m) = lim m 1 − (1 − d) m
m→∞ m→∞
1
1 − (1 − d) m
= lim 1
m→∞
m
1
− 12 · ln(1 − d) · (1 − d) m
= lim m by l’Hospital’s Rule
m→∞ − m12
= − ln(1 − d) = + ln(1 + i) = δ ¤
1
We could avoid the use of l’Hospital’s Rule above by changing the variable to x = m
.
For example,
1
(1 + i) m − 1 (1 + i)x − 1
lim 1 = lim+
m→∞
m
x→0 x
(1 + i)x − (1 + 0)x
= lim+
x→0
¯x
d ¯
= ((1 + i)x )¯¯
dx x=0
= (1 + i)x · ln(1 + i)|x=0 = ln(1 + i) .
Information for Students in MATH 329 2009 01 2029

A.7 Supplementary Notes for the Lecture of January 19th,


2009
Distribution Date: Monday, January 19th, 2009
(subject to revision)

A.7.1 §1.9 FORCES OF INTEREST AND DISCOUNT (conclusion)


Motivation for Definition A.13 Under constant compound interest i the growth of
the amount function A(t) in a small period ∆t is given by A(t + ∆t) = A(t) · (1 + i)∆t ,
so
A(t+∆t)−A(t)
∆t (1 + i)∆t − 1
= .
A(t) ∆t
A0 (t)
In the limit, as ∆t → 0, the left side approaches , while the right side approaches
A(t)
ln(1 + i) = δ. This motivates Definition A.5 (equation (29)),

A0 (t) a0 (t) d d
δt = = = ln A(t) = ln a(t)
A(t) a(t) dt dt

when the interest rate is now permitted to be time-dependent; the force of interest also
becomes time-dependent, and we denote it by δt . In general we can solve differential
equation (A.13) by integrating:
Rt
δr dr
A(t) = A(0) · e 0 ,

which implies that Rt


δr dr
a(t) = e 0 . (30)
We can interpret the product A(t) · δt · ∆t = A0 (t) · ∆t ≈ A(t + ∆t) − A(t) = ∆A(t) as
the increment in A(t) over the time interval from t to t + ∆t, i.e., as the interest earned
in the account in that time interval. Then we have

Theorem A.7
Z n Z n
A(t) · δt dt = A0 (t) dt = A(t)]n0 = A(n) − A(0) ,
0 0

giving an integral formula for the interest earned over a time interval. This formula may
be applied under conditions of variable interest.
For the present, please omit the discussion of force of discount on [4, p. 30].
Information for Students in MATH 329 2009 01 2030

Force of interest under simple interest and simple discount While δt is constant
under compound interest and compound discount, it is not constant under simple interest
and simple discount: under the former it decreases with time, and, under the latter, it
increases with time.
i
Theorem A.8 1. When a(t) = 1 + it, where i is constant, δt = 1+it
for t ≥ 0.
1 d
2. When a(t) = 1−dt
, then δt = 1−dt
, for 0 ≤ t < d1 .

Relations between interest, discount, and force of interest We have already


seen that, under constant compound interest and compound discount,

δ = ln(1 + i) .

Since (1 + i)(1 − d) = 1, this relationship implies that

δ = − ln(1 − d) .

These equations could be expressed, equivalently, in the forms

1 + i = eδ (31)
1 − d = e−δ (32)
i = eδ − 1 (33)
d = 1 − e−δ . (34)

[4, Exercise 34, p. 45], [5, Exercise 42, p. 33] “Fund A accumulates at a simple in-
terest rate of 10%. Fund B accumulates at a simple discount rate of 5%. Find the
point in time at which the forces of interest on the two funds are equal.”
Solution: For a unit of capital in Fund A, A(t) = 1 + 0.1t, so the force of interest is

A0 (t) 0.1
δtA = = .
A(t) 1 + 0.1t

For a unit of capital in Fund B, A(t) = (1 − 0.05t)−1 , so the force of interest is

A0 (t) 0.05
δtB = = .
A(t) 1 + 0.05t
Equating these forces of interest yields

0.1(1 − 0.05t) = 0.05(1 + 0.1t) ⇔ t = 5,

so the forces of interest are equal after precisely 5 years.


Information for Students in MATH 329 2009 01 2031

[4, Exercise 35, p. 45]. [5, Exercise 43, p. 33] “An investment is made for one year
in a fund whose accumulation function is a second degree polynomial. The nominal
rate of interest earned during the first half of the year is 5% convertible semiannu-
ally. The effective rate of interest earned for the entire year is 7%. Find δ0.5 .”
Solution: We are to assume that a(t) has the form a(t) = Kt2 + Lt + M ; since
A(0) = 1, M = 1.
a(0.5) = 1.025 ⇒ 0.25K + 0.5L = 0.025
a(1) = 1.07 ⇒ K + L = 0.07
½
K = 0.04

L = 0.03
⇒ a(t) = 0.04t2 + 0.03t + 1
⇒ a0 (t) = 0.08t1 + 0.03
a0 (0.5) 0.04 + 0.03 14
⇒ δ0.5 = = =
a(0.5) 0.01 + 0.015 + 1 205
[5, Exercise 36 p. 32] Derive the formula lim d(m) = δ.
m→∞
Solution:
³ 1
´
lim i(m) = lim m (1 + i) m − 1
m→∞ m→∞
1
(1 + i) m − 1
= lim 1
m→∞
m
x
(1 + i) − 1
= lim+
x→0 x ¯
d ¯
= ((1 + i)x )¯¯
dx x=0
= (1 + i)x ln(1 + i)|x=0 = ln(1 + i)
Analogously,
³ 1
´
lim d(m) = lim m 1 − (1 − d) m
m→∞ m→∞
1
1 − (1 − d) m
= lim 1
m→∞
m
1 − (1 − d)x
= lim+
x→0 x ¯
d ¯
x ¯
= − ((1 − d) )¯
dx x=0
= − ln(1 − d) = ln(1 + i)
Information for Students in MATH 329 2009 01 2032

A.7.2 §1.10 VARYING INTEREST


[4, Exercise 37, p. 45] “Find the level, effective rate of interest over a three-year pe-
riod which is equivalent to an effective rate of discount of 8% the first year, 7% the
second year, and 6% the third year.”
Solution: Suppose the rate of interest sought is si effective per annum. Then (1 +
1
i)−3 = (1 − 0.06)(1 − 0.07)(1 − 0.08) ⇒ 1 + i = 3 = 1.0753103 ⇒
(0.94)(0.93)(0.92)
i = 7.53%.
[4, Exercise 39, p. 46] “An investor makes a deposit today, and earns an average con-
tinuous return (force of interest) of 6% over the next five years. What average
continuous return must be earned over the subsequent five years in order to double
the investment at the end of ten years?”
Solution: Let i be the average continuous return earned over the subsequent five
years. Then
¡ 0.006 ¢5 ¡ i ¢5
e e = 2 ⇔ e0.3+5i = 2
⇔ 0.3 + 5i = ln 2
ln 2 − 0.3
⇔ i= = 0.0786294
5
or 7.86%.
[4, Exercise 40, p. 46] “In Fund X money accumulates at a force of interest δt =
0.01t + 0.1 for 0 ≤ t ≤ 20. In Fund Y money accumulates at an annual effective
interest rate i. An amount of 1 is invested in each fund for 20 years. The value
of Fund X at the end of 20 years is equal to the value of Fund Y at the end of 20
years. Calculate the value of Fund Y at the end of 1.5 years.”
Solution: After 20 years the value of the dollar invested in Fund X is
R 20 2 20
e 0 (0.01t+0.1) dt
= e[0.005t + 0.1t]0 = e2+2 = e4 .

The dollar in Fund Y has grown to (1 + i)20 , and these two amounts are postulated
to be equal. Accordingly, the dollar in Fund Y grows to
1.5×4
(1 + i)1.5 = e 20 = 1.3498588

after one and one-half years. While this appears to be a high rate of growth, one
must not overlook that the rate of growth of Fund X is very high; for example, in
its final year that fund grows by a factor of more than e0.19+0.1 = 1.336, i.e., more
than 33%.
Information for Students in MATH 329 2009 01 2033

[4, Exercise 41, p. 46] “If the effective rate of discount in year k is equal to 0.01k+0.06
for k = 1, 2, 3, find the equivalent rate of simple interest over the three-year period.”
Solution: Let the equivalent (annual) rate of simple interest be i. Then a principal
of 1 grows under simple interest to 1 + 3i. This must be equal to
1 1 1
· · = 1.2843631 ,
1 − 0.07 1 − 0.08 1 − 0.09
implying that i = 0.0947877 or 9.48%.

A.7.3 §1.11 SUMMARY OF RESULTS


Table A.7.3, page 2033 is reproduced from the textbook [4, Table 1.1, p. 38][5, Table 1.1,
p. 29].

Rate of interest The accumulated value The present value of 1


1
or discount of 1 at time t = a(t) at time t =
a(t)
Compound interest
i (1 + i)t v t = (1 + i)−t
µ ¶mt µ ¶−mt
i(m) i(m)
i(m) 1+ 1+
m m
d (1 − d)−t (1 − d)t
µ ¶−mt µ ¶mt
d(m) d(m)
d(m) 1− 1−
m m
δ eδt e−δt
Simple interest
i 1 + it (1 + it)−1

Simple discount
d (1 − dt)−1 1 − dt

Table 2: Summary of Relationships in Chapter 1


Information for Students in MATH 329 2009 01 2034

A.8 Supplementary Notes for the Lecture of January 21st, 2009


Distribution Date: Wednesday, January 21st, 2009
(subject to revision)

Textbook Chapter 2. Solution of problems in interest

A.8.1 §2.1 INTRODUCTION


Chapter 2 “discusses general principles to be followed in the solution of problems in
interest. The purpose of this chapter is to develop a systematic approach by which the
basic principles from Chapter 1 can be applied to more complex financial transactions.”
“Successive chapters have two main purposes:
1. “to familiarize the reader with more complex types of financial transactions, includ-
ing definitions of terms, which occur in practice;
2. “to provide a systematic analysis of these financial transactions, which will often
lead to a more efficient handling of the problem than resorting to basic principles.”

A.8.2 §2.2 THE BASIC PROBLEM


As the textbook observes, most interest problems can be broken down into problems
that involve 4 parameters:
1. The principal amount originally invested. (In these notes I will normally not show
the dollar sign for amounts invested.)
2. The length of time during which the amount will be held in the account.
3. The rate of interest or discount to be applicable; in the limit, this could be expressed
as the force of interest of discount.
4. The amount accumulated in the account at the end of the investment period.
These parameters are not independent: when three of them have been specified, the
fourth is determined; but finding the value of that 4th parameter may not be trivial.
Numerical results can be obtained by
• direct calculation
• direct calculation by hand, typically using series expansions.18
18
Students in MATH 329 do not often have background from Calculus 3, where the computation using
series is normally introduced. Thus any applications of this type will have to be justified to students
whose background cannot be assumed to include more than Calculus 1 and Calculus 2. Examples of
Information for Students in MATH 329 2009 01 2035

• use of compound interest tables, as in [5, Appendix I]; in this case we may interpolate
between entries in the tables; more information will be provided — you are not
expected to have any prior knowledge about interpolation;19
The following problem was not discussed at the lecture.
[5, Exercise 1, p. 53] “10000 is invested for 4 months at 12.6%, where interest is com-
puted using a quadratic to approximate an exact calculation. Find the accumulated
value.”
Solution: The exact value using compound interest is
4
10000(1.126) 12 = 10403.50 .
Alternatively, when we approximate by a quadratic, we truncate the MacLaurin
series after the 2nd degree term (cf. (35))
µ ¶ µ ¶ µ ¶
1 1 1 2 1 2 1 2 5 1
(1 + i) 3 = 1 + · i + · − · ·i + · − · − · · i3 + . . .
3 3 3 2! 3 3 3 3!
µ ¶
1 1 2 1
≈ 1+ ·i+ · − · · i2
3 3 3 2!
= 1.040236
hence the accumulated value is approximately 10402.36.
Note that a linear approximation, which would be equivalent to simple interest,
would give 10420: as the period of time is less than one unit, a linear approximation
is higher than the value given by compound interest — equivalently, the line joining
the points (x, y) = (0, (1.043)0 ) and (x, y) = (1, (1.043)1 ), passes over the graph of
y = (1.043)x .
series of this type are
k(k − 1) 2 k(k − 1)(k − 2) 3
(1 + i)k = 1 + ki + i + i + ... (35)
2! 3!
x2 x3
ex = 1+x+ + + ... (36)
2! 3!
x2 x3
ln(1 + x) = x− + − ... (37)
2 3
which are “MacLaurin” series (a special case of “Taylor” series), and have restrictions on the numbers
for which they are valid. (The first is always valid if |i| < 1, and also for any i if k is a non-negative
integer; the second is valid for all x; and the third is valid for −1 < x ≤ 2.) The error that occurs when
one “truncates” a series — i.e., when one stops adding at a particular term — can be estimated.
“Using series expansions for calculation purposes is cumbersome and should be unnecessary except in
unusual circumstances.”
19
The current edition of the textbook does not contain interest tables, although I may discuss some
examples of interpolation and provide specific tables at that time, for historical reasons, even though
the technique may be obsolete in these applications.
Information for Students in MATH 329 2009 01 2036

A.8.3 §2.3 EQUATIONS OF VALUE


We will often express the relationship between different sums in a problem in an “equation
of value”, which is simply a statement obtained by bringing all amounts to a particular
time (by accumulating or discounting according to whatever accumulation functions are
appropriate)”. Sometimes we will have an inequality rather than an equation. The time
where the various amounts are compared is the comparison date. Under compound inter-
est equations of value for the same payments, made a different comparison dates, should
be equivalent: you should be able to extract one from the other simply by multiplying
or dividing by the appropriate accumulation factors.

Time diagrams A time diagram is a one-dimensional diagram where the only variable
is time, shown on a single coordinate axis. We may show above or below the coordinate
of a point on the time-axis values of money intended to be associated with different
funds. As in any mathematical exercises, the diagram is not a formal part of a solution,
but may be very helpful in visualizing the solution. Some authors use variants of time
diagrams where there may be several parallel horizontal axes, representing several funds;
alternatively, you could use the usual type of graph that you have seen in calculus,
where the values of the fund will be shown as points in the plane, with the horizontal
axis representing time, and the second coordinate giving the function value. (The system
I am using for these notes accommodates figures with great difficulty; I will usually not
attempt to show time diagrams for that reason.)
In a typical problem there will exist more than one way in which to view the various
sums of money involved, at any given point in time (the comparison date). In an Equation
of Value we equate two such representations, and thereby may obtain an equation which
we can solve for one of the variables in terms of the others.
If the prevailing rate of interest or discount is non-zero, the equations obtained at
different points in time will be different. Under compound interest or compound discount
— the environments we will be considering most of the time — the information we
derive from equations of value will be the same no matter which comparison date we will
be using; under simple interest and discount, and other environments, the comparison
date could be relevant. Sometimes we represent such a situation by a one-dimensional
diagram, where time is represented along a horizontal axis, the direction of increase
being to the right. Notwithstanding the one-dimensionality, there could be several lines
above and below the time-axis on which we indicate amounts of money being added or
removed, as well as the time labels. The comparison date is often indicated by a vertical
arrow.
[4, Exercise 1, p. 67], [5, Exercise 8, p. 55] “In return for payments of 2000 at the
end of four years and 5000 at the end of ten years, an investor agrees to pay 3000
Information for Students in MATH 329 2009 01 2037

immediately and to make an additional payment at the end of three years. Find
the amount of the additional payment if i(4) = 0.06.”
Solution: If we denote the additional payment by X, then the equation of value at
time t = 0 is

3000 + X(1.015)4×(−3) = 2000(1.015)4×(−4) + 5000(1.015)4×(−10)

which we can solve to yield

X = −3000(1.015)12 + 2000(1.015)−4 + 5000(1.015)−28


= −3586.854514 + 1884.368461 + 3295.496247 = 1593.010194

so the additional payment should be 1593.01. Depending on where rounding has


been applied, the amount could vary slightly from this figure.
[4, Exercise 2, p. 67] “You have an inactive credit card with a 1000 outstanding un-
paid balance. This particular credit card charges interest at the rate of 18% com-
pounded monthly. You are able to make a payment of 200 one month from today,
and 300 two months from today. Find the amount that you will have to pay three
months from today to completely pay off this credit card debt.”
18%
Solution: The effective interest rate per month is = 1.5%. Denote the payment
12
3 months hence by X. The equation of value at time t = 3 months is

1000(1.015)3 = 200(1.015)2 + 300(1.015)1 + X(1.015)

which we can solve, to yield

X = 1000(1.015)3 − 200(1.015)2 − 300(1.015)


= 535.133375

so the final payment must be in the amount of 535.13.


Information for Students in MATH 329 2009 01 2038

A.9 Supplementary Notes for the Lecture of January 23rd,


2009
Distribution Date: Friday, January 23rd, 2009
(subject to revision)

A.9.1 §2.3 EQUATIONS OF VALUE (conclusion)


[4, Exercise 3, p. 67], [5, Exercise 9, p. 54] “At a certain interest rate the present
values of the following two payment patterns are equal:
(i) 200 at the end of 5 years plus 500 at the end of 10 years;
(ii) 400.94 at the end of 5 years.
At the same interest rate 100 invested now plus 120 invested at the end of 5 years
will accumulate to P at the end of 10 years. Calculate P .
Solution: Payment schemes (i) and (ii), both with comparison date t = 0, give rise
to the following equations of value:

200(1 + i)−5 + 500(1 + i)−10 = 400.94(1 + i)−5 (38)


100 + 120(1 + i)−5 = P (1 + i)−10 (39)

Equation (38) implies that

(1 + i)−5 = 0.40188 . (40)

Substituting in (39) yields


100 120
P = 2
+ = 917.76.
(0.40188) 0.40188

[4, Exercise 4, p. 67], [5, Exercise 10, p. 54] “An investor makes three deposits into
a fund, at the end of 1, 3, and 5 years. The amount of the deposit at time t is
100(1.025)t . Find the size of the fund at the end of 7 years, if the nominal rate of
4
discount convertible quarterly is 41 .”
4
Solution: The nominal rate of discount of d(4) = 41 convertible quarterly produces
¡ ¢
1 −4
an annual accumulation factor of 1 − 41 = (1.025)4 . At the end of 7 years the
3 payments accumulate to

100(1.025)1+(4×6) + 100(1.025)3+(4×4) + 100(1.025)5+(4×2) = 483.11 .


Information for Students in MATH 329 2009 01 2039

[4, Exercise 5, p. 67], [5, Exercise 11, p. 54] “Whereas the choice of a comparison
date has no effect on the answer obtained with compound interest, the same cannot
be said of simple interest. Find the amount to be paid at the end of 10 years which
is equivalent to two payments of 100 each, the first to be paid immediately, and the
second to be paid at the end of 5 years. Assume 5% simple interest is earned from
the date each payment is made, and use a comparison date of
1. The end of 10 years.
2. The end of 15 years.”
Solution:
1. With a comparison date at t = 10, the payment at the end of 10 years will be
100(1 + 10 × 0.05) + 100(1 + 5 × 0.05) = 275.
2. With a comparison date at t = 15, the amount P that must be paid at the end
of 10 years satisfies the equation of value
100(1 + 15 × 0.05) + 100(1 + 10 × 0.05) = P · (1 + 5 × 0.05) ,
which implies that P = 260.
20

A.9.2 §2.4 UNKNOWN TIME


Method of Equated Time Omit

Rule of 72 How long does it take money to double? Solving the equation (1 + i)n = 2
yields
ln 2 0.6931471806 i
n= ≈ · . (41)
ln(1 + i) i ln(1 + i)
The last ratio can be approximated using the MacLaurin expansion (37):
i i
= i2 3
ln(1 + i) i− + i3 − . . .
2
1
= ¡ i i2 ¢
1 − 2 + 3 − ...
µ ¶ µ ¶2
i i2 i i2
= 1+ + − ... + + − ... + ...
2 3 2 3
20
The question is: What is the meaning of equivalent?. The meaning taken here is that all amounts
are considered from the point of view of the comparison date, at which time the single amount P is to
equal the sum of the values of the two payments made at different times and transported to the specified
comparison date using simple interest.
Information for Students in MATH 329 2009 01 2040

µ ¶ µ ¶
i 2i 2i2 i2 2i 2i2
= 1+ 1− + − ... + 1− + − ... + ...
2 3 4 4 3 4
i i2
≈ 1+ − + ...
2 12
which is approximately equal to 1.0395 when i = 8%. Applying this approximation to
equation (41) yields
72
n≈ , (42)
100i
which is called the “Rule of 72”. The textbook reports that this rule is “surprisingly
accurate over a wide range of interest rates.”
[4, Exercise 6, p. 68], [5, Exercise 13, p. 55] “Find how long 1000 should be left
to accumulate at 6% effective in order that it will amount to twice the accumulated
value of another 1000 deposited at the same time at 4% effective.”
Solution: If we let t denote the number of years, the equation of value at t = n is

1000(1.06)t = 2000(1.04)t

so
2
n= = 36.39
ln 106 − ln 104
years.
[4, Exercise 7, p. 68] “You invest 3000 today, and plan to invest another 2000 two
years from today. You plan to withdraw 5000 in n years, and another 5000 in n + 5
years exactly, liquidating your investment account at that time. If the effective rate
of discount is equal to 6%, find n.”
Solution: The effective rate of discount is 6%. One version of the equation of value
as of time t = 0 is

3000 + 2000(1 − 0.06)2 = 5000(1 − 0.06)n + 5000(1 − 0.06)n+5 ,

which implies that


3000 + 2000(0.94)2
(0.94)n = .
5000 (1 + (0.94)5 )
This implies that

ln (3000 + 2000(0.94)2 ) − ln(5000 (1 + (0.94)5 ))


n =
ln 1.06
= 9.665461121 .

Thus n is approximately 9 2/3 years.


Information for Students in MATH 329 2009 01 2041

[4, Exercise 8, p. 68], [5, Exercise 14, p. 55] “The present value of two payments
of 100 each to be made at the end of n years and 2n years is 100. If i = 0.08, find
n.”
Solution:
√ Solving the equation of value, 100v 2n + 100v n = 100, we obtain v n =
−1± 5
2
, in which only the + sign is acceptable, since v n > 0. Taking logarithms
gives √
ln −1+2 5
n= = 6.2527 years.
− ln(1.08)
[4, Exercise 10, p. 68], [5, Exercise 16, p. 55] “You are asked to develop a rule of
n to approximate how long it takes money to triple. Find n.”
Solution: On page 2039 of these notes I have considered the “Rule of 72.” Let us
consider the rationale of that rule:
(1 + i)n = 2
ln 2
⇒n =
ln(1 + i)
ln 2 i
= ·
i ln(1 + i)
ln 2 i
= ·
i i − 2 + 3 − i44 + . . .
i 2 i3

ln 2 1
= ·
i 1 − 2 + 3 − i43 + . . .
i i 2

ln 2 1
= · ¡ i i2 i3 ¢
i 1 − 2 − 3 + 4 − ...
à µ ¶ µ ¶2 !
ln 2 i i2 i3 i i2 i3
= · 1+ − + − ... + − + − ... + ...
i 2 3 4 2 3 4
µ ¶
ln 2 i i2
= · 1+ − + ...
i 2 12
³ ´¯
i2 ¯
(ln 2) 1 + 2i − 12 + ... ¯
i=0.08

i
The only change if we wish to obtain a rule for the time for money to triple is that
we obtain ³ ´
i i2
(ln 3) 1 + 2 − 12 + . . .
n≈ .
i
If we approximate i in the numerator by 8%, we obtain a numerator of approxi-
mately 1.14; the rule could be called the “Rule of 114”.
Information for Students in MATH 329 2009 01 2042

[4, Exercise 11, p. 68] “A deposits 10 today and another 30 in five years into a fund
paying simple interest of 11% per year. B will make the same two deposits, but the
10 will be deposited n years from today, and the 30 will be deposited 2n years from
today. B’s deposits earn an annual effective rate of 9.15%. At the end of 10 years
the accumulated value of B’s deposits equals the accumulated value of A’s deposits.
Calculate n.”
Solution: At time 10 the value of A’s deposits is 10(1 + 10(0.11)) + 30(1 + 5(0.11)) =
21+46.5 = 67.5. At the same time the present value of B’s deposits is 10(1.0915)10−n +
30(1.0915)10−2n . Equating the two amounts yields a quadratic equation for X =
(1.0915)−n :
67.5
3X 2 + X − ,
10(1.0915)10
whose only positive solution is
q
12(67.5)
−1 + 1+ 10(1.0915)1 0
X =
6
= 0.8157901067
ln 0.8157901067
implying that n = − = 2.325430525 or 2.33 years.
ln 1.0915
[4, Exercise 12, p. 68], [5, Exercise 18, p. 55] “Fund A accumulates at a rate of
t
12% convertible monthly. Fund B accumulates with a force of interest δt = . At
6
time t = 0 equal deposits are made in each fund. Find the next time that the two
funds are equal.”
Solution: Without limiting generality I can take the amount of each deposit to be
1. The equation of value is
Rt Rtr t2
(1.01)12t = e 0 δr dr
=e 0 6 dr
= e 12 ,

t2
of which one solution is t = 0. Taking logarithms yields 12t ln 1.01 = , so the
12
funds are next equal when t = 144 ln 1.01 = 1.432847643 years.
Information for Students in MATH 329 2009 01 2043

A.10 Supplementary Notes for the Lecture of January 26th,


2009
Distribution Date: Monday, January 26th, 2009
(subject to revision)

A.10.1 §2.5 UNKNOWN RATE OF INTEREST


The textbook and its previous edition describe four methods for determining an unknown
rate of interest:
• Sometimes — particularly if only one payment is involved — it may be possible to
solve the equation of value by using the fact that the logarithm and exponential
functions are mutual inverses.
• Sometimes the equation may be solved by algebraic techniques, for example when
the equation is equivalent to a polynomial equation that factorizes.
• This method is not even mentioned in the current edition. Sometimes the equation
can be solved by interpolation in interest tables.
• When all else fails, it may be necessary to solve by successive approximation or
iteration. There are various methods, and some converge much faster than others.
The justification of the better methods is beyond this course, but is the subject
matter of courses like MATH 317 (Numerical Analysis).
[4, Exercise 13, p. 68], [5, Exercise 19, p. 55] “Find the nominal rate of interest
convertible semiannually at which the accumulated value of 1000 at the end of 15
years is 3000.”
¡ ¢
i 2(15)
Solution: The
³ 1 equation
´ of value at time t = 15 is 1000 1 + 2
= 3000, implying
that i = 2 3 30 − 1 = 7.4598394% .
[4, Exercise 14, p. 68], [5, Exercise 20, p. 55] “Find an expression for the exact ef-
fective rate of interest at which payments of 300 at the present, 200 at the end of
one year, and 100 at the end of two years will accumulate to 700 at the end of two
years.”
Solution: For the unknown interest rate i, which we assume to be non-negative, the
equation of value at time 2 is
300(1 + i)2 + 200(1 + i)1 + 100 = 700 ,
yielding a quadratic equation in 1 + i:
3(1 + i)2 + 2(1 + i) − 6 = 0 ,

−1+ 19
whose only positive solution is 1 + i = 3
= 1.119632981, so i = 11.9632981%.
Information for Students in MATH 329 2009 01 2044

[4, Exercise 15, p. 69] “You can receive one of the following two payment streams:
(i) “100 at time 0, 200 at time n, and 300 at time 2n;
(ii) “600 at time 10.
“At an annual effective interest rate of i, the present values of the two streams are
equal. Given v n = 0.75941, determine i.”
Solution: At time t = 0 an equation of value is
600v 10 = 100 + 200v n + 300v 2n = 100 + 151.882 + 173.011 = 424.893 ,
implying that v 10 = 0.7082, so v = 0.9661, i = 3.51%.
[4, Exercise 16, p. 69], [5, Exercise 21, p. 55] “It is known that an investment of
1000 will accumulate to 1825 at the end of 10 years. If it is assumed that the
investment earns simple interest at rate i during the 1st year, 2i during the second
year, . . . , 10i during the 10th year, find i.”
Solution: The intended interpretation was that the equation of value is
1 + i + 2i + 3i + ... + 10i = 1.825 ,
which implies that i = 0.15 exactly.
[4, Exercise 17, p. 69] “It is known that an amount of money will double itself in 10
years at a varying force of interest δt = kt. Find an expression for k.”
Solution: An equation of value is
R 10
kr dr
2=e 0 = e50k ,
ln 2
implying that k = = 0.01386.
50
[4, Exercise 18, p. 69], [5, Exercise 23, p. 55] “The sum of the accumulated value
of 1 at the end of three years at a certain effective rate of interest, and the present
value of 1 to be paid at the end of three years at an effective rate of discount
numerically equal to i is 2.0096. Find the rate.”
Solution: The equation of value at time t = 3 of 1 payable at time 0 and 1 payable
at time 6 is (1 + i)3 + (1 − i)3 = 2.0096, which implies that i = 0.04 (the only
positive solution).

A.10.2 §2.6 DETERMINING TIME PERIODS


The syllabus now reads “Students should peruse §2.6, but will not be expected to mem-
orize the definitions contained therein.”
A number of schemes are in common use for calculating interest for fractions of a
period:
Information for Students in MATH 329 2009 01 2045

• Under exact simple interest or “actual/actual” one counts the exact number of days,
and assumes the year has 365 days.
• Under ordinary simple interest or “30/360” one assumes that each calendar month
has 30 days, and the year has 360 days.
• Under Banker’s Rule or “actual/360” one uses the exact number of days but treats
a year as having 360 days. The treatment of February 29th in leap years (like the
present) is not completely standardized. These calculation bases can be used for
either simple or compound interest.

Example A.9 Here is a statement I found at the bottom of a bank statement that I
received from a large Canadian bank the last time I was teaching this course: “(name of
bank) calculates interest daily using a 365-day year, including leap years. The interest
rate charged in a leap year will be equal to the Annual Interest Rate in effect on each
day in that year multiplied by 366 and divided by 365. Although this results in slightly
more interest being charged, the effective annual rate is the same when rounded to the
nearest 1/8th of 1%.”

[4, Exercise 19, p. 69], [5, Exercise 5, p. 53] “If an investment was made on the
date the United States entered World War II, i.e., December 7, 1941, and was
terminated at the end of the war on August 8, 1945, for how many days was the
money invested:
1. “on the actual/actual basis?
2. on the 30/360 basis?”
Solution:
1. “From December 7, 1941 to December 7, 1944, 3 years passed, the last of which
was a leap year. The number of days is 3(365) + 1. From December 7, 1944
to August 7, 1945, the numbers of days in the months December through July
were 31 + 31 + 28 + 31 + 30 + 31 + 30 + 31. Add one day, from 7th to 8th August.
The total is 1340.
2. From December 7, 1941 to December 7, 1944, 3 years passed, the last of which
was a leap year. The number of days is 3(360). From December 7, 1944 to
August 7, 1945, the numbers of days in the months December through July
were 8(30). Add one day, from 7th to 8th August. The total is 1321.
[4, Exercise 20, p. 69], [5, Exercise 6, p. 54] “A sum of 10,000 is invested for the
months of July and August at 6% simple interest. Find the amount of interest
earned:
1. “Assuming exact simple interest.
Information for Students in MATH 329 2009 01 2046

2. “Assuming ordinary simple interest.


3. “Assuming the Banker’s Rule.”
Solution:
1. The number of days is 31 + 31 = 62; exact simple interest is 621
365
· 10000 · (0.06) =
101.92.
60
2. Ordinary simple interest is 360 · 10000 · (0.06) = 100.
62
3. Interest under the Banker’s Rule is 360 · 10000 · (0.06) = 103.33.
[4, Exercise 21, p. 69], [5, Exercise 7, p. 54] 1. “Show that the Banker’s Rule is
always more favourable to the lender than is exact simple interest.
2. “Show that the Banker’s Rule is usually more favourable to the lender than is
ordinary simple interest.
3. “Find a counterexample to (the preceding) for which the opposite relationship
holds.”
Solution:
1. Under the Banker’s Rule the denominator of the fraction is decreased, so the
fraction is increased.
2. Suppose that k months have passed. The Banker’s Rule counts the days as
30k. How does this compare with the actual number of days? To answer this
question correctly, we need to compare all terms of lengths under a year. But
the question is not precisely stated, and it’s not clear how we would have to
weight the results. As the computations are extensive, we will not bother.
3. A loan between February 1 and March 1 provides a counterexample.

A.10.3 §2.7 PRACTICAL EXAMPLES


[4, Exercise 22, p. 70], [5, Exercise 25, p. 56] “A bill for 100 is purchased for 96
three months before it is due. Find
1. “The nominal rate of discount convertible quarterly earned by the purchaser.
2. “The annual effective rate of interest earned by the purchaser.”
Solution:
³ ´
d(4)
1. 100 1 − 4
= 96 ⇒ d(4) = 16%.
1 ¡ ¢4
2. 96(1 + i) 4 = 100 ⇒ i = 100
96
− 1 = 17.7375701%.
[4, Exercise 23 p. 70], [5, Exercise 26 p. 56] “A two-year certificate of deposit pays
an annual effective rate of 9%. The purchaser is offered two options for prepayment
penalties in the event of early withdrawal:
Information for Students in MATH 329 2009 01 2047

A - a reduction in the rate of interest to 7%


B - loss of three months’ interest.
In order to assist the purchaser in deciding which option to select, compute the
ratio of the proceeds under Option A to those under Option B if the certificate of
deposit is surrendered:
1. at the end of 6 months
2. at the end of 18 months”
Solution:
1. Under Option A the effective annual interest rate is now 7%, so a principal of 1
1 1 1
returns (1.07) 2 . Under Option B a principal of 1 returns (1.09) 2 − 4 . The ratio
1
(1.07) 2
of returns under Option A to those under option B is 1 1 = 1.012360669.
(1.09) 2 − 4
2. Under Option A the effective annual interest rate is now 7%, so a principal of 1
3 3 1
returns (1.07) 2 . Under Option B a principal of 1 returns (1.09) 2 − 4 . The ratio
3
(1.07) 2
of returns under Option A to those under option B is 3 1 = .9937852442.
(1.09) 2 − 4

[4, Exercise 24 p. 70] “The ABC Bank has an early withdrawal policy for certificates
of deposit (CD’s), which states that interest sill be credited for the entire length
the money actually stays with the bank, but that the CD nominal interest rate will
be reduced by 1.8% for the same number of months as the CD is redeemed early.
An incoming college freshman invests 5000 in a two-year CD with a nominal rate
of interest equal to 5.4% compounded monthly on September 1, at the beginning
of the freshman year. The student intended to leave the money on deposit for the
full two-year term to help finance the junior and senior years, but finds the need to
withdraw it on May 1 of the sophomore year. Find the amount that the student
will receive for the CD on that date.”
Solution: The CD is redeemed after 18 months, although it was issued for 24 months.
Since it is being redeemed 6 months early, the nominal interest rate will be reduced
after 18 − 6 = 12 months: the effective monthly rate is 5.4
12
% for the first 12 months,
and 5.4−1.8
12
% = 3.6
12
% for the last 6 months. The student receives
µ ¶1 µ ¶6
0.054 0.036
5000 1 + 2 1+ = 5000(1.055356752)(1.074496218) = 5372.48 .
12 12

[4, Exercise 25 p. 70] “Many banks quote two rates of interest on certificates of de-
posite (CD’s). If a bank quotes 5.1% compounded daily, find the ratio of the APY
(annual percentage yield) to the quoted rate for this CD.”
Information for Students in MATH 329 2009 01 2048

µ ¶3
0.051
Solution: Since 1 + 65 = 1.05231914, the ratio of the APY to the quoted
365
5.231914
rate is = 1.0259 .
5.1
[4, Exercise 26, p. 70], [5, Exercise 27, p. 56] “A savings and loan association pays
7% effective on deposits at the end of each year. At the end of every 3 years a 2%
bonus is paid on the balance at that time. Find the effective rate of interest earned
by an investor if the money is left on deposit
1. “Two years.
2. “Three years.
3. “Four years.”
Solution: Let i denote the effective rate of interest in each case.
1. There being no bonus, i = 7%.
1
2. Here (1 + i)3 = (1.07)3 (1.02), so i = (1.07)(1.02) 3 − 1 = 7.7086300%.
1
3. This time i = (1.07)(1.02) 4 − 1 = 7.5410377%.
[4, Exercise 27 p. 71], [5, Exercise 28 p. 56] “A bank offers the following certifi-
cates of deposit:
Nominal annual interest rate
Term in years (convertible semiannually)
1 5%
2 6%
3 7%
4 8%
The bank does not permit early withdrawal. The certificates mature at the end of
the term. During the next six years the bank will continue to offer these certificates
of deposit. An investor deposits 1000 in the bank. Calculate the maximum amount
that can be withdrawn at the end of six years.”
Solution: It is instructive to begin first with an interpretation that resulted from
a misreading of the problem. In this misreading, I overlooked the fact that the
rates were convertible semi-annually. the terms are correct, but, where I used
factors 1.05, 1.06, 1.07, 1.08, I should have used (1.025)2 , (1.03)2 , (1.035)2 , and
(1.04)2 respectively; but fractional years were still not permitted. “We consider the
partitions of 6 into sums of integers: 6 = 4 + 2 = 4 + 1 + 1 = 3 + 3 = 3 + 2 + 1 =
3+1+1+1 = 2+2+2 = 2+2+1+1 = 2+1+1+1+1 = 1+1+1+
1 + 1 + 1. Because the rates increase for periods of increased length, there is no
advantage to partitioning a summand further; so we need not consider any partition
containing 1 + 1, since 2 will always be better, etc. This leaves the partitions
Information for Students in MATH 329 2009 01 2049

6 = 4 + 2 = 3 + 3 = 3 + 2 + 1 to consider. These three partitions correspond to


2
accumulations of 1000(1.08)4 (1.06)2 , 1000 ((1.07)3 ) , and 1000(1.07)3 (1.06)2 (1.05)1 ,
which are respectively equal to 1528.645395, 1500.730352, and 1445.281231, so the
best partition is into 4 + 2 years, and the best return is 1528.645395.”
Now I repeat the computations, this time correctly. I must compare 1000(1.04)8 (1.03)4 ,
2
1000 ((1.035)6 ) , and 1000(1.035)6 (1.03)4 (1.025)2 , which are respectively equal to
1540.336523, 1511.068657, 1453.579295; thus the best partition is again into 4 + 2
years, and the best return is 1540.336523.
Information for Students in MATH 329 2009 01 2050

A.11 Supplementary Notes for the Lecture of January 28th,


2009
Distribution Date: Wednesday, January 28th, 2009; corrected Thursday, January 29th,
2009
(subject to further correction or revision)

A.11.1 §2.8 MISCELLANEOUS PROBLEMS


[4, Exercise 28, p. 71] “A store is running a promotion during which customers have
two options for payment. Option One is to pay 90% of the purchase price two
months after the date of sale. Option Two is to deduct X% off the purchase price
and pay cash on the date of sale. Determine X such that a customer would be
indifferent between the two options when valuing them using an effective interest
rate of 8%.”
2
Solution: The present value of a payment of 0.9 two months hence is (0.9) (1.08)− 12 =
0.8885296, corresponding to an immediate discount of X = 11.15%.
[4, Exercise 29, p. 71], [5, Exercise 29, p. 56] “A manufacturer sells a product to
a retailer who has the option of paying 30% below the retail price immediately, or
25% below the retail price in six months. Find the annual effective rate of interest
at which the retailer would be indifferent between the two options.
1 ¡ ¢2
Solution: We solve the equation of value, 70% = 75% · (1 + i) 2 . i = 7570
−1 =
14.7959183%.
[4, Exercise 30, p. 71] “You deposit 1000 into a bank account. The bank credits
interest at a nominal annual rate of i, convertible semiannually for the first 7 years,
and a nominal annual rate of 2i, convertible quarterly for all years thereafter. The
accumulated amount in the account at the end of 5 years is X. The accumulated
amount in the account at the end of 10.5 years is 1980. Calculate X to the nearest
dollar.”
Solution: After 10.5 years the accumulation is
µ ¶2×7 µ ¶4×(10.5−7)
i 2i
1000 1 + · 1+ = 1908
2 4
µ ¶14+14
i
⇒ 1+ = 1.980
2
i 1
⇒ 1 + = 1.980 28
2
µ ¶2×5
i 10
⇒ 1000 1 + = 1000 × 1.980 28 = 1276.30 .
2
Information for Students in MATH 329 2009 01 2051

[4, Exercise 31, p. 71], [5, Exercise 31, p. 56] “Fund A accumulates at 6% effec-
tive, and Fund B accumulates at 8% effective. At the end of 20 years the total of
the two funds is 2000. At the end of 10 years the amount in Fund A is half that in
Fund B. What is the total of the two funds at the end of 5 years.?”
Solution: Denote the principals in funds A and B by A and B respectively. Then
we know that
A(1.06)20 + B(1.08)20 = 2000
1
A(1.06)10 = · B(1.08)10
2
and wish to determine A(1.06)5 + B(1.08)5 . Solving the equations yields
2000(1.06)−10
A = = 182.8195812
(1.06)10 + 2(1.08)10
2000(1.08)−10
B = 1 = 303.3009728 .
2
(1.06)10 + (1.08)10
Hence
2000(1.06)−5 2000(1.08)−5
A(1.06)5 + B(1.06)5 = + 1
(1.06)10 + 2(1.08)10 2
(1.06)10 + (1.08)10
= 182.8195812(1.06)5 + 303.3009728(1.08)5 = 690.3024748.

[4, Exercise 32, p. 71], [5, Exercise 32, p. 57] “An investor deposits 10,000 in a
bank. During the first year the bank credits an annual effective rate of interest
i. During the second year the bank credits an annual effective interest i − 0.05%.
At the end of two years the account balance is 12,093.75. What would the account
balance have been at the end of three years if the annual effective rate of interest
were i + 0.09 for each of the three years?”
Solution: The equation of value is
10000(1 + i)(1 + i − 0.05) = 12093.75 ,
which we interpret as a quadratic equation in 1 + i:
(1 + i)2 − 0.05(1 + i) − 1.209375 = 0
whose only positive solution is
p
0.05 + (0.05)2 + 4(1.209375)
1+i= = 1.125000000
2
from which we conclude that i = 12.5%, and that the account balance after 3 years
would be 10000(1.125 + 0.09)3 = 10000(1.215)3 = 17936.13.
Information for Students in MATH 329 2009 01 2052

[4, Exercise 33, p. 71], [5, Exercise 33, p. 57] “A signs a one-year note for 1000
and receives 920 from the bank. At the end of six months, A makes a payment of
288. Assuming simple discount, to what amount does this reduce the face amount
of the note?”
80
Solution: The rate of simple discount is 1000 = 8%. At the due date, 1 year from
288
now, the 288 accumulates to 1−0.04 = 300, which is the reduction in the face value
of the note, reducing the value to 700.
[5, Exercise 30, p. 56] “If an investment will be doubled in 8 years at a force of interest
δ, in how many years will an investment be tripled at a nominal rate of interest
numerically equal to δ and convertible once every three years?”
¡ ¢
Solution: The given information implies that eδ = 2, so δ = ln82 . We need
n
to determine the number of years n with the property that (1 + 3δ) 3 = 3, so
3 ln 3
n = ln(1+3δ) = 14.26421450 years.

Textbook Chapter 3. Basic Annuities

A.11.2 §3.1 INTRODUCTION


Definition A.14 1. An annuity is a series of payments, usually made at equal inter-
vals of time.
2. The total period during which payments will be made is called the term.
3. The interval between payments is called the payment period ; the name “annuity”
suggests a default period of 1 year, but, in some specific applications, the default
period may have some other length; there can, in fact, be two distinct periods
involved — one associated with the interest calculations, and one associated with
the payments under the annuity.
4. An annuity-certain consists of payments that are all certain to be made. Otherwise
an annuity is a contingent annuity. An annuity whose payments are contingent on
whether a given individual is alive is a life annuity; the study of such annuities
is concerned with “life contingencies”, and is outside of this course; following the
textbook, we will usually suppress the suffix -certain.

A.11.3 §3.2 ANNUITY-IMMEDIATE


In an annuity-immediate the payments are made at the end of each period. The present
value of an annuity-certain for n periods is denoted by an , or, if the interest rate needs
to be stated explicitly an i . The author uses a diagram to denote annuities, with 2
arrows, one sometimes labelled t1 and the other sometimes labelled t2 . The first shows
Information for Students in MATH 329 2009 01 2053

the beginning of the first period, at the end of which a payment is due under the annuity-
certain. The second arrow, labelled t2 , indicates the last payment date — more precisely,
just after the payment has been made. I shall usually not include diagrams in these notes,
as they are very time-consuming to arrange using the text-preparation software that I
am using.
The accumulated value of the payments as of time t2 is denoted by sn , or, if the
interest rate needs to be stated explicitly sn i . We can prove the following basic formulæ:

an = v + v 2 + . . . + v n−1 + v n
1 − vn
= v· when i 6= 0
1−v
1 − vn
= (43)
i
sn = 1 + (1 + i) + (1 + i)2 + . . . + (1 + i)n−1
(1 + i)n − 1
= when i 6= 0
(1 + i) − 1
(1 + i)n − 1
= (44)
i
Note that we have assumed — as is usually the case — that i 6= 0.21 We can also prove
the following identities, both algebraically and verbally:

1 = ian + v n (45)
sn = an · (1 + i)n (46)
1 1
= +i (47)
an sn

A verbal proof of the identity sn = an · (1 + i)n . The value at time n of n


payments of 1 at the ends of the years, the last payment being at time n, is sn . Instead
of shifting each of the payments of 1 immediately to time n, we could first view them
at time 0, where they are worth an . That amount at time 0 accumulates at an annual
accumulation factor of 1 + i, and is worth an · (1 + i)n at time n.

21
When i = 0 an = n = sn .
Information for Students in MATH 329 2009 01 2054

A.12 Supplementary Notes for the Lecture of January 30th,


2009
Distribution Date: Friday, January 30th, 2009
(subject to revision)

A.12.1 §3.2 ANNUITY-IMMEDIATE (conclusion)


A verbal proof of the identity 1 = ian + v n . The borrower of 1 today can
repay the debt n years from now by paying 1 at that time (worth only v n today, when
discounted back to the present) after paying i in interest at the end of each year for the
use of that 1 unit. Those interest payments constitute an n-payment annuity-immediate,
and are today worth i · an i .

1 1
A verbal proof of the identity = + i. In the previous two verbal proofs I
an sn
interpreted the quantities on one or both of the sides of the equal sign as the accumulation
of several payments. In this case I propose to interpret the identity as a decomposition
1
of an annual payment of at the end of each year for n years. n annual payments of
an
1
, one at the end of each year, can be interpreted as being the sole payments to repay
an
a loan of 1 today. Alternatively, we can think of delaying repayment, and instead paying
the lender annual interest payments of i, while the debt of 1 remains at the same level,
1
unpaid. The remainder of the annual payment, can be thought of as being held in
sn
a separate fund, also accumulating interest at the annual effective rate i, accumulating
until the end of the nth year. At that time the payments have accumulated a fund of
1
value · sn = 1, and the principal of the loan can also be repaid. We will return to this
sn
formalization later, when we introduce the concept of a sinking fund [4, §5.4].

Remember to sketch a time diagram as you read each of the following problems.
[4, Exercise 1, p. 107], [5, Exercise 1, p. 88] “A family wishes to accumulate 50,000
in a college education fund (by) the end of 20 years. If they deposit 1000 into the
fund at the end of each of the first 10 years, and 1000 + X at the end of each of the
second 10 years, find X to the nearest unit if the fund earns 7% effective.”
Solution: The equation of value at time t2 = 20 is

1000s20 + X · s10 = 50000 ,


Information for Students in MATH 329 2009 01 2055

which we may solve to yield


50000 s
X = − 1000 20 0.07
s10 0.07 s10 0.07
50000 (1.07)20 − 1
= − 1000
s10 0.07 (1.07)10 − 1
= 50000s−1
10 0.07
− 1000((1.07)10 + 1)
= 50000(0.072378) − 1000(2.96715) from the tables in [5]
= 651.75.

A more precise computation would yield 651.724; the difference is due to round-
ing errors and/or the limited precision of the tables. (The current edition of the
textbook does not provide tables.)
[4, Exercise 2, p. 107], [5, Exercise 2, p. 88] “The cash price of a new automobile
is 10,000. The purchaser is willing to finance the car at 18% convertible monthly
and to make payments of 250 at the end of each month for 4 years. Find the down
payment which will be necessary.”
Solution: Let X be the down payment. Then the equation of value at time t = 0
is X + 250a48 1.5% = 10000, from which we determine the down payment, X =
1489.361590. (Here again, the answer obtained using the tables in the 2nd edition
of the text-book is slightly incorrect, at 1489.35.)
Information for Students in MATH 329 2009 01 2056

A.13 Supplementary Notes for the Lecture of February 02nd,


2009
Distribution Date: Monday, February 02nd, 2009
(subject to revision)

A.13.1 §3.3 ANNUITY-DUE


In an annuity-due the payments are made at the beginning of each period. The present
value of an annuity-certain for n periods is denoted by än , or, if the interest rate needs
to be stated explicitly än i . Again the author uses a diagram to denote annuities, with
2 arrows, one labelled t1 and the other labelled t2 . The first shows the beginning of the
first period, just before a payment is due under the annuity-due. The second arrow,
labelled t2 , indicates the end of the last period at the beginning of which a payment was
made. The accumulated value of the payments as of time t2 is denoted by s̈n , or, if the
interest rate needs to be stated explicitly s̈n i . We can prove the following basic formulæ:

än = 1 + v + v 2 + . . . + v n−2 + v n−1


1 − vn
= when i 6= 0
1−v
1 − vn
= (48)
d
s̈n = (1 + i) + (1 + i)2 + . . . + (1 + i)n
(1 + i)n − 1
= when i 6= 0 . (49)
d
Note that we have assumed — as is usually the case — that i 6= 0; when i = 0, we can
see that än = an = 1 + 1 + . . . + 1 = n.22 . We can also prove the following identities,
both algebraically and verbally:

än = (1 + i)an (50)


s̈n = (1 + i)sn (51)
än = 1 + an−1 (52)
s̈n = −1 + sn+1 (53)
s̈n = (1 + i)n än (54)

(Discount, Due) vs. (Interest, Immediate). There are analogies between annuities-
immediate and annuities-due that can be explained by the two points of view of interest
22
Since this is the limit of an i as i → 0, we have proved the right continuity of an i and än i at i = 0.
Information for Students in MATH 329 2009 01 2057

and discount. So we have the following analogous formulæ:

1 − (1 + i)−n 1 − (1 − d)n
an = än =
i d
n
(1 + i) − 1 (1 − d)−n − 1
sn = s̈n =
i d

Note that, in spite of this analogy, the notation is “prejudiced” to expression in terms
of interest rather than discount. Thus, for example, in the symbols än k , än k k always
represents the interest rate — never the rate of discount.
[4, Exercise 7, p. 107], [5, Exercise 11, p. 89] “Find ä8 if the effective rate of dis-
count is 10%.”
1 − (1 − d)n
Solution: Using the discount version of the formula, i.e., än = , we
d
have
1 − (0.9)8
ä8 = = 5.695327900.
0.1
[4, Exercise 8, p. 108], [5, Exercise 9, p. 89] “Find the present value of payments
of 200 every six months starting immediately and continuing through four years
from the present, and 100 every six months thereafter through ten years from the
present, if i(2) = 0.06.”
Solution: The reader is likely to make assumptions in a casual reading that were
not intended by the author. Reading carefully, one might see that he intends that,
though the payments start immediately, they continue to the end of 4 years from
now — that is, that there be 9 payments of 200, not 8. Similarly, it is the payments
that end after 10 years, not the years for which they are prepaid — so there are
payments over 21 half-years in all. With this interpretation, at the effective rate of
1 (2)
2
i = 3% per half-year,

Present value = 100ä21 3% + 100ä9 3%


¡ ¢
= 100(1.03) 100a21 3% + 100a9 3%
= 2389.716705

Note that my solution here shows my prejudice (or laziness) in defaulting to the
interest version of the formulæ; but as we are given the rate of interest, rather than
the rate of discount, this turns out to be the easiest approach.
Information for Students in MATH 329 2009 01 2058

A.14 Supplementary Notes for the Lecture of February 04th,


2009
Distribution Date: Wednesday, February 04th, 2009
(subject to revision)

A.14.1 §3.4 ANNUITY VALUES ON ANY DATE


Three cases are considered: the date is always an integral number of periods from each
payment date.
1. present values more than one period before the first payment date
2. accumulated values more than one period after the last payment date;
3. current values at a date strictly between the first and last payment dates.

Present values more than one period before the first payment date An annuity
is said to be deferred by m time units if the first payment is m time units later than
the type of annuity in question would normally be paid, and the subsequent payments
occur at the expected intervals. The value of an annuity that is deferred through k
periods is denoted by the earlier symbol, prefixed by k |. Thus k |an is the present value
of an annuity-immediate whose first payment has been deferred through k time periods;
analogous symbols can be used for än . It can be seen that

k |an = v k an
= an+k − ak , (55)
k |än = v k än
= än+k − äk . (56)

Accumulated values more than 1 period after the last payment date. Here
we can express the current values by multiplying by the appropriate power of 1 + i; or,
alternatively, by taking the difference of two annuity values.

Summary. You are urged to follow the textbook’s suggestion, “The reader should not
try to work problems by memorizing formulas...” Remember the reasoning that was used
to derive the formulæ, and apply that reasoning “from first principles” in each case.

Some exercises from the textbook. The textbook asks you to prove several for-
mulæ. There are two levels at which such exercises should be approached:
• an algebraic proof
Information for Students in MATH 329 2009 01 2059

• a verbal justification
Usually an algebraic proof should not be difficult; I will not normally include proofs in
these notes, but you can see me if you have difficulty working through a proof. The
issue is not to find an elegant proof — just to show that the two sides of the equation
are equal. As for a verbal proof, that will be much harder, and I will spend increasing
amounts of time at the lectures discussing problems of this type.
[4, Exercise 12, p. 108], [5, Exercise 17, p. 89] “Payments of 100 per quarter are
made from June 7, year Z through December 7, year Z + 11, inclusive. If the
nominal rate of interest, convertible quarterly, is 6%:
1. find the present value on September 7, year Z − 1;
2. find the current value on March 7, year Z + 8;
3. find the accumulated value on June 7, year Z + 12”.
Solution:
1. As of September 7, year Z − 1, no payments have yet been made. The present
value is23
(1.015)−2 100a44+3 1.5% = 3256.879998 .
2. As of March 7, year Z + 8, the value as of March 7, year Z will accumulate by
a factor (1.015)(8×4) , for an accumulated value of 5403.152103.
3. As of June 7, year Z + 12, the originally computed value will accumulate by a
factor (1.015)(13×4)−1 , for an accumulated value of 6959.369761.
[4, Exercise 13, p. 108], [5, Exercise 22, p. 90] “Simplify a15 (1 + v 15 + v 30 ) to one
symbol.”
Solution: Here is an algebraic solution:
¡ ¢ 1 − v 15 ¡ ¢
a15 1 + v 15 + v 30 = · 1 + v 15 + v 30
i
1 − v 45
= = a45
i
A verbal proof could be based on the equation
¡ ¢
a15 1 + v 15 + v 30 = a15 + v 15 · a15 + v 30 · a15
¯ ¯
= a15 + 15 ¯a15 + 30 ¯a15
23
Why is the exponent −2 and not −3 even though the the evaluation is being made 9 months before
the first payment? Because I am viewing the payments as an annuity-immediate: the clock starts
ticking one period before the first payment. And why have 3 periods been added, even though June
and December are only 2 quarter-years apart? Again because the first payment is associated with the
3-month period ending with the payment, so the actual length of the annuity period is 3 months longer.
Information for Students in MATH 329 2009 01 2060

In the last member the first summand is the present value of an annuity of 15
annual payments of 1, the first one year from now and the last 15 years from now;
the second summand is the present value of a sequence of 15 payments which begin
one year after the last represented by the first summand and the last being paid
30 years from now; and the last summand is the present value of a final subseries
of 15 payments of 1, the first to be paid one year after the last of the 30 payments
mentioned above, and the last to be paid 45 years from now. We know that the
present value of such a series of 45 payments of 1 is a45 .
[4, Exercise 14, p. 108], [5, Exercise 21, p. 90] “It is known that
a7 a + sx
= 3 . (57)
a11 ay + sz

Find x, y, and z.”


Solution: Intuitively we usually expect that the determination of 3 variables requires
3 constraints. While this is not always the case, the fact that only one equation
has been presented here should ring an alarm bell. The variable i has not been
mentioned, and we might also wish to know whether the solution we are asked to
find should be dependent on i.24 It appears from the textbook that the values given
above are the only solution the textbook was seeking — but there exist others!.
At time t = 0 the present values of annuities-due of 1 per year for respectively 7
a
and 11 years have value in the ratio 7 . Both of these values should increase by
a11
a factor of 1 + i per year under compound interest. Hence, 4 years later, the ratio
a + s4
will not have changed. But, at that time, it can be interpreted as 3 . Thus
a7 + s4
one solution to the problem is

(x, y, z) = (4, 7, 4), (58)

and this solution is valid for all i. I would not expect students to be able to generate
the rest of this solution!
Having found one solution, which we can see from [4, p. 605] to be the solution
the author is seeking, we might be expected to stop. But, to a mathematician, an
instruction like “Find x, y, and z” means, implicitly,
“Find all possible sets of values for x, y, and z.”
24
This can explain the apparent paucity of equations: to assume that the solution holds for all i is
equivalent to assuming an equation for every value of i — infinitely many equations, for the 3 unknowns
we are trying to determine. In such a situation we should not be surprised if there is no solution at all.
Information for Students in MATH 329 2009 01 2061

So let’s investigate whether we have all solutions. This raises new issues. The
solution we found above in (58) is valid for all i. I have two subquestions:
• Could there be other solutions for all i?
• Could there be solutions that hold for specific values of i, but not for all i?
Could there be other solutions for all i? When i = 0, equation
a7 a + sx
= 3 (59)
a11 ay + sz
becomes
7 3+x
= ,
11 y+z
implying that
11x − 7y − 7z = −33 . (60)
As i → ∞, v → 0, and the left side of equation (59) approaches
1 − (1 + i)−7
lim = 1;
i→∞ 1 − (1 + i)−11

the limit of the right side will be 1 only if

x = z. (61)

Then (60) becomes


4x − 7y = −33 . (62)
Our earlier solution (58) satisfies this last “diophantine” equation; some other so-
lutions are
(x, y, z) = (11, 11, 11), (18, 15, 18), (−3, 3, −3) .
If we define
y−3 x+3
w= = ,
4 7
the conditions we have determined on x, y, z have the general solution

(x, y, z) = (7w − 3, 4w + 3, 7w − 3)

and the case (4, 7, 4) corresponds to w = 1. What about the other solutions? All
we have shown is that, if there are any solutions valid for all i, then they have to be
of the preceding form. But, when we substitute these general values into equation
(57), we may reduce the equation to the following condition on w:
1 − v7 1 − v 7w
= .
1 − v 11 1 − v 11w
Information for Students in MATH 329 2009 01 2062

All values of w other than w = 1 produce a polynomial equation which constrains


v, so the equality will not hold for all interest rates. Thus solution (58) is the only
solution valid for all i.
You are not expected to be able to reproduce this detailed argument.
[4, Exercise 15, p. 108], [5, Exercise 19, p. 89] “Annuities X and Y provide the
following payments:
End of Year Annuity X Annuity Y
1–10 1 K
11–20 2 0
21–30 1 K
“Annuities X and Y have equal present values at an annual effective interest rate
i such that v 10 = 12 . Determine K.”
Solution: ¡As of today,¢ the present values
¡ of annuities
¢ X and Y are respectively
1 · a30 + 1 a20 − a10 and K · a30 − K a20 − a10 . Setting these amounts equal and
solving, we obtain
a30 + a20 − a10
K =
a30 − a20 + a10
1 − v 30 − v 20 + v 10
=
1 − v 30 + v 20 − v 10
1 − 18 − 14 + 12 9
= 1 1 1 = = 1.8.
1− 8 + 4 − 2 5
¯ ¯
[4, Exercise 16, p. 108] “You are given that 5 ¯a10 = 3 ·10 ¯a5 . Find (1 + i)5 .”
Solution:
¯ ¯ 1 − v 10 1 − v5
5
¯a = 3 ·10 ¯a5 ⇒ v 5 · = 3v 10 ·
10
i i
⇒ 2v 15 − 3v 10 + v 5 = 0
¡ ¢¡ ¢
⇒ v 5 2v 5 − 1 v 5 − 1 = 0 ,

provided i 6= 0. Aside from the trivial solution i = −1, there are two non-trivial
solutions to this equation: v 5 = 1, 21 , equivalent to (1 + i)5 = 1 (when i = 0) and
(1 + i)5 = 2.
The solution v 5 = 1 is not valid, since we assumed earlier that i 6= 0. The issue here
is with the formula used for an , which is the sum of a geometric progression on the
assumption that the common ratio is non-zero. When i = 0, the correct formula
Information for Students in MATH 329 2009 01 2063

would be an0% = n. In that case the derivation would be


¯ ¯
¯ ¯ 5
5 a100% = 3 ·10 a5 0% ⇒ v · 10 = 3v
10
·5
5
¡ 5
¢
⇒ v 3v − 2 = 0 ,

which would imply that v 5 = 0 or v 5 = 32 , neither of which is consistent with


i = 0 ⇔ v = 1. Thus there is one non-trivial solution, (1 + i)5 = 2. The solution
v = 0, i.e., i = −1, would correspond to a system where money invested for 1 year
is lost, and so the original equation would correspond to 0 = 0.
[4, Exercise 17, p. 108], [5, Exercise 23, p. 90] “Find the present value to the near-
est dollar on January 1 of an annuity which pays 2000 every six months for five
years. The first payment is due on the next April 1, and the rate of interest is 9%
convertible semiannually.”
Solution: On April 1st the annuity is worth 2000 · ä10 4.5% . Discounting back to
January 1st, i.e. through half of a half-year period, we obtain a value of
1 1
(1.045)− 2 · 2000ä10 4.5% = (1.045) 2 · 2000a10 4.5%
1 1 − (1.045)−10
= (1.045) 2 · 2000 · = 16177.59053
0.045
or 16178 to the nearest dollar.
Information for Students in MATH 329 2009 01 2064

A.15 Supplementary Notes for the Lecture of February 06th,


2009
Distribution Date: Friday, February 06th, 2009
(subject to revision)

A.15.1 §3.5 PERPETUITIES


The symbol a∞ is used for the present value of a perpetual annuity — an infinite sequence
of payments, the first one period from now; where the interest rate is not clear from the
context, we may write a∞ i . When the interest rate i is 0 the present value would be
infinite. Otherwise we sum an infinite geometric series, obtaining
v v 1
v + v2 + v3 + . . . = = = .
1−v iv i
It can be seen that this is the limit of the usual formula for an as n → ∞. An infinite
annuity of this type is called a perpetuity-immediate, or usually just a perpetuity.
Analogously we may define a perpetuity-due to be an infinite sequence of equal pay-
ments, where the first is made immediately. We use the symbol ä∞ , and can prove that
1
its present value is . While perpetuities do exist in the real world, for example in the
d
bond market, they are also useful in providing verbal explanations of identities. For
n
example, we can explain the formula an = 1−v i
by expressing the quotient on the right
1 n1
as a difference, i − v i and interpreting the n-payment annuity as the difference of 2
perpetuities, one beginning a year from now, and the other n + 1 years from now; i.e.,

an = a∞ − (n |a∞ )

[4, Exercise 18, p. 109], [5, Exercise 25, p. 90] “Deposits of 1000 are placed into
a fund at the beginning of each year for the next 20 years. After 30 years, annual
payments commence, and continue forever, with the first payment at the end of the
30th year. Find an expression for the amount of each payment.”
Solution: The deposits accumulate to a fund worth 1000s̈20 a year after the last
payment, and 1000(1 + i)10 s̈20 10 years after the last payment. A perpetuity-due
bought with this amount will have annual payments of

10 (1 + i)21 − (1 + i)
10
1000s̈20 (1 + i) d = 1000(1 + i) · · iv
¡ i ¢
= 1000 (1 + i)30 − (1 + i)10 .

Alternatively, we could set up an equation of value at any other time, for example
at time t = 0. The value of the deposits will be 1000ä20 . Suppose that the level
Information for Students in MATH 329 2009 01 2065

amount of the withdrawals is X. These withdrawals may be interpreted as either a


perpetuity-due for which the clock starts at the time of the first payment, 30 years
from now; or as a perpetuity-immediate for which the clock starts 29 years from
now in order that the first payment occur at the end of that year. This leads to
equations, either
1000ä20 = Xv 30 ä∞
or
1000ä20 = Xv 29 a∞
which may be solved for the same value of X as determined earlier.
[4, Exercise 19, p. 109] “A deferred perpetuity-due begins payments at time n with
annual payments of 1000 per year. If the present value of this perpetuity is equal
to 6561, and the effective rate of interest is i = 19 , find n.”
Solution:
¯ vn
¯
1000 ·n ¯ä∞ 1 = 6561 ⇔ = 6.561
9 d
⇔ v n−1 = 6.561i = 0.729
µ ¶n−1 µ ¶3
10 10
⇔ =
9 9
10
⇔ n − 1 = 3 taking logarithms to base
9
⇔ n = 4.

[4, Exercise 20, p. 109] “A woman has an inheritance in a trust fund for family mem-
bers left by her recently deceased father that will pay 50,000 at the end of each
year indefinitely into the future. She has just turned 60, and does not think that
this perpetuity-immediate meets her retirement needs. She wishes to exchange the
value of her inheritance in the trust fund for one which will pay her a 5-year deferred
annuity-immediate, providing her a retirement annuity with annual payments at the
end of each year for 20 year following the 5-year deferral period. She would have
no remaining interest in the trust fund after 20 payments are made. If the trustee
agrees to her proposal, how much annual retirement income would she receive? The
trust fund is earning an annual effective rate of interest equal to 5%. Answer to the
nearest dollar.”
Let X be the annual retirement income the woman would receive annually for 20
years under the new, deferred annuity-immediate. The interest rate for the new
annuity is not stated, so I am assuming it will be the same as the rate for the trust
fund.
¯
50000 · a∞5% = X ·5 ¯a205%
Information for Students in MATH 329 2009 01 2066

1 1 − (1.05)−20
⇔ X = 50000 · (1.05)−5 ·
0.05 0.05
25
50000(1.05)
⇔ X= = 102, 412.1345 ,
1.0520 − 1
so the annual payment under the new scheme will be 102,412.13.
[4, Exercise 21, p. 109], [5, Exercise 26, p. 90] “A benefactor leaves an inheritance
to 4 charities, A, B, C, and D. The total inheritance is a series of level payments
at the end of each year forever. During the first n years, A, B, and C share each
payment equally. All payments after n years revert to D. If the present values of
the shares of A, B, C, and D are all equal, find (1 + i)n .”
Solution: Imposing the condition that the sum of the first n payments is equal to 3
times the present value of payments ##n + 1, n + 2, . . ., we obtain an = 3v n a∞ ⇒
v n = 41 ⇒ (1 + i)n = 4.
[4, Exercise 22, p. 109], [5, Exercise 27, p. 90] “A level perpetuity-immediate is
to be shared by A, B, C, and D. A receives the first n payments, B the second
n payments, C the third n payments, and D the payments thereafter. It is known
that the ratio of the present value of C’s share to A’s share is 0.49. Find the ratio
of the present value of B’s share to D’s share.”
Solution: The present values of the shares of A, B, C, D are, respectively, an , v n an ,
v 2n an , and v 3n a∞ . The fact that C’s share, divided by A’s share is to equal 0.49
implies that v n = 0.7. The ratio of B’s share to D’s is then seen to be
v n an 0.7 × 0.3
3n
=
v a∞ (0.7)3
30
= .
49
Information for Students in MATH 329 2009 01 2067

A.16 Supplementary Notes for the Lecture of February 09th,


2009
Distribution Date: Monday, February 09th, 2009
(subject to revision)

A.16.1 §3.6 UNKNOWN TIME


In this section the textbook considers situations where n, the number of annuity pay-
ments, is not known.

Fractions of a payment period It is possible to give a reasonable interpretation to


a non-integer number of level annuity payments. The textbook observes that, if we take
k to lie in the interval 0 ≤ k ≤ 1, then, using the (43) on page 2053 of these notes, we
have
1 − v n+k 1 − vk (1 + i)k − 1
an+k = = an + v n · = an + v n+k · , (63)
i i i
which can be interpreted as either an n − period level annuity of 1 with an additional
k (1+i)k −1
payment of 1−v i
at time n; or with a final payment of i
at time n + k. In the first
case the additional payment can be shown to equal, for small i, approximately
µ ¶
1+k
k 1− i + ... ;
2
in the second case the final payment is approximately
µ ¶
1−k
k 1− i + ... ;
2
thus the payment is approximately consistent with a payment of 1 per period. But,
because of the approximations involved, and the possibility that a court might order a
specific interpretation — such problems are usually resolved in advance by having a final
payment in an otherwise regular sequence of payments cover the amount necessary to
make up the difference. Where the final payment needed to meet a goal is larger than
a regular payment, it is called a balloon payment; where it is smaller, it is called a drop
payment. Solving problems in this section typically involves two phases:
• Determination of the number of regular payments.
• Determination of the value of the final payment, subject to given constraints (e.g.
that it be a balloon or drop payment).
We can proceed by first solving an inequality. Read [4, Example 3.7, pp. 92-93], where
the author considers a problem where a planned final payment turns out to be negative.
Information for Students in MATH 329 2009 01 2068

[4, Exercise 23, p. 110], [5, Exercise 29, p. 90] “Compute a5.25 if i = 5% using the
following definitions:
1. Formula [5, (3.22)].
2. A payment of 0.25 at time 5.25.
3. A payment of 0.25 at time 6.
Solution:
1
1. a5 + v 5.25 (1.05)
4
0.05
= 4.519457520.
5.25
2. a5 + v 0.25 = 4.522983461.
3. a5 + v 6 0.25 = 4.516030529.
[4, Exercise 24, p. 110], [5, Exercise 32, p. 91] “A loan of 1000 is to be repaid by
annual payments of 100 to commence at the end of the 5th year, and to continue
thereafter for as long as necessary. Find the time and amount of the final payment
if the final payment is to be larger than the regular payments. Assume i = 4.5%.”
Solution: Let the time of the last — balloon — payment be n, and let the amount
of the last payment be X. Then n is the largest integer solution to the inequality

1 − (1.045)−(n−4)
1000 ≥ 100(1.045)−4 an−4 = 100(1.045)−4 ·
0.045
4
1000 × 0.045 × (1.045)
⇔ (1.045)−(n−4) ≥ 1 −
¡ 100 ¢
⇔ −(n − 4) ln 1.045 ≥ ln 1 − 10 × 0.045 × (1.045)4
ln (1 − 0.45(1.045)4 )
⇔ −(n − 4) ≥
ln 1.045
ln (1 − 0.45(1.045)4 )
⇔ n≤4− = 21.47594530.
ln 1.045
Thus we conclude that the balloon payment is made at time t = 21. The equation
of value at time t = 21 is

1000(1.045)21 = 100s17 + (X − 100)

implying that
100 ¡ ¢
X = 100 + 1000(1.045)21 − (1.045)17 − 1 = 146.070467 .
0.045

[5, Exercise 33, p. 91] “A fund of 2000 is to be accumulated by n annual payments


of 50, followed by n annual payments of 100, plus a smaller final payment made 1
Information for Students in MATH 329 2009 01 2069

year after the last regular payment. If the effective rate of interest is 4.5%, find n
and the amount of the final irregular payment.”
Solution: We shall interpret the payments to be made under two annuities-due: the
first, for 2n years, consists of an annual deposit of 50 in advance; the second, for
n years, deferred n years after the first, also consists of an annual deposit of 50 in
advance. It is at the end of year 2n that the final, drop payment is to be made,
and it is to be under 100. (Note that this is the type of problem where the drop
payment could turn out to be negative. We seek the smallest n for which
50s̈2n + 50s̈n > 2000 − 100
(1.045)2n + (1.045)n − 2
⇔ 50(1.045) · > 1900
0.045
1900 0.045
⇔ (1.045)2n + (1.045)n − 2 > ·
50 1.045
µ ¶2
1 1900 0.045
⇔ (1.045)n + > · + 2.25 = 3.886363636
2 50 1.045
Since the exponential is positive, the preceding inequality is equivalent to (1.045)n >
1.471386222, and, in turn, to
ln 1.471386222
n> = 8.774018446 .
ln 1.045
Thus the drop payment will be when t = 2 × 9, i.e., 18 years after the first pay-
ment under the annuity with payments of 50. Just before the drop payment the
accumulated value of all previous payments is
¡ ¢ (1.045)18 + (1.045)9 − 2
50 s̈9 + s̈18 = 50(1.045) · = 1967.588591
0.045
so the drop payment at time t = 18 is 2000 − 1967.588591 = 32.411409.
Note that there is an error in the answers in the textbook: while n = 9 is correct,
the payment of 32.41 is not “at time n = 9”.
(If tables like those in the textbook were available, one could determine the value of
n by inspecting the value of s2n + sn . We observe from the 4.5% tables the following
values:
n s2n + sn
8 32.0993
9 37.6572
10 43.6596
We seek the smallest n such that
50s̈2n + 50s̈n > 2000 − 100
Information for Students in MATH 329 2009 01 2070

i.e., such that


s̈2n + s̈n > 38 ,
equivalently,
38
s2n + sn > = 36.37 ,
1.045
and so can conclude that n = 9.)
At the beginning of the next lecture I will consider a variation of this last problem, where
it is insisted that the last payment can’t be more than 25.
Information for Students in MATH 329 2009 01 2071

A.17 Supplementary Notes for the Lecture of February 11th,


2009
Distribution Date: Wednesday, February 11th, 2009
(subject to revision)

A.17.1 §3.6 UNKNOWN TIME (conclusion)


At the end of the last lecture I was discussing [5, Exercise 33, p. 91]; a solution is
included in the notes for that lecture. Suppose now that we change the problem slightly,
as follows:

Example A.10 (cf. [5, Exercise 33, p. 91]) A fund of 2000 is to be accumulated by
n annual payments of 50, followed by n annual payments of 100, plus a smaller final
payment of not more than 25 made 1 year after the last regular payment. If the effective
rate of interest is 4.5%, find n and the amount of the final irregular payment.”
Solution: I will adapt my solution for the original version. Again I interpret the payments
to be made under two annuities-due: the first, for 2n years, consists of an annual deposit
of 50 in advance; the second, for n years, deferred n years, also consists of an annual
deposit of 50 in advance. It is at the end of year 2n that the final, drop payment is to
be made, and it is to be under 25. We seek the smallest integer n for which

50s̈2n + 50s̈n > 2000 − 25


(1.045)2n + (1.045)n − 2
⇔ 50(1.045) · > 1975
0.045
1975 0.045
⇔ (1.045)2n + (1.045)n − 2 > ·
50 1.045
µ ¶2
n 1 1975 0.045
⇔ (1.045) + > · + 2.25 = 3.950956938.
2 50 1.045

Since the exponential is positive, the preceding inequality is equivalent to (1.045)n >
1.487701421, and, in turn, to
ln 1.487701421
n> = 9.024542616 .
ln 1.045
Thus the drop payment will be when t = 2 × 10, i.e., 20 years after the first payment
under the annuity with payments of 50. Just before the drop payment the accumulated
value of all previous payments is
¡ ¢ (1.045)20 + (1.045)10 − 2
50 s̈10 + s̈20 = 50(1.045) · = 2281.215780 ,
0.045
Information for Students in MATH 329 2009 01 2072

so the drop payment at time t = 20 is 2000 − 2281.215780 = −281.215780. Thus, by


insisting that the last payment be very small, the payment schedule was extended so
that the borrower actually gets money back after the n + n payments as prescribed.

A.17.2 §3.7 UNKNOWN RATE OF INTEREST


I do not expect you to be able to use the iteration algorithms built into financial calcu-
lators.

Approximation formulæ for i, given sn i or an i In what follows, assume that


n > 0 is given, as well as either an i or sn i . It is trivial to test whether i = 0, since then
an 0 = n = sn 0 . Now assume i > 0, and that i is “small”.

Approximation formula for i, given an i


1 1 1 1
− = −n

an i n 1 − (1 + i) n
i 1
= ³ ´−
1 − 1 − ni + n(n+1) i2 + . . . n
2

(binomial expansion)
µ ¶
1 1
= −1
n 1 − n+1 2
i + ...
µ ¶
1 n+1
≈ 1+ i−1
n 2
(geometric series)
n+1
= i
2n
Solving this approximate equation for i we obtain

2 (n − an i )
i= . (64)
an i · (n + 1)

Approximation formula for i, given sn i


1 1 1 1
− = −
n sn i n (1 + i)n − 1
1 i
= −³ ´
n n(n−1) 2
1 + ni + 2 i + . . . − 1
Information for Students in MATH 329 2009 01 2073

(binomial expansion)
µ ¶
1 1
= 1−
n 1 + n−1
2
i + ...
µ ¶
1 n−1
≈ 1−1+ i
n 2
(geometric series)
n−1
= i
2n
2 (sn i − n)
Solving this approximate equation for i we obtain i ≈ .
sn i · (n − 1)

Approximating compound interest for fractions of a measurement period In


the previous edition of the textbook [5, §2.2] the author observed that a method that
is often used when a fraction of a measurement period is involved is to approximate
compound interest by simple interest. This amounts to using series (35) above, but
stopping after the 1st degree term:

(1 + i)k ≈ 1 + ki .

It can also be interpreted as linear interpolation between values of (1 + i)x for integer
x. Suppose that we are interested in the accumulation factor for compound interest
between t = n and t = n + k, where 0 ≤ k ≤ 1. If we think of a line joining the points
(n, (1 + i)n ) and (n + 1, (1 + i)n+1 ), and take as a value approximating (1 + i)n+k , the
ordinate of the point where this line meets the line x = n + k, we have similar triangles,
leading to the equation
k approximation − (1 + i)n
=
1 (1 + i)n+1 − (1 + i)n
from which we determine that the approximation is

k(1 + i)n+1 + (1 − k)(1 + i)n = (1 + i)n (k(1 + i) + (1 − k)) = (1 + i)n (1 + ki).

The textbook remarks that linear approximation between successive integer values of v x
is equivalent to simple discount. We can obtain a better approximation by truncating
the MacLaurin expansion after a higher term than the first degree term. In the following
example the accumulation factor is approximated by a quadratic — not a linear —
function.

Unknown rate of interest Where it is the interest rate that is not known, there can
be several different approaches:
Information for Students in MATH 329 2009 01 2074

• Algebraic methods:
– Where possible, one tries to find a solution using algebraic methods. Since
the various formulæ we work with are usually polynomial in i or ratios of
polynomials in i, it may be possible to find the particular solution(s) we seek
by algebraic means.
– Algebraic means may still be available where solving for i fails, if we can find
another convenient intermediary variable.
• It may be possible to solve by interpolation on tables, provided the functions we
are interested in are tabulated.
• The favoured method is by successive approximation, to obtain a solution to any
desired accuracy.
[4, Exercise 29, p. 110], [5, Exercise 38, p. 92] “If a2 = 1.75, find an exact expres-
sion for i.”
Solution:

(1 + i)2 − 1
a2 = 1.75 ⇔ = 1.75(1 + i)2
i
⇔ 1.75i2 + 2.5i − 0.25 = 0 (65)
⇔ 7i2 + 10i − 1 = 0 √ √

−10 + 100 + 28 −5 − 4 2 −5 + 4 2
⇔ i= = or . (66)
14 7 7
Since i > 0, the second solution is inadmissible, and i = 0.09383632129.
[4, Exercise 30, p. 111], [5, Exercise 42, p. 92] “A beneficiary receives a 10,000 life
insurance benefit. If the beneficiary uses the proceeds to buy a 10-year annuity-
immediate, the annual payout will be 1,538. If a 20-year annuity-immediate is
purchased, the annual payout will be 1,072. Both calculations are based on an
annual effective interest rate of i. Find i.”
Solution: The equations of value at time 0 are

10000 = 1538 · a10 = 1072 · a20

The last equation alone implies that 1 + v 10 = 1.434701492, which implies that
i = 8.6878222%. I don’t see why the amount of the insurance benefit was given
(although it is consistent with the other information.)
[5, Exercise 38, p. 92] “If a2 = 1.75, find an exact expression for i.”
Information for Students in MATH 329 2009 01 2075

Solution:

(1 + i)2 − 1
a2 = 1.75 ⇔ = 1.75(1 + i)2
i
⇔ 1.75i2 + 2.5i − 0.25 = 0 (67)
⇔ 7i2 + 10i − 1 = 0 √ √

−10 + 100 + 28 −5 − 4 2 −5 + 4 2
⇔ i= = or . (68)
14 7 7
Since i > 0, the second solution is inadmissible, and i = 0.09383632129.
Information for Students in MATH 329 2009 01 2076

A.18 Supplementary Notes for the Lecture of February 13th,


2009
Distribution Date: Friday, February 13th, 2009
(subject to revision)

A.18.1 §3.7 UNKNOWN RATE OF INTEREST (conclusion)


[4, Exercise 31, p. 111] “The present values of the following three annuities are equal:
(i) “perpetuity-immediate paying 1 each year, calculated at an annual effective
interest rate of 7.25%;
(ii) “50-year annuity-immediate paying 1 each year, calculated at an annual effec-
tive interest rate of j%;
(iii) “n-year annuity-immediate paying 1 each year, calculated at an annual effective
interest rate of j − 1%.
“Calculate n.”
Solution: We are given that

a∞ 7.25% = a50 j% = an (j−1)% ,

which is equivalent to

1 1 − (1 + j)−50 1 − (0.99 + j)−n


13.79310345 = = = .
0.0725 j j − 0.01

Equality of (i) and (ii) yields, using the approximation formula in [4, Appendix 3,
p. 106] (equation (64) on page 2072 of these notes),

2(50 − 13.7931625)
j≈ = .1029410086 .
13.79310345(51)

For these large values of n, i, the approximation is not very good, since

1 − (1.102941)−50
a50 0.102941 = = 9.6419 ,
0.102941
which is quite far from the known value of 13.7931625. However, since the per-
petuity is calculated at a rate of 7.25%, and is being truncated, the interest rate
for a50 should be less than 7.25%. By trial and error and then by an iterative
procedure such as bisection or something more sophisticated, we can show that
j = 0.07004382216, or approximately j = 7.00%.
Information for Students in MATH 329 2009 01 2077

Now we need to solve an equation

an (7.00−1)% = 13.79310345 ⇔ 1.06−n = 1 − (0.06)(13.79310345) = 0.1724137930


⇔ n = 30.168

so n is approximately 30.2. (There remains, of course, the issue of what is meant


by an when n is not an integer.)

A.18.2 §3.8 VARYING INTEREST


I will omit this section, except for stating the following definition.
Suppose that the interest rate ik applies only to the kth period of time — from
time k − 1 to time k. The textbook considers two different ways of interpreting the
applicability of these different rates. Of course there is a need for clarity when more
than one interest rate is stated, as the reader must be able to infer precisely which of
these methods is applicable, if not an entirely different method.

Definition A.15 1. In the Portfolio Rate Method rate ik applies to all interest trans-
actions that cross this time interval, even if the actual payment was made later than
the kth period. Thus, for example,
1 1 1 1 1 1
an = + · + ... + · · ... · .
1 + i1 1 + i1 1 + i2 1 + i1 1 + i2 1 + in
and

sn = (1 + in ) + (1 + in )(1 + in−1 ) + . . . + (1 + in )(1 + in−1 ) · (1 + i1 ) .

2. In the Yield Curve Method rate ik applies only to the payments made during the
kth period, and follow those payments back to the present. Thus, for example,
µ ¶2 µ ¶n
1 1 1
an = + + ... + .
1 + i1 1 + i1 1 + in

and
sn = (1 + in ) + (1 + in−1 )2 + . . . + (1 + i1 )n .

A.18.3 §3.9 ANNUITIES NOT INVOLVING COMPOUND INTEREST


I wish to consider only one problem from this section:
Information for Students in MATH 329 2009 01 2078

1
[4, Exercise 38, p. 112] “Given that δt = 20−t (t ≥ 0), find s10 .”
Solution: Note that the theory of [4, §4.5] does not apply here, as it is for continuous
annuities with a constant rate. In this case the payments are discrete — at 1-year
intervals, but the interest rate is changing continuously. The payment at time n is
worth, at time 10,
R
10
δt dt 10 20 − n
en = e−[ln(20−t)]n = .
10
Hence
10
1 X 19 · 20 9 · 10
s10 = (20 − n) = − = 14.5 .
10 n=1 20 20

A.18.4 §3.10 MISCELLANEOUS PROBLEMS


[4, Exercise 40, p. 112] “At an annual effective interest rate of i, both of the following
annuities have a present value of X:
(i) a 20-yeear annuity-immediate with annual payments of 55;
(ii) a 30-year annuity-immediate with annual payments that pay 30 per year for
the first 10 years, 60 per year for the second 10 years, and 90 per year for the
final 10 years.
Calculate X.”
Solution: There are two unknowns here: i and X. The hypotheses yield two equa-
tions for these unknowns:

X = 55a20 i
X = 90a30 i − 30a20 i − 30a10 i

Could i = 0? If that were the case, the annuities could be evaluated, and the two
equations would reduce to

X = 55(20) = 1100
X = 90(30) − 30(20) − 30(10) = 1800 .

Since the equations are contradictory, we conclude that the hypothesis that i = 0
is invalid.
Now, using the property that v n = 1 + ian , we find from the second equation that

iX = 90(1 − v 30 ) − 30(1 − v 20 ) − 30(1 − v 10 )


= 30(1 − v 10 )(3v 20 + 2v 10 + 1)
Information for Students in MATH 329 2009 01 2079

so the second of our original equations may be rewritten in the form


¡ ¢
X = 30a10 · 3v 20 + 2v 10 + 1 .
The first original equation may be similarly rewritten in the form
¡ ¢
X = 55a10 · 1 + v 10 .
Dividing one equation by the other yields
¡ ¢ ¡ ¢
55 v 10 + 1 = 30 3v 20 + 2v 10 + 1 ,
which is equivalent to the factorized quadratic equation
¡ 10 ¢¡ ¢
2v − 1 9v 10 + 5 = 0 .
1

Of the two roots, the negative is extraneous, so v 10 = 2
and i = 10
2. Hence
1− 1
a20 = √ 4
10
2−1
= 10.44954456, so X = 55a20 = 574.7249508.
[4, Exercise 41, p. 112] “To accumulate 8000 at the end of 3n years, deposits of 98
are made at the end of the first n years, and 196 at the end of the next 2n years.
The annual effective rate of interest is i. You are given that (1 + i)n = 2. Determine
i.”
Solution: An equation of value, at time 3n is
¡ ¢
8000 = 98 s3n + s2n
98 ¡¡ ¢ ¡ ¢¢
⇒ 8000 = (1 + i)3n − 1 + (1 + i)2n − 1
i
98
⇒ 8000 = ((8 − 1) + (4 − 1) = 980 ,
i
which implies that i = 12.5%.
[4, Exercise 42, p. 112], [5, Exercise 51, p. 93] “A loan of 10,000 is to be repaid,
with annual payments at the end of each year for the next 20 years. For the first
5 years the payments are k per year; the second 5 years, 2k per year; the third 5
years 3k per year; and the fourth 5 years, 4k per year. Find an expression for k.”
Solution: The trick is to interpret the payment scheme in terms of annuities-certain,
all beginning at the present. We find that
¡ ¢
10000 = k 4a4k − a3k − a2k − ak ,
so
10000
k= .
4a4k − a3k − a2k − ak

A.18.5 APPENDIX 3
This section was discussed in connection with [4, §3.7]
Information for Students in MATH 329 2009 01 2080

A.19 Supplementary Notes for the Lecture of February 16th,


2009
Distribution Date: Monday, February 16th, 2009
(subject to revision)

Textbook Chapter 4. More General Annuities.

A.19.1 §4.1 INTRODUCTION


The textbook considers
• annuities whose payments do not have the same frequency as the interest conversion
periods;
• annuities whose payments are not constant.
For the first of these topics, contained in the following three sections of the textbook, we
shall not study all the material carefully, but shall consider ad hoc solutions to problems.

A.19.2 §4.2 ANNUITIES PAYABLE AT A DIFFERENT FREQUENCY


THAN INTEREST IS CONVERTIBLE
A typographical error in [5] (corrected in [4]). The following example, given in the
textbook [5, Example 4.1, p. 96] contains a serious error. “Example 4.1 Find the accumulated
value at the end of 45 years of an investment fund in which 100 is deposited at the beginning
of each quarter for the first 2 years, and 200 is deposited at the beginning of each quarter for
the second 2 years, if the fund earns 12% convertible quarterly.”
Solution: If the interest rate were as stated the problem would be of a type studied in Chapter
2. A reading of the solution shows that the author intended the last word to be monthly.
[4, Exercise 1, p. 146], [5, Exercise 1, p. 122] “Find the accumulated value 18 years
after the first payment is made of an annuity on which there are 8 payments of 2000
each made at 2-year intervals. The nominal rate of interest convertible semiannually
is 7%. Answer to the nearest dollar.”
Solution:
1. First Solution: Working with 2-year periods. The nominal biennial
interest rate i( 2 ) is given by
1

à ! 12 µ ¶2
i( 2 )
1
i(2)
1+ 1 = 1+i= 1+
2
2
µ ¶2
0.07
= 1+
2
Information for Students in MATH 329 2009 01 2081

1 ¡ ¢
⇒ i( 2 ) =
1
· (1.035)4 − 1
2
and the effective biennial rate — call it j — is

i( 2 )
1

j= 1 = (1.035)4 − 1 .
2

Using this interest rate and a scale with time unit of a 2 year interval, the
payments — 2 intervals after the last — are worth
¡ ¢ 2000 ¡ ¢
2000 s10 − s2 = (1 + j)10 − (1 + j)2
j
µ ¶
(1.035)40 − (1.035)8
= 2000
(1.035)4 − 1
= 35824.25354
which is 35824 to the nearest unit. (Why, then, does the textbook give the value
as 35825? Because the author is using his tables. If we work with the 3.5% tables [5,
p. 383], we obtain
µ ¶
¡ ¢ s − s8 0.035
2000 s10 − s2 = 2000 40 0.035 (69)
s4 0.035
µ ¶
84.5503 − 9.0517
= 2000 (70)
4.2149
= 35824.62217 . (71)

The tables also contain values of the inverse of s4 0.035 :


µ ¶
s − s8 0.035 ¡ ¢
2000 40 0.035 = 2000s4−10.035 s40 0.035 − s8 0.035
s4 0.035
= 2000(0.237251)(84.5503 − 9.0517)
= 35824.23670 ,

which gives an answer which is closer to the correct one.)


2. Second Solution: Working with half-year periods. The effective interest
rate for half-year periods is 12 (7%) = 3.5%. A payment of 2000 at the end of
a 2-year period could be replaced by 4 equal payments at the ends of the first,
second, third, and 4th half year (which is the time of the payment of 2000). If
we denote the amount of these equal payments by X, we have the equation of
value at the end of the 2-year period,

X · s4 3.5% = 2000
Information for Students in MATH 329 2009 01 2082

2000
so X = . There will be 4 × 8 = 32 such payments of X, the last of them
s4 3.5%
8 half-years before the date at which we require the accumulated value. We
can, instead think of a series of 32 + 8 = 40 payments, and then subtract the
accumulated value of the last 8 payments that we have added. We obtain as
the accumulated value
¡ ¢ 2000 ¡ ¢
X s40 3.5% − s8 3.5% = · s40 3.5% − s8 3.5%
s4 3.5%

which agrees with equation (69) above.


[4, Exercise 2, p. 146], [5, Exercise 2, p. 122] “Find the present value of a 10-year
annuity which pays 400 at the beginning of each quarter for the first 5 years, in-
creasing to 600 per quarter thereafter. The annual effective rate of interest is 12%.
Answer to the nearest dollar.”
1
Solution: With 1 + i = (1.12) 4 ,

Present Value = 600ä40 − 200ä20


µ ¶ µ ¶
1 − v 39 1 − v 19
= (600 − 200) + 600 − 200
i i
= 11466.12687

[4, Exercise 3, p. 146], [5, Exercise 3, p. 122] “A sum of 100 is placed into a fund
at the beginning of every other year for 8 years. If the fund balance at the end of
8 years is 520, find the rate of simple interest earned by the fund.”
Solution: The wording of this problem is not as precise as it could be — while it
is clear that the payments into the fund are 2 years apart, it is not clear whether
they are at the beginnings of years ##0, 2, 4, 6 or the beginnings of years ##1, 3,
5, 7. From the author’s answer we see that he intended the former interpretation.
Let i be the annual rate of simple interest. Then the equation of value at time t = 8
is
100[(1 + 8i) + (1 + 6i) + (1 + 4i) + (1 + 2i)] = 520 ⇒ i = 6%
(The other interpretation mentioned would have given

100[(1 + 7i) + (1 + 5i) + (1 + 3i) + (1 + 1i)] = 520 ⇒ i = 7.5% .)

[4, Exercise 4, p. 146] “An annuity-immediate that pays 400 quarterly for the next 10
years costs 10,000. Calculate the nominal interest rate convertible monthly earned
by this investment.”
Information for Students in MATH 329 2009 01 2083

Solution: Let i be the effective quarterly interest rate. Then

1 − v 40 10000
a40i = = = 25,
i 400
which can be solved by iteration to yield i = 0.025243849. The effective monthly
rate is, therefore, (1.025243849)0.25 − 1 = 0.006252085, and the nominal annual
interest rate, convertible monthly, is 12 × 0.006252085 = 7.5025020%, presumably
7.5%.
Information for Students in MATH 329 2009 01 2084

A.20 Supplementary Notes for the Lecture of February 18th,


2009
Distribution Date: Wednesday, February 18th, 2009
(subject to revision)

A.20.1 §4.3 ANNUITIES PAYABLE LESS FREQUENTLY THAN INTER-


EST IS CONVERTIBLE
[4, Exercise 5, p. 146], [5, Exercise 4, p. 122] “Rework [4, Exercise 1, p. 146] us-
ing the approach developed in [4, §4.3].”
Solution: There is more than one approach in the section. It would appear that
the textbook wishes the student to apply the formulæ developed in the section,
whereby a scaling factor is used. Preferring to work from first principles, I interpret
this instruction to refer to the replacement of the biennial payments by payments
spaced according to the interest compounding interval — here a half-year. If we let
R denote the amount of such a payment under an annuity-immediate paying 2000
every 2 years, we have
R · s4 3.5% = 2000
2000
so R = s
. The value 18 years after the first biennial payment is made — i.e., 4
4 3.5%
years after the last biennial payment of 2000 is made will be equal to the value of
a semi-annual 32-payment annuity-immediate of R per half-year, 8 half-years after
the last payment, i.e.
2000
(1.035)8 · R · s32 3.5% = (1.035)8 · ·s
s4 3.5% 32 3.5%
s
= 2000(1.035)8 · 32 3.5%
s4 3.5%
57.3345
= 2000(1.035)8 · = 35824.61636.
4.2149

[4, Exercise 6, p. 146] “Give an expression in terms of functions assuming a rate of


interest per month for the present value, 3 years before the first payment is made,
of an annuity on which there are payments of 200 every 4 months for 12 years:
1. expressed as an annuity-immediate;
2. expressed as an annuity-due.”
Solution: (Since any scheme of equal payments can be viewed from the appropriate
time as either an annuity-immediate or annuity-due, the instruction isn’t completely
clear. While I obtain the answers the author appears to expect, my solution to each
Information for Students in MATH 329 2009 01 2085

of the parts gives rise to a solution to the other when we recognize that the addition
or removal of the double-dots can be effected in a fraction of the type I obtain as
solutions without any other change.)
In order to use these functions, we need to replace the regular payment every 4
months by an equivalent monthly payment. Let i represent the effective monthly
interest rate, and A and B respectively be the monthly payments at the end of the
month/in advance equivalent to a payment under an annuity-immediate/annuity-
due of 200 every 4 months. Then

As4 i = 200
Bä4 i = 200

Both replacement annuities will pay 12 × 12 = 144 monthly payments.


1. The annuity is deferred 3 years, i.e., 3 × 12 = 36 months, before the first
payment is made — that means 32 months before the beginning of annuity-
immediate at 4-month intervals, and also 32 months before the beginning of
the replacement annuity-immediate with monthly payments of A.
¡ ¯ ¢
Value = A · 32 ¯a144i
v 32 a144
= 200 ·
s4
a − a32
= 200 · 176
s4
ä − ä32
= 200 · 176
s̈4

2. This annuity is deferred 36 months.


¡ ¯ ¢
Value = B · ¯
36 ä144i
v 36 ä144
= 200 ·
ä4
ä − ä36
= 200 · 180
ä4
a − a36
= 200 · 180
a4

[4, Exercise 7, p. 146], [5, Exercise 8, p. 122] “Find an expression for the present
value of an annuity-due of 600 per annum payable semiannually for 10 years, if
d(12) = 0.09.”
Information for Students in MATH 329 2009 01 2086

Solution: The effective discount rate per month is 0.09


12
= 34 %. The effective discount
(2)
rate per 6 months is, therefore, d0 = d 2 = 1−(1−0.0075)6 . The textbook wishes us
to interpret the statement “...600 per annum payable semiannually” to mean “...300
paid per half-year”. With this interpretation the present value of the 20-half-year
annuity-due is, therefore,

1 − (1 − d0 )20
300 · ä20 = 600 ·
d0
1 − (1 − 0.0075)120
= 300 ·
1 − (1 − 0.0075)6

[4, Exercise 8, p. 146], [5, Exercise 9, p. 122] “The present value of a perpetuity
125
paying 1 at the end of every 3 years is . Find i.”
91
Solution: A payment of 1 at the end of the year for 3 years is equivalent to a
payment of s3−1 at the end of every year. The equation of value is

125 1
= s−1 ·
91 3 i
125 1
⇒ =
91 (1 + i)3 − 1
µ ¶3
3 6
⇒ (1 + i) =
5
⇒ i = 20%.

[4, Exercise 9, p. 146], [5, Exercise 10, p. 123] “Find an expression for the present
value of an annuity on which payments are 100 per quarter for 5 years, just before
the first payment is made, if δ = 0.08.”
0.08
Solution: The effective interest rate per quarter is i = e 4 − 1, so v = e−0.02 .
Accordingly, the present value of the annuity-due is

(1 + i) · (1 − v 20 )
100ä20 i = 100 ·
i
1 − e−0.4
= 100 · −0.02
e · (−1 + e0.02 )

[4, Exercise 10, p. 146], [5, Exercise 12, p. 123] “Find an expression for the present
value of an annuity on which payments are 1 at the beginning of each 4-month pe-
riod for 12 years, assuming a rate of interest per 3-month period.”
Information for Students in MATH 329 2009 01 2087

Solution: If the effective interest rate per 3-month period is i, then the equivalent
4
effective rate per 4-month period will be j = (1 + i) 3 − 1. The present value of the
annuity-due at this rate for 12 × 3 4-month periods will be
(1 + j) − (1 + j)−35
ä36 j =
j
4 140
(1 + i) 3 − (1 + i)− 3
= 4
(1 + i) 3 − 1
1 − v 48
= 4
1 − v3
[5, Exercise 6, p. 122] “Show that the present value at time 0 of 1 payable at times
7, 11, 15, 19, 23, and 27 is
a28 − a4
.
s3 + a1
Solution: s3 + a1 is the present value — just after the third payment — of a 4-
payment annuity of 1 for which the first payment was 1 unit ago. Hence a single
1
payment of 1 at time 3 is equivalent to payments of at times 1, 2, 3, 4.
s3 + a1
Hence an n-payment annuity that pays 1 every 4 years, starting in 2 years with first
payment in 3 years, may be replaced by a 4n-payment annuity-immediate starting
now, with first payment in 4 years, whose present value is
a4n
.
s3 + a1
In the problem the first payment of the given annuity is excluded; this corresponds
to the first 4 payments of the replacement annuity. Thus the present value of the
annuity described is
a28 − a4
.
s3 + a1
¤
[5, Exercise 7, p. 122] “A perpetuity of 750 payable at the end of every year, and a
perpetuity of 750 payable at the end of every 20 years are to be replaced by an
annuity of R payable at the end of every year for 30 years. If i(2) = 0.04, show that
µ ¶
1 v 40 s
R = 37500 · + · 2
s2 a40 a60
where all functions are evaluated at 2% interest.”
Solution: The given nominal annual interest rate of 4%, compounded semi-annually,
is equivalent to an effective semi-annual rate of 2%.
Information for Students in MATH 329 2009 01 2088

750
1. Each payment of 750 at the end of a year is equivalent to 2 payments of s
,
2 2%
one at the end of 6 months, the other at the end of the year. Thus the present
750
value of the first perpetuity is · a∞ 2% .
s2 2%
2. The perpetuity of 750 payable at the end of 20-year intervals, is, analogously,
750
worth at present · a∞ 2% .
s40 2%
3. Hence the amount available to purchase the annuity is
µ ¶ µ ¶
1 1 1 1 v 40
750 · a∞ 2% · + = 750 · +
s2 2% s40 2% 0.02 s2 2% v 40 s40 2%
µ ¶
1 v 40
= 37500 · + (72)
s2 2% a40 2%
R
4. A payment of R at the end of a year is equivalent to a payment of s
at the
2
end of every 6 months. The present value of the 30-year annuity is, therefore,
a
R · 60 (73)
s2
since we are replacing it by a 60-half-year annuity-immediate.
5. The equation of value equates amounts (72) and (73), implying that
µ ¶
1 v 40
37500 · +
s2 2% a40 2%
R = a 60
s
µ 2 ¶
1 v 40 s
= 37500 · + · 2
s2 2% a40 2% a60
¤
[5, Exercise 11, p. 123] “A perpetuity paying 1 at the beginning of each year has a
present value of 20. If this perpetuity is exchanged for another perpetuity paying R
at the beginning of every 2 years, find R so that the values of the two perpetuities
are equal.”
Solution: An equation of value now for the perpetuity is d1 = 20, implying that
1
i = 19 . At an effective annual discount rate of d, a payment of 1 now is equivalent
1
to a 2-year annuity-due paying 1+v = 1+i
2+i
each year, in advance. The perpetuity-
due paying R at the beginning of each 2-year period is equivalent to a perpetuity-
1+i
due paying R · 2+i at the beginning of every year. Equating this to 20 yields
39
R = 20 = 1.95.
Information for Students in MATH 329 2009 01 2089

A.21 Supplementary Notes for the Lecture of February 20th,


2009
Distribution Date: Friday, February 20th, 2009
(subject to revision)

A.21.1 §4.4 FURTHER ANALYSIS OF ANNUITIES PAYABLE MORE


FREQUENTLY THAN INTEREST IS CONVERTIBLE
In §4.4 we have a new notation. A right-superscribed (m) indicates that the annuity
is payable m times in a time unit for compounding of interest; more than that, we will
assume that the unit previously associated with one payment is divided into m parts; this
(m)
usage is analogous to the use of the upper right parentheses in i(m) . Thus an i represents
the present value of an annuity of m1 payable in m time-subintervals of length m1 : there
will be immediate and due versions of the symbol, analogous symbols for perpetuities,
and corresponding symbols for s.
(m)
We can develop formulæ for functions like a i , where i is the interest rate for a
period which contains m payments of m1 . We find, for example, that

(m) 1 − vn 1 − vn
an i = = ³ ´;
i(m) 1
m (1 + i) − 1
m

(m) 1 − vn 1 − vn 1 − vn
än i = = ³ ´ = ³ ´;
d(m) m 1 − (1 − d)
1
m m 1 − (1 + i)
1
−m

but we can also solve problems involving this function from first principles, by converting
interest rates or payments so that the intervals of interest and payment coincide; and it
is this latter practice that I shall try to follow, rather than making extensive use of the
new functions. We can also generalize these functions to perpetuities.
[4, Exercise 11, p. 147], [5, Exercise 13, p. 123] “Rework [4, Exercise 2, p. 146]
using the approach developed in [4, Section 4.4].”
Solution: The annuity can be expressed as the difference

(4) (4) 4(600) (1 − (1.12)−10 ) − 4(200) (−(1.12)−5 )


4 × 600ä10 12% − 4 × 200ä5 12% = ³ 1
´
4 1 − 1.12− 4
= 11466.12691 .
m
(m) 1 X t
[4, Exercise 12, p. 147], [5, Exercise 19, p. 123] 1. “Show that an = v m än .
m t=1
Information for Students in MATH 329 2009 01 2090

2. “Verbally interpret the (preceding) result.”


Solution:
1.
(m) 1 ³ 1 2 nm−1 nm
´
an = vm + vm + ... + v m + v m
m
1 2
vm ¡ 2 n−1
¢ vm ¡ ¢
= 1 + v + v + ... + v + 1 + v + v 2 + . . . + v n−1 + . . .
m m
m−1 m
v m ¡ 2 n−1
¢ vm ¡ ¢
+ 1 + v + v + ... + v + 1 + v + v 2 + . . . + v n−1
m m
1 ³ 1 2 m
´¡ ¢
= v m + v m + . . . + v m 1 + v + v 2 + . . . + v n−1
m
¤.
1 (m)
2. a is the present value of an annuity-due whose m × n payments of m1
m n
occur m1 th of an interest conversion period apart, the first payment being im-
mediate. We can decompose this sequence of payments into m subsequences of
n payments of m1 , the first occurring t mths of a conversion period from now
(t = 1, 2, . . . , m − 1). For the tth of these subsequences, the payments may be
viewed as constituting an annuity-due deferred t m’ths of a conversion period.
1 t
The present value of that subsequence is then · v m än .
m
[4, Exercise 13, p. 147], [5, Exercise 20, p. 123] “A sum of 10000 is used to buy a
deferred perpetuity-due paying 500 every 6 months forever. Find an expression for
the deferred period expressed as a function of d.”
1
Solution: The effective discount rate per 6 months is 1 − v 2 . If the perpetuity-due
has been deferred for n full years, the equation of value is
1
10000 = 500 · v n · √
1− v

from which it follows that



ln(20(1 − v))
n =
ln v √
ln(20(1 − 1 − d))
=
ln(1 − d)

(2) (2) (2)


[4, Exercise 14, p. 147], [5, Exercise 21, p. 124] “If 3 · an = 2 · a2n = 45 · s1 , find
i.”
Information for Students in MATH 329 2009 01 2091

Solution:
(2) (2) (2)
3 · an = 2 · a2n = 45 · s1
3 2¡ ¢ 45
⇔ (1 − v n ) = 1 − v 2n = i
2 2 2
3
The first equation implies that 1 + v n = 2
⇒ (1 + i)n = 2. Substitution in the
1
equations yields i = 30 .
[4, Exercise 15, p. 147], [5, Exercise 22, p. 124] “Find an expression for the present
value of an annuity which pays 1 at the beginning of each 3-month period for 12
years, assuming a rate of interest per 4-month period.”
Solution: If i be the effective interest rate per 4-month period, the effective rate
3
per 3-month period will be j = (1 + i) 4 − 1. Accordingly the value of the desired
annuity is

1 − (1 + j)−48
ä48 j = (1 + j) ·
j
3 1 − (1 + i)−36
= (1 + i) 4 · 3
(1 + i) 4 − 1
1 − (1 + i)−36
= 3
1 − (1 + i)− 4

An alternative approach to this problem would be to define A to be the amount


of payment that would have to be made under an annuity-due every four months
for 12 years to give the same present value as the annuity-due described in the
problem. We can look at a single year in order to determine A. The present value
of payments of A now, 4 months from now, and 8 months from now, is
¡ ¢ 1 − (1 + i)−3
A 1 + (1 + i)−1 + (1 + i)−2 = A · (1 + i) ·
i
and the value of the 4-payment annuity-due at 3 month intervals is

1 − (1 + j)−4
1(1 + (1 + j) + (1 + j)−2 + (1 + j)−3 = (1 + j) · .
j
i(1 + j)
Equating the two, and recalling that (1 + i)3 = (1 + j)4 , gives A = . The
j(1 + i)
value of the annuity will then be

ä48 j = A · ä36 i
Information for Students in MATH 329 2009 01 2092

i(1 + j) 1 − (1 + i)−36
= · (1 + i) ·
j(1 + i) i
−36
1 − (1 + i)
= 3
1 − (1 + i)− 4

as found previously.
[5, Exercise 14, p. 123] “Derive formula [5, (4.9), p. 122].”

(m) 1 ³ 1 2 1
´
än = 1 + v m + v m + . . . + v n− m
m
1 1 − vn
= ·
m d(m)
Information for Students in MATH 329 2009 01 2093

A.22 Supplementary Notes for the Lecture of March 02nd,


2009
Distribution Date: Wednesday, March 04th, 2009
(subject to revision)

At this lecture the students present were consulted on the date for the
class test, tentatively shown in the course outline as 11 March, 2009.
The majority favoured delaying the test until the following Friday, 13
March, 2009, and so that will be the date for the 45-minute class test.

A.22.1 §4.5 CONTINUOUS ANNUITIES


In the previous section we considered annuities in which a payment of 1 per period was
split into m payments of m1 paid regularly during the period. We can pass to the limit
as m → ∞ and generalize to one unit being paid continuously over the period, where
the payment in an infinitesimal subinterval of time is equal in magnitude to the length
of the subinterval. We denote this situation by placing a bar above the corresponding
symbol, as in an and sn , which we represent by integrals with respect to time, in which
we discount or accumulate the infinitesimal payments respectively to times 0 or n. Then
we can show that
Z n ¸n
t vt 1 − vn
an = v dt = =
0 ln v 0 δ
Z n ¸n
t (1 + i)t (1 + i)n − 1
sn = (1 + i) dt = = ,
0 ln(1 + i) 0 δ

both of which formulæ could have been obtained by passing to the limit in analogous
formulæ in the preceding section of the textbook. We can prove that

(m) 1 − vn 1 − vn 1 − e−nδ
an = lim a = lim = =
m→∞ n m→∞ i(m) δ δ
n n
(m) 1−v 1−v
an = lim ä = lim =
m→∞ n m→∞ d(m) δ
n
(m) (1 + i) − 1 (1 + i)n − 1 enδ − 1
sn = lim sn = lim = =
m→∞ m→∞ i(m) δ δ
n n
(m) (1 + i) − 1 (1 + i) − 1
sn = lim s̈ = lim = .
m→∞ n m→∞ d(m) δ
As always in these situations we need to be careful to distinguish the interest rate which
is applied continuously and the nominal annual rate to which it is equivalent.
Information for Students in MATH 329 2009 01 2094

[4, Exercise 17, p. 147] “There is 40,000 in a fund which is accumulating at 4% per
annum, convertible continuously. If money is withdrawn continuously at the rate
of 2400 per annum, how long will the fund last?”
Solution: Let the unknown time be n years. The fund is accumulating continuously
at 4% per annum, so δ = 0.04, and v = e−δ = e−0.04 . An equation of value is
1 − vn
40000 = 2400an = 2400 ·
δ
40000 × 0.04 1
⇒ e−0.04n = 1 − =
2400 3
ln 3
⇒ n= = 27.46530722 ,
0.04
so the fund will last 27.27 years.
[4, Exercise 18, p. 147] “If an = 4 and sn = 12, find δ.”
Solution: The hypotheses imply that

1 − v n = 4δ
(1 + i)n − 1 = 12δ

which together imply that (1 + i)n = 3. Substitution of this result into the second
1
equation yields δ = .
6

Review on instantaneous compounding

Nominal and Effective Rates To say that we are considering compound interest
is to say that we are assuming that the function a(t) is exponential The constant base of
the exponential function is the accumulation factor, as it represents the factor of increase
over one time interval; we normally require that this base be no less than 1, so that a(t)
does not decrease with time; the rate of interest is the excess of this accumulation factor
over 1. The word nominal alerts the reader to the fact that an associated rate of interest
is not the true rate associated with the given function a(t), but is stated as a multiple of
the true rate; where a nominal rate i is described as compounded m times per period , the
intention is that the effective rate, i.e., the true rate of interest, for m1 th of a time period
is to be mi ; in this usage we permit m to be any positive number. Once the accumulation
factor for the effective rate of interest is established for a given time interval, we can
determine the accumulation factors for other lengths of time, and can determine the
corresponding effective rate of interest by subtracting 1. We can compute an effective
rate of interest for any time interval, not only for intervals of unit length.
Information for Students in MATH 329 2009 01 2095

Instantaneous Compounding As we increase the frequency of compounding we


obtain, in the limit, instantaneous compounding. We have already seen that the equation
µ ¶m
i(m)
1+ = 1 + i,
m

or, equivalently, ³ ´
1
i(m) = m (1 + i) m − 1
leads, in the limit as m → ∞, to

lim i(m) = ln(1 + i) = δ .


m→∞

What this equation states is that δ = ln(1 + i) is the nominal annual rate of interest
which, when compounded instantaneously, produces an effective annual rate of interest
of i. Note that we can recover the nominal rate from the effective annual rate by solving
the equation δ = ln(1 + i), to obtain i = eδ − 1.

A.22.2 §4.6 PAYMENTS VARYING IN ARITHMETIC PROGRESSION


A general formula was be developed for an annuity-immediate with a term of n periods,
with payments beginning at P and increasing by Q per period thereafter, at interest rate
i, showing that its present value A is
an − nv n
A = P · an + Q · . (74)
i
Information for Students in MATH 329 2009 01 2096

A.23 Supplementary Notes for the Lecture of March 04th, 2009


Distribution Date: Wednesday, March 04th, 2009
(subject to revision)

A.23.1 §4.6 PAYMENTS VARYING IN ARITHMETIC PROGRESSION


(continued)
Basic formulæ I observed at the end of the notes for the last lecture that the value A
at time 0 of an annuity-immediate with a term of n periods, with payments beginning at
P and increasing by Q per period thereafter, at interest rate i, showing that its present
value is
an − nv n
A = P · an + Q · ,
i
and the accumulated value just after the last payment is
sn − n
S = P · sn + Q · .
i
When P = Q = 1, a specific symbol is used:
än − nv n
(Ia)n = (75)
i
s̈n − n sn+1 − (n + 1)
(Is)n = = (76)
i i
Similarly, when P = n and Q = −1, we have a decreasing annuity for which the present
and accumulated values are given by
n − an
(Da)n = ,
i
n(1 + i)n − sn
(Ds)n = .
i
Other symbols of interest (with obvious definitions) are (Iä)n , (I s̈)n , (Dä)n , (Ds̈)n ,
(Iä)∞ , (Ia)∞ . You may omit the discussion of Fn , Gn , Hn in [4, Appendix 4, pp. 143-
145], and any exercises based on these functions. However, you should be able to derive
formulæ
än − nv n 1+i
(Ia)∞ = lim = 2
n→∞ i i
(1 + i)2
(Iä)∞ = (1 + i)(Ia)∞ =
i2
using the calculus (l’Hospital’s Rule, or even simply methods).
Information for Students in MATH 329 2009 01 2097

The sum of the payments valued by (Ia)n and (Da)n is an annuity-immediate with
constant payments all equal to n + 1, and similar statements may be made about the
other pairs of functions. We have the identities

(Ia)n + (Da)n = (n + 1)an


(Is)n + (Ds)n = (n + 1)sn
(Iä)n + (Dä)n = (n + 1)än
(I s̈)n + (Ds̈)n = (n + 1)s̈n
Information for Students in MATH 329 2009 01 2098

A.24 Supplementary Notes for the Lecture of March 06th, 2009


Distribution Date: Friday, March 06th, 2009
(subject to revision)

A.24.1 §4.6 PAYMENTS VARYING IN ARITHMETIC PROGRESSION


(conclusion)
Verbal Interpretations. A number of the identities we can develop in connection
with increasing and decreasing annuities admit interesting verbal interpretations.
1.
än = i(Ia)n + nv n (77)
This equation can be obtained from (75): that provides an algebraic proof. One
verbal interpretation is as follows. The annual payments of 1 under an n-payment
annuity-due of present value än may be each invested for repayment n years after
the first of the payments. The interest earned by these investments will be multiples
of i: 1i at the end of year 1, 2i at the end of year 2, . . . , ni at the end of year n;
the principal of n repaid after n years is worth nv n at time 0.
2. Equation (75)
än − nv n
(Ia)n = ,
i
can be interpreted directly as describing a decomposition of the increasing annuity.
Think of a perpetuity paying än − nv n at the end of every year. That annual
payment may be interpreted as the value of an n-payment annuity-due of regular
payments of 1, the first payment at that time, followed by a charge of n the year
after the last payment. These payments, when summed, yield 0 for years n + 1, . . .,
and yield an increasing annuity with payments 1, 2, . . . , n for the first n payments.
Analogous verbal interpretations can be constructed for the identities related to (Da)n ,
(Is)n , (Ds)n , etc.

Expressing annuities with payments in Arithmetic Progression in terms of


functions we have defined. Suppose, for example, that you wish to evaluate the
present value of an annuity-immediate whose first payment is P , and whose subsequent
payments are each obtained from its predecessor by adding Q. If we begin with an
annuity whose payments are Q, 2Q, 3Q, . . ., whose present value is Q(Ia)n , we will be
left with residual payments of P − Q on each payment date. Hence the present value of
this annuity-immediate is
Q(Ia)n + (P − Q)an .
Information for Students in MATH 329 2009 01 2099

[4, Exercise 21, p. 147], [5, Exercise 31, p. 124] “Show algebraically, and by means
of a time diagram, the following relationship between (Ia)n and (Da)n :

(Da)n = (n + 1) · an − (Ia)n .”

Solution: In this method of presenting notes it is difficult to present a time diagram;


instead, I give a verbal explanation. To simplify, I shall move the subtracted term
from the right side to an added term on the left side of the identity. Then we are
summing two variable annuities: as the amount of one decreases by 1 unit, the
amount of the other takes up the slack and increases by one unit, so the sum of
the two remains constant for n payments. And that sum begins with value n + 1,
so that is the amount that remains constant, giving an annuity-immediate of that
constant payment for n payments.
Algebraically, we have
1 − v n än − nv n
(n + 1) · an − (Ia)n = (n + 1) · −
i i
n
(n + 1) − v − än
=
i
(n + 1) − v n − 1 − an−1
=
i ¡ ¢
(n + 1 − 1) − an−1 + v n
=
i
n − an
=
i
= (Da)n

20
X
[5, Exercise 22, p. 148] “Simplify (t + 5)v t .”
t=1
Solution: One method for solving this is as follows. Represent the unknown sum
by S. Then
20
X 20
X
t
S= (t + 5)v ⇔ vS = (t + 5)v t+1
t=1 t=1
X21
⇔ vS = (s + 4)v s taking s = t + 1
s=2
X21
⇔ vS = (t + 4)v t replacing s by t
t=2
Information for Students in MATH 329 2009 01 2100

21
X 21
X 21
X
⇔ vS = ((t + 5) − 1)v t = (t + 5)v t − vt
t=2 t=2 t=2
¡ ¢ v (1 − v 20 )
⇔ vS = S + v 26v 20 − 6 −
i
¡ ¢ 20
v (1 − v )
⇔ dS = v 6 − 26v 20 +
¡ ¢ i
⇔ iS = 6 − 6v 20 + a20 − 20v 20
6 − 6v 20 a20 − 20v 20 a − 20v 20
⇔ iS = + = 6a20 + 20 .
i i i

[4, Exercise 23, p. 148], [5, Exercise 32, p. 124] “The following payments are made
under an annuity: 10 at the end of the 5th year, 9 at the end of the 6th year, de-
creasing by 1 each year until nothing is paid. Show that the present value is

10 − a14 + a4 (1 − 10i)
.”
i
Solution: The payments decrease until a payment of 1 at the end of the 14th year.
We can think of a decreasing annuity starting with a payment of 14 at the end of
the 1st year, and then make corrections. We can subtract a 4-payment decreasing
annuity-immediate beginning with a payment of 4 at the end of the year, and a
4-payment annuity-immediate with a constant payment of 10. Thus we have
¡ ¢ ¡ ¢
14 − a14 − 4 − a4 − 10i · a4
(Da)14 − (Da)4 − 10a4 =
i
10 − a14 + a4 (1 − 10i)
=
i

[4, Exercise 24, p. 148], [5, Exercise 33, p. 124] “Find the present value of a per-
petuity under which a payment of 1 is made at the end of the 1st year, 2 at the end
of the 2nd year, increasing until a payment of n is made at the end of the nth year,
and thereafter payments are level at n per year forever.”
Solution: We can begin with a perpetuity-immediate of n per year, and subtract
from it a decreasing annuity-immediate which begins with a payment of n − 1 and
decreases by 1 unit per year.
¡ ¢
n − (n − 1) − an−1
n · a∞ − (Da)n−1 =
i
1 + an−1
=
i
Information for Students in MATH 329 2009 01 2101

än
=
i
(1 + i)an
=
i
an an
= =
vi d

[4, Exercise 25, p. 148], [5, Exercise 34, p. 124] “A perpetuity-immediate has an-
nual payments of 1, 3, 5, 7, . . . . If the present value(s) of the 6th and 7th payments
are equal, find the present value of the perpetuity.”
Solution: We equate the values of the 6th and 7th payments:

(1 + (6 − 1)2)v 6 = (1 + (7 − 1)2)v 7
13
⇔ 1+i=
11
2
⇔ i=
11
The perpetuity can be viewed as the sum of an increasing perpetuity with pay-
ments increasing by 2 each year, i.e., 2(Ia)∞ diminished by a constant perpetuity-
immediate a∞ :
³ ´
n
lim än − (1+i) n
1
2 · (Ia)∞ − a∞ = 2 n→∞ −
i i
2ä∞ − 1
= by l’Hôpital’s Rule
i
= 66

[4, Exercise 26, p. 148], [5, Exercise 35, p. 124] “If X is the present value of a
perpetuity of 1 per year with the first payment at the end of the 2nd year and
20X is the present value of a series of annual payments 1, 2, 3, . . . with the first
payment at the end of the 3rd year, find d.”
Solution: The constraints are:
1 v
X = −v =
i i
20X = (Ia)∞ − 2a∞ + v
v
= 2,
i
1 1
implying that i = 20
, d= 21
.
Information for Students in MATH 329 2009 01 2102

[4, Exercise 27, p. 148], [5, Exercise 36, p. 125] “An annuity-immediate has semi-
annual payments of 800, 750, 700, . . . , 350, at i(2) = 0.16. If a10 0.08 = A, find the
present value of the annuity in terms of A.”
Solution: We will be working with an effective semi-annual interest rate of j =
i(2)
2
= 8%. The truncated decreasing annuity-immediate that we wish to evaluate
has present value
¡ ¢ ¡ ¢
10 16 − a16 − v 10 6 − a6
50(Da)10 − 50v (Da)6 = 50 ·
i
50 ¡¡ ¢ ¡ ¢¢
= · 16 − 6v 10 − a16 − v 10 a6
i
50 ¡ ¢
= · 10 + 6ia10 − a10
i
50 ¡ ¢
= · 10 + (6i − 1)a10
i
= 625(10 − 0.52A) = 6250 − 325A .

Alternatively we could express the payments as a constant annuity of payment size


300, superimposed on a decreasing annuity of payments 500, 450, 400, . . . , 50. This
has value
10 − A
300 · a10 8% + 50(Da)10 8% = 300A + 50 ·
0.08
= 300A + 6250 − 625A = 6250 − 325A.

[5, Exercise 29, p. 124] In [5, Example 4.13, p. 116] it is shown that the present value
of an annuity-immediate such that payments start at 1, increase by annual amounts
of 1 to a payment of n, and then decrease by annual amounts of 1 to a final payment
of 1, is an · än . The present exercise is to justify this value verbally.
Solution: Consider a sequence of n annuities-due, each of them consisting of n
payments of 1. The first of these annuities is to make its first payment 1 year
from now, the second 2 years from now, ..., the nth n years from now. The total
payments made will increase from 1 to n, then decrease to a payment of 1, 2n − 1
years from now. The value of the payments under each of these annuities-due is än
just before the first payment. These values, when discounted to the present, have
value än · an .

A.24.2 §4.7 PAYMENTS VARYING IN GEOMETRIC PROGRESSION


I shall work some problems to illustrate that problems of this type are not hard to solve,
since the effect of the geometric progression is equivalent to altering the interest rate.
Information for Students in MATH 329 2009 01 2103

[4, Exercise 28, p. 148], [5, Exercise 38, p. 125] “Find the present value of a 20-
year annuity with annual payments which pays 600 immediately and each subse-
quent payment is 5% greater than the preceding payment. The annual effective rate
of interest is 10.25%. Answer to the nearest dollar.”
Solution:
19
X
Present Value = 600v n (1.05)n
n=0
¡ 1.05 ¢20
1 − 1.1025
= 600 · 1.05
1 − 1.1025
= 7851.1926 ,

or 7,851 to the nearest dollar.


[4, Exercise 29, p. 148] “ In [4, Exercise 28, p. 148]] find the interest rate i0 such
that the present value would be equal to the present value of the level annuity-due
600ä20i0 .”
7851.1926
Solution: We have to solve for i0 the equation ä20i0 = . We find by
600
iteration, interpolation, or other methods that i = 5.0%.
[4, Exercise 30, p. 148], [5, Exercise 37, p. 125] “Annual deposits are made into a
fund at the beginning of each year for 10 years. The first 5 deposits are 1,000 each,
and deposits increase by 5% per year thereafter. If the fund earns 8% effective, find
the accumulated value at the end of 10 years.”
Solution: The first 5 deposits are today worth

1000ä5 8% = 4312.12684 .

I see 2 ways of interpreting the words “increase by 5% per year thereafter”: either
the increase is geometric, by a factor of 1.05 applied repeatedly; or the deposits
increase in arithmetic progression (before discounting). If the deposits increase in
geometric progression, then the present value will be
¡ ¢
1000 v 5 (1.05) + v 6 (1.05)2 + . . . + v 9 (1.05)5
¡ ¢
5 1 − (v(1.05))5
= 1000v (1.05) ·
1 − v(1.05)
à µ ¶5 !
1.05
= 36000(1.05)(1.08)−5 · 1 −
1.08
= 3379.996182.
Information for Students in MATH 329 2009 01 2104

The present value will then be 4312.1268+3379.9962 = 7692.1230; the accumulated


value at the end of 10 years will be 16606.72. This is the answer given by the
textbook, so, presumably, our interpretation is the one the author intended, (which
could be expected by the presence of the exercise in this section of the chapter).
(But the language is ambiguous, and the other interpretation is plausible also. If
the deposits increase in arithmetic progression, then the present value will be
1000(1.08)−5 ä5 8% + 50(1.08)−5 (Iä)5 8% = 3510.09347
so the present value is 7822.2203, and the value after 10 years is 16,887.59.)
[4, Exercise 31, p. 149] “A perpetuity makes payments starting five years from today.
The first payment is 1000, and each payment thereafter increases by k% per year.
The present value of this perpetuity is equal to 4096, when compounded at i = 25%.
Find k.”
Solution: Five years from now the value of the perpetuity is
à µ ¶1 µ ¶2 µ ¶3 !
1 + 0.01k 1 + 0.01k 1 + 0.01k
1000 1 + + + + ...
1.25 1.25 1.25
1000 125000
= 1+0.01k
= .
1 − 1.25 25 − k
125000
An equation of value is, therefore = 4096(1.25)5 = 4 · 55 , implying that
25 − k
k = 15.
[4, Exercise 32, p. 149] “An employee currently is aged 40, earns 40,000 per year, and
expects to receive 3% annual raises at the end of each year for the next 25 years.
The employee decides to contribute 4% of annual salary at the beginning of each
year for the next 25 years into a retirement plan. How much will be available for
retirement at age 65 if the fund can earn a 5% effective rate of interest? Answer to
the nearest dollar.”
Solution: The accumulation into the fund by age 65 is
¡ ¢
(0.04)(40000) (1.05)25 (1.03)0 + (1.05)24 (1.03)1 + . . . + (1.05)1 (1.03)24
à µ ¶2 µ ¶24 !
1.03 1.03 1.03
= 1600(1.05)25 1 + + + ... +
1.05 1.05 1.05
¡ 1.03 ¢25
25 1 − 1.05
= 1600(1.05) · 1.03
1 − 1.05
à µ ¶25 !
25 105 103
= 1600(1.05) · · 1− = 108576.4687
2 105
Information for Students in MATH 329 2009 01 2105

or 108,576 to the nearest unit.

A.24.3 §4.8 MORE GENERAL VARYING ANNUITIES


We will not formally study this section and its interesting generalizations. If you meet
any problems of these types, they should be solvable “by first principles”. To repeat:
you don’t need to remember derivations or formulæ, but you have already been exposed
to machinery that can be used to solve problems of these types — no new machinery is
required.

A.24.4 §4.10 MISCELLANEOUS PROBLEMS


[4, Exercise 44, p. 150] “A perpetuity paying 1 at the beginning of each 6-month
period has a present value of 20. A second perpetuity pays X at the beginning of
every 2 years. Assuming the same annual effective interest rate, the two present
values are equal. Determine X.”
Solution: In terms of the effective annual discount rate d, the rate of discount for
√ 1
one-half year is 1 − 1 − d, and the value of the first perpetuity is √ . The
1− 1−d
rate of discount for a 2-year period is 1 − (1 − d)2 = 2d − d2 , so the value of the
X
second perpetuity is . Equating the values of the two perpetuities yields
2d − d2
2d − d2 ³ √ ´
X= √ = (2 − d) 1 + 1 − d .
1− 1−d
We are told that the first perpetuity is worth 20, so
1 39 761 39
√ = 20 ⇒ d = ⇒X= · = 3.709875 .
1− 1−d 400 400 20
[4, Exercise 45, p. 150], [5, Exercise 54, p. 126] “ For a given n it is known that
Zn
an = n − 4 and δ = 10%. Find at dt.”
0
Solution: Note that the exercise does not say that an = n − 4 for all n; in any case,
this would be absurd for n ≤ 4. (The hypothesis that δ = 10% should not be linked
to the previous statement: it is part of the general background for the exercise.)
n
By [4, equation (4.14), p. 125] an = 1−vδ
, so the first hypothesis is equivalent to
1 − vn
= n − 4. This equation also enables us to find an antiderivative:
δ
Z n Z n
1 − vn
at dt = dt
0 0 δ
Information for Students in MATH 329 2009 01 2106

· ¸n
1 1 t
= t+ ·v
δ δ 0
µ n

1 v −1
= n+
δ δ
1−v n
n− δ
=
δ
n − an 4
= = = 40.
δ δ

[4, Exercise 46, p. 150], [5, Exercise 56, p. 127] “A family wishes to provide an
annuity of 100 at the end of each month to their daughter, now entering college.
The annuity will be paid for only nine months each year, for four years. Show that
(12)
the present value one month before the first payment is 1200ä4 · a .”
9/12
Solution: The proposed factor ä4 represents a 4-payment annual annuity-due. We
need only determine the value of the annuity-immediate of 9 monthly payments.
These result from division of the year into 12 months — that’s the message of the
superscript (12) — but the payments stop after the 9th — that’s the message of
the subscript 9/12 .
[4, Exercise 47, p. 150], [5, Exercise 58, p. 127] “There are two perpetuities. The
first has level payments of p at the end of each year. The second is increasing such
that the payments are q, 2q, 3q, . . . . Find the rate of interest that will make the
difference in present value between these perpetuities
a) “Zero;
b) “A maximum.”
Solution: I assume that the second perpetuity is a perpetuity-immediate, like
p
the first. The present value of the first perpetuity-immediate is . The second
i
perpetuity-immediate is worth
ä∞ q
q · (Ia)∞ = q · = .
i di
a) For the present values to be equal,
p q
=
i id
p q(1 + i)
⇒ =
i i2
q
⇒ i=
p−q
Information for Students in MATH 329 2009 01 2107

b) The difference is
µ ¶2
p q (p − q)2 1 p−q
− = −q −
i id 4q i 2q

which is maximized when


1 p−q
− =0
i 2q
2q
i.e., when i = .
p−q
Information for Students in MATH 329 2009 01 2108

A.25 Supplementary Notes for the Lecture of March 09th, 2009


Distribution Date: Monday, March 09th, 2009
(subject to revision)

Textbook Chapter 5. Amortization schedules and sinking


funds.

A.25.1 §5.1 INTRODUCTION


We consider methods of repaying a loan, in particular
The Amortization Method: In this method the borrower makes instalment payments
to the lender. Usually these payments are at regularly spaced periodic intervals;
the progressive reduction of the amount owed is described as the amortization of
the loan.
The Sinking Fund Method: In this method the loan will be repaid by a single lump
sum payment at the end of the term of the loan. However the borrow may prepare
himself for the repayment by making deposits to a fund called a sinking fund to
accumulate the repayment amount. (Sometimes the lender may be aware of the
existence of the sinking fund; for example, an institutional borrower that issues a
series of bonds may let the public know that the accumulation of funds to redeem
the bonds may be disciplined by a sinking fund.)

A.25.2 §5.2 FINDING THE OUTSTANDING LOAN BALANCE


When a loan is being amortized the outstanding balance is being reduced by the amor-
tization payments. Each payment may be analyzed and interpreted as consisting of an
interest component and a component for reduction of principal . An equation of value
can be set up at any time during the amortization, equating

Current value of payments = Accumulated value of Loan

where “Payments” consists of both past and future payments. Decomposing the term
and rearranging the equation gives

Present Value of Future Payments


=Accumulated Value of Loan − Accumulated Value of Past Payments

Synonymous terms:
• outstanding loan balance
• outstanding principal
Information for Students in MATH 329 2009 01 2109

• unpaid balance
• remaining loan indebtedness
[4, Exercise 1, p. 185], [5, Exercise 1, p. 195] “A loan of 1,000 is being repaid with
quarterly payments at the end of each quarter for 5 years, at 6% convertible quar-
terly. Find the outstanding loan balance at the end of the 2nd year.”
1000
Solution: The level payments under this annuity-immediate will be .
a20 1.5%
Retrospective method: The value of the payments already made is
1000
·s = 491.1769.
a20 1.5% 8 1.015

Subtracting this from 1000(1.015)8 yields 635.3157.


Prospective method: The value of the 12 remaining payments is
1000
·a = 635.3157 .
a20 1.5% 12 1.5%

[4, Exercise 2, p. 185], [5, Exercise 2, p. 195] “A loan of 10,000 is being repaid by
instalments of 2,000 at the end of each year, and a smaller final payment made one
year after the last regular payment. Interest is at the effective rate of 12%. Find
the amount of outstanding loan balance remaining when the borrower has made
payments equal to the amount of the loan.”
Solution: The problem asks for the outstanding loan balance just after payments
totalling 10,000 have been made; this will be immediately after the 5th payment.
We shall use the retrospective method only here. The accumulated value of the
loan at time t = 5 is 10000(1.12)5 . The accumulated value of the payments made
is 2000s5 . The outstanding loan balance will, therefore be

2000 ¡ ¢
10000(1.12)5 − 2000s5 = 10000(1.12)5 − (1.12)5 − 1
0.12
= 4917.72212

Were we to use the prospective method, we would need to determine the value of
the last drop payment. This is interesting information, and we could have been
asked for it. But it has not been requested, and so we shall not bother finding it.
(But you should know how to do that if it is necessary.)
[4, Exercise 3, p. 185], [5, Exercise 3, p. 195] “A loan is being repaid by quarterly
instalments of 1,500 at the end of each quarter, at 10% convertible quarterly. If the
loan balance at the end of the first year is 12,000, find the original loan balance.
Information for Students in MATH 329 2009 01 2110

Solution: Denote the original loan balance by L. Here the retrospective method is
the most appropriate, since we don’t know how many future payments have to be
made. The equation of value at time 1 year is

L(1.025)4 − 1500s4 2.5% = 12000

which we solve to yield


¡ ¡ ¢¢
L = (1.025)−4 1200 + 1500 (1.025)4 − 1
= −48000(1.025)−4 + 60000 = 16514.36905.

[4, Exercise 4, p. 185], [5, Exercise 6, p. 196] “A 20,000 loan is to be repaid with
annual payments at the end of each year for 12 years. If (1 + i)4 = 2, find the
outstanding balance immediately after the fourth payment.”
20000
Solution: The annual payment is, by the prospective method, . Again by the
a12 i
prospective method, the outstanding balance after the 4th payment is
µ ¶
20000 1 − (1 + i)−8 6
· a8 i = 20000 −12
= · 20000 = 17142.85714.
a12 i 1 − (1 + i) 7

[4, Exercise 5, p. 185], [5, Exercise 7, p. 196] “A 20,000 mortgage is being repaid
with 20 annual instalments at the end of each year. The borrower makes 5 payments,
and then is temporarily unable to make payments for the next 2 years. Find an
expression for the revised payment to start at the end of the 8th year if the loan is
still to be repaid at the end of the original 20 years.”
20000
Solution: The original payments are (by the prospective method) . The
a20
outstanding balance at the end of the 7th year (with no payment then or at the
end of the previous year) is the value then of all unpaid payments, i.e.,
20000 ¡ ¢
· a13 + s2 .
a20

It follows that the level payment needed to repay the loan in 13 payments (under
an annuity-immediate) is

20000 ¡ ¢ 1 20000 · (1 + i)2 · a15


· a13 + s2 · = .
a20 a13 a20 · a13

[4, Exercise 6, p. 185][5, Exercise 8, p. 196] “A loan of 1 was originally scheduled


to be repaid by 25 equal annual payments at the end of each year. An extra payment
Information for Students in MATH 329 2009 01 2111

K with each of the 6th through the 10th scheduled payments will be sufficient to
repay the loan 5 years earlier than under the original schedule. Show that
a − a15
K = 20 .”
a25 · a5
1
Solution: The level payment of this loan of 1 is . The extra payments are equal
a25
in value to the value of the last 5 payments, so, at time t = 5,
a − a15
K · a5 = 20
a25
which yields the desired value for K.
[4, Exercise 7, p. 185] “A husband and wife buy a new home and take out a 150,000
mortgage loan with level annual payments at the end of each year for 15 years, on
which the effective rate of interest is equal to 6.5%. At the end of 5 years they
decide to make a major addition to the house, and want to borrow an additional
80,000 to finance the new construction. They also wish to lengthen the overall
length of the loan by 7 years (i.e., until 22 years after the date of the original loan).
In the negotiations the lender agrees to these modifications, but only if the effective
interest rate for the remainder of the loan after the first 5 years (be) raised to 7.5%.
Find the revised annual payment which would result for the remainder of the loan.
Answer to the nearest dollar.”
Solution: The statement “after the first 5 years” is ambiguous — is the interest rate
to begin immediately with the new loan, or is it to begin after 5 years of the new
loan? I will assume the intention is that the new interest rate begins immediately.
150000
The initial payment is . Immediately after the 5th payment the amount
a15 6.5%
outstanding on the loan is
µ ¶
5 s5 6.5%
150000 (1.065) − .
a15 6.5%
The principal of the new loan will be
µ ¶
5 s5 6.5%
150000 (1.065) − + 80000 .
a15 6.5%
The 17 level payments on the new loan will therefore be
µ ¶
5
s
5 6.5%
³ ´
150000 (1.065) − a + 80000 150000 (1.065)5
− (1.065)5 −1
+ 80000
15 6.5% 1−(1.065)−15
= 1−(1.075)−17
a17 7.5%
0.075
= 20636.38388 ,
Information for Students in MATH 329 2009 01 2112

so the new level payment should be 20,636.


[5, Exercise 4, p. 195] A loan is being repaid by annual payments at the ends of 15
successive years. The first 5 instalments are 4,000 each, the next 5 are 3,000 each,
and the final 5 are 2,000 each. Find expressions for the outstanding loan balance
immediately after the second 3,000 instalment.
1. prospectively;
2. retrospectively.
Solution:
1. Prospective Method. The value at time 7 of the payments yet to be made
is 2000a8 + 1000a3 .
2. Retrospective Method. We have to use the prospective method at some time
to obtain the initial value of the loan. This will be the value of all payments
at time t = 0, i.e., 2000a15 + 1000a10 + 1000a5 . The value of all payments
made before and at time t = 7 is 4000s − 2s2 . The outstanding loan balance
is, therefore
¡ ¢
2000a15 + 1000a10 + 1000a5 (1 + i)7 − 4000s7 + 1000s2

[5, Exercise 5, p. 196] “A loan is to be repaid with level instalments payable at the
end of each half-year for 3 12 years, at a nominal rate of interest of 8% convertible
semiannually. After the fourth payment the outstanding loan balance is 5,000. Find
the initial amount of the loan.”
Solution: The effective semiannual rate is 4%. There are to be 7 level payments in
all. The amount of the payments is, by the prospective method
5000 5000 × 0.04
= .
a3 4% 1 − (1.04)−3

It follows that the amount of the loan, now by the prospective method, is
µ ¶
5000 1 − (1.04)−7
·a = 5000 = 10814.15817 .
a3 2% 7 2% 1 − (1.04)−3
Information for Students in MATH 329 2009 01 2113

A.26 Supplementary Notes for the Lecture of March 11th, 2009


Distribution Date: Wednesday, March 11th, 2009
(subject to revision)

A.26.1 §5.3 AMORTIZATION SCHEDULES


An amortization schedule is a chart showing, for each payment date, the information
shown in the columns of the following beginning of such a schedule with n payments of
size x:
Year Payment Interest Principal Outstanding
amount paid repaid loan balance
0 x · an i
1 x x(1 − (1 + i)−n ) x(1 + i)−n x · an−1 i
2 x x(1 − (1 + i)−n+1 ) x(1 + i)1−n x · an−2 i
··· ··· ··· ··· ···
In practice there will be rounding errors as the table is generated line by line, and the
last line may not quite balance. “Standard practice is to adjust the last payment so that
it is exactly equal to the amount of interest for the final period plus the outstanding
loan balance at the beginning of the final period,” in order to bring the outstanding loan
balance to 0. In order to determine the entries in one row of the table, we do not need to
generate the table line by line; the table is useful when a number of rows are of interest.
Few of the exercises in the textbook require extensive computation of schedules.
Students should, however, be capable of completing a full table for a loan when all the
information needed is available.
[4, Exercise 8, p. 186], [5, Exercise 10, p. 196] “A loan is being repaid with quar-
terly instalments of 1,000 at the end of each quarter for 5 years at 12% convertible
quarterly. Find the amount of principal in the 6th instalment.”
Solution: The principal is
1000 ¡ ¢
1000a20 3% = 1 − (1.03)−20 = 14877.47486 .
0.03
After the 5th payment, the outstanding principal is (by the prospective method)
1000a20−5 3% , so the interest component of the next payment is
30 ¡ ¢
30a20−5 3% = 1 − (1.03)−15 = 358.1380526 ,
0.03
and the amount of principal reduction is
1000 − 358.1380526 = 641.8619474 .
Information for Students in MATH 329 2009 01 2114

[4, Exercise 9, p. 186], [5, Exercise 12, p. 196] “A loan of 10,000 is being repaid
with 20 instalments at the end of each year, at 10% effective. Show that the amount
of interest in the 11th instalment is
1000
.”
1 + v 10
Solution: The amount of each payment is, by the prospective method,
10000
.
a20 10%
By the prospective method, the unpaid balance after the 10th payment is
10000 10000
· a10 10% =
a20 10% 1 + (1.1)10
so the interest component of the 11th payment is
µ ¶
10000 100
0.01 10
= .
1 + (1.1) 1 + (1.1)10
[4, Exercise 10, p. 186], [5, Exercise 14, p. 197] “A loan is being repaid with a se-
ries of payments at the end of each quarter, for 5 years. If the amount of principal
in the 3rd payment is 100, find the amount of principal in the last 5 payments.
Interest is at the rate of 10% convertible quarterly.”
Solution: The loan is being repaid in 5 × 4 = 20 quarterly payments, and the
effective interest rate per quarter is 41 (10%) = 2.5%. Let’s assume that the amount
of each level payment is x, and begin by compiling the first part of the amortization
table.
Payment Payment Interest Principal Outstanding
amount paid repaid loan balance
0 x · a20 2.5%
1 x x(1 − (1.025)−20 ) x(1.025)−20 x · a19 2.5%
−19 −19
2 x x(1 − (1.025) ) x(1.025) x · a18 2.5%
−18 −18
3 x x(1 − (1.025) ) x(1.025) x · a17 2.5%
from which we see that x(1.025)−18 = 100, so
x = 100(1.025)18 = 155.9658718 .
(We didn’t need to use the schedule here. By the Prospective Method, the unpaid
balance just after the 2nd payment is x · s18 , so the interest component of the 3rd
payment is ix · s18 and the residue for reduction of principal is
¡ ¡ ¢¢
x 1 − 1 − v 18 = xv 18 .)
Information for Students in MATH 329 2009 01 2115

We could now compile the last lines of the amortization table backwards. Alter-
natively, if the author is requesting the total amount of principal in the last 5
payments, that is
x(v + v 2 + v 3 + v 4 + v 5 ) = 155.9658718 · a5 0.025 = 724.5906916 .

[4, Exercise 11, p. 186], [5, Exercise 15, p. 197] “A loan is being repaid with in-
stalments of 1 at the end of each year for 20 years. Interest is at effective rate i for
the first 10 years, and effective rate j for the second 10 years. Find expressions for
“a) the amount of interest paid in the 5th instalment;
“b) the amount of principal repaid in the 15th instalment.”
Solution: The present value of the last 10 payments is (1 + i)−10 a10 j ; the principal
of the loan is, therefore,
P = a10 i + (1 + i)−10 a10 j .
We compile the first lines of the amortization table:
Year Payment Interest Principal Outstanding
amount paid repaid loan balance
0 P
1 1 iP 1 − iP P (1 + i) − s1 i 1
2 1 i(P¡¡ 1) (1 + i)(1 − iP ) P (1 + i)2 − s2 i
(1 + i) − ¢¢
3 1 i(1 + i)2 ¡¡P − a2 i ¢¢ s2 i − i(1 + i)2 P P (1 + i)3 − s3 i
4 1 i(1 + i)3 ¡¡P − a3 i ¢¢ s3 i − i(1 + i)3 P P (1 + i)4 − s4 i
5 1 i(1 + i)4 P − a4 i s4 i − i(1 + i)4 P P (1 + i)5 − s5 i
1. The amount of interest in the 5th instalment is
¡ ¢ ¡ ¢
i(1 + i)4 ( a10 i + (1 + i)−10 a10 j − a4 i = i a6 i + (1 + i)−6 a10 j .
More simply, we can observe (using the Prospective Method) that the outstand-
ing balance just after the 4th instalment is
(1 + i)−6 a10 j + a6 i .
The interest component of the 5th payment is obtained by multiplying this
amount by i.
2. After the 10th instalment has been paid, we shift to the second interest rate.
The outstanding balance after the 14th payment is, again by the Prospective
Method, a6 j ; interest one payment later will be
j · a6 j = 1 − v 6
so the payment will reduce principal by 1 − (1 − v 6 ) = v 6 .
Information for Students in MATH 329 2009 01 2116

[4, Exercise 12, p. 186], [5, Exercise 17, p. 197] “A borrower has a mortgage which
calls for level annual payments of 1 at the end of each year for 20 years. At the time
of the 7th regular payment an additional payment is made equal to the amount
of principal that, according to the original amortization schedule, would have been
repaid by the 8th regular payment. If payments of 1 continue to be made at the end
of the 8th and succeeding years until the mortgage is fully repaid, show that the
amount saved in interest payments over the full term of the mortgage is 1 − v 13 .”
Solution: At time 0 the amount owing is a20 . By the 8th regular payment on the
original schedule, the principal repaid would have been

A − A(1 + i)8 + s8 = (Ai − 1)s8 .

It is intended that this amount is added to the 7th payment. Immediately after the
original 7th payment the principal owing would have been

A(1 + i)7 − s7 .

The additional payment at time 7 reduces this amount to


¡ ¢ ¡ ¢
A(1 + i)7 − s7 − Ai − 1)s8 = (1 + i)7 + A 1 − i(1 + i)7

[4, Exercise 13, p. 186], [5, Exercise 18, p. 197] “A loan of L is being amortized
with payments at the end of each year for 10 years. If v 5 = 32 , find the following:
“a) The amount of principal repaid in the first 5 payments.
“b) The amount due at the end of 10 years if the final 5 payments are not made as
scheduled.”
Solution:
1. The annual level payments constitute an annuity-immediate with annual pay-
ment of
L Li 9iL
= ¡ 2 ¢2 = .
a10 1− 3 5
By the prospective method, the amount of principal remaining to be paid im-
mediately after the 5th annual payment is
µ ¶
9iL 9L 2 3L
· a5 = · 1− = .
5 5 3 5

Hence
µ the
¶ amount of principal that has already been paid at that time is
3 2L
1− L= .
5 5
Information for Students in MATH 329 2009 01 2117

2. If no further payments are made, the amount repayable at the end of 10 years
is
3L 3 3L 9L
(1 + i)5 · = · = .
5 2 5 10
[4, Exercise 14, p. 187], [5, Exercise 19, p. 197] “A 35-year loan is to be repaid
with equal instalments at the end of each year. The amount of interest paid in the
8th instalment is 135. The amount of interest paid in the 22nd instalment is 108.
Calculate the amount of interest paid in the 29th instalment.”
Solution: Suppose the amount of the loan at time 0 was L. The annual instalments
L
are each . By the prospective method the amount of principal outstanding just
a35
after the 7th instalment is aL · a28 . This will incur an interest payment of i · aL · a28
35 35
in the 8th instalment; we thus have the equation

1 − v 28
iL · = 135 .
1 − v 35
A similar computation involving instalments 21 and 22 gives

1 − v 14
iL · = 108 .
1 − v 35
Taking the ratio of the two equations, we obtain

1 − v 28 135 5
1 + v 14 = = =
1 − v 14 108 4
279
so v 7 = 12 ; substitution in either equation above yields iL = . The amount of
2
interest in the 29th instalment is
a7 1 − v7 16iL
iL · = iL · 35
= = 72 .
a35 1−v 31

[4, Exercise 15, p. 187] “A 10-year loan of L is repaid by the amortization method,
with payments of 1000 at the end of each year. The annual effective interest rate is
i. The total amount of interest repaid during the life of the loan is also equal to L.
Calculate the amount of interest paid during the first year of the loan.”
Solution: The level payments under the amortized loan are
L
1000 = ;
a10i
Information for Students in MATH 329 2009 01 2118

the total amount of interest paid during the life of the loan is 10000 − L = L, so
L = 5000, and a10i = 5. Solving this equation (how?) yields i = 0.1509841448 =
15.09841448%, so the amount if interest paid at the end of the first year is 0.1509841448×
5000 = 754.9207240 or 754.92.
[4, Exercise 16, p. 187] “A bank customer borrows X at an annual effective rate of
12.5%, and makes level payments at the end of each year for n years.
(i) “The interest portion of the final payment is 153.86.
(ii) “The total principal repaid as of time n − 1 is 6009.12.
(iii) “The principal repaid in the first payment is Y .
Calculate Y .”
X
Solution: The level payments are each equal to . The interest portion of the
an 12.5%
final payment is
0.125
dX 1.125
·X X
153.86 = = = ,
an 12.5% an 12.5% 9an 12.5%
implying that
X X
an12.5% = = ,
153.86 × 9 1384.74
so the level payments are 1384.74. The unpaid principal at time n − 1 — is it clear
1384.74
whether this is before or after the payment? — 1384.74a1 = 1384.74v = =
1.125
1230.88, so the total loan is X = 6009.12 + 1230.88 = 7240.; hence
7240.
an12.5% = = 5.2284
1384.74
so 1 − (1.125)−n = (0.125)(5.2284) = 0.65355, and v n = 0.34645. But this implies
that the principal component of the first payment is Y = 0.34645×1384.74 = 479.74.
[5, Exercise 11, p. 196] “Consider a loan which is being repaid with instalments of 1
at the end of each period for n periods. Find an expression at issue for the present
value of the interest which will be paid over the life of the loan.”
Solution: We can use the information in [4, Table 5.1, p. 157]. The sum of the
Xn
interest portions of the payments is, as shown in the table, (1 − v r ) = n − an .
r=1
This, however, is not what the problem requests. The present value of the interest
payments — presumably as of time t = 0, is
n
X
v n−r+1 (1 − v r ) = an − nv n+1 .
r=1
Information for Students in MATH 329 2009 01 2119

[5, Exercise 13, p. 196] “A loan is being repaid with 20 instalments at the end of each
year at 9% effective. In what instalment are the principal and interest portions most
nearly equal to each other?”
Solution: Without limiting generality, assume the payments are all of size 1, so that
the loan is for a20 9% , and [4, Table 5.1, p. 157] applies. In that table we see that the
repayments of principal range between v 20 in the first payment to v 1 in the last. The
two portions under consideration sum to a constant, so they cannot both exceed 12
at any time. Both sequences — interest payments and reductions of principal —
are monotone: there will be only one pair of values t = a, t = a + 1, where they
reverse their positions from being greater than/less than 21 to the reverse. But it
is not clear which of those two will have the closest values! We see from [4, Table
5.1, p. 157] that the difference between principal and interest components in the
tth payment is |1 − 2v n−t+1 |. We shall begin by determining the largest value of t
— if any — for which
1
v 20−t+1 ≤ ,
2
equivalently, the largest value of t such that

(1 + i)21−t ≥ 2 ,

equivalently, the largest value of t such that


ln 2.
t ≤ 21 − = 12.95676827 ,
ln 1.09
i.e., t = 12. This is the “best” choice where the difference 1 − 2v n−t+1 is positive.
We need also to consider the “best” choice when the difference is negative, i.e.,
t = 13. We compute the differences:

t = 12 1 − 2(1.09)−9 = 0.0791444410
t = 13 1 − 2(1.09)−8 = −0.003732559

and see that the values are closest when t = 13.


Information for Students in MATH 329 2009 01 2120

A.27 Supplementary Notes for the Lecture of March 16th, 2009


Distribution Date: Monday, March 16th, 2009
(subject to revision)

Negative final payment? I begin this lecture with an example from an earlier chapter
which can be relevant to sinking fund problems. In [4, Example 3.7, pp. 92–93] there is
a fund that has a designated target value, where the payments are not designed so that
an integer number of them will exactly bring the fund to its desired value. This means
that there can be a final payment which is less than a prescribed maximum value. But it
can also happen that, at the time of a payment, the outstanding balance is just slightly
larger than a prescribed maximum payment, so that, after the payment is made, a very
small balance remains. One waits for the next payment, and, during that time, the value
of the fund grows by an accumulation factor, and may exceed the target value. So the
next payment is negative! Read the example. This type of situation can occur in other
ways, so you should be prepared for it, even though it is not the most likely outcome.

A.27.1 §5.4 SINKING FUNDS


Differences between the Amortization and Sinking Fund Methods of repay-
ment In the Amortization Method for repaying a loan, the borrower makes regular
payments — often level payments — directly to the lender. The Outstanding Loan Bal-
ance at any time is then the net amount owing on the lender’s books — either the excess
of the accumulated value of the loan minus the accumulated value of the payments made
Retrospective Method or the present value of the payments yet to be made Prospective
Method .
Where a loan is repaid by a Sinking Fund , portions of the borrower’s payments are
not transmitted to the lender until a later date, usually when they have accumulated in
fund — the Sinking Fund — often at an interest rate different from the rate associated
with the loan. Where the borrower is required or permitted to make regular payments
directly to the lender in addition to those paid into the fund, those are described as
service on the loan; where those regular payments cover the interest costs on the loan,
the principal outstanding will remain constant. This should not be confused with the
Net Amount of the loan, which will be the excess of the outstanding principal over the
accumulated value of the sinking fund.
Where the interest rate associated with the Sinking Fund is the same rate as is being
paid on the loan, the Sinking Fund Method is equivalent to the Amortization Method.
Information for Students in MATH 329 2009 01 2121

1 1
The identity = + i. While the identity can easily be proved algebraically
an sn
(as was done in the lecture), it admits an interesting verbal proof in terms of a loan of
1
1 being repaid over n periods. is the regular payment necessary under an annuity-
an
1
immediate. is the portion of that regular payment that will accumulate in a sinking
sn
fund to a value of 1 just after the nth payment; the complement — i — is the amount
necessary to service the loan annually until the time that the sinking fund matures.
[4, Exercise 17, p. 187], [5, Exercise 20, p. 197] “A has borrowed 10,000 on which
interest is charged at 10% effective. A is accumulating a sinking fund at 8% effective
to repay the loan. At the end of 10 years the balance in the sinking fund is 5000.
At the end of the 11th year A makes a total payment of 1500.
“a) How much of the 1500 pays interest currently on the loan?
“b) How much of the 1500 goes into the sinking fund?
“c) How much of the 1500 should be considered as interest?
“d) How much of the 1500 should be considered as principal?
“e) What is the sinking fund balance at the end of the 11th year?”
Solution: This problem requires attention to the terminology used.
“a) Presumably we are to assume that the borrower is servicing the loan so that
the outstanding balance remains constant. The interest payment necessary at
the end of the 11th year will, therefore, be 0.10 × 10000 = 1000.
“b) The remainder of the payment of 1500 is a contribution of 500 to the sinking
fund.
“c) The net interest paid is the excess of the interest paid — 1000 — over the
8% × 5000 = 400 interest earned by the sinking fund, or 600.
“d) The excess of the contribution over the net interest payment can be assigned
to reducing the principal of the loan. The amount is 1500 − 600 = 900. This
can be considered as made up of two components: 500 which is paid into the
sinking fund, and 8% of 5000, which is 400 — the interest earned by the sinking
fund, and which will ultimately be paid to the lender to retire the loan.
“e) At the end of the 11th year, the sinking fund balance is

5000(1.08) + 500 = 5900 .


Information for Students in MATH 329 2009 01 2122

A.28 Supplementary Notes for the Lecture of March 18th, 2009


Distribution Date: Wednesday, March 18th, 2009
(subject to revision)

A.28.1 §5.4 SINKING FUNDS (conclusion)


[4, Exercise 18, p. 187], [5, Exercise 21, p. 198] “A loan of 1000 is being repaid
with level annual payments of 120 plus a smaller final payment made one year after
the last regular payment. The effective rate of interest is 8%. Show algebraically
and verbally that the outstanding loan balance after the 5th payment has been
made is:
“a) 1000(1.08)5 − 120 · s5
“b) 1000 − 40s5 .”
Solution: I give only the verbal explanations.
“a) Interpret the loan as being amortized by the regular payments and the final drop
payment. The amount of 1000(1.08)5 −120·s5 is that given by the retrospective
method.
“b) Now interpret the loan as being repaid by the sinking fund method. As there
is no formal sinking fund, one may simply view a portion of the payments as
being contributed to a sinking fund in the hands of the lender, where the sinking
fund earns interest at the same rate as the principal. Each annual payment of
120 may be interpreted as being the sum of the service cost of 80, plus a
contribution of 40 to the sinking fund which will repay the loan at maturity.
After the 5th payment the value of the sinking fund is 40 · s5 . The “outstanding
loan balance” will be the excess of the face value of the loan (which has been
serviced annually, so there is no additional accumulation of interest) over the
value of the sinking fund.
[4, Exercise 19, p. 187], [5, Exercise 23, p. 198] “On a loan of 10,000, interest at
9% effective must be paid at the end of each year. The borrower also deposits X
at the beginning of each year into a sinking fund earning 7% effective. At the end
of 10 years the sinking fund is exactly sufficient to pay off the loan. Calculate X.”
Solution: An equation of value at time t = 10 is X · s̈10 7% = 10000, so

10000 × 0.07
X= = 676.4252593 .
((1.07)10 − 1) 1.07

The interest rate of 9% is totally irrelevant.


Information for Students in MATH 329 2009 01 2123

[4, Exercise 20, p. 188], [5, Exercise 24, p. 198] “A borrower is repaying a loan
with 10 annual payments of 1,000. Half of the loan is repaid by the amortiza-
tion method at 5% effective. The other half of the loan is repaid by the sinking
fund method, in which the lender receives 5% effective on the investment and the
sinking fund accumulates at 4% effective. Find the amount of the loan.”
Solution: Let the amount of the loan be L. The amortization of the loan of L2 entails
L 0.025L
an annual payment of = . For the other half of the loan the
2a10 5% 1 − (1.05)−10
borrower must pay interest annually in the amount of 0.05 × L2 = 0.025L. These
two expenses — the amortization of half the loan, and the servicing of the other
half — leave from his annual payment a balance of
0.025L
1000 − − 0.025L
1 − (1.05)−10
L
which must accumulate at 4% in the sinking fund to produce a balance of 2
at
maturity. We have the equation of value
0.025L L 0.02L
1000 − −10
− 0.025L = =
1 − (1.05) 2s10 4% (1.04)10 − 1
40000
⇔ L= 1 0.8 = 7610.479836
1−(1.05)−10
+ 1 + (1.04)10 −1

[4, Exercise 21, p. 188], [5, Exercise 25, p. 198] “A borrows 12,000 for 10 years,
and agrees to make semiannual payments of 1,000. The lender receives 12% con-
vertible semiannually on the investment each year for the first 5 years and 10%
convertible semiannually for the second 5 years. The balance of each payment is
invested in a sinking fund earning 8% convertible semiannually. Find the amount
by which the sinking fund is short of repaying the loan at the end of the 10 years.”
Solution: The interest payments for the first 10 half-years are 6% of 12,000, i.e. 720
per half-year; and, for the second 10 half-years, 600 per half-year. This leaves 280
at the end of each of the first 10 half-years, and 400 at the end of each of the second
10 half-years to accumulate in the sinking fund, which earns 4% effective every half
year. The accumulated balance in the sinking fund at maturity will be
1 ¡ ¡ ¢ ¡ ¢¢
120s10 4% + 280s20 4% = 120 (1.04)10 − 1 + 280 (1.04)20 − 1
0.04
¡ ¢
= 25 120(1.04)10 + 280(1.04)20 − 400
= 9778.594855

implying that the shortfall to repay the loan will be 12, 000 − 9778.59 = 2221.41.
Information for Students in MATH 329 2009 01 2124

[4, Exercise 22, p. 188], [5, Exercise 26, p. 198] 1. “A borrower takes out a loan
of 3000 for 10 years at 8% convertible semiannually. The borrower replaces one-
third of the principal in a sinking fund earning 5% convertible semiannually,
and the other two-thirds in a sinking fund earning 7% convertible semiannually.
Find the total semiannual payment.
2. “Rework (a) if the borrower each year puts one-third of the total sinking fund
deposit into the 5% sinking fund and the other two-thirds into the 7% sinking
fund.
3. “Justify from general reasoning the relative magnitude of the answers to (a)
and (b).”
Solution:
1. The semiannual contribution to the sinking funds is
1000 2000
+
s20 2.5% s20 3.5%
and the semiannual interest payment is 4% of 3, 000, or 120. Hence the total
semiannual payment is
1000 2000 25 70
+ + 120 = + + 120
s20 2.5% s20 3.5% (1.025) − 1 (1.035)20 − 1
20

= 229.8692824
2. Let the total sinking fund deposit be D. Then the equation of value at maturity
is
D 2D
· s20 2.5% + · s20 3.5% = 3000 ,
3 3
implying that
9000
D =
s20 2.5% + 2s20 3.5%
9000
= (1.025)20 −1 (1.035)20 −1
+2·
0.025 0.035
= 109.6170427 ,
so the total semi-annual payment is 109.6170427+120=229.6170427.
3. In the original repayment scheme the portion of the payment contributed to
the 5% sinking fund grows more slowly than that to the 7% fund. Thus, while
the final accumulations in the funds will be in the ratio of 1:2, the proportion
of the contribution to the 5% fund would have been more than 13 . By reducing
that proportion to 31 we increased the interest earned by the fund, so a smaller
total contribution was required for the sinking fund.
Information for Students in MATH 329 2009 01 2125

[4, Exercise 23, p. 188], [5, Exercise 27, p. 198] “A payment of 36,000 is made at
the end of each year for 31 years to repay a loan of 400,000. If the borrower replaces
the capital by means of a sinking fund earning 3% effective, find the effective rate
paid to the lender on the loan.”
Solution: The annual contribution to the sinking fund is
400000 12000
= = 7999.571516.
s31 3% (1.03)31 − 1

Hence the annual interest payment is 36, 000 − 7, 999.57 = 28000.43, i.e., 7% of the
principal of 400,000.
[5, Exercise 28, p. 198] “A 20-year annuity-immediate has a present value of 10,000,
where interest is 8% effective for the first 10 years, and 7% effective for the second
10 years. An investor buys this annuity at a price which, over the entire period,
yields 9% on the purchase price; and, further, allows the replacement of capital by
means of a sinking fund earning 6% for the first 10 years and 5% for the second 10
years. Find an expression for the amount that is placed in the sinking fund each
year.”
Solution: The level annual payments under the annuity will be
10000
.
a10 8% + (1.08)−10 a10 7%

It appears to be intended that the sinking fund payments be level also. If their
value is S, and the purchase price is P , then
¡ ¢
S (1.05)10 s10 6% + s10 5% = P .

The purchase price satisfies the following equation of value at time 0:


µ ¶
10000
−10
− S a20 9% + (1.09)−20 P = P ,
a10 8% + (1.08) a10 7%

which implies that


µ ¶
10000
−S = 0.09P .
a10 8% + (1.08)−10 a10 7%

Solving with the earlier equation yields


10000
S=¡ ¢¡ ¡ ¢¢ .
a10 8% + (1.08) a10 7% 1 + 0.09 (1.05)10 s10 6% + s10 5%
−10
Information for Students in MATH 329 2009 01 2126

[5, Exercise 29, p. 199] “A loan of 1 yields the lender rate i per period for n periods,
while the borrower replaces the capital in a sinking fund earning rate j per period.
Find expressions for the following if 1 ≤ t ≤ n:
1. Periodic interest paid to the lender.
2. Period sinking fund deposit.
3. Interest earned on sinking fund during the tth period.
4. Amount in sinking fund at end of the tth period.
5. Net amount of loan at the end of the tth period.
6. Net interest paid in period t.
7. Principal repaid in period t.
Solution:
1. The yield rate is i, so the lender must be receiving an amount of i each period.
2. The capital is 1, so the borrower is depositing s−1n j regularly into the sinking
fund. (I understand that interest will be paid regularly, and the capital will be
repaid at maturity.)
s
t−1 j
3. At the beginning of the tth period the balance in the sinking fund is sn j
;
s
t−1 j
during the period it earns interest in the amount of j · sn j
, payable at the end
of the period, i.e., at time t.
s
tj
4. The amount in sinking fund at end of the tth period is sn j
.
5. The net amount of loan at the end of the tth period is the excess of 1 over the
s
balance in the sinking fund, i.e., 1 − s t j .
nj
6. The net interest paid in the tth period is the excess of interest paid over interest
s
earned, i.e. i − j · t−1
s
j

nj
7. By 5. above, the change in the amount of the loan between the t − 1th and the
tth payment is
µ ¶ µ ¶
st j st−1 j st j − st−1 j (1 + i)t−1
1− − 1− = =
sn j sn j sn j sn j
.

A.28.2 §5.5 DIFFERING PAYMENT PERIODS AND INTEREST CON-


VERSION PERIODS
Omit this section.
Information for Students in MATH 329 2009 01 2127

A.28.3 §5.6 VARYING SERIES OF PAYMENTS


Omit this section.

A.28.4 §5.7 AMORTIZATION WITH CONTINUOUS PAYMENTS


Omit this section.

A.28.5 §5.8 STEP-RATE AMOUNTS OF PRINCIPAL


Omit this section.
Information for Students in MATH 329 2009 01 2128

A.29 Supplementary Notes for the Lecture of March 20th, 2009


Distribution Date: Friday, March 20th, 2009
(subject to revision)

Textbook Chapter 6. Bonds and other securities.

A.29.1 §6.1 INTRODUCTION


The chapter is concerned with relations between the price of a security and its yield rate,
and with the value of a security at any time after it has been purchased, even at a time
that is not an interest compounding date.

A.29.2 §6.2 TYPES OF SECURITIES


We shall confine our study to bonds, which are a commitment by the issuer to repay
a loan at a particular time, with interest payments according to a prescribed rate and
schedule. Many variations are possible, and this type of instrument is still evolving.
Read the book and become familiar with the following terms concerning bonds:

Definition A.16 • The word bond originally had a much more general meaning: we
are using the word in the sense of a security that commits a borrower to pay one
or more specific sums at specific times, subject to detailed requirements of interest
and/or bonuses.
• The term of the bond is the length of time from the date of issue until the date of
final payment, which date is the maturity date.
• The detailed conditions of the bond may permit the bond to be called at some date
prior to the maturity date, at which time the issuer (=the lender) will repay the
commitment, possibly with some additional amounts. Such a bond is callable.
• While the calling of a bond is at the initiative of the borrower, the lender (=the
purchaser) may possibly have the right to redeem the bond prior to the date of
maturity. Or, he may have some other type of right, e.g., to exchange the bond for
shares of the stock of the issuing company; this is a convertible bond.
• A bond may be supplied with coupons — portions of the paper bond that are to be
cut (=coupes) from the bond and exchanged for interest payments. Nowadays the
coupons may no longer be printed, but the purchaser may receive regular payments
from the lender. This can be the case if the bond is fully registered , so that the
issuer has the coordinates of the purchaser. (A bond could also be registered only
as to principal , in which case the coupons are of the traditional type.)
Information for Students in MATH 329 2009 01 2129

• A coupon bond may be “stripped”, separating the coupons from the commitment
to repay the face value of the bond, following which the two parts may be sold to
separate purchasers.

The preceding is just a brief introduction: this is not the course in which to learn about
the variety of investment vehicles available in today’s financial markets.
Unlike the problems we have been considering in earlier chapters, those here some-
times involve technical definitions that are not necessarily intuitive, and need simply
to be memorized. I will eventually discuss with the class what definitions need to be
absorbed for examination purposes.
[4, Exercise 1, p. 240], [5, Exercise 1, p. 240] “Find the price which should be paid
for a zero coupon bond which matures for 1000 in 10 years to yield:
1. 10% effective
2. 9% effective
3. Thus a 10% reduction in the yield rate causes the price to increase by what
percentage?”
Solution:
1. The bond is now worth 1000(1.10)−10 = 385.5432894.
2. When the interest rate is reduced to 9%, the present value of the bond increases
to 422.4108069.
3. The 10% decrease in the interest rate thereby increases the price by
422.4108069
− 1 = 9.5624846%.
385.5432894
[4, Exercise 2, p. 240], [5, Exercise 2, p. 240] “A 10-year accumulation bond with
an initial par value of 1000 earns interest of 8% compounded semiannually. Find
the price to yield an investor 10% effective.”
Solution:
Definition A.17 [5, p. 205] An accumulation bond is one in which the redemption
price includes the original loan plus all accumulated interest.
Solution: The only return payment is at maturity. The price to yield 10% interest
will therefore be ¡ ¢
(1.10)−10 1000(1.04)20 = 844.7728240.
[4, Exercise 3, p. 240], [5, Exercise 3, p. 240] “A 26-week (U.S.) T(reasury)-bill is
bought for 9,600 at issue, and will mature for 10,000. Find the yield rate computed
as:
Information for Students in MATH 329 2009 01 2130

1. A discount25 rate, using the typical method for counting days on a T-bill26 .
2. An annual effective rate of interest, assuming the investment period is exactly
half a year.”
Solution: Here is an example of a problem requiring some technical preparation.
While we have encountered the use of discount rather than interest in isolated
problems, this is the first time we have met it in a complex transaction.
1. The time is 26 weeks, i.e., 26 × 7 = 182 days, or, under the actual/360 system,
182
of a year. The discount rate will, therefore, be
360
360 400
× = 7.912087912% .
182 10000
2. The effective interest rate for half a year is
400 1
= .
9600 24
The effective rate for a full year will be
µ ¶2
1 1 1
1+ −1= + = 8.506944444% .
24 12 576

25
simple discount
26
i.e., “actual/360” [5, p. 39], using the exact number of days, but assuming 360 days in the year.
Information for Students in MATH 329 2009 01 2131

A.30 Supplementary Notes for the Lecture of March 23rd, 2009


Distribution Date: Monday, March 23rd, 2009
(subject to revision)

A.30.1 §6.3 PRICE OF A BOND


Remember that a bond is essentially a contract to pay a large amount “at maturity”,
and smaller amounts of interest periodically on “coupon dates” until maturity. It differs
from the types of loans we have studied hitherto in that the contract is often (but not
always)27 transferable from one owner to another, and so it is reasonable to investigate
the value of the entire contract under conditions of varying yield.

Further definitions Familiarize yourself with the following terms, defined in the text-
book, and with the symbols usually used for them.
Definition A.18 1. The price P paid for a bond. In practice bond prices are usually
quoted in terms of a bond with face value (see next item) of 100.
2. The par value or face value or face amount. This amount is usually printed in
the bond contract, but may not be the amount paid at maturity. Its function is to
determine, once the coupon rate r has been specified, the magnitude of the coupons.
3. The redemption value C is the amount paid when the bond is redeemed. When a
bond is “redeemable at par”, C = F . Where the redemption value exceeds the face
value, the word premium may be used for the excess; this word premium is also
used to denote the excess of the price paid for a bond over what would have been
the value if the yield rate was the same as the coupon rate.
4. The coupon rate r is the effective rate per coupon payment period, based on which
the amount of the coupon is calculated. The default payment period is a half-year.
5. The amount of a coupon is the product F r.
Fr
6. The modified coupon rate g = is the coupon rate per unit of redemption value,
C
rather than per unit of par value.
7. The yield rate or yield to maturity i is the actual interest rate earned by the investor.
8. The number of coupon payment periods from the date of calculation until maturity
is denoted by n.
27
Of course, the issuer of a security cannot prevent the purchaser from arranging privately to transfer
the proceeds to another person, and to receive payment for that. What the issuer may be able to do is to
restrict the establishment of a secondary market at which the security may be routinely bought and sold,
and which would create a market value for the security. Some securities have restricted transferability
conditions, e.g., only on the death of the owner. And no security is immune from a court order.
Information for Students in MATH 329 2009 01 2132

9. The present value of the redemption value, discounted back to the present by the
yield rate, is denoted by K; so K = C(1 + i)−n .
Fr
10. The base amount G is : the amount which, if invested at the yield rate i, would
i
produce periodic interest payments equal to the coupons.
11. A callable bond is one where the lender has the right to declare that interest pay-
ments will stop and a bond may be redeemed at certain dates before the maturity
date; there could be a premium paid in addition to the redemption value, to en-
courage lenders to cash in the bond.
12. The word discount is often used where we have been using the word premium if the
premium is negative: the discount is the negative of the premium.

Four formulæ for price. We shall consider four different ways of determining the
present value, or price of a bond. The formulae are derivable one from the other: there
are situations where one may be more useful than another for specific applications; as
with many of the other formula we have met, the relative advantages were often linked to
the number of times that tables had to be consulted in computing the bond value; such
distinctions may no longer be significant. The most “basic” of the formulæ computes the
price of a bond as the present value of the coupons, interpreted as an annuity-immediate,
to which is added the present value K of the redemption value C of the bond, the face
value plus any premium that is payable upon redemption. This formula may be applied
either at the maturity date, or, if the bond can be called (by the issuer) or redeemed (by
the purchaser), at some earlier date.

Theorem A.11 (The “Basic” Formula) P = F r · an i + Cv n = F r · an i + K

Theorem A.12 (The Premium/Discount Formula) P = C + (F r − Ci) · an

Proof: This formula may be derived from the preceding by recalling that 1 = v n + i ·
an i . ¤
Fr
The Base Amount G was defined above to be .
i
Theorem A.13 (The Base Amount Formula) P = G + (C − G)v n
g
Theorem A.14 (Makeham’s Formula) P = K + (C − K)
i
There are interesting verbal explanations of the preceding formulæ, to which we may
return.
Information for Students in MATH 329 2009 01 2133

[4, Exercise 5, p. 240], [5, Exercise 8, p. 241] “Two 1000 bonds redeemable at par
at the end of the same period are bought to yield 4% convertible semiannually. One
bond costs 1136.78, and has a coupon rate of 5% payable semiannually. The other
bond has a coupon rate of 2 12 % payable semiannually. Find the price of the second
bond.”
Solution: For the first bond we have F1 = C1 = 1000, i1 = 2%, P1 = 1136.78,
r1 = 2.5%. By the Premium/Discount formula,

1136.78 = 1000 + (25 − 20) · an 2% ,

implying that an = 27.356. For the second bond we have F2 = C2 = 1000, i2 = 2%,
r2 = 1.25%, n2 = n1 ,

P2 = 1000 + (12.5 − 20) · an


= 794.83 .
Information for Students in MATH 329 2009 01 2134

A.31 Supplementary Notes for the Lecture of March 25th, 2009


Distribution Date: Wednesday, March 25th, 2009
(subject to revision)

A.31.1 §6.4 PREMIUM AND DISCOUNT

Premium = P − C = (F r − Ci)an i
Discount = C − P = −(F r − Ci)an i

In practice the word premium is used when P − C > 0, and discount when P − C < 0;
when P = C we speak of a purchase at par .

Book Value For accounting purposes it is necessary to show a gradual progression of


the value of a bond from purchase to maturity that is in some reasonable relationship
with its market value. There are a number of possible methods for doing this in practice,
but we shall adhere to the method whereby the value is shown to be what would be the
price if the yield rate does not change after the purchase of the bond. Of course, the yield
rate could very well change after the purchase, and so this is not a completely realistic
assumption. There are other methods for assigning a value to the bond. In this section
we shall consider the value only at coupon payment dates; generalization to other dates
will be considered in the next section.
When a bond is purchased at a premium28 the value of the redemption value and
the unpaid coupons just after the payment of the tth coupon — i.e., of the book value
Bt — will decrease until the maturity date. Thus the coupons may be interpreted as
consisting of the interest which is earned at the yield rate i and an amount to amortize
the premium. The redemption value C is C = Bn , and the price is P = B0 . We will use
this notation even when the bond is purchased with some of its coupons already paid
(to the vendor). The writing down of the premium may be shown on a schedule like

Coupon Coupon Interest Amount for Amortization Book


Number amount earned of Premium value
0 ... ...
1 Fr
... Fr ... ... ...
The following example illustrates a problem where the price P of a bond is not determined
explicitly until an equation for P is determined and solved.
28
mutatis mutandis, at a discount
Information for Students in MATH 329 2009 01 2135

Example A.15 [4, Example 6.4, p. 212], [5, Example 7.4, p. 219] “Find the price of a
1,000 par value 2-year 8% bond with semi-annual coupons bought to yield 6% convertible
semiannually, if the investor plans to replace the premium by means of a sinking fund
earning 5% convertible semi-annually. (Note: The intention is that this plan for a sinking
fund has been considered in calculating the yield rate of the bond.)
Solution: Note that, following the usual convention, the 8% rate is interpreted as a
nominal interest rate compounded twice a year. The unknown is the price, P . The
coupons each have value 0.04 × 1000 = 40, but the interest earned is 3% of the price P ,
which remains to be determined. The sinking fund is to mature at value P − 1000 after
2 years, i.e., after 4 contributions. Thus an equation of value is

(40 − .03P )s4 2.5% = P − 1000 . (78)

This can be solved for P :


1000 + 40s4 2.5%
P =
1 + 0.03s4 2.5%
= 1, 036.93 .

Note the assumption used in equation (78): all of the coupon that exceeds the interest
“earned” at the yield rate is contributed to the sinking fund; this was not explicitly stated
in the problem, but is the author’s interpretation. If, for example, the purchaser had
decided that he would allocate only half of the excess to his sinking fund contributions,
then the price of the bond would be about 1082.43.
Information for Students in MATH 329 2009 01 2136

A.32 Supplementary Notes for the Lecture of March 27th, 2009


Distribution Date: Friday, March 27th, 2009
(subject to revision)

A.32.1 §6.4 PREMIUM AND DISCOUNT (conclusion)


Example A.16 Let’s return to the Example A.15 on page 2135 of these notes, con-
sidered last day, [4, Example 6.4, p. 212], [5, Example 7.4, p. 219] “Find the price of a
1,000 par value 2-year 8% bond with semi-annual coupons bought to yield 6% convertible
semiannually, if the investor plans to replace the premium by means of a sinking fund
earning 5% convertible semi-annually.” We found that the price of this bond would be
1,036.93. Now let’s modify the problem so that there is no 5% sinking fund. Then, by
the Premium/Discount Formula, the price of the bond would be simply

P = C + (F r − Ci) · a4 3%
1 − (1.03)−4
= 1000 + (40.00 − 30.00) ·
0.03
= 1037.17 .

At first glance this appears to be erroneous: how could the price without the sinking
fund, which is earning an inferior interest rate, be less than the price with the assumption
of the sinking fund? The point is that, when one assumes apriori that part of the coupon
will not be available, but will be trapped in a sinking fund earning an inferior interest
rate for a period of time ranging from 1.5 years to 0 years, then the value of the bond is
less than if the entire coupon was assumed to be readily available, and that all interest
calculations would be made at a rate of 3% per half-year. You can see this, for example,
by tracing the contribution from the first coupon to the sinking fund. The coupon is for
40.00, of which (under the sinking fund assumption) the amount that was “earned” was
only 0.03 × 1, 036.93 = 31.11, so 8.89 was contributed to the sinking fund. That amount
earns interest for 1.5 years at 2.5% per half-year, and grows to 8.89(1.025)3 = 9.57 at
maturity. But, if the prevailing interest rate is to be taken to be 3% per half year, then
the present value of that amount is 8.89(1.025)3 (1.03)−4 = 8.51, and its value on the
date the coupon was received was only 8.89(1.025)3 (1.03)−3 = 8.76, 13 cents less than
the amount deposited in the sinking fund at that time.
For the bond purchased at 1037.17 without the sinking fund, let’s set up an amorti-
zation table at 3% per half-year:
Information for Students in MATH 329 2009 01 2137

Coupon Coupon Interest Amount for Amortization Book


Number amount earned of Premium value
0 1,037.17
1 40.00 31.12 8.88 1,028.29
2 40.00 30.85 9.15 1,019.14
3 40.00 30.57 9.43 1,009.71
4 40.00 30.29 9.71 1,000.00
TOTAL 160.00 122.88 37.16

Before leaving this example, let’s work out an analogous table for the sinking fund
variation considered last day. In this case the premium is not amortized until the date
of the last coupon. We have
Coupon Coupon Interest S. Fund S. Fund Book
Number amount earned Contribution Balance Value
0 1,036.93
1 40.00 31.11 8.89 8.89 1,036.93
2 40.00 31.11 8.89 18.00 1,036.93
3 40.00 31.11 8.89 27.34 1,036.93
4 40.00 31.11 8.89 36.91 1,036.93

where the balance in the Sinking Fund has reduced the book value of the bond to
(approximately) the maturity value.

A.32.2 §6.5 VALUATION BETWEEN COUPON PAYMENT DATES (Omit)


A.32.3 §6.6 DETERMINATION OF YIELD RATES (omit)
A.32.4 §6.7 CALLABLE AND PUTABLE BONDS
[4, Exercise 24, p. 243], [5, Exercises 31, 32, p. 243] “A 1000 par value bond has
8% semiannual coupons, and is callable at the end of the 10th through the 15th
years at par.
1. “Find the price to yield 6% convertible semiannually.
2. “Find the price to yield 10% convertible semiannually.”
3. “If the bond in 2. is actually called at the end of 10 years, find the yield rate.
Solution: The question did not mention the length of term. Does that matter? It
doesn’t matter in the case of 6% yield because, as we will see, the worst value of
the bond is when it is called as early as possible, and that determines the minimum
price to pay to obtain a yield of 6%. However, when the yield is 10% compounded
semi-annually, the value of the bond decreases as the time to redemption increases;
Information for Students in MATH 329 2009 01 2138

so, if the bond is not called, the owner is stuck with a poor investment that he must
keep until it matures, and the price he should pay for the bond depends on knowing
that maturity date.
1. Let n be the coupon number at whose date the bond is called matures. Then,
by the Premium/Discount Formula

P = 1000 + (40 − 30)an 3% , (n = 20, 22, 24, 26, 28, 30).

Without knowing which will be the date of call, we take the worst possible date
in order to minimize the price; since an 3% is an increasing function of n, and
is multiplied by a positive number, 10, we minimize by making n as small as
possible, i.e., 2 × 10 = 20:

P = 1000 + (40 − 30)a20 3% = 1000.00 + 148.78 = 1148.78 .

2. When the semi-annual yield rate is 5%, the multiplier is negative, 40−50 = −10,
and we must choose the largest value of n, i.e., n = 30, for a price of

P = 1000 + (40 − 50)a30 5% = 1000.00 − 153.72 = 846.28 .

3. The yield rate i satisfies the equation

846.28 = 1000 + 1000(0.04 − i)a20 i ,

and can be found by iteration to be 10.52%%.


Information for Students in MATH 329 2009 01 2139

A.33 Supplementary Notes for the Lecture of March 30th, 2009


Distribution Date: Monday, March 30th, 2009
(subject to revision)

A.33.1 §6.7 CALLABLE AND PUTABLE BONDS (continued)


Example A.17 [4, Example 6.8, p. 225][5, Example 7.9, p. 231] “Consider a $100 par
value 4% bond with semiannual coupons callable at $109 on any coupon date starting 5
years after issue for the next 5 years, at $104.50 starting 10 years after issue for the next
5 years, and maturing at $100 at the end of 15 years. What is the highest price which
an investor can pay and still be certain of a yield of: (1) 5% convertible semiannually,
and (2) 3% convertible semiannually?”
Solution: I recommend that you use the Premium/Discount Formula, and remember
that the function an i is an increasing function of n; this means that, in a problem of this
type, the values least favourable to the investor will always be at the end of an interval
of payment periods: at the lower end when F r − Ci is positive, and at the upper end
when F r − Ci is negative. The intervals to be considered will be consecutive payment
dates when the call premium has the same value. Each of these intervals need to be
treated separately, and then the least favourable from each interval taken together and
the least favourable of them determined. The previous edition of the textbook stated, in
connection with the part of the problem where the yield rate is 5%, that “it is immediately
clear that the latest possible redemption date is less favourable to an investor, since the
bond will be selling at a discount.” When I was using that version in my course, I felt the
statement may not have been clear, and offered the following explanation; the wording
has changed in the current edition, although it may not be any more transparent. The
factor that determines which end of the interval is least favourable is F r − Ci = C(g − i),
where g is the modified coupon rate (cf. Definition A.18.4, p. 2131). The given data lead
to finding the minimum of the following three values:

100 + (2 − 2.50)a30 0.025 = 89.53


109 + (2 − 2.725)a29 0.025 = 94.17
104.50 + (2 − 2.6125)a19 0.025 = 95.33

And, indeed, the minimum will be when n = 30. The situation is best viewed through
the Base amount formula:
P = G + (C − G)v n
Fr
where G = i
(cf. Definition A.18.10). Now v n is a decreasing function of n, and the
Fr
sign of C − G = C − is +: the factor v n is smallest when n is as large as possible;
i
and, because the values at which the bond is called or redeemed are not increasing with
Information for Students in MATH 329 2009 01 2140

time, the factor C − G is also non-increasing. Thus, in this particular case, the bond
will be worth least if it is not called. Thus we did not need to evaluate the prices for the
bond being called when n = 19, 29, as they could not be less than the price for n = 30.
When the yield rate is 3% there is no such claim of an obvious solution. In this case
the Premium/Discount Formula gives prices

109.00 + (2.0000 − 1.6350)an 0.015 for n = 10, . . . , 19


104.50 + (2.0000 − 1.5675)an 0.015 for n = 20, . . . , 29
100.00 + (2.0000 − 1.5000)an 0.015 for n = 30.

We must consider the smallest value of n for each of the intervals associated with the
different redemption values, i.e.,

109.00 + (2.0000 − 1.6350)a10 0.015 = 112.37


104.50 + (2.0000 − 1.5675)a20 0.015 = 111.93
100.00 + (2.0000 − 1.5000)a30 0.015 = 112.01 ,

so, in this case the lowers price occurs for n = 20.

[4, Exercise 25, p. 243], [5, Exercise 33, p. 243] “A 1,000 par value 8% bond with
quarterly coupons is callable five years after issue. The bond matures for 1,000 at
the end of 10 years, and is sold to yield a nominal rate of 6% convertible quarterly,
under the assumption that the bond will not be called. Find the redemption value
at the end of 5 years that will provide the purchaser the same yield rate.”
Solution: If the bond is certain not to be called, its present value (i.e., its price) is

1000 + (20 − 15)a40 1.5% = 1000 + 5(29.9258) = 1149.63.

Let x denote the redemption value after 5 years that will provide the same yield
rate. Then
1149.63 = x + (20 − 0.015(x))a20 1.5%
which implies that
1149.63 − 20a20 1.5%
x= = 1085.91 .
1 − 0.05a20 1.5%
[4, Exercise 26, p. 243], [5, Exercise 34, p. 243] “A 1000 par value 4% bond with
semiannual coupons matures at the end of 10 years. The bond is callable at 1050
at the ends of years 4 through 6, at 1025 at the ends of years 7 through 9, and at
1000 at the end of year 10. Find the maximum price that an investor can pay and
still be certain of a yield rate of 5% convertible semiannually.”
Information for Students in MATH 329 2009 01 2141

Solution: We have to find the minimum of the following prices based on the given
call dates and premiums:

1050.00 + (20.000 − 26.250)an 2.5% (n = 8, 10, 12) (79)


1025.00 + (20.000 − 25.625)an 2.5% (n = 14, 16, 18) (80)
1000.00 + (20.000 − 25.000)a20 2.5% . (81)

In each case the coefficient of an 2.5% is negative, so the lowest value will be when n
is as large as possible; that is, we have to compare the following three amounts

1050.00 + (20.000 − 26.250)a12 2.5% = 1050 − (6.250)10.2578 = 985.89


1025.00 + (20.000 − 25.625)a18 2.5% = 1025 − (5.625)14.3534 = 944.26
1000.00 + (20.000 − 25.000)a20 2.5% = 1000 − (5.000)15.5892 = 922.05 ,

whose minimum is the last, the price of the bond if not called before maturity. That
is the highest price the investor may pay if she wishes to be sure that the yield will
not be less than 5% convertible semiannually.
[5, Exercise 35, p. 243] “A 1,000 par value 6% bond with semiannual coupons is
callable at par 5 years after issue. It is sold to yield 7% under the assumption
that the bond will be called. The bond is not called, and it matures at the end
of 10 years. The bond issuer redeems the bond for 1000 + X without altering the
buyer’s yield rate of 7% convertible semiannually. Find X.”
Solution: Under the assumption that the bond will be called at par 5 years after
issue, its price, when yielding 3.5% effective semi-annually, would be

1000 + (30 − 35)a10 3.5% = 958.42 .

The premium upon maturity will be given by the equation

958.42 = 1000 + X + (30 − (0.035)(1000 + X))a20 3.5% .

which implies that


958.42 − 30a20 3.5%
X= − 1000 = 58.66 .
1 − 0.035a20 3.5%

Textbook Chapter 7. Yield Rates.


Information for Students in MATH 329 2009 01 2142

A.33.2 §7.1 INTRODUCTION (omit)


A.33.3 §7.2 DISCOUNTED CASH FLOW ANALYSIS (omit)
A.33.4 §7.3 UNIQUENESS OF THE YIELD RATE (part)
Definition A.19 The yield rate is that rate of interest at which the present value of
returns from the investment is equal to the present value of contributions into the in-
vestment.

Finding the yield rate may require the use of various approximation methods, since the
equations that have to be solved may be polynomial of high degree.

Example A.18 [4, p. 255] [5, p. 133] A person makes payments of 100 immediately and
132 at the end of 2 years, in exchange for a payment in return of 230 at the end of 1
year. The yield rate i can be shown to satisfy the equation

((1 + i) − 1.1)((1 + i) − 1.2) = 0

which has two distinct solutions.

Example A.19 [4, Example 7.3, p. 258] “A is able to borrow 1000 from B for 1 year at
8% effective, and to lend it to C for 1 year at 10% effective. What is A’s yield rate on
this transaction.”
Solution: The equation of value at time 0 is

1000 − v(1080) = 1000 − v(1100) ,

which has no finite solution. We can say that the yield rate is infinite.

Read [4, Example 7.4, p. 258], in which the borrower cannot possibly earn sufficient
interest to cover her payments; in this case we might wish to speak of an “imaginary”
yield rate.
Information for Students in MATH 329 2009 01 2143

A.34 Supplementary Notes for the Lecture of April 01st, 2009


Distribution Date: Wednesday, April 01st, 2009
(subject to revision)

A.34.1 §7.4 REINVESTMENT RATES


Single payment with interest reinvested Suppose 1 is invested at time 0, with
interest being paid at rate i at the ends of n years, but where the interest can be
reinvested only at rate j. At the end of n years the total accumulated value of the
investment is
1 + isn j

Annuity-immediate at rate i, with reinvestment at rate j If an annuity-immediate


of 1 per period for n periods pays interest at rate i, but the interest can be reinvested
only at rate j, the accumulated value at time n is
i
n + i · (Is)n−1 j = n + · (sn j − n)
j
[4, Exercise 11, p. 301], [5, Exercise 10, p. 161] “It is desired to accumulate a fund
of 1,000 at the end of 10 years by equal deposits at the beginning of each year. If
the deposits earn interest at 8% effective, but the interest can only be reinvested at
only 4% effective, show that the deposit necessary is
1000
.
2s11 0.04 − 12

Solution: Let x denote the necessary deposit. We will sum the principal payments
(x×10) and treat the interest payments as forming an increasing annuity-immediate
with increments of 0.08x.
¡ ¢
x 10 + 0.08 · (Is)10 4% = 1000
µ ¶
s̈10 4% − 10
⇔ x 10 + 0.08 · = 1000
0.04
1000
⇔ x= . ¤
2s11 0.04 − 12

[4, Exercise 12, p. 301][5, Exercise 11, p. 161] “A loan of 10,000 is being repaid
with payments of 1,000 at the end of each year for 20 years. If each payment is
immediately reinvested at 5% effective, find the effective annual rate of interest
earned over the 20-year period.”
Information for Students in MATH 329 2009 01 2144

Solution: Let the effective yield rate be i. The payments do not become available
until the maturity date, after 20 years. Until that time they are locked into a
payment-scheme that accumulates to value 1000s20 5% . We are asked for the interest
rate that was earned. There are thus just two transactions: the loan at time 0, in
the amount of 10,000, and the repayment at time 20, in the amount given above.
The equation of value at time t = 0 is
1000s20 5% (1 + i)−20 = 10000
10(0.05)
⇔ (1 + i)−20 =
(1.05)20 − 1
⇔ i = 6.1619905%.

[4, Exercise 13, p. 301], [5, Exercise 12, p. 161] “An investor purchases a 5-year
financial instrument having the following features:
“(i) The investor receives payments of 1000 at the end of each year for 5 years.
“(ii) These payments earn interest at an effective rate of 4% per annum. At the end
of the year, this interest is reinvested at the effective rate of 3% per annum.
“Find the purchase price to the investor to produce a yield rate of 4%.”
Solution: To determine the yield we need to consider when the payment is finally
released to the investor. The payments of 1000 are to be invested at 4%; they will
generate an increasing annuity whose payments start at 40 at the end of year 2, up
to 160 at the end of year 5. But they are not released to the investor; rather, they
earn interest at 3%. At the end of 5 years — and only then — the investor receives
5(1000) + 40(Is)4 3%
and these amounts have to be discounted to the present at 4%, giving a present
value of
¡ ¢ 40 ¡ ¢
(1.04)−5 5(1000) + 40(Is)4 3% = 5000 + s5 3% − 5
0.03
µ µ ¶¶
−5 40 (1.03)5 − 1
= (1.04) 5000 + −5
0.03 0.03
= 4448.418326
if the yield is to be 4%.
[5, Exercise 13, p. 161] “An investor deposits 1,000 at the beginning of each year
for five years in a fund earning 5% effective. The interest from this fund can be
reinvested at only 4% effective. Show that the total accumulated value at the end
of ten years is ¡ ¢
1250 s11 0.04 − s6 0.04 − 1 .”
Information for Students in MATH 329 2009 01 2145

¡ ¢
Solution: The 5 deposits of 1000 would be worth 1000 s̈10 4% − s̈5 4% at the end of
10 years if they were earning interest together with the reinvested annual interest
payments. But they are locked into a fund where they earn 5%, and are not released
until time 10, still worth 5,000. The interest payments constitute an increasing
annuity to the investor, beginning with 50 at time 1, increasing to 250 at time 5,
and then remaining constant until time 10. They are available to the investor as
they
¡ are paid, but she¢reinvests them at 4%. Their value at time 10 is, therefore
50 (Is)10 4% − (Is)5 4% . Summing yields
¡ ¢ 50 ¡ ¢
5000 + 50 (Is)10 4% − (Is)5 4% = 5000 + s11 4% − 11 − s6 4% + 6
0.04 ¡ ¢
= (5000 − 6250) + 1250 s11 4% − s6 4%
¡ ¢
= 1250 s11 0.04 − s6 0.04 − 1
= 7316.719914

Using the tables in the textbook, we would find the answer to be


¡ ¢
1250 s11 0.04 − s6 0.04 − 1 = 1250(13.4864 − 6.6330 − 1)
= 7316.7500 .

[5, Exercise 14, p. 161] “A invests 2,000 at an effective interest rate of 17% for 10
years. Interest is payable annually and is reinvested at an effective rate of 11%. At
the end of 10 years the accumulated interest is 5,685.48. B invests 150 at the end
of each year for 20 years at an effective interest rate of 14%. Interest is payable
annually and is reinvested at an effective rate of 11%. Find B’s accumulated interest
at the end of 20 years.”
Solution: A’s interest payments begin at the end of year 1, in the amount of 17% ×
2000 = 340 and continue at the same level for 10 years. The equation of value at
the end of year 10 is
340s10 11% = 5685.48 ,
implying that
5685.48
s10 11% = = 16.722.
340
It follows that (1.11)10 = (16.722)(0.11) + 1 = 2.83942, and that
(1.11)20 − 1
s20 11% =
0.11¡ ¢
= s10 11% · (1.11)10 + 1
¡ ¢
= s10 11% · s10 11% + 2
= 16.722 × 3.83942 = 64.20278124.
Information for Students in MATH 329 2009 01 2146

B’s interest payments constitute an increasing annuity-immediate whose first pay-


ment is at the end of year 2, in the amount of 21. The accumulated value of B’s
accumulated interest is
s20 11% − 20
21(Is)19 11% = 21
0.11
64.20278124 − 20
= 21 ·
0.11
= 8438.712782.

A.34.2 §7.5 INTEREST MEASUREMENT OF A FUND (Omit)


A.34.3 §7.6 TIME-WEIGHTED RATES OF INTEREST (Omit)
A.34.4 §7.7 PORTFOLIO METHODS AND INVESTMENT YEAR METH-
ODS (Omit)
A.34.5 §7.8 SHORT SALES (Omit)
A.34.6 §7.9 CAPITAL BUDGETING - BASIC TECHNIQUES (Omit)
A.34.7 §7.10 CAPITAL BUDGETING - OTHER TECHNIQUES (Omit)
A.34.8 Appendix 7 (Omit)
Information for Students in MATH 329 2009 01 2147

A.35 Supplementary Notes for the Lecture of April 03rd, 2009


Distribution Date: Friday, April 03rd, 2009
(subject to revision)

This hour was devoted to discussion of Problems ##1,2(a) on the Final Examination in
MATH 329 2005 01, included without solutions in these notes, beginning on page 3145.
Solutions will not be reproduced in these notes.

A.36 Supplementary Notes for the Lecture of April 06th, 2009


Distribution Date: Monday, April 06th, 2009
(subject to revision)

This hour was devoted to discussion of Problems ##3,6 on the Final Examination in
MATH 329 2005 01, included without solutions in these notes, beginning on page 3145.
Solutions will not be reproduced in these notes.

A.37 Supplementary Notes for the Lecture of April 08th, 2009


Distribution Date: Wednesday, April 08th, 2009
(subject to revision)

This hour was devoted to discussion of Problems on the Final Examination in MATH
329 2005 01 and on another final examination, included without solutions in these notes,
beginning on page 3145. Solutions will not be reproduced in these notes.

A.38 Supplementary Notes for the Lecture of April 14th, 2009


Distribution Date: Tuesday, April 14th, 2009
(subject to revision)

I was asked by a student to discuss an example on sinking funds. I chose to discuss the
following example from the textbook.

Example A.20 [4, Example 6.4, p. 212]. “Find the price of a 1,000 par value 2-year
8% bond with semi-annual coupons, bought to yield 6% convertible semi-annually, if the
investor can replace the premium by means of a sinking fund earning 5% convertible
semi-annually.”
Solution: In this course we have encountered certain linguistic conventions. Here we
possibly have one in the intention the textbook attaches to the word “can”: the author’s
Information for Students in MATH 329 2009 01 2148

interpretation is that the “replacement of the principal” must be undertaken in the


manner described. The purchaser is, by means of the sinking fund, going to convert
his investment into one that emulates a bond which is purchased at the price he is
paying, but will mature at precisely that amount, and which, in addition, will be paying
semi-annual amounts that could be interpreted as 3% coupons. Thus the word “can” is
being interpreted as meaning that the vendor is pricing his bond so that the purchaser
is permitted to invest part of the coupon monies into a lower-paying sinking fund: the
purchaser is subsidizing this “poor” investment by charging less for the bond.
Had the purchaser not been permitted to invest in this way, the price would have had
to be [4, Table 6.2, p. 209]
1, 037.17 = 100 + (40 − 30)a4 3% = 1037.171 ,
by the Premium/Discount Formula.
Denote the price paid under the present scenario by P . After investing P the pur-
chaser will be receiving semi-annual payments of 40, and a final payment of 1,000; but
we are obliged to supplement that final payment by the proceeds of a sinking fund whose
maturity value will be P −1, 000; semi-annual level payments into that sinking fund have
1000 − P
to be in the amount of , and these amounts have only one possible source —
s4 2.5%
they are deducted from the regular semi-annual interest payments of 40. The purchaser
is obliged to interpret the investment as one of P , at a yield rate of 3%. The premium
paid by the purchaser is P − 1, 000. The solution of the textbook is based on solving the
equation
(40 − .03P )s4 0.025 = P − 1000 ,
which yields P = 1, 036.93.
A more naive solution could proceed as follows. From each semi-annual coupons of
1000 − P
40, the balance left after the required payment into the sinking fund is 40 − .
s4 2.5%
An equation of value at time 0 is, therefore
µ ¶
−4 −4 1000 − P
1000(1.03) + (P − 1000)(1.03) 40 − a4 3% = P
s4 2.5%
µ ¶
1000 − P ¡ ¢
⇔ 40 − a4 3% = P 1 − (1.03)−4
s4 2.5%
µ ¶
1000 − P
⇔ 40 − a4 3% = P (0.03)a4 0.03
s4 2.5%
1000 − P
⇔ 40 − = P (0.03) ,
s4 2.5%
which is equivalent to the textbook equation.
Information for Students in MATH 329 2009 01 3001

B Problem Assignments, Tests, and Examinations


from Previous Years
B.1 2002/2003
B.1.1 First 2002/2003 Problem Assignment, with Solutions
In all of the following problems students were are expected to show your work.
1. (a) What principal will earn interest of 100 in 7 years at a simple interest rate of
6%?
(b) What simple interest rate is necessary for 10,000 to earn 100 interest in 15
months?
(c) How long will it take for money to double at a simple interest rate of 8%?
(d) For the rate stated and the period of time computed in the previous part of the
question, what would 1 grow to if interest were compounded annually?
Solution:
(a) Let P denote the unknown principal. Equating the interest earned, P · (0.06) · 7
100
to 100 and solving for P , we obtain that P = 7×0.06 = 238.10.
(b) If the interest rate is i, the amount of interest earned will be i · 15
12
· 10000 = 100.
Solving yields i = 54 · 10000
100
= 0.008 or 0.8%.
(c) Let the number of years for money to double be denoted by t. We solve 1 +
(0.08)t = 2: t = 2−1
0.08
= 12.5. Money doubles in 12 12 years.
(d) (1.08)12.5 = 2.62.
2. The total amount of a loan to which interest has been added is 20,000. The term
of the loan was four and one-half years.
(a) If money accumulated at simple interest at a rate of 6%, what was the amount
of the loan?
(b) If the nominal annual rate of interest was 6% and interest was compounded
semi-annually, what was the amount of the loan?
(c) If the rate of interest was 6%, interest was compounded annually for full years,
but simple interest was paid for the last half-year, what was the amount of the
loan?
(d) If the rate of interest was 6%, interest was compounded annually for full and
part years, what was the amount of the loan?
(e) If the effective annual rate of interest was 6%, but interest was compounded
semiannually, what was the amount of the loan?
Information for Students in MATH 329 2009 01 3002

(f) If the nominal annual rate of interest was 6%, but interest was compounded
continuously, what was the amount of the loan?
(g) If interest was compounded continuously, and the force of interest was 6%, what
was the amount of the loan?
Solution:
(a) If the amount of the loan is P , then 20000 = (1 + 0.06 × 4.5)P = 1.27P , so
P = 20000
1.27
= 15, 748.03.
(b) If the amount of the loan is P , then 20000 = (1 + 0.06
2
)2×4.5 P ,
P = 20000(1.03)−9 = 15, 328.33 .
¡ 0.06
¢
(c) If the amount of the loan is P , then 20000 = (1 + 0.06)4 · 1 + 2
P,
P = 20000(1.06)−4 (1.03)−1 = 15, 380.46 . (82)

(d) If the amount of the loan is P , then 20000 = (1+0.06)4.5 P , P = 20000(1.06)−4.5 =


15, 386.99.
(e) If the semi-annual rate of interest is denoted by i, then (1+i)2 = 1.06, so 1+i =
1
(1.06) 2 . If the amount of the loan is P , then 20000 = (1 + i)9 P = (1.06)4.5 P ,
so P = 20000 × (1.06)−4.5 = 15, 386.99.
Note that this is exactly the same principal as in the preceding version of the
problem, since it is precisely the same problem! Note also that the amount of
the principal is slightly more than in the version in part 2c, since simple interest
for a fraction of a year in that case will have a higher yield than compound
interest in this case.
(f) We are told that the nominal rate of interest, compounded instantaneously, is
6%; thus, if i is the effective annual rate, 0.06 = δ = ln(1 + i); equivalently,
e0.06 = 1 + i, so i = 6.18365%. In a full year √ an amount of 1 will grow by a
factor 1.0618365; in a half year, by a factor 1.0618365. In 4 12 years we have
P (1.0618365)4.5 = 20000, so P = 20000(1.0618365)−4.5 = 20000(0.763379) =
15267.59.
(g) The force of interest is the nominal rate of interest which is convertible contin-
uously [9, p. 17]. We are told that interest was compounded continuously. The
effective annual rate of interest, which we shall denote by i, has the property
that 0.06 = ln(1 + i), or that 1 + i = e0.06 . If the amount of the loan is P , then
20000 = (1 + i)4.5 P = e4.5×0.06 = e0.27 P , so P = 20000e−0.27 = 15267.59, the
same result as in the preceding part.
3. (cf. [9, Exercise 1-13, p. 24]) Henry plans to have an investment of 10,000 on January
1, 2006, at a compound annual rate of discount d = 0.11.
Information for Students in MATH 329 2009 01 3003

(a) Find the value that he would have to invest on January 1, 2003.
(b) Find the value of i corresponding to d.
(c) Using your answer to part (b), rework part (a) using i instead of d. Do you get
the same answer?
Solution:
(a) The accumulation of 10,000 will have to be discounted by a factor of (1 − 0.11)
three times to reduce it by compound discount to January 1, 2003. The amount
to be invested is, accordingly, 10000(1 − 0.11)3 = 7049.69.
(b) The relationship between d and i is given, for example, by (1 + i)(1 − d) = 1,
d
which implies that i = 1−d . Here

0.11 11
i= = = 0.1233596.. = 12.36..%.
1 − 0.11 89
1 1
(c) When i = 12.36%, v = 1+i = 1.1236 = 0.89. The value on January 1, 2003 of
the 10,000 expected on January 1, 2006 will then be 10000(0.89)3 = 7049.69,
as before.
4. (cf. [9, Exercise 1-24, p. 26]) Recall that (cf. [9, (1.21)])
· ¸m · ¸−m
i(m) 1 d(m)
1+ =1+i= = 1− . (83)
m 1−d m

(a) Determine whether there is an integer n such that


i(2)
i(n) 1+ 2
1+ = (84)
n 1+ i(3)
3

and, if there is such an integer, find it.


(b) Replace the right member of (84) by a product
µ ¶µ ¶
i(2) d(3)
1+ 1−
2 3
(n)
and then interpret this product verbally to show that it must be equal to 1+ i n
if a suitable n exists.
Solution:
(a) i(2) is the nominal annual interest rate which, when compounded semi-annually,
(2) √ (3)
yields an effective annual rate of i. Thus 1 + i 2 = 1 + i; similarly, 1 + i 3 =
Information for Students in MATH 329 2009 01 3004

√ (2)
1+ i 2 1 1 1
3
1 + i. The ratio (3) is, therefore, equal to (1 + i) 2 − 3 = (1 + i) 6 , which is
1+ i 3
1
the accumulation of 1 after a period of 6
of a year; this is, by definition, equal
(6)
to 1 + i 6 .
³ ´
i(2)
(b) Under an effective annual interest rate of i, the factor 1 + 2
is the value
12 12
of 1 after = 6 months. If this amount
2 ³ is´discounted back 4 months, it 3
=
d(3)
decreases by a reduction factor of 1 − 3 . The result is equivalent to a net
1
accumulation period of 6 − 4 = 2 months, i.e. 6
of a year, under which it would
(6)
grow by a factor 1 + i 6 .
5. (cf. [9, Exercise 1-30, p. 27]) Show that f (t) = (1 + i)t − (1 + it) is minimized at
t = ln i−ln
δ
δ
.
Solution:
If only elementary calculus is used, this problem is more difficult than
it looks. Students were accorded a full grade for showing that the point
claimed is, indeed, a local minimum; the proof that it is a global = abso-
lute minimum, is more difficult; one possible solution is given below. No
attempt has been made to produce a compact solution.
Applying elementary calculus, we find that

f 0 = (1 + i)t ln(1 + i) − i = (1 + i)t δ − i


f 00 = (1 + i)t δ 2

To find the critical points of the function, we solve for t the equation f 0 (t) = 0.
Taking natural logarithms yields

t ln(1 + i) + ln δ = ln i

which is satisfied only for

ln i − ln δ ln i
t0 = = δ. (85)
δ δ
The second derivative, f 00 (t0 ) is positive everywhere, since it is the product of an
exponential — always positive — and the square of a real number; this tells us that
the point t = t0 (85) is a local minimum.
Does this completely solve the problem? Not yet! To solve an extremum problem
we need to interpret local extremum information with reference to the domain of
the function. For example, if the domain is infinite, then the function might not
Information for Students in MATH 329 2009 01 3005

even have a global or absolute minimum, even though it has a local minimum.29
And, if the domain of the function is a closed interval, we need to investigate the
behavior at the end points of that interval. The function f is meaningful for all real
values of t. One interpretation would be to take the domain to be t ≥ 0; another
interpretation would be to take the domain to be −∞ ≤ t ≤ +∞.
What follows is just one possible way of completing this problem. We observe that
f (0) = f (1) = 0. Could f (t) = 0 for t different from 0, 1? Rolle’s theorem implies
the existence of a point with zero slope between any two zeros of the function; as we
have seen that there is only one such point with zero slope, there cannot exist more
than two zeros of the function: and thus the point t0 is the only local extremum.
Thus, by the Intermediate Value Theorem, f has the same sign throughout each
of the intervals −∞ < t < 0, 0 < t < 1, 1 < t. As t → ∞, lim f (t) → ∞; hence
f (t) > 0 for all t > 1; as t → −∞, lim f (t) → ∞; so the function is positive
in the interval −∞ < t < 0 also. Thus the global minimum is in the interval
0 ≤ t ≤ 1; and, from our investigation of the critical point, we know that the
minimum is attained at one (or more) of t = 0, t = 1 or t = t0 . We can complete
this investigation if we can argue that f (t0 ) < 0. As we know the sign of the function
will be the same throughout the interval 0 < t < 1, we can take any convenient
value of t in that interval.
µ ¶ µ ¶
1 √ i
f = 1+i− 1+
2 2
¡√ ¢ ¡√ ¢
1 + i − (1 + 2i ) · 1 + i + (1 + 2i )
= √
1 + i + (1 + 2i )
³ ´
i2
(1 + i) − 1 + i + 4
= √ ¡ ¢
1 + i + 1 + 2i
i2 1
= − ·√ ¡ ¢ <0
4 1 + i + 1 + 2i

Thus the global minimum is attained at t0 .


6. (cf. [9, Exercise 1-33, p. 27]) Find the accumulation function a(t) if it is known that
δt = 0.04(1 + t)−1 for t > 0.
Solution: Applying definition [9, (1.28), p. 19], we solve the differential equation

d
ln(a(t)) = 0.04(1 + t)−1 :
dt
29
Consider, for example, the function t3 − t, which has a local minimum at t = 1, a local maximum at
t = −1, but has neither a global maximum nor a global minimum over its entire domain −∞ < t < +∞.
Information for Students in MATH 329 2009 01 3006

Integration gives
Z
0.04
ln(a(t)) = dt = 0.04 ln(1 + t) + C
1+t
where C is the constant of integration. Setting t = 0, where we know, by definition,
that a(0) = 1, we have
0 = ln 1 = 0.04 ln 1 + C
so C = 0, and
¡ ¢0.04
a(t) = e0.04 ln(1+t) = eln(1+t) = (1 + t)0.04 .
7. Let φ(λ) denote the value of 1 at the end of 3 years, accumulated at an effective
rate of interest λ; let ψ(λ) denote the present value of 1, to be paid at the end of 3
years at an effective rate of discount numerically equal to λ. Suppose it is known
that φ(λ) + ψ(λ) = 2.0294. Determine λ.
Solution: (cf. [6, Exercise 52, p. 30]) φ(λ) = (1 + λ)3 ; ψ(λ) = (1 − λ)3 . Summing
2
yields φ(λ)
q+ ψ(λ) = 2 + 6λ , which we equate to 2.0294, and from which we infer
0.0294
that λ = 6
= .07 = 7%.
8. Showing your work, determine a formula — in terms of the force of interest, δ, for
the number of years that are needed for a sum of money to double itself. Verify
your answer by determining the value of δ when the annual interest rate is 100%.
Solution: Let the number of years needed be t, the interest rate be i, and the force
of interest δ. We solve the equation (1 + i)t = 2 by taking logarithms of both sides:
t ln(1 + i) = ln 2, so
ln 2 ln 2
t= = .
ln(1 + i) δ
When the interest rate is 100% money doubles in one year; here δ = ln 2.

B.1.2 Second 2002/2003 Problem Assignment, with Solutions


1. (cf. Exercise 2-5, p. 36) A vendor has three offers for a house:
(a) three equal payments — one now, one 1 year from now, and the other 2 years
from now;
(b) a single cash payment now of 120,000;
(c) two payments, 45,000 a year from now, and 90,000 two years from now.
He makes the remark that “one offer is just as good as another”. Determine the
interest rate and the sizes of the equal payments that will make this statement
correct.
Information for Students in MATH 329 2009 01 3007

Solution: Let the interest rate be i, and the equal payments be k. Equating the
present value of the payments of 45,000 and 90,000 to 120,000 yields

45000v + 90000v 2 = 120000

which we solve for v, obtaining


q
− 12 ± 1
4
+ 16
3
v =
√2
−1 ± 22.33333
=
4
The lower sign yields a negative value of v, and so that solution is extraneous. We
obtain v = 0.931454, so i = 0.0736 = 7.36%.
The present value of the equal payments is then k(1+v +v 2 ) = 2.79906k = 120, 000,
so the equal payments will each be 42,871.53.
2. (cf. [9, Exercise 2-9, p. 37]) Fund A accumulates at 9% effective, and Fund B at 8%
effective. At the end of 12 years the total of the two funds is 50,000. At the end of
6 years the amount in Fund B is 4 times that in Fund A. How much is in Fund A
after 15 years?
Solution: Let a and b denote the initial amounts in funds A and B. We have two
constraints relating a and b:

a(1.09)12 + b(1.08)12 = 50000


b(1.08)6 = 4a(1.09)6

From the second of these we can determine the relative sizes of a and b; substituting
in the first equation and solving gives
50000
a = ¡ ¢6
(1.09)12 + 4 1.09
1.08
(1.08)12
50000
= = 3, 715.25.
(1.09) ((1.09)6 + 4(1.08)6 )
6

¡ ¢6
Hence b = 4 109
108
a = 15, 705.96. The value of Fund A after 15 years is, therefore,
15
3, 715.25(1.09) = 13, 532.73.
3. The initial balance in an investment fund was 100,000. At the end of 3 months it
had increased to 105,000; at that time 25,000 was added to the fund. Six months
later the fund had increased to 143,000, and this time 30,000 was removed. Finally,
Information for Students in MATH 329 2009 01 3008

at the end of a year, the fund had a balance of 120,000. What was the time-weighted
rate of return?
Solution: [8, Example 2.3.2, p. 39] The balances and withdrawals are respectively
B0 = 100, 000, B1 = 105, 000, B2 = 143, 000, B3 = 120, 000; W0 = 0, W1 = 25, 000,
W2 = −30, 000. Hence the rates of interest in the successive time periods are given
by
B1 105, 000
1 + i1 = = = 1.05
B0 + W 0 100, 000 + 0
B2 143, 000
1 + i2 = = = 1.10
B1 + W 1 105, 000 + 25, 000
B3 120, 000
1 + i3 = = = 1.062
B2 + W 2 143, 000 − 30, 000
The time-weighted rate of return i is, by definition, given by the product
1 + i = (1.05)(1.10)(1.062) = 1.227
so i = 22.7%.
4. (cf. [9, Exercise 2-15, p. 38]) A trust company pays 5% effective on deposits at the
end of each year. At the end of every 3 years a 2% bonus is paid on the balance at
the time. Find the effective rate of interest earned by an investor if she leaves her
money on deposit
(a) for 2 years;
(b) for 3 years (until after the bonus payment is made);
(c) for 4 years;
(d) forever — take a limit!
Solution:
(a) Since there are no bonus payments, the effective rate of interest is 5%.
(b) A deposit of 1 grows to 1.05 at the end of the first year, (1.05)2 at the end of
the second year, and, after the bonus, (1.05)3 (1.02) at the end of the 3rd year.
If the effective rate of interest is i, then we must solve the equation
(1 + i)3 = (1.05)3 (1.02) .
Taking logarithms, we obtain 3 ln(1 + i) = 3 ln(1.05) + ln(1.02), so
1
ln(1 + i) = (3 ln(1.05) + ln(1.02))
3
1
1 + i = e 3 (3 ln(1.05)+ln(1.02))
1
i = e 3 (3 ln(1.05)+ln(1.02)) − 1
√3
= 1.05 1.02 − 1 = .056953846 = 5.70%.
Information for Students in MATH 329 2009 01 3009

(c) Analogously to the preceding,


1
i = e 4 (4 ln(1.05)+ln(1.02)) − 1
√4
= 1.05 1.02 − 1 = .0552 = 5.52%.

(d) The effects of the bonuses depend on whether the remainder of the number of
years is, upon division by 3, 0, 1, or 2. We have, for any non-negative integer
n,

(1 + i)3n = (1.05)3n (1.02)n


(1 + i)3n+1 = (1.05)3n+1 (1.02)n
(1 + i)3n+2 = (1.05)3n+2 (1.02)n

By taking logarithms and dividing, or, equivalently, by taking the appropriate


(positive) root of both sides of the equation, we obtain

1 + i = (1.05)(1.02)1/3
n
1 + i = (1.05)(1.02) 3n+1
n
1 + i = (1.05)(1.02) 3n+2

As n → ∞, the exponent of 1.02 approaches 13 , and so the interest rate ap-


proaches the value of 5.70% we obtained in case of 3 years.
5. Alice borrows 5000 from The Friendly Finance Company, at an annual rate of
interest of 18% per year, where the company compounds interest annually, but
charges simple interest for fractions of a year.
(a) She plans to pay the company 5000 at the end of 2 years.
i. How much will she continue to owe the company at that time?
ii. What is the present value of that residual amount, assuming the same 18%
interest rate?
(b) Alice discovers that she doesn’t need the loan, so she offers to lend the money
to her brother, at an effective annual rate of 18%. When her brother pays off
his loan, Alice will pay off hers. How much will Alice still owe Friendly if
i. Her brother pays off his loan exactly 3 years from now?
ii. Her brother pays off his loan 3.5 years from now?
Solution:
(a) i. The residual amount immediately after the payment will be
¡ ¢
5000 (1.18)2 − 1 = 1962 .
Information for Students in MATH 329 2009 01 3010

ii. The present value of the residual amount is 5000 ((1.18)2 − 1) (1.18)−2 =
5000 (1 − (1.18)−2 ) = 1409.08.
(b) i. If her brother pays off his loan after an integer number of years, and Alice
immediately repays her loan, she will owe nothing to Friendly.
ii. After 3.5 years Alice will receive 5000(1.18)3.5 , but will be owing
5000(1.18)3 (1.09); after making her payment, she will continue to owe

−5000(1.18)3.5 + 5000(1.18)3 (1.09) = 5000(1.18)3 (1.09 − 1.18) = 30.58 .

6. (cf. [9, Exercise 2-12, p. 37])


(a) Find an equation that gives information about the effective rate of interest i if
payments of 200 at the present, 300 at the end of 1 year, and 400 at the end of
3 years, are to accumulate to 1000 at the end of 4 years.
(b) Use the Intermediate Value Theorem to argue that there exists a positive rate of
interest less than 100% which can solve this problem. Then use the Mean Value
Theorem to show that there is just one solution to the problem (by showing
that a certain derivative is positive).
(c) While there exist more efficient algorithms for solving problems like this, a
naive solution could be found by successively subdividing an interval at whose
ends a certain function would have values with opposite signs. Apply this idea
to find the interest rate i to within an error of 0.1%.
Solution:
(a) The equation of value at the end of 4 years30 is

200(1 + i)4 + 300(1 + i)3 + 400(1 + i) = 1000

Define f (x) = 200x4 + 300x3 + 400x − 1000. Then f (1) = −100 < 0 <
5400 = 3200 + 2400 + 800 − 1000 = f (2). By the Intermediate Value Theorem
function f , which, being a polynomial, is continuous, has a zero somewhere
in 1 < x < 2. If there were 2 or more zeroes, then, by Rolle’s Theorem,
(since f , being a polynomial, is differentiable), there would be a point between
them where f 0 would be 0. But f 0 (x) = 800x3 + 900x2 + 400 > 0 for 1 <
x < 2. From this contradiction we know that there is at most one zero for
f , hence exactly one solution i for our effective interest rate. We can apply
30
Non-trivial equations are never unique; also, we could have found an equation of value at another
time. For example, an equation of value at the present could be

200 + 300v + 400v 3 = 1000v 4 .


Information for Students in MATH 329 2009 01 3011

the Intermediate Value Theorem between any two points in the domain. The
most naive solution would be to repeatedly halve the interval. We find that
f (1.5) = 1625, so we may confine ourselves to the interval 1 < x < 1.5; then
f (1.25) = 574.22, f (1.125) = 197.51, f (1.0625) = 39.72, f (1.03125) = −32.29.
We evaluate f at the midpoint of the interval [1.03125, 1.0625]: f (1.046875) =
3.17, so we next use the interval [1.046875, 1.0625], whose midpoint is 1.0546875,
where f (1.0546875) = 21.30. As there will is a sign change in the interval
[1.03125, 1.0546875], we next evaluate f at its mid-point: f (1.042968750) =
−5.80.
In the course of these calculations we have not bothered to round the decimal
expansions of the midpoints. There is nothing to be gained by this persis-
tence, as the procedure will work even if we do not take the precise midpoints.
Having now confined
¡ 1.043+1.049 ¢ the root to the interval [1.043, 1.055].
¡ 1.043+1.046 ¢ f (1.049) = 8.07,
we try f 2
= f (1.046) = 1.15, f 2
= f (1.0445) = −2.29,
f (1.045) = −1.15, f (1.0455) = 0.002, f (1.04549913) = −0.0000013. Thus the
rate is approximately 4.55%.

B.1.3 Third 2002/2003 Problem Assignment, with Solutions


1. (a) Find the sum of the positive integers 1, 2, . . . , N .
(b) Find the sum of the odd integers 1, 3, 5, . . . , 2N + 1.
(c) In an arithmetic progression x1 , x2 , . . ., xn , . . . the third term is 4 times the
first term, and the sixth term is 17. Find the general term xn .
(d) The sum of n terms of the arithmetic series 2, 5, 8, . . . is 950. Find n.
(e) The sum of the first 6 terms of a geometric progression is equal to 9 times the
sum of the first 3 terms. Find the common ratio. Is it possible to determine
the sequence from this information?
(f) Use your knowledge of the sum of geometric series to determine a “vulgar”
fraction of integers m
n
which is equal to the repeating decimal number

3.157157157157...

Do not use a calculator for this problem.


Solution: These topics were once part of the standard high school curriculum. These
problems were adapted from [3].
(a) The common difference is 1 and the first term is also 1; the sum of N terms is,
therefore, N2 (1 + N ) = N (N2+1) .31
31
This expression is known, from other considerations,¡ to¢ be the number of ways of choosing 2 objects
from a set of N distinct objects; it is often denoted by N2 .
Information for Students in MATH 329 2009 01 3012

(b) The common difference is 2, the first term is 1, and the N + 1st term is 2N + 1.
The sum of N + 1 terms is N 2+1 (2 · 1 + N · 2) = (N + 1)2 .
[Many students failed to notice that the number of summands was N + 1 —
not N . An error of this type might have been detected by checking one’s
computations for small values of N , e.g. N = 0 or N = 1. Carry out the
summation mechanically, then compare the sum that you obtain with the value
of the formula you have derived; if the values are different, you need to check
every step of your work carefully.]
(c) Let the first term be a and the common difference be d. We have to solve the
equations:
x3 = 4x1 ⇔ a + 2d = 4a
x6 = 17 ⇔ a + 5d = 17
which yield a = 2, d = 3. Hence xn = 2 + 3(n − 1) = 3n − 1.
(d) We solve the equation n2 (2 · 2 + (n − 1) · 3) = 950, which reduces to 3n2 + n −
1900 = 0, whose only positive solution is n = 25.
(e) If the first term is a and the common ratio is r, the given information implies
that
r6 − 1 r3 − 1
a· = 9a · if r 6= 1 , (86)
r−1 r−1
6a = 9a if r = 1 . (87)
When a = 0, both equations are satisfied: the sequence is 0, 0, 0, . . . ; the
common ratio is indeterminate. When r 6= 1, (86) yields r6 − 1 = 9(r3 − 1),
so r3 = 1 (which contradicts the hypothesis) or r3 = 8, hence r = 2 and the
sequence is then
a, 2a, 4a, ..., 2n−1 a, ...
But the sequence is not completely determined, since any value of a is acceptable
— including the value 0 which we already saw as the solution to (87).
(f)
µ ¶ µ ¶
1 5 7 1 1 5 7
3.157157157157... = 3 + + + + + +
10 100 1000 1000 10 100 1000
µ ¶
1 1 5 7
+ + + + ...
10002 10 100 1000
157 1 157 1 157
= 3+ + · + 2
· + ...
1000 1000 1000 1000 1000
157 1 157 3154
= 3+ · 1 =3+ = .
1000 1 − 1000 999 999
Information for Students in MATH 329 2009 01 3013

2. (cf. [9, Exercise 3-6, p. 66] An annuity pays 1000 per year for 8 years. If i = 0.05,
find each of the following
(a) The value of the annuity one year before the first payment.
(b) The value of the annuity one year after the last payment.
(c) The value of the annuity at the time of the 4th payment.
(d) If possible, the number of years an annuity-immediate would have to run in
order that its value, viewed one year before the first payment should be twice
that of the 8-payment annuity whose value at the same time was determined
above.
(e) If possible, the number of years an annuity-immediate would have to run in
order that its value, viewed one year before the first payment, should be three
times that of the 8-payment annuity whose value at the same time was deter-
mined above.
(f) If possible, the number of years an annuity-immediate would have to run in or-
der that its value, viewed one year before the first payment, should be four times
that of the 8-payment annuity whose value at the same time was determined
above.32
(g) (cf. [9, Exercise 3-58, p. 73]) Redo part (a), assuming now that the annuity is
continuous. (The effective annual interest rate remains 5%, and the time is still
8 years.)
Solution:
1−(1.05)−8
(a) 1000a8 5% = 1000 · 0.05
= 20000 (1 − (1.05)−8 ) = 6463.21.
(1.058 −1)(1.05)
(b) 1000s̈8 5% = 1000 · 0.05
= 10026.56.
(c)

1000s4 5% + 1000a4 5% = 1000(1.05)4 a45%


= (1.05)4 (6463.21) = 7856.07 .

(d) We have to solve for n:

an 5% = 2a8 5%
⇒ 1 − v n = 2 − 2v 8
⇒ v n = 2v 8 − 1
ln(2v 8 − 1)
⇒ n= = 21.30 years.
ln v
32
Note that the wording of the cited questions in the textbook required a number of assumptions that
have been made more explicit in the present questions.
Information for Students in MATH 329 2009 01 3014

(e) We have to solve for n:

an 5% = 3a8 5%
⇒ 1 − v n = 3 − 3v 8
⇒ v n = 3v 8 − 2
ln(3v 8 − 2)
⇒ n= = 71.52 years.
ln v
(f) In this case we observe that the value of a perpetuity of 1000 per year at
5% is only 1000
0.05
= 20000 < 4(6463.21); so the problem will have no solution.
If we attempt to solve as in the preceding case, we will obtain the equation
1 − v n = 4 − 4v 8 ⇒ v n = 4v 8 − 3 = −0.29, which has no solution.
(g)

Z8
1000a8 5% = 1000 v t dt
0
1 − (1.05)−8
= 1000 · = 6623.48.
ln(1.05)

3. [9, Exercise 3.15] Prove each of the following identities


• algebraically; and
• verbally
(a) än = an + 1 − v n
(b) s̈n = sn − 1 + (1 + i)n
Solution:
(a) än differs from an in that it has an immediate payment of 1 but lacks the final
payment of 1 n years hence, whose value now is v n .
Algebraically,

än = an (1 + i)
= an + ian
1 − vn
= an + i · = an + (1 − v n )
i
(b) s̈n differs from sn in that is lacks a payment of 1 at time t = 0, but has a
payment of 1 that has accumulated interest over n years, so that its present
value is (1 + i)n .
Information for Students in MATH 329 2009 01 3015

Algebraically,

s̈n = sn (1 + i)
= sn + isn
(1 + i)n − 1
= sn + i · = sn + ((1 + i)n − 1)
i
4. [9, Exercise 3-45, p. 71] Wilbur leaves an inheritance to four charities: A, B, C, D.
The total inheritance is a series of level payments at the end of each year, payable
forever. During the first 20 years, A, B, C share each payment equally. All payments
after 20 years are to revert to charity D. The present value of the shares of A, B,
C, and D are all equal. Showing all your work , prove that i = 0.07177.
Solution: It does not limit generality to assume that the level payments are all of
1. The present value of the payments to each of A, B, C is 31 a20 i% . The payments
to D constitute a perpetuity-immediate of 1 deferred 20 years; its value is v 20 · 1i .
Accordingly we have to solve the following equation for i:
1 1
· a20 i% = v 20 ·
3 i
1 − v 20
⇔ = v 20
3
1
⇔ v 20 =
4
1
⇔ 1 + i = 4 20 = 1.0717735

so i = 7.177%.
5. Find the present value at i effective of a perpetuity whose annual payments of 1000
begin with a payment of 1000 after one year, with the property that each payment
thereafter is reduced by 10% from the preceding payment. In particular, determine
the present value when i = 2.5%.
Solution: [STUDENTS WERE ASKED NOT TO SUBMIT A SOLUTION TO
THIS PROBLEM.] The present value is

1000(v + 0.9v 2 + 0.92 v 3 + 0.93 v 4 + ... + 0.9n−1 v n + ...)


X∞ µ ¶n
0.9 1000v
= 1000v =
n=0
1+i 1 − 0.9v
1000 1000
= =
(1 + i) − 0.9 0.1 + i
1000
When i = 2.5%, the present value is 0.125
= 8000.
Information for Students in MATH 329 2009 01 3016

6. A fund of 10,000 is to be accumulated by means of deposits of 1000 made at the


end of every year, as long as necessary. If the fund earns an effective rate of interest
of 2 21 %, find how many regular deposits will be necessary, and the size of a final
deposit to be made one year after the last regular deposit.
Solution: [6, Example 6.5, pp. 59-60] There are often tacit assumptions in interest
problems; usually there is an “obvious” intended interpretation, while other inter-
pretations might be justified by some unusual reading of the wording. In the present
problem one is to assume that the deposits are not permitted to exceed 1000, and
that all deposits (the “regular” deposits) but the last are to be exactly 1000. The
last deposit can be smaller, but not larger.
Let n be the number of regular deposits required. Then n is the largest integer that
satisfies the “inequality of value”

1000sn 2.5% ≤ 10000 ;

equivalently, sn 2.5% ≤ 10 . Computing the values of sn 2.5% , we find that s8 2.5% =


8.73612, s9 2.5% = 9.95452, s10 2.5% = 11.20338. Hence n = 9. The final partial
deposit will have to be the excess of 10,000 over the accumulated value of s9 2.5% after
one year, i.e., the excess of 10,000 over (1.025)s9 2.5% = 10, 000 − (1.025)(9954.52) =
−203.38. So rather than a final deposit, there will be a final refund of 203.38. This
situation could also have been seen from the value of 10,000s10 2.5% = 11203.38,
which would be the value after a 10th deposit; since this exceeds 11,000, no 10th
deposit would be required.
[ADDED March 10th, 2003] Another possible interpretation of the instructions in
this problem is to treat the 8th as the last “regular” deposit, and to reduce the 9th
deposit so that, when the time arrives for a possible 10th deposit, the balance in
the fund is exactly 10000. If we define the value of the 9th deposit to be x, then
¡ ¢
(1 + i) (1 + i)s8 + x = 1000

10000
⇔x = − 1025s8
1.025
= 9756.0976 − 8954.5188 = 801.58.

We can verify the correctness of this computation by observing that the excess
payment of 1000−801.58 = 198.42 accumulated at 2.5% to 1.025×198.42 = 203.38,
which was computed earlier as the amount refunded one later.
[This assignment was intended as a learning exercise, rather than a testing exercise.
Students were not expected to have seen an example of this type before.]

UPDATED TO April 14, 2009


Information for Students in MATH 329 2009 01 3017

7. A deferred annuity is one that begins its payments later than might otherwise have
been expected. We define
m |an = v m an (88)
m
m |än = v än (89)
Prove, both algebraically and verbally, that, for non-negative integers m and n,
m|an = am+n − am (90)
m
(1 + i) · sn = sm+n − sm (91)
1 |än = an (92)
Solution:
(a)
m |an = v m (v + v 2 + . . . + v n )
= (v m+1 + v m+2 + . . . + v m+n
= (v 1 + v 2 + . . . + v m+n − (v 1 + v 2 + . . . + v m
= am+n − am
In deferring an n-payment annuity-immediate by m years we are planning for
the first payment to be made m + 1 years from now, and the last m + n years
from now. These can be viewed as the last n payments of an m + n-payment
annuity-immediate whose first payment begins one year hence; thus we obtain
the value of m |an by subtracting am from am+n .
(b)
¡ ¢
(1 + i)m · sn = (1 + i)m 1 + (1 + i)1 + . . . + (1 + i)n−1
= (1 + i)m + (1 + i)m+1 + . . . + (1 + i)m+n−1
¡ ¢
= 1 + (1 + i)1 + . . . + (1 + i)m+n−1
¡ ¢
− 1 + (1 + i)1 + . . . + (1 + i)m−1
= sm+n − sm
The payments associated with (1 + i)m · sn can be interpreted as the first m
payments of an m+n-payment annuity whose last payment has just been made.
If we subtract from sm+n the value of the last n payments as viewed from the
day of the last payment, we obtain the value of those first m payments.
(c)
¡ ¢
1 |än = v 1 + v + v 2 + . . . + v n−1
= v + v 2 + . . . + v n = an
When we defer an annuity-due one year it becomes an annuity-immediate.
Information for Students in MATH 329 2009 01 3018

B.1.4 Fourth 2002/2003 Problem Assignment, with Solutions


1. (cf. [9, Exercise 3-60, p. 73]) A student attending engineering school has increasing
amounts of income as she advances through her programme. Accordingly she agrees
to borrow a decreasing annual amount from her parents during her 5 training years,
and to repay the loan with increasing amounts for 15 years after graduation. She
receives amounts 5X, 4X, 3X, 2X and X at the beginning of each of 5 years, where
the last payment is paid at the beginning of her final year. At the end of her first
year after graduation she pays 500, and then increases the amount by 200 each year
until a final payment of 3300.33 If the interest rate is 5%, determine X.
Solution: An equation of value at the time of graduation is

(1 + i)X(Ds)5 = 300a15 + 200(Ia)15

implying that
300a15 + 200(Ia)15
X =
(1 + i)(Ds)5
300(1 − v 15 ) + 200((1 + i)a15 − 15v 15 )
=
5(1 + i)6 − (1 + i)6 a5
= 993.11.

For students who corrected the error in the problem by increasing the number of
years by 1, here is a solution:
Solution: An equation of value at the time of graduation is

(1 + i)X(Ds)5 = 300a16 + 200(Ia)16

implying that
300a16 + 200(Ia)16
X =
(1 + i)(Ds)5
300(1 − v 16 ) + 200((1 + i)a16 − 16v 16 )
=
5(1 + i)6 − (1 + i)6 a5
= 1082.33.

2. (cf. [9, Exercise 4-2, p. 85]) A loan is being repaid by 36 monthly payments. The
first 12 installments are 250 each; the next 18 are 300 each; and the last 6 are 500
each. Assuming a nominal annual interest rate of 12% compounded monthly,
33
The original version of this problem gave the final payment as 3500, which would have required 16
years of payments. A correction was announced at the lecture of March 3rd, 2003.
Information for Students in MATH 329 2009 01 3019

(a) Find the principal, A(0), of the loan.


(b) Using the Prospective Method, find the loan balance immediately after the 6th
payment.
(c) Using the Retrospective Method, find the loan balance immediately after the
6th payment.
(d) Divide the 7th and 8th payments into principal and interest.
Solution:
(a) Using the Prospective Method, the principal is seen to be

A(0) = 250a12 + 300v 12 · a18 + 500v 30 · a6


= 9, 329.46

(b) Immediately after the 6th payment, the value of the remaining 30 payments
(at an interest rate of 1% per period) is

250a6 + 300v 6 · a18 + 500v 24 · a6


1 − v6 1 − v 18 1 − v6
= 250 + 300v 6 · + 500v 24 ·
i i i
1 ¡ ¢
= · (250 + 500v 24 )(1 − v 6 ) + 300v 6 (1 − v 18 )
i
1 ¡ ¢
= · 250 + 50v 6 + 200v 24 − 500v 30
i
= 8365.40

(c) The principal of the loan has been determined above. The outstanding principal
is the accumulated value of this principal decreased by the accumulated values
of the payments that have been made, i.e.

(1.01)6 − 1
(1.01)6 A(0) − 250s6 = (1.016 )(9, 329.46) − 250 ·
0.01
= 9, 805.38 − 1, 538.00 = 8, 365.40.

(d) The principal owing immediately after the 6th payment is known to be 8,365.40.
At the time of the 7th payment, this will have accumulated interest of 1%, or
83.65; the balance of the payment, i.e. 250 − 83.65 = 166.35, will be applied to
reduction of principal. The reduced balance of 8365.40 − 166.35 = 8199.05 will
accumulate interest in the amount of 0.01 × 8199.05 = 81.99 in the 8th month.
The 8th payment will include, in addition to this amount of interest, an amount
of 250 − 81.99 = 168.01 for the reduction of principle; the outstanding principal
after the 8th payment will be 8199.05 − 168.01 = 8031.04.
Information for Students in MATH 329 2009 01 3020

3. (a) [9, Exercise 4-16, p. 87] Harriet is repaying a car loan with payments of 2,000
every three months and a final payment 3 months after the last full payment of
2,000. If the amount of interest in the 4th installment (paid at the end of the
first year) is 1,100, find the principal of the loan, the time and amount of the
final payment, and the amounts of principal and interest in that final payment.
Assume that interest is compounded monthly, at a nominal annual rate of 18%.
(b) Construct an amortization schedule for the first year of this loan.
Solution:
(a) The interest rate being charged monthly is 0.18/12 = 1.5%. Let the principal
of the loan be A. The Retrospective Method shows that the amount owing
immediately after the 3rd installment (paid at 9 months) is
¡ ¢
−2000 (1.015)6 + (1.015)3 + (1.015)0 + (1.015)9 A .

This unpaid balance will, in 3 months, earn the lender interest in the amount
of
¡ ¢¡ ¡ ¢ ¢
1100 = (1.015)3 − 1 −2000 (1.015)6 + (1.015)3 + (1.015)0 + (1.015)9 A .

Thus
1100
A =
(1.015)12
− (1.015)9
¡ ¢
+2000 (1.015)−9 + (1.015)−6 + (1.015)−3 (93)
= 26552.32

We can consider the payments as constituting an annuity, with time interval 3


months, and interest rate per 3 months of (1.015)3 − 1 = 0.045678375. Using
the Prospective Method, we see that the value of n installments, as of the day
of the loan, is
1 − (1.015)−3n
2000an0.045678375 = 2000 · .
0.045678375
We seek the smallest n such that
26552.32 × 0.045678375
1 − (1.015)−3n ≥ = 0.6064334
2000
i.e., such that
1.015−3n ≤ 0.3935666
or
− ln 0.3935666
n≥ = 20.88
3 ln 1.015
Information for Students in MATH 329 2009 01 3021

Thus there will be 20 full payments of 2,000, the last full payment being made
5 years after the beginning of the loan. At that time the amount outstanding
will be

26552.32(1.015)60 − 2000s200.045678375
20
(1.0153 ) − 1
= 64873.15 − 2000 ·
(1.015)3 − 1
= 64873.15 − 63190.50 = 1682.65

The last payment will be 1682.65(1.015)3 = 1759.51; of this, the interest com-
ponent will be 1682.65 ((1.015)3 − 1) = 76.86, and the balance will be the
outstanding principal of 1682.65.
(b)
Duration Payment Interest Principal Repaid Outstanding Principal
(Months)
0 26552.32
3 2000.00 1212.87 787.13 25765.19
6 2000.00 1176.91 823.09 24942.10
9 2000.00 1139.31 860.69 24081.41
12 2000.00 1100.00 900.00 23181.41
4. John has borrowed 10000, on which he is paying interest at 10% effective per year.
He is required to pay the interest on the loan annually, and is permitted to repay
only the entire loan, and only on an anniversary. He decides to accumulate a
sinking fund to accumulate the funds to repay the loan. Suppose that John has
2400 available at the end of each year, out of which to pay both the interest on the
loan and an annual contribution to his sinking fund. If the sinking fund accumulates
at 6%, complete a table under the following headings to determine when John will
be able to repay the loan.
Duration Contribution Interest Interest Earned Balance of
(Years) to Sinking Fund on Loan in Sinking Fund Sinking Fund
0 0 0 0 0
... ... ... ... ...
Solution: The instructions asked that the student “complete a table...to determine
when John will be able to repay the loan”. The information could have been
obtained without the table, however, by finding the smallest value of n for which
1400sn ≥ 10000; this can be seen to be n = 6, where

10000 − 1400sn = 234.55 . (94)


Information for Students in MATH 329 2009 01 3022

From (94) we see that the shortfall in the balance of the sinking fund after the last
payment of 1400 is 234.55. The value of the sinking fund is not yet sufficient to
repay the loan. Even without a 7th payment the sinking fund will exceed 10000
by the time when that payment is due. It will, however, be necessary to pay the
interest charge of 1000 on the loan. If a full 6th payment of 1400 was made into the
sinking fund, there would be a refund of (1.06)(1400)s6 − 1000 = 351.38. However,
a better solution would have been for John to make a smaller 6th deposit into the
sinking fund — just sufficient to bring the fund up to the level of 10000 at the time
of the 7th interest payment. The balance just after such a 6th payment would need
to be 10000
1.06
, and the balance just prior to the 6th deposit would be (1.06)1400s5 ; so
the appropriate 6th deposit would be
10000
− (1.06)1400s5 = 9433.963 − 8365.446 = 1068.52 .
1.06
The first table below shows what would happen if John made a full 6th contribution:

Duration Contribution Interest Interest Earned Balance of


(Years) to Sinking Fund on Loan in Sinking Fund Sinking Fund
0 0.00 0. 0.00 0.00
1 1400.00 1000. 0.00 1400.00
2 1400.00 1000. 84.00 2884.00
3 1400.00 1000. 173.04 4457.04
4 1400.00 1000. 267.42 6124.46
5 1400.00 1000. 367.47 7891.93
6 1400.00 1000. 473.52 9765.45
7 -351.38 1000. 585.93 10351.38
The following table shows the result of a reduced 6th contribution:

Duration Contribution Interest Interest Earned Balance of


(Years) to Sinking Fund on Loan in Sinking Fund Sinking Fund
0 0.00 0. 0.00 0.00
1 1400.00 1000. 0.00 1400.00
2 1400.00 1000. 84.00 2884.00
3 1400.00 1000. 173.04 4457.04
4 1400.00 1000. 267.42 6124.46
5 1400.00 1000. 367.47 7891.93
6 1068.51 1000. 473.52 9433.96
7 0.00 1000. 566.04 10000.00
NOTE TO THE GRADER: PLEASE ACCEPT EITHER OF THESE TABLES.
Information for Students in MATH 329 2009 01 3023

5. (This is a complicated variant of [9, Exercise 4-6, p. 86]. It requires considerable


persistence, but is a very thorough exercise. Don’t panic! This is not a typical
examination question.) Garfield is repaying a debt with 25 annual payments of
1000 each, at an annual interest rate of i = 10%. The terms of his loan permit
him to make additional payments on the date of any regular payment. After any
such additional payment, the terms of the loan require the borrower to continue
with payments of 1000 until a last payment of 1000 or less which settles the debt
completely.
(a) At the end of the 7th year Garfield proposes to make, in addition to his regular
annual payment of 1000, an extra payment of 5000. At that time he also
proposes to reduce his remaining payment period by 4 years, and to make
level payments over that time (replacing the originally agreed payments of
1000). Find the revised annual level payment, computed using the interest rate
i = 10%.
(b) The lender is obliged to accept Garfield’s extra payment. But he is not obliged
to accept Garfield’s proposed method to repay the loan in fewer payments. If,
at the time of the change in the payment scheme, the lender insists on charging
an interest rate of i = 12% when the remainder of the loan will be repaid over
14 equal annual payments, what will be the revised annual level payment that
will have to be paid at the end of the each of the next 14 years?
(c) Determine the premium Garfield is being asked to pay as a result of the in-
creased interest rate in part 5b. Express the amount as of the date of the
proposed change in the payment scheme. Make two sets of calculations:
i. when the cost of money34 is 10% per annum;
ii. when the cost of money is 12% per annum.
(d) As the loan is repaid, the lender is able to put his money to work. Suppose
that money now costs 12%, instead of the 10% that prevailed when the loan
was written. One might have expected the lender to encourage the borrower to
repay the loan faster. Faced with the lender’s intransigence, Garfield makes the
supplementary payment of 5000, but decides to abandon his plans to change
the payment size; his payments will be 1000 per year until possibly the last
payment. Determine whether the lender has suffered from his own stubborn-
ness: express his loss (or gain) as of the time of the supplementary payment
made with the 7th payment.
(e) Suppose that, learning that the cost of money is 12% when he is about to make
his supplementary payment, Garfield changes his plans. He makes no change
34
By the statement The cost of money is i we intend that Garfield is able — as of this particular date
— to either borrow or lend money in any amount and for any period of time commencing immediately
— at the interest rate i.
Information for Students in MATH 329 2009 01 3024

to his loan, but invests his 5000 elsewhere in an annuity which will provide him
with payments of 1000 to apply to as many of the final payments under his
loan as possible. As of the beginning of the 8th year of the loan (immediately
following the 7th payment and any supplementary payment) compare the cost
of this scheme with
i. his commitment under the original loan contract;
ii. his proposed scheme, whereby he would pay 5000 immediately and pay the
rest of the loan over 14 years at 10%;
iii. the lender’s proposal, where an immediate payment of 5000 would be fol-
lowed by equal payments for 14 years, computed at a rate of 12%.
(f) Determine the yield earned by the lender under each of the following repayment
schemes:
i. the loan as originally written — 25 annual payments of 1000;
ii. the repayment scheme proposed by Garfield: 1000 per year for 7 years, 5000
additional at the end of the 7th year; level payments for 14 years thereafter,
amount as computed in part 5a above;
iii. the repayment scheme proposed by the lender, in part 5b, where the level
payments are recomputed at 12% charged from the time of the 7th payment;
(in this case it suffices to write down an equation that must be satisfied by
the yield);
iv. the repayment scheme finally followed by Garfield in part 5d, where he
invests in an annuity to provide him with payments of 1000 for the final
payments, and pays the rest annually from his savings.
Solution:
(a) Using the Prospective Method, we find that the unpaid balance immediately
after the 7th payment, but prior to the extra payment, is 1000a180.1 ; after the
extra payment the amount owed is
1000a180.1 − 5000 = 8201.41 − 5000 = 3, 201.41 .
The level payment to repay this principal in 18 − 4 = 14 years is (with i = 10%
1
and v = 1.1 )
1000a180.1 − 5000 1000(1 − v 18 ) − 5000i
= (95)
a140.1 1 − v 14
= 434.58.
(b) We will have to evaluate the same ratio as in (95), but where numerator and
denominator involve different interest rates.
1000a180.1 − 5000 1000(1 − (1.1)−18 ) − 5000(0.1) 0.12
= ·
a140.12 1 − (1.12)−14 0.1
Information for Students in MATH 329 2009 01 3025

= 483.

(c) Let’s first determine the nature of Garfield’s commitment under the loan af-
ter he makes his supplementary payment. We have determined that the loan
balance is 3,201.41. We note that 1, 000a4.10 = 3, 169.87, while 1, 000a5.10 =
3, 790.79. Garfield’s loan contract requires him to make 4 payments of 1000;
and, at the end of the 5th year, to pay the balance of principal that would be
owing at that time. That balance would be
¡ ¢
(1.1)5 (3201.41) − 1000 s5.1 − 1 = 5155.90 − 5105.10 = 50.80 .

These 5 payments are prescribed under his contract, and the calculation of their
values is not affected by the cost of money today. What is affected is the way
in which Garfield finances these payments; or, equivalently, the present value
of these payments, which may not be equal to the loan balance.
i. If the cost of money is 10%, the present value of the 14 payments Garfield
would have to make would be 483a14.1 = 3558.11; the present value of the
payments required under the loan contract is the outstanding principal,
3201.41; so the premium would be 356.70.
ii. If the cost of money is 12%, the present value of the 14 payments would be
the outstanding principal, 3,201.41. The value of the 4 payments of 1000
and one final payment (i.e. the cost of financing them at 12%) is

1000a4.12 + (1.12)−5 50.80 = 3037.35 + 28.83 = 3066.18 .

In this case he would be paying a premium of

3201.41 − 3066.18 = 135.23 .

(d) After his supplementary payment, Garfield owes an unpaid balance of 3201.41.
Had he been permitted to repay this with 14 annual payments of 434.58, the
present value of those payments would be 2880.47. But Garfield has now been
driven to repay the loan by continuing the planned payments of 1000 until a
final payment. In part 5c we have determined that the number of payments of
1000 is 4, and these are followed by a payment one year later of 50.80. The
value of these payments today, when money costs 12%, is

1000a40.12 + (1.12)−5 50.80 = 3037.35 + 28.83


= 3066.18 .
Information for Students in MATH 329 2009 01 3026

While neither of these repayment schemes yields the full amount owed — be-
cause interest rates are higher than at the outset — the lender was wise to be
unwilling to accept Garfield’s offer: he has reduced his losses under the loan by
3066.18 − 2880.47 = 185.71.
(e) i. Garfield’s commitment under the original contract is for payments of 1000
for 18 more years. At a rate of 12%, Garfield could buy an annuity to cover
his payments at a present cost of 1000a18.12 = 7249.67. We are asked to
compare this cost with the use of the 5000 to purchase a deferred annuity
to cover the last payments due under the contract. We will answer this
question naively and then, when the answer looks “interesting”, observe
that there is a much simpler solution.
The present value of an annuity that will cover the payments due in years
##k + 1, k + 2, ..., 18 is
¡ ¢
1000 a18.12 − ak.12 .
Since
1000a18.12 = 7249.67 − 5000 = 2249.67
1000a3.12 = 2401.83
1000a2.12 = 1690.05 ,
Garfield’s 5000 will buy him a deferred annuity paying 1000 per year, start-
ing at the end of 4 years from now until 18 years from now, costing him
1000s18.12 −1000s3.12 = 7249.67−2401.83 = 4847.84 and he will have 152.16
left over. The cost of the payments not covered by his 4847.84 is 2401.83;
the total of his commitments today is therefore 7249.67, precisely the same
as computed above. This should be no surprise, as both sets of computa-
tions are being made with an interest rate of 12%. Thus the excess of one
over the other is zero.
ii. An annuity to cover the payments of 434.58 per year for 14 years would
cost Garfield today 434.58a14.12 = 2880.47; under this scheme he would also
be making a payment of 5000, for a total of 7880.47: the deferred annuity
method would cost 7880.47 − 7249.67 = 630.80 less.
iii. The payments of 483 per year for 14 years are worth today 483a14.12 =
1000a180.1 − 5000 = 3201.41; the sum of the value of these payments and
the supplementary payment is 1000a180.1 = 8201.41 : the deferred annuity
method would cost 8201.41 − 7249.67 = 951.74 less.
(f) i. The loan was written to provide a yield of 10%. The fact that the cost of
money may have changed does not affect the yield, which is influenced only
by the lender’s payments and receipts under the loan.
Information for Students in MATH 329 2009 01 3027

ii. Since Garfield’s computation of the new level payment is based on an in-
terest rate of 10%, there has been no change in the yield to the lender: it
remains 10%.
iii. When the lender demands that the computation of the replacement level
payment be based on an interest rate of 12%, he effects a partial improve-
ment of the yield; but it cannot affect those funds that were already repaid.
Setting up an equation of value at time 7, just after the supplementary
payment and the 7th payment of 1000, we find that the yield rate, i, will
satisfy the equation:

−(1 + i)7 1000a2510% + 1000s7i + 5000 + 483a14i = 0

which is equivalent to
µ ¶ µ ¶
7 (1 + i)7 − 1 (1 + i)14 − 1
−9077.04(1 + i) + 1000 + 5000 + 483 = 0.
i i(1 + i)14

It can be shown that i = 0.10345 approximately. Thus even the increase in


the interest rate for the final payments does not effect a marked increase in
the yield rate.
iv. In this case the yield rate is 10%, as there are, from the lender’s perspective,
no changes.

B.1.5 Fifth 2002/2003 Problem Assignment, with Solutions


1. (a) (cf. [9, Exercise 5.1, p. 105]) A 15-year bond with face value 20000, redeemable
at par, earns interest at 7.5%, convertible semiannually. Find the price to yield
an investor 8% convertible semiannually.
(b) What is the premium or discount at which the bond will be purchased?
(c) (cf. [9, Exercise 5-13, p. 109]) For the bond in part 1a find the market price35
and flat price at each of the following dates and times:
i. Just after the 7th coupon has been paid.
ii. 3 months after the 7th coupon has been paid. (Use simple interest for
fractions of a period.)
iii. Just before the 8th coupon is paid.
iv. Just after the 8th coupon is paid.
35
Market price=amortized value [9, p. 98] is obtained by interpolating linearly between book values
on coupon dates. Thus the market price is a continuous function of time.
Information for Students in MATH 329 2009 01 3028

(d) What would the market price and flat price have been just before and just after
payment of the 8th coupon if the bond had been purchased at par?
Solution:
(a) Both of the interest rates are nominal annual rates convertible semiannually;
we must divide each by 2. Using the “general formula”, we find the price of
the bond to be

(20000)(1.04)−30 + (0.0375 × 20000)a30.04 (96)


µ ¶
−30 750 ¡ ¢
= (20000)(1.04) + 1 − (1.04)−30 (97)
.04
−30
¡ ¢
= (20000)(1.04) + 18750 1 − (1.04)−30 (98)
= 18750 + (20000 − 18750)(1.04)−30 (99)
= 18750.00 − 385.40 = 19135.40. (100)

Alternatively, using the ”alternate” formula, we find it to be

200 + (750 − 800)a304%


50 ¡ ¢
= 2000 − 1 − (1.04)−30 = 19135.40.
0.04
(b) The bond is selling at a discount of 20000.00 − 19135.40 = 864.60 less than its
redemption value.
(c) The book value at the time of an interest payment is the present value of the
unpaid portions of the bond; the coupon payments will enter into the accounting
in some other way, e.g., as income. These computations make use of the yield
rate associated with the owner’s acquisition of the bond. The market price at
these times will equal the book value.
i. The remaining 30−7 coupons are worth 750a234% = 11142.53; the principal
is worth 20000(1.04)−30+7 = 8114.53. The market price is the book value,
i.e., the sum, 19257.16.
ii. The book value immediately after the payment of the 8th coupon is the sum
of the value of the unpaid coupons, 750a224% = 10838.34 and the present
value of the principal, 20000(1.04)−30+8 = 8439.11; together, 19277.45. The
average of this book value and that after the payment of the 7th coupon is
19267.30, and this is what we define to be the market price.
iii. By our definition, market price is a continuous function of time: the market
price immediately before a coupon payment will be equal to that after the
payment — here, 19277.45.
Information for Students in MATH 329 2009 01 3029

iv. As seen above, the book value is 19277.45. This could also have been
computed by subtracting the value of the coupon from 1.04 times the book
value after the payment of the 7th coupon:

(1.04 × 19257.16) − 750.00 = 20027.45 − 750.00 = 19277.45 .

We compute the flat prices:


i. The flat price associated with yield rate 4% per interest period, just after
payment of the 7th coupon, is the same as the market price, as there is no
accrued interest. Here the value is, as above, 19257.16.
ii. The flat price is the book value at the time of the preceding coupon payment
plus accrued simple interest. That is,
µ ¶
1
1 + · 0.04 19257.16 = 19642.30 .
2
In practice this is often quoted as the market price of 19267.30 plus accrued
interest of 375.00, (half of the next coupon).
iii. The flat price just before the payment of the 8th coupon can be determined
in several different ways:
• Viewed as book value plus accrued interest,

(1 + 0.04) 19257.16 = 20027.45 .

• Viewed as the book value just after the payment of the coupon, plus the
value of the coupon, it is

19277.45 + 750.00 = 20027.45 .

iv. The flat price just after payment of a coupon is the book value, here
19277.45. (The flat price is discontinuous at such points in time: the limit
as time approaches the point from the right is different from the limit from
the left: they differ by the value of the coupon.)
(d) The flat price just after payment of the 8th coupon would have been

20000(1.0375)−22 + 750a223.75%
= 8897.99 + 11102.01 = 20000.00 ;

are you surprised by this result? The flat price just before payment of the
coupon would have been 20000.00 increased by the interest that had been
earned but not paid, i.e. 20000.00 + 750.00 = 20750.00.
The market price would remain constant at 20000 throughout.
Information for Students in MATH 329 2009 01 3030

2. (a) [9, Exercise 5-3, p. 106] Prove the “Alternate Price Formula”:
P = C + (F r − Ci)an
algebraically.
(b) (cf. [9, Exercise 5.5, p. 106]) Two bonds with face value 10000 each, redeemable
at par at the end of the same period, are bought to yield 10%, convertible
semiannually. The first bond costs 8246.56, and pays coupons at 7% per year,
convertible semiannually. The second bond pays coupons at 6% per half-year.
Find
i. the price of the second bond;
ii. the number of coupons remaining on each of the bonds.
Solution:
(a) We can derive the Alternate Price Formula from the “General Formula” [9,
(5.1)] as follows:
P = (F r)an + Cv n
= (F r)an + C(1 − ian ) [9, (3.6), p. 45]
= C + (F r − Ci)an ¤
(b) We apply the Alternate Price Formula proved above to the two bonds. Denote
the price of the second bond by P2 , and the number of coupons remaining by
n. Then
8246.56 = 10000 + (10000(0.035) − 10000(0.05))an (101)
P2 = 10000 + (10000(0.06) − 10000(0.05))an (102)
From (101) we find that
8246.56 − 10000
an5% = = 11.6896 . (103)
10000(0.035 − 0.05)
i. Substituting in (102) yields P2 = 11168.96 as the price of the second bond.
ii. We solve (103) for n:
1 − vn
= 11.6896
0.05
⇒ 1 − v n = 0.58448
⇒ v n = 0.41552
⇒ −n ln(1.05) = ln(0.41552)
⇒ n = 18
There are 18 coupons remaining: the bonds mature in 9 years.

UPDATED TO April 14, 2009


Information for Students in MATH 329 2009 01 3031

3. (cf. [9, Exercise 5-16, p. 108])


(a) Construct a bond amortization schedule for a 3 year bond of face amount 5000,
redeemable at 5250 with semiannual coupons, if the coupon rate is 5% and the
yield rate is 6% — both converted semiannually. Use the format
Time Coupon Interest Principal Book
Value Adjustment Value
0
..
.
(b) Construct a bond amortization schedule for a 3 year bond of face amount 5000,
redeemable at 5250 with semiannual coupons, if the coupon rate is 6% and the
yield rate is 5% — both converted semiannually.
Solution:
(a) The purchase price of the bond will be 5250(1.03)−6 + 125a63% = 4396.79 +
677.15 = 5073.94.
Time Coupon Interest Principal Book
Value Adjustment Value
0 5073.94
1 125.00 152.22 -27.22 5101.16
2 125.00 153.03 -28.03 5129.19
3 125.00 153.88 -28.88 5158.07
4 125.00 154.74 -29.74 5187.81
5 125.00 155.63 -30.63 5218.44
6 125.00 156.55 -31.55 5249.99
−6
(b) The purchase price of the bond will be 5250(1.025) + 150a62.5% = 4527.06 +
826.22 = 5353.28.
Time Coupon Interest Principal Book
Value Adjustment Value
0 5353.28
1 150.00 133.83 16.17 5337.11
2 150.00 133.43 16.57 5320.54
3 150.00 133.01 16.99 5303.55
4 150.00 132.59 17.41 5286.14
5 150.00 132.15 17.85 5268.29
6 150.00 131.71 18.29 5250.00
4. (cf. [9, Exercise 5-22, p. 109]) A 10-year bond of face value 12000 with semiannual
coupons, redeemable at par, is purchased at a premium to yield 10% convertible
semiannually.
Information for Students in MATH 329 2009 01 3032

(a) If the book value (just after the payment of the coupon) six months before the
redemption date is 11828.57, find the total amount of premium or discount in
the original purchase price.
(b) Determine the nominal annual coupon rate of the bond, compounded semian-
nually.
(c) Give the amortization table for the last one and one-half years.
Solution:
(a) The book value just after the pænultimate36 coupon is
11828.57 = 12000v + F r · a10.05
12000 + F r
= 12000v + F r · v =
1.05
so
F r = 1.05 × 11828.57 − 12000 = 420 .
Knowing the amount of each coupon we can now evaluate the purchase price
of the bond to have been
12000(1.05)−20 + 420a200.05 = 4522.67 + 5234.13
= 9756.80 .
The bond was purchased at a discount of 12000 − 9756.80 = 2243.20.
420
(b) The rate per period was 12000 = 3.5%; hence the nominal rate compounded
semi-annually, is 2 × 3.5% = 7%.
(c) For convenience we will compile this table backwards, beginning with Time=20.
We were given that B19 = 11828.57. Hence the Principal Adjustment contained
in the 20th coupon is
11, 828.57 − 12, 000 = −171.43 .
book value at Time=18 will be (12000)(1.05)−2 + 420(1.05−1 + (1.05)−2 ) =
11665.31; the book value at Time=17 will be (12000)(1.05)−3 + 420(1.05−1 +
(1.05)−2 + (1.05)−3 ) = 11509.81.
Time Coupon Interest Principal Book
Value Adjustment Value
20 420.00 591.43 -171.43 12000.00
19 420.00 583.26 -163.26 11828.57
18 420.00 575.50 -155.50 11665.31
17 420.00 ... ... 11509.81
36
2nd last
Information for Students in MATH 329 2009 01 3033

5. A 4.5% bond37 with par value of 100 and semiannual coupons is issued on July 1,
2003. It is callable at 110 on any coupon date from July 1, 2008 through January 1,
2011; at 105 on any coupon date from July 1, 2011 through January 1, 2013; and at
102.50 on any coupon date from July 1, 2013 through January 1, 2015; thereafter
it is callable without premium on any coupon date up to January 1, 2018 inclusive;
its maturity date is July 1, 2018. Determine the highest price that an investor can
pay and still be certain of a yield of
(a) 5% convertible semiannually;
(b) 4% convertible semiannually.
(c) 3% convertible semiannually.
[Hint: For each interest rate, and each range of payments for a given premium,
express the price of the bond as a function of the payment number.]
Solution: As a first step towards organizing data, the student should determine the
payment numbers being referred to. If we label the payment dates with natural
numbers, and define the issue date to be (non)-payment #0, the July dates will
have even numbers, and the January dates odd numbers. The premium of 10 is
payable when the calling date is ##10-15; the premium of 5 when the calling date is
##16-19; the premium of 2.5 when the calling date is ##20-23; and no premium
is payable when the calling date is ##24-29 nor on the maturity date, which is
payment #30.
(a) We tabulate the applicable price formulæ, based on the call or maturity date:

First Date Last Date Price


−n
10 15 110.00 · (1.025) + 2.25an2.5% = 90 + 20.00(1.025)−n
16 19 105.00 · (1.025)−n + 2.25an2.5% = 90 + 15.00(1.025)−n
20 23 102.50 · (1.025)−n + 2.25an2.5% = 90 + 12.50(1.025)−n
24 30 100.00 · (1.025)−n + 2.25an2.5% = 90 + 10.00(1.025)−n

One way to solve the problem would be to laboriously compute the price for
every possible call date, and then take the minimum. However, as the above
formulæ express the value in terms of a decreasing function v n , it suffices to
consider the smallest value in each interval, i.e. the largest value of n. So we
37
The convention in bonds is that, lacking any indication to the contrary, the term an r% bond refers
to a bond whose coupon rate is a nominal rate of r%; the rate is compounded (or converted ) as often as
indicated in the description of the bond, with the default being half-yearly if there is no indication to
the contrary. Under this convention the coupon rate for this bond is 2.25%. This convention is stated
in the textbook [9, p. 93, 1st paragraph]; however, the author usually supplies additional, redundant,
information in his problems.
Information for Students in MATH 329 2009 01 3034

have to compare the following four prices:

Call Date Price


15 103.81
19 99.38
23 97.08
30 94.77

Thus the highest price that the investor may safely pay is 94.77. Because
the prices were expressible in the form 90 + A(1.025)−n , where A is a non-
increasing function of n and (1.025)−n also a non-increasing function of n, we
could have stated immediately that the lowest price would be that for the bond
held to maturity: it was not necessary to carry out all these computations. The
situation is not so clear when the yield rate is less than the coupon rate.
(b) Again we tabulate the applicable price formulæ, based on the call or maturity
date:
First Date Last Date Price
10 15 110.00 · (1.02) + 2.25an2% = 112.50 − 2.50(1.02)−n
−n

16 19 105.00 · (1.02)−n + 2.25an2% = 112.50 − 7.50(1.02)−n


20 23 102.50 · (1.02)−n + 2.25an2% = 112.50 − 10.00(1.02)−n
24 30 100.00 · (1.02)−n + 2.25an2% = 112.50 − 12.50(1.02)−n

These formulæ express the value in terms of an increasing function −v n , it


suffices to consider the largest value in each interval, i.e. the smallest value of
n. So we have to compare the following four prices:

Call Date Price


10 110.45
16 107.04
20 105.77
24 104.73

Thus the highest price that the investor may safely pay is 104.73.
(c) As before, we tabulate the applicable price formulæ, based on the call or ma-
turity date:

First Date Last Date Price


10 15 110.00 · (1.015)−n + 2.25an1.5% = 150.00 − 40.00(1.015)−n
16 19 105.00 · (1.015)−n + 2.25an1.5% = 150.00 − 45.00(1.015)−n
20 23 102.50 · (1.015)−n + 2.25an1.5% = 150.00 − 47.50(1.015)−n
24 30 100.00 · (1.015)−n + 2.25an1.5% = 150.00 − 50.00(1.015)−n
Information for Students in MATH 329 2009 01 3035

As in the case of 4% we have to compare four prices:

Call Date Price


10 115.53
16 114.54
20 114.73
24 115.02

This time the highest price that the investor may safely pay is 114.54.

B.1.6 2002/2003 Class Tests, with Solutions


Versions 2 and 4 appear to have been slightly more difficult than Versions 1
and 3, and the grades were adjusted to compensate for this.

Versions 1 (white) and 3 (yellow)


1. Showing your work, solve each of the following problems:
(a) [2 MARKS] Determine the nominal annual interest rate, i1 , compounded ev-
ery 3 months, which is equivalent to a nominal annual interest rate of 12%
compounded every 4 months.
(b) [4 MARKS] Determine the nominal annual interest rate compounded semi-
annually, i2 , which is equivalent to an effective annual discount rate of 6%.
(c) [4 MARKS] Determine the nominal annual interest rate, i3 , compounded in-
stantaneously (=convertible continuously), which is equivalent to an effective
monthly discount rate of 1%.
Solution:
¡ ¢4
(a) In one year a sum of 1 will grow, under the first rate, to 1 + i41 , and under
¡ ¢3 3
the second to 1 + 0.12
3
. Equating these two yields 1 + i41 = (1.04) 4 , so
³ 3
´
i1 = 4 (1.04) 4 − 1 = 0.1194 = 11.94% .

(b) We know several relationships between i and the corresponding d. For example,
i 1
d = iv = 1+i = 1 − 1+i . Solving these equations for i when d = 0.06, we obtain,
corresponding to an effective annual discount rate of 6%, an effective annual
1 6
interest rate of 0.94 − 1 = 94 = 0.06383. The effective semi-annual interest rate
q
6 12
corresponding to this effective annual rate will be (1 + 94 ) − 1 = 100 94
−1=
0.03142. Corresponding to this semi-annual rate, the nominal annual interest
rate compounded semi-annually will be twice this rate, i.e. 6.284 %.
Information for Students in MATH 329 2009 01 3036

99
(c) Since d + v = 1, the value of v corresponding to d = 1% is 100 , so 1 + i =
100 1 1
99
= 1 + 99 and i = 99 ; hence the corresponding effective annual rate of
¡ ¢
1 12
interest is 1 + 99 − 1 = 12.81781%. The “nominal annual interest rate, i3 ,
compounded instantaneously (=convertible continuously)” will be the force of
interest
õ ¶12 !
1
i3 = δ = ln 1+
99
µ ¶
1 100
= 12 ln 1 + = 12 ln = 0.1206 = 12.06%.
99 99
2. (a) [2 MARKS] Define the sequence of payments whose value is represented by the
symbol sni , using a time diagram showing the payments, and indicating the
point in time where the value of the various payments is being calculated.
(b) [4 MARKS] Derive a formula for sni by using formulæ known to you for the
summation of arithmetic or geometric progressions. Your final formula should
P
be expressed in closed form, i.e., without using summation symbols ( ) or
dots (. . .),
(c) [4 MARKS] Define what is meant by s̈ni . Give, without proof, a formula which
expresses the value of s̈ni in terms of i and n.
Solution:

1 1 1 1 1

0 1 2 3 ··· n

(a)
(b)
sni = 1 + (1 + i) + (1 + i)2 + . . . + (1 + i)n−1
(1 + i)n − 1
= 1·
(1 + i) − 1
(1 + i)n − 1
=
(1 + i) − 1
Information for Students in MATH 329 2009 01 3037

(1 + i)n − 1
=
i
(c) s̈ni is the value of the sum of n payments of 1 at the beginning of each year,
n+1
evaluated one year after the last payment. Its value is sn+1i − 1 = (1+i) i −(1+i) ;
other formulæ would also have been acceptable.
3. A loan of 10,000 at i = 10% is to be repaid by ten equal annual payments.
(a) [5 MARKS] Determine the annual payment.
(b) [10 MARKS] Determine an amortization schedule for the first 5 payments,
showing, for each payment, the interest portion and the portion for reduction
of principal. Use the following format for your table.
Duration Payment Interest Principal Outstanding
(Years) Repaid Principal
...
(c) [5 MARKS] If the loan is sold to an investor immediately after the 5th payment
at a price to yield 12% effective annual interest, determine the price paid by
the investor.
Solution: (Source = Deferred/Supplemental Examination in Math 329, August,
2000, Problem 3.)
(a) If the annual payment is denoted by X, it must satisfy the equation X ·a1010% =
1000 1000
10, 000. Solving this equation yields X = 1−(1.1) −10 = 0.61445671 = 1627.45 as

the level annual payment.


(b) If we were interested only in the interest portion of the 5th payment, we might
recall having proved that [9, p. 79] to be
X(1 − (1.1)10−5+1 ) = 1627.45 × (1 − 0.56447393) = 708.80.
The portion for reduction of principal would then be
X − 708.80 = 1627.45 − 708.80 = 918.65 .
However, the problem required the construction of an amortization table, so
these data can be used only to verify our computations in the table:
Duration Payment Interest Principal Outstanding
(Years) Repaid Principal
0 10000.00
1 1627.45 1000.00 627.45 9372.55
2 1627.45 937.26 690.19 8682.36
3 1627.45 868.24 759.21 7923.15
4 1627.45 792.31 835.14 7088.01
5 1627.45 708.80 918.65 6169.36
Information for Students in MATH 329 2009 01 3038

(c) While the outstanding principal is shown as 6169.35, that will equal the present
value of the remaining 5 payments of 1627.45 each only if the interest rate
remains at 10%. If the interest rate changes to 12%, the present value of the
−5
remaining 5 payments falls to 1627.45 · a512% = 1627.45 × 1−(1.12)
0.12
= 5866.59.
This will be the price paid by an investor who expects the 5 remaining payments
to yield 12% effective interest.

Versions 2 (blue) and 4 (green)


1. [20 MARKS] A borrower takes out a loan of 2000 to be paid by one payment with
full interest at the end of two years. Construct a sinking fund schedule using the
headings
Duration Contribution to Interest Interest Earned Balance of Balance of
(Years) Sinking Fund on Loan in Sinking Fund Sinking Fund Principal
...
assuming that the lender receives 10% convertible semi-annually on the loan, and
the borrower replaces the amount of the loan with equal semi-annual deposits in a
sinking fund to mature when the loan becomes due, where the sinking fund earns
8% convertible semi-annually.
Solution: (Source = Final Examination in Math 329, April, 2000, Problem 3; the
present problem is simplified from that on the examination.)
(a) [7 MARKS] The sinking fund must attain the value of 2000(1.05)4 ; if we de-
note the value of the semi-annual payments into this fund, than X · s40.04 =
2000(1.05)4 , so

2000 × (1.05)4 × 0.04 97.2405


X= 4
= = 572.48.
(1.04) − 1 0.16986

(b) [13 MARKS] The schedule is as follows:


Duration Contribution to Interest Interest Earned Balance of Balance of
(Years) Sinking Fund on Loan in Sinking Fund Sinking Fund Principal
0.0 0.00 0.00 0.00 0.00 2000.00
0.5 572.48 100.00 0.00 572.48 2100.00
1.0 572.48 105.00 22.90 1167.86 2205.00
1.5 572.48 110.25 46.71 1787.05 2315.25
2.0 572.48 115.76 71.48 2431.01 2431.01
2. Showing your work, solve each of the following problems:
(a) [4 MARKS] Determine the nominal annual interest rate compounded semi-
annually, i1 , which is equivalent to an effective annual discount rate of 4%.
Information for Students in MATH 329 2009 01 3039

(b) [2 MARKS] Determine the nominal annual interest rate, i2 , compounded ev-
ery 6 months, which is equivalent to a nominal annual interest rate of 24%
compounded every 3 months.
(c) [4 MARKS] Determine the nominal annual interest rate, i3 , compounded in-
stantaneously (=convertible continuously), which is equivalent to an effective
quarterly discount rate of 2%.
Solution:
(a) We know several relationships between i and the corresponding d. For example,
i 1
d = iv = 1+i = 1 − 1+i . Solving these equations for i when d = 0.04, we obtain,
corresponding to an effective annual discount rate of 4%, an effective annual
1 4
interest rate of 0.96 − 1 = 96 = 0.04167. The effective semi-annual interest rate
q
4 12
corresponding to this effective annual rate will be (1 + 96 ) − 1 = 100 96
−1=
0.02062. Corresponding to this semi-annual rate, the nominal annual interest
rate compounded semi-annually will be twice this rate, i.e. i1 = 4.124%.
¡ ¢2
(b) In one year a sum of 1 will grow, under the first rate, to 1 + i22 , and under
¡ ¢4
the second to 1 + 0.24 4
. Equating these two yields 1 + i21 = (1.06)2 , so
¡ ¢
i2 = 2 (1.06)2 − 1 = 0.0472 = 24.72% .
98
(c) Since d + v = 1, the value of v corresponding to d = 2% is 100 , so 1 + i = 100
98
=
2 2
1 + 98 and i = 98 ; hence the corresponding effective annual rate of interest is
¡ ¢
2 4
1 + 98 − 1 = 8.4166%. The “nominal annual interest rate, i3 , compounded
instantaneously (=convertible continuously)” will be the force of interest
õ ¶4 !
2
i3 = δ = ln 1+
98
µ ¶
2 100
= 4 ln 1 + = 4 ln = 0.08081 = 8.081%.
98 98
3. (a) [2 MARKS] Define the sequence of payments whose value is represented by the
symbol ani , using a time diagram showing the payments, and indicating the
point in time where the value of the various payments is being calculated.
(b) [4 MARKS] Derive a formula for ani by using formulæ known to you for the
summation of arithmetic or geometric progressions. Your final formula should
P
be expressed in closed form, i.e., without using summation symbols ( ) or
dots (. . .).
(c) [4 MARKS] By allowing n to approach infinity, determine a (closed form) for-
mula for the value of a∞i .
Solution:
Information for Students in MATH 329 2009 01 3040

1 1 1 1 1

0 1 2 3 ··· n

(a)
(b)

an = v + v 2 + . . . + v n
1 − vn
= v·
1−v
1 − vn
= (1 + i)v ·
(1 + i) − (1 + i)v
n
1−v 1 − vn
= =
(1 + i) − 1 i
lim 1− lim v n
1 1−v n 1−0
(c) Since 1 + i > 1, 0 < 1+i
< 1, so a∞i = lim = n→∞ n→∞
= = 1i .
n→∞ i i i

B.1.7 Final Examination, 2002/2003


1. (a) [3 MARKS] The total amount of a loan to which interest has been added is
5,000. The term of the loan was 4 years. If the nominal annual rate of interest
was 6% and interest was compounded semi-annually, determine the original
amount of the loan, showing all your work.
(b) [3 MARKS] Showing all your work, determine the simple interest rate under
which a sum of money will double in 5 years.
(c) [4 MARKS] Showing all your work, determine the effective annual compound
discount rate under which a sum of money will double in 8 years.
(d) [5 MARKS] Showing all your work, determine the rate of interest, convertible
continuously, that is equivalent to an effective interest rate of 1% per month.
2. (a) [8 MARKS] To repay a loan, X is obliged to pay Y 1,000 at the end of December,
2004, and 1,200 at the end of December, 2006. He proposes to replace these
two payments by a single payment of 2,196 at the end of December, 2005. If Y
accepts this proposal, what yield rate will he be earning on his loan? Show all
your work.
(b) [7 MARKS] Showing all your work, determine the value at time t = 0 of a
continuous annuity that pays 10,000 per year for 2 years, at an effective annual
interest rate of 5%.
Information for Students in MATH 329 2009 01 3041

3. Express each of the following only in terms of `x , and v.


(a) [2 MARKS] d27
(b) [2 MARKS] 4 q24
(c) [2 MARKS] ä20:25
(d) [2 MARKS] A120:25
(e) [2 MARKS] A20:25
(f) [2 MARKS] The probability that a 25-year old will survive 40 years, but will
die before reaching age 75.
(g) [3 MARKS] 12 |a20:25
4. The Wallace Widget Company is planning to borrow 150,000 from the Bank of
Antigonish, and to undertake to pay interest annually at a rate of 12%; they plan
to contribute equal annual payments to a sinking fund that earns interest at the rate
of 9%. The sinking fund will repay the principal at the end of 10 years. Showing
all your work, determine
(a) [2 MARKS] the annual interest payment,
(b) [3 MARKS] the annual payment into the sinking fund
At the end of 4 years, when Wallace has made its annual interest payment and its
4th payment to the sinking fund, it proposes that this should be the last payment
to the sinking fund. It will apply the balance X accumulated to date in the sinking
fund to repay principal, and it will amortize the remainder of the principal by equal
annual payments over the next 5 years, at a rate of 10%.
(c) [4 MARKS] Determine the annual level payment Y under this proposal.
(d) [6 MARKS] Construct an amortization table for this proposal, under the fol-
lowing headings, beginning immediately after the 4th and last payment to the
sinking fund; assume also that all outstanding interest on the loan has been
made annually to date:
Duration Payment Interest Principal Outstanding
Repaid Principal
4 0.00 0.00 0.00 150000.00−X
5 Y =
.. .. .. .. ..
. . . . .
9 0.00
5. Consider a 100 par-value 15-year bond, with semi-annual coupons at the nominal
annual interest rate of 4%, convertible every six months. Let t represent time in
half-years; assume that the bond is callable at 109.00 on any coupon date from
t = 10 to t = 20 inclusive, at 104.50 from t = 21 to t = 29 inclusive, but matures at
Information for Students in MATH 329 2009 01 3042

100.00 at t = 30. In each of the following cases, determine what price an investor
should pay to guarantee himself
(a) [7 MARKS] a nominal annual yield rate of 5%, convertible semi-annually;
(b) [8 MARKS] an effective annual yield rate of 3%.
6. In addition to her down payment, Mary’s purchase of her new home is financed
by a mortgage of 60,000 payable to the vendor; the mortgage is amortized over 20
years, with a level payment at the end of each month, at a nominal annual rate of
6% compounded monthly.
(a) [3 MARKS] Determine the monthly payments under this mortgage.
(b) [2 MARKS] Divide the first payment into principal and interest.
(c) [3 MARKS] Determine the outstanding principal immediately after the 60th
payment.
(d) [4 MARKS] Divide the 60th payment into principal and interest.
(e) [3 MARKS] Determine the payment that Mary could make at the end of each
year which would be equivalent to the year’s 12 monthly payments.
7. (a) [5 MARKS] Define what is meant by (Da)n and (Ia)n , and explain verbally
why
(Da)30 + (Ia)30 = 31a30 .
(b) [10 MARKS] Showing all your work, find the present value (using effective
annual interest rate i = 6%) of a perpetuity which pays 100 after 1 year, 200
after 2 years, increasing until a payment of 2000 is made, after which payments
are level at 2000 per year forever. [For this problem you may assume that
än − nv n
(Ia)n = (104)
i
ä∞
(Ia)∞ = (105)
i
s̈n − n
(Is)n = .] (106)
i
8. In order to complete the sale of his home in Vancouver, John accepted, in partial
payment, a 200,000 mortgage amortized over 15 years with level semi-annual pay-
ments at a nominal annual rate of 5% compounded semi-annually. Fred has cash
available, and is prepared to buy the mortgage from John and to invest a fixed
portion of the semi-annual payments he receives in a sinking fund that will replace
his purchase capital in 15 years. The sinking fund will earn interest at only 4%,
compounded semi-annually. Showing all your work, determine the following:
(a) [3 MARKS] the amount of the semi-annual mortgage payments
Information for Students in MATH 329 2009 01 3043

(b) [4 MARKS] as a fraction of the purchase price Fred pays for the mortgage, the
semi-annual payment into the sinking fund
(c) [8 MARKS] the amount that Fred should pay for the mortgage in order to obtain
an overall yield rate of 6%, compounded semi-annually on his investment. (Note
that the sinking fund earns 4% compounded semi-annually.)

B.2 2003/2004
B.2.1 First 2003/2004 Problem Assignment, with Solutions
Distribution Date: Solutions mounted on the Web on Wednesday, 4 February, 2004
Assignment was mounted on the Web on Thursday, January 15th, 2005
Hard copy was distributed on Monday, January 19th, 2004
Solutions were to be submitted by Monday, January 26th, 2004
(This is a short assignment. Subsequent assignments can be expected to be
longer.)
1. It is known that the accumulation function a(t) is of the form b · (1.1)t + ct2 , where
b and c are constants to be determined.
(a) If $100 invested at time t = 0 accumulates to $170 at time t = 3, find the
accumulated value at time t = 12 of $100 invested at time t = 1.
(b) Show that this function satisfies the requirement [5, p. 2, #2] that it be non-
decreasing.
(c) Determine a general formula for in , and show that lim in = 10%. (Use
n→∞
L’Hôpital’s Rule.)
Solution: [5, Exercise 4, p. 30] Denote the corresponding amount function by A(t).
(a) An accumulation function must have the property that a(0) = 1; this implies
that 1 = a(0) = b + 0, so b = 1.
The given data imply that
170 = 100(a(3)) = 100(1(1.331) + c · 32 ) (107)
which implies that c = 0.041. We conclude that
A(t)
= a(t) = (1.1)t + 0.041t2 , (108)
A(0)
implying that a(1) = 1.141, a(12) = 9.042428377. Then
A(12) a(12)
A(12) = A(1) · = A(1) ·
A(1) a(1)
9.042428377
= A(1) · = A(1)(7.925002959) = 792.5002959
1.141
Information for Students in MATH 329 2009 01 3044

so $100 at time t = 1 grows to $792.50 at time t = 12.


(b) It follows from (108) that a0 (t) = (1.1)t ln 1.1 + 0.082t, which is positive for
positive t; thus a(t) is an increasing function of t for positive t.
(This property may also be proved “from first principles”. Let t1 ≥ t2 . Then

a(t2 ) − a(t1 ) = (1.1)t2 + 0.041t22 − (1.1)t1 + 0.041t21


¡ ¢
= (1.1)t1 (1.1)t2 −t1 − 1 + (0.041) (t2 − t1 ) (t2 + t1 )

where both of the summands are non-negative for 0 ≤ t1 ≤ t1 .


(c)
a(n) − a(n − 1) (0.1)(1.1)n−1 + (0.041)(2n − 1)
in = =
a(n − 1) (1.1)n−1 − (0.041)(n − 1)2
³ ´
2n−1
0.1 + 0.041 (1.1) n−1
= ³ ´
(n−1)2
1 − 0.041 (1.1) n−1

By L’Hôpital’s Rule
2x − 1 2
lim = lim =0
x→∞ (1.1)x−1 x→∞ (1.1)x−1
ln 1.1
(n − 1)2 2(x − 1)
lim = lim
x→∞ (1.1)n−1 x→∞ (1.1)x−1 ln 1.1
2
= lim x−1
=0
x→∞ (1.1) (ln 1.1)2
Hence
0.1 + 0.041 (0)
lim in = = 0.1 = 10%.
n→∞ 1 − 0.041 (0)
2. It is known that 1000 invested for 4 years will earn 250.61 in interest, i.e., that the
value of the fund after 4 years will be 1250.61. Determine the accumulated value
of 3500 invested at the same rate of compound interest for 13 years.
Solution: [5, Exercise 14, p. 30] Let i be the rate of compound interest. Then
1000(1 + i)4 = 1250.61. The accumulated value of 3500 after 13 years will be
µ ¶ 134
13 1250.61
3500(1 + i) = 3500 = 7239.57 .
1000

3. It is known that an investment of 750 will increase to 2097.75 at the end of 25 years.
Find the sum of the present values of payments of 5000 each which will occur at
the ends of 10, 15, and 25 years.
Information for Students in MATH 329 2009 01 3045

Solution: [5, Exercise 21, p. 31] Let i be the interest rate. The known fact is that
750(1 + i)25 = 2097.75. Hence (1 + i)25 = 2.797 , so v 25 = 0.357535924. The present
value of three payments of 5000 after 10, 15, and 25 years will, therefore, be

5000(v 10 + v 15 + v 25 )
³ 10 15 25
´
= 5000 (0.357535924) 25 + (0.357535924) 25 + (0.357535924) 25
= 5000(0.662709221 + 0.539500449 + 0.357535924)
= 7798.73.

4. Find the accumulated value of 1000 at the end of 10 years:


(a) if the nominal annual rate of interest is 6% convertible monthly;
(b) if the nominal annual rate of discount is 5% convertible every 2 years.
Solution: [5, Exercise 32, p. 31]
(a) The accumulation factor for each month is 1 + 6%
12
= 1.005. After 10 years 1000
grows to
1000(1.005)10×12 = 1819.40.
(b) The discount factor for each 2 years is 1 − 2 × 5% = 0.09 (moving backwards),
1
corresponding to an accumulation factor of 0.9 . After 10 years 1000 grows to
10
1000(0.09)− 2 = 1693.51 .
√ √
5. Given that i(m) = 5 6 6 − 2 and d(m) = 2 − 8 0.06, find m, the equivalent annual
compound interest rate, and the equivalent annual compound discount rate.
Solution: [5, Exercise 30, p. 32] For an mth of a year the relationship between i(m)
and d(m) is given by µ ¶µ ¶
i(m) d(m)
1+ 1− =1
m m
which is equivalent to
(m + i(m) )(m − d(m) ) = m2
or
i(m) d(m)
m= .
i(m) − d(m)
Substituting the given values

i(m) = 0.041241452
d(m) = 0.040408205
Information for Students in MATH 329 2009 01 3046

gives m = 2. It follows that


µ ¶2
i(2)
i = 1+ −1
2
1 1
= (1.020620726)2 − 1 = 4.1666667% = 4 % =
6 24
µ ¶
(2) 2
d
d = 1− 1−
2
1
= 1 − 0.96 = 4% =
25

B.2.2 Second 2003/2004 Problem Assignment, with Solutions


Distribution Date: Mounted on the Web on Friday, February 20th, 2004
Assignment was mounted on the Web on January 19th, 2004.
Hard copy was distributed on Wednesday, January 28th, 2004
Solutions were due by Monday, February 9th, 2004
(Solutions presented subject to correction of errors and omissions.)

1. Find the present value of 1000, to be paid at the end of 37 months under each of
the following scenarios:
(a) Assume compound interest throughout, and a (nominal) rate of discount of 6%
payable quarterly.
(b) Assume compound interest for whole years only at a (nominal) rate of discount
of 6% payable quarterly, and simple discount at the rate of 1.5% per 3 months
during the final fractional period.
(c) Assume compound interest throughout, and a nominal rate of interest of 8%
payable semi-annually.
Solution: (cf. [5, Exercise 2, p. 53])
µ ¶ 37
0.06 3
(a) Present value = 1000 1 − = 829.94.
4
µ ¶ 36 µ ¶
0.06 3 0.015
(b) Present value = 1000 1 − 1− = 1000×0.872823×0.9995 =
4 3
829.96.
37
(c) Present value = (1.04)− 6 = 1000 × 0.785165257 = 785.17.
2. The sum of 5,000 is invested for the months of April, May, and June at 7% simple
interest. Find the amount of interest earned
Information for Students in MATH 329 2009 01 3047

(a) assuming exact simple interest in a non-leap year


(b) assuming exact simple interest in a leap year (with 366 days);
(c) assuming ordinary simple interest;
(d) assuming the “Bankers’ Rule”.
Solution: (cf. [5, Exercise 6, p. 54])
91
(a) The number of days is 30 + 31 + 30 = 91; exact simple interest is 365
· 5000 ·
(0.07) = 87.26.
91
(b) Exact simple interest is 366 · 5000 · (0.07) = 87.02
(c) Ordinary simple interest is 30+30+30
360
· 5000 · (0.07) = 87.50
91
(d) Interest under the Banker’s Rule is 360 · 5000 · (0.07) = 88.47.
3. Find how long 4,000 should be left to accumulate at 5% effective in order that it
will amount to 1.25 times the accumulated value of another 4,000 deposited at the
same time at a nominal interest rate of 4% compounded quarterly.
Solution: (cf. [5, Exercise 13, p. 55]) The equation of value at n years is
4000(1.05)n = (1.25)(4000)(1.01)4n
so
ln 1.25
n= = 24.82450822 .
ln 105 − 4 ln 101
4. The present value of two payments of 100 each, to be made at the end of n years
and 2n years is 63.57. If i = 6.25%, find n.
Solution: (cf. [5, Exercise 14, p. 55]) Solving the equation of value, 100v 2n +100v n =
63.57, we obtain √
n −1 ± 3.5428
v = ,
2
in which only the + sign is acceptable, since v n > 0. Taking logarithms gives
0.818446587
n= = 13.50023411.
ln 1.0625
We conclude, to the precision of the problem, that n = 13.5 years.
5. (a) Find the nominal rate of interest convertible quarterly at which the accumulated
value of 1000 at the end of 12 years is 3000.
(b) Find the nominal rate of discount convertible semi-annually at which a payment
of 3000 12 years from now is presently worth 1000.
(c) Find the effective annual rate of interest at which the accumulated value of
1000 at the end of 12 years is 3000.
Solution:

UPDATED TO April 14, 2009


Information for Students in MATH 329 2009 01 3048

(a) (cf. [5, Exercise 19, p. 55]) The equation of value at time t = 12 is
µ ¶4×12
i(4)
1000 1 + = 3000 ,
4

implying that ³ 1 ´
i(4) = 4 3 48 − 1 = 9.260676%.

(b) The equation of value at time t = 12 is


µ ¶2×12
d(2)
3000 1 − = 1000 ,
2

implying that ³ ´
1
d(2) = 2 1 − 3 24 = 9.3678763%.

(c) The equation of value at time t = 12 is

1000 (1 + i)12 = 3000 ,

implying that
1
i = 3 12 − 1 = 9.5872691%.
6. An investor deposits 20,000 in a bank. During the first 4 years the bank credits an
annual effective rate of interest of i. During the next 4 years the bank credits an
annual effective rate of interest of i − 0.02. At the end of 8 years the balance in the
account is 22,081.10. What would the account balance have been at the end of 10
years if the annual effective rate of interest were i + 0.01 for each of the 10 years?
Solution: (cf. [5, Exercise 32, p. 57]) The equation of value is

20000(1 + i)4 (1 + (i − 0.02))4 = 22081.10 ,

which we interpret as a polynomial equation. The equation is of degree 8, and we


don’t have a simple algebraic method for solving such equations in general. But
this equation has the left side a pure 4th power, so we can extract the 4th roots of
both sides, obtaining
1
(1 + i)(1 + (i − 0.02)) = (1.104055) 4 = 1.025056201 ,

which may be expressed as a quadratic equation in 1 + i:

(1 + i)2 − 0.02(1 + i) − 1.025056 = 0


Information for Students in MATH 329 2009 01 3049

whose only positive solution is


p
0.02 + (0.02)2 + 4(1.025056)
1+i= = 1.0225
2
from which we conclude that i = 2.25%, and that the account balance after 10 years
would be 20000(1.0225 + 0.01)10 = 20000(1.0325)10 = 27, 737.89.
7. A bill for 1000 is purchased for 950 4 months before it is due. Find
(a) the nominal rate of discount convertible monthly earned by the purchaser;
(b) the annual effective rate of interest earned by the purchaser.
Solution: (cf. [5, Exercise 25, p. 56])
(a) If d(12) be the nominal discount rate, then
µ ¶4
d(12)
1000 1 − = 950
12
implying that
d(12)
1− = 0.9872585
12
so d(12) = 15.29%.
(b) Let i be the effective annual interest rate. Then
1
950(1 + i) 3 = 1000

implies that µ ¶3
1000
i= − 1 = 16.635% .
950
8. A signs a 2-year note for 4000, and receives 3168.40 from the bank. At the end
of 6 months, a year, and 18 months A makes a payment of 1000. If interest is
compounded semi-annually, what is the amount outstanding on the note at the
time if falls due?
Solution: If i0 be the rate of interest charged semi-annually, then

3168.40(1 + i0 )4 = 4000

so i0 = 6.00%; that is i(2) = 12.00%. The value of the 3 payments at the time the
note matures is
¡ ¢
1000 (1.06)3 + (1.06)2 + (1.06)1 = 3374.62

so the amount outstanding before the final payment is 625.38.


Information for Students in MATH 329 2009 01 3050

9. The Intermediate Value Theorem for continuous functions tells us that such a func-
tion f (x) whose value at x = a has the opposite sign from its value at x = b will
assume the value 0 somewhere between a and b. By computing the value of f at
the point 21 (a + b), we can infer that there is a 0 of f in an interval half as long as
[a, b], and this procedure may be repeated indefinitely to determine a zero of f to
any desired accuracy. Assuming that polynomials are continuous, use this idea to
determine the nominal quarterly compound interest rate under which the following
payments will accumulate to 1000 at the end of 4 years:
• 300 today

• 200 at the end of 1 year

• 300 at the end of 2 years


Your answer should be accurate to 3 decimal places, i.e., expressed as a percentage
to 1 decimal place.
Solution: (We will carry the computations to an accuracy greater than requested
in the problem.)
(a) Let the effective annual interest rate be i. The equation of value at the end of
4 years is
300(1 + i)4 + 200(1 + i)3 + 300(1 + i)2 = 1000 . (109)
(b) We need a continuous function to which to apply the Intermediate Value The-
orem. Some choices may be better than others. We will choose

f (x) = 3x4 + 2x3 + 3x2 − 10 .

We observe that f (0) = −10, that f (2) = 48 + 16 + 12 − 10 = 66 > 0, and that


f (−2) = 48 − 16 + 12 − 10 = 34 > 0. This tells us that there is a solution to
equation (109) for −2 ≤ x ≤ 0, equivalently for −3 ≤ i ≤ −1: such a solution
is of no interest to us, as it does not fit the constraints of this problem. But
the function is a cubic polynomial, and has 2 other zeros. We see that it also
has a solution in the interval 0 ≤ x ≤ 2, and we proceed to progressively halve
intervals.
(c) The midpoint of interval [0, 2] is 1;

f (1) = 3 + 2 + 3 − 10 = −2 < 0 < 66 = f (2) ,

so there must be a root in the interval [1, 2].


Information for Students in MATH 329 2009 01 3051

(d) The midpoint of [1, 2] is 1.5;


f (1.5) = 3(1.5)4 + 2(1.5)3 + 3(1.5)2 − 10
= 15.1875 + 6.75 + 6.75 − 10 = 18.6875
> 0
so there must be a zero in the interval [1, 1.5], whose midpoint is 1.25.
(e)
f (1.25) = 3(1.25)4 + 2(1.25)3 + 3(1.25)2 − 10
= 7.3242 + 3.9063 + 4.6875 − 10 = 5.918
> 0
so there must be a zero in the interval [1, 1.25], whose midpoint is 1.125.
(f)
f (1.125) = 3(1.125)4 + 2(1.125)3 + 3(1.125)2 − 10
= 4.8054 + 2.8477 + 3.7969 − 10 = 1.45
> 0
so there must be a zero in the interval [1, 1.125], whose midpoint is 1.0625.
(g)
f (1.0625) = 3(1.0625)4 + 2(1.0625)3 + 3(1.0625)2 − 10
= 3.8233 + 2.3989 + 3.3867 − 10 = −0.3911
< 0
so there must be a zero in the interval [1.0625, 1.125], whose midpoint is 1.09375.
(h)
f (1.09375) = 3(1.09375)4 + 2(1.09375)3 + 3(1.09375)2 − 10
= 4.2933 + 2.6169 + 3.5889 − 10 = 0.4991
> 0
so there must be a zero in the interval [1.0625, 1.09375], whose midpoint is
1.078125.
(i)
f (1.078125) = 3(1.078125)4 + 2(1.078125)3 + 3(1.078125)2 − 10
= 4.053197 + 2.506325 + 3.487061 − 10 = 0.046582
> 0
Information for Students in MATH 329 2009 01 3052

so there must be a zero in the interval [1.0625, 1.078125], whose midpoint is


1.0703125.
(j)

f (1.0703125) = 3(1.0703125)4 + 2(1.0703125)3 + 3(1.0703125)2 − 10


= 3.936984 + 2.452233 + 3.436707 − 10 = −0.174076
< 0

so there must be a zero in the interval [1.0703125, 1.078125], whose midpoint is


1.07421875.
(k) f (1.07421875) = −.064207958 < 0 so there must be a zero in the interval
[1.07421875, 1.078125], whose midpoint is 1.076172.
(l) f (1.076172) = −.008925 < 0 so there must be a zero in the interval
[1.076172, 1.078125], whose midpoint is 1.077149.
(m) f (1.077149) = .018814 > 0 so there must be a zero in the interval
[1.076172, 1.077149], whose midpoint is 1.076661.
(n) f (1.076661) = .004952 > 0 so there must be a zero in the interval
[1.076172, 1.076661], whose midpoint is 1.076417.
(o) f (1.076417) = −.001974 < 0 so there must be a zero in the interval
[1.076417, 1.076661], whose midpoint is 1.076539.
(p) f (1.076539) = .001488 > 0 so there must be a zero in the interval
[1.076417, 1.076539], whose midpoint is 1.076478.
(q) f (1.076478) = −.000243 < 0 so there must be a zero in the interval
[1.076478, 1.076539], whose midpoint is 1.076509.
(r) f (1.076509) = .000637 > 0 so there must be a zero in the interval
[1.076478, 1.076509], whose midpoint is 1.076494.
(s) f (1.076494) = .000211 > 0 so there must be a zero in the interval
[1.076478, 1.076494], whose midpoint is 1.076486.
(t) f (1.076486) = −.000016 > 0 so there must be a zero in the interval
[1.076486, 1.076494], whose midpoint is 1.07649.
(u) f (1.07649) = .000097 > 0 so there must be a zero in the interval
[1.076486, 1.07649], whose midpoint is 1.076488.
(v) f (1.076488) = .000041 > 0 so there must be a zero in the interval
[1.076486, 1.076488], whose midpoint is 1.076487.
(w) f (1.076487) = .000012. One zero will be approximately x = 1.07649. Thus the
effective annual interest rate is approximately 7.649%. This, however, is not
what the problem asked for. The accumulation function for 3-months will then
1
be (1.0749) 4 = 1.01822, so the effective interest rate for a 3-month period will
Information for Students in MATH 329 2009 01 3053

be 1.822%, and the nominal annual interest rate, compounded quarterly, will
be 7.288, or 7.3% to the accuracy requested.
THE FOLLOWING PROBLEM WAS CONSIDERED FOR INCLUSION IN THE AS-
SIGNMENT, BUT WAS (FORTUNATELY) NOT INCLUDED.
10. [5, Exercise 6, p. 88]
(a) Show that
am−n = am − v m sn = (1 + i)n am − sn
where 0 < n < m.
(b) Show that
sm−n = sm − (1 + i)m an = v n sm − an
where 0 < n < m.
(c) Interpret the results in (a) and (b) verbally.
Solution:
(a) We prove the first of these identities by technical substitutions in sums, analo-
gous to changes of variables in a definite integral. For the second identity we
give a less formal proof.
m−n
X
am−n = vr
r=1
m
X m
X
r
= v − vr
r=1 r=m−n+1
m
X m
X
= vr − v m
v r−m
r=1 r=m−n+1
m
X 1
X
r m
= v −v v 1−s
r=1 s=n
under the change of variable s = m − r + 1
Xm Xn
= vr − vm v 1−s
r=1 s=1
reversing the order of the 2nd summation
Xm Xn
r m
= v −v (1 + i)s−1
r=1 s=1
m
= am − v · sn
am−n = v + v 2 + . . . + v m−n
Information for Students in MATH 329 2009 01 3054

= v −n+1 + v −n+2 + . . . + v 0 + v 1 + v 2 + . . . + v m−n


¡ ¢
− v −n+1 + v −n+2 + . . . + v 0
¡ ¢
= v −n v 1 + v 2 + . . . + v n + v n+1 + v n+2 + . . . + v m
¡ ¢
− (1 + i)n−1 + (1 + i)n−2 + . . . + (1 + i)v 0
¡ ¢
= (1 + i)n v 1 + v 2 + . . . + v n + v n+1 + v n+2 + . . . + v m
¡ ¢
− (1 + i)0 + (1 + i)1 + . . . + (1 + i)v n−1
= (1 + i)n am − sn

(b) These identities could be proved in similar ways to those used above. Instead,
we shall show that these identities can be obtained from the preceding simply
by multiplying the equations by (1 + i)m−n :

sm−n = (1 + i)m−n am−n [5, (3.5), p. 60]


m−n n
= (1 + i) ((1 + i) am − sn )
= (1 + i) am − (1 + i)m · (1 + i)−n sn
m

= sm − (1 + i)m an
sm−n = (1 + i)m−n am−n
= (1 + i)m−n (am − v m sn )
= (1 + i)−n · (1 + i)m am − (1 + i)−n sn
= (1 + i)−n sm − an

(c) i. An (m − n)-payment annuity-immediate of 1 has the same present value as


an annuity for a total term of m = (m−n)+n years minus a correction paid
today equal to the value of the deferred n payments. Those n payments are
worth sm at time t = m, which amount can be discounted to the present
by multiplying by v m .
The preceding explanation was based on values at the commencement of
the first year of an m-year annuity-immediate. Let us now interpret the
m − n payments as being the last payments of an m-year annuity whose
mth payment has just been made. That m-payment annuity was worth am ,
n years ago — a year before its first payment; today it is worth (1 + i)n an ,
including the payments we attached at the beginning. Those payments are
worth sn today, for a net value as claimed.
ii. Consider an value of the first m − n payments of an m-payment annuity-
immediate of 1, just after the (m − n)th payment. Since these could be
considered simply the accumulated value of an (m − n)-payment annuity,
they are worth sm−n . But the payments of the m-payment annuity not
yet made are worth an , and the entire annuity is worth sm at termination,
Information for Students in MATH 329 2009 01 3055

hence v n sm today; hence v m sm − an is also the value today. This gives the
equality between the extreme members of the alleged inequality.
Now let’s evaluate the same m − n payments, but this time consider them
to be the last m − n payments of an m-payment annuity-certain; again, the
m−nth payment has just been made. From first principles, the accumulated
value of the payments actually received is sm−n , and we are viewing them
from the context of an annuity-certain of m payments that would be worth
sm today: let’s determine the amount that would have to be paid out today
to correct for that expanded annuity. The value of the n payments we have
tacked on in the past was an one year before the payments began, and
(1 + i)m sn today; so we can also view the value today as sm − (1 + i)m an .

B.2.3 Third 2003/2004 Problem Assignment, with Solutions


Distribution Date: Mounted on the Web on Wednesday, March 3rd, 2004.
Assignment was mounted on the Web on February 8th, 2004,
hard copy of assignment was distributed on Wednesday, February 11th, 2004.
Solutions were to be submitted by 9 a.m., Monday, March 3rd, 2004
SUBJECT TO CORRECTION OF TYPO’S AND OTHER ERRORS

Sketch a time diagram to accompany your solution of all problems except the last.
1. A skier wishes to accumulate 30,000 in a chalet purchase fund by the end of 8 years.
If she deposits 200 into the fund at the end of each month for the first 4 years, and
200 + X at the end of each month for the next 4 years, find X if the fund earns a
nominal (annual) rate of 6% compounded monthly.
Solution: The equation of value at the end of 8 × 12 = 96 months is

200s96 + X · s48 = 30000 ,

which we may solve to yield


30000 s
X = − 200 96 0.005
s48 0.005 s48 0.005
30000 (1.005)96 − 1
= − 200
s48 0.005 (1.005)48 − 1
= 30000s−1
48 0.005
− 200((1.005)48 + 1)
= 30000(0.018485) − 200(2.27049) from the tables
= 100.452.
Information for Students in MATH 329 2009 01 3056

2. A fund of 2500 is to be accumulated by n annual payments of 50, followed by n + 1


annual payments of 75, plus a smaller final payment of not more than 75 made 1
year after the last regular payment. If the effective annual rate of interest is 5%,
find n and the amount of the final irregular payment.
Solution: We shall interpret the payments to be made under two annuities-due: the
first, for 2n + 1 years, consists of an annual deposit of 50 in advance; the second,
for n + 1 years, deferred n years after the first, consists of an annual deposit of 25
in advance. It is at the end of year 2n + 1 that the final — drop — payment is to
be made, and it is to be under 75. (Note that this is the type of problem where the
drop payment could turn out to be negative.) We seek the smallest n for which
50 · s̈2n+1 + 25 · s̈n+1 > 2500 − 75 = 2425
µ ¶
n 2 n 0.05(2425)
⇔ 2(1.05) ((1.05) ) + (1.05) − 3 +
25(1.05)
µ ¶
n 2 1 n 1 0.05(2425)
⇔ ((1.05) ) + (1.05) − 3+
2.1 2.1 25(1.05)
µ ¶2 µ ¶
n 1 1 1 0.05(2425)
⇔ (1.05) + > + 3+
4.2 (4.2)2 2.1 25(1.05)
µ µ ¶¶ 21
n 1 1 1 0.05(2425)
⇒ (1.05) + > + 3+ = 1.919585178
4.2 (4.2)2 2.1 25(1.05)
ln 1.919585178
⇒ n> = 10.65133267
ln 1.05
Thus the drop payment will be when t = 11 + 12, i.e., 23 years after the first
payment under the annuity with payments of 50. Just before the drop payment the
accumulated value of all previous payments is
1.05 ¡ ¢
50s̈23 + 25s̈12 = 50 · (1.05)23 + 25 · (1.05)12 − 75 = 2592.924516
0.05
so the drop payment at time t = 23 is 2500 − 2592.924516 = −92.92.
Thus we have an example here of a negative drop payment. Could this mean that
we should have taken n = 10? No. In that case we would find that the final payment
would be larger than the permitted 75. (If tables like those in the textbook were
available, one could determine the value of n by inspecting the value of 2s2n+1 +sn+1 .
We observe from the 5% tables the following values:
n 2s2n+1 + sn
10 84.0165
11 97.0678
12 111.3713
Information for Students in MATH 329 2009 01 3057

We seek the smallest n such that

50s̈2n+1 + 25s̈n > 2500 − 75

i.e., such that


2s̈2n+1 + s̈n > 97 ,
equivalently,
97
2s2n+1 + sn > = 92.38 ,
1.05
and so can conclude that n = 11.)
3. On his 30th birthday, a teacher begins to accumulate a fund for early retirement
by depositing 5,000 on that day and at the beginnings of the next 24 years as well.
Since he expects that his official pension will begin at age 65, he plans that, starting
at age 55 he will make an annual level withdrawal at the beginning of each of 10
years. Assuming that all payments are certain to be made, find the amount of these
annual withdrawals, if the effective rate of interest is 6% during the first 25 years,
and 7% thereafter.
Solution: Let the constant amount of the withdrawals beginning at age 55 be X.
The equation of value at age 55, just before the first withdrawal, is

5000 · s̈25 6% = X · ä10 7%


5000 · s̈25 6%
⇒ Annual Withdrawal X =
ä10 7%
1.06 0.07 (1.06)25 − 1
= 5000 · · ·
1.07 0.06 1 − (1.07)−10
= 33477.74

4. At an effective annual interest rate of i it is known that


(a) The present value of 5 at the end of each year for 2n years, plus an additional
3 at the end of each of the first n years, is 64.6720.
(b) The present value of an n-year deferred annuity-immediate paying 10 per year
for n years is 34.2642.
Find i.
Solution: It is convenient to distinguish two cases.
Case i 6= 0: From (4a) we have an equation of value

64.6720 = 5 · a2n + 3 · an ; (110)

UPDATED TO April 14, 2009


Information for Students in MATH 329 2009 01 3058

from (4b) we have the equation of value


¡ ¢
34.2642 = v n · 10 · an = 10 a2n − an . (111)

Solving these equations, we obtain

a2n = 9.3689 (112)


an = 5.9425 , (113)

implying that
1 − v 2n 9.3689
n
=
1−v 5.9425
⇒ 1 + v n = 1.5766
⇒ v n = 0.5766.

We can substitute in equation (113) to obtain

i = 0.07125 = 7.125%

which implies that


ln 0.5766
n=− = 8.000 years.
ln 1.07125
Case i = 0: Here Equations (110) and (111) become

64.6720 = 5(2n) + 3n = 13n


34.2642 = 10n = 10(2n − n)

which are inconsistent. Thus this case is impossible.


5. (a) Find ä12 if the effective rate of discount is 5%.
(b) Charles has inherited an annuity-due on which there remain 12 payments of
10,000 per year at an effective discount rate of 5%; the first payment is due
immediately. He wishes to convert this to a 25-year annuity-immediate at the
same effective rates of interest or discount, with first payment due one year
from now. What will be the size of the payments under the new annuity?
Solution:
(a) Since (1 − d)(1 + i) = 1, v = 1 − d = 0.95 when d = 0.05.

1 − (0.95)12
ä12 = = 9.19279825.
0.05
Information for Students in MATH 329 2009 01 3059

(b) i = d(1 + i) = vd = 0.05


0.95
1
= 19 . Let X be the size of the new payments. We must
solve the equation of value,

91927.9825 = X · a25i

where v = 0.95. Hence


91927.9825
X =
a25i
91927.9825i
=
1 − v 25
91927.9825
=
(1 − (0.95)25 ) 19
= 6695.606220

So the new annuity-immediate will pay 25 annual payments of 6695.61, begin-


ning one year from now.
6. Give an algebraic proof and a verbal explanation for the formula

m |an = a∞ − am − v m+n a∞ .

Solution:
(a)
1 1 − vm 1
a∞ − am − v m+n a∞ = − − v m+n ·
i i i
1 − (1 − v m ) − v m+n
=
i
v m (1 − v n ) 1 − vn
= = vm · = v m · an = m |an
i i
(b) a∞ i is the present value of a perpetuity at rate i of 1 per year, payments
starting a year from now. am is the present value of the first m payments of
that perpetuity; if we subtract this we have the present value of a perpetuity-
immediate that starts m years from now, i.e., where the first payment is m + 1
years from now. v m+n a∞ is the value of a perpetuity-immediate of 1 starting
m + n years from now, i.e., where the first payment is m + n + 1 years from now;
if we subtract this term as well, we are left with the present value of payments
at the ends of years m + 1, m + 2, . . ., m + n, i.e., with the present value of an
n-payment annuity-certain of 1, deferred m years, i.e., of m |an
Information for Students in MATH 329 2009 01 3060

7. A level perpetuity-immediate is to be shared by A, B, C, and D. A receives the first


n payments, B the next 2n payments, C payments ##3n + 1, . . . , 5n, and D the
payments thereafter. It is known that the present values of B’s and D’s shares are
equal. Find the ratio of the present value of the shares of A, B, C, D.
Solution: The present values of the shares of A, B, C, D are, respectively, an ,
v n a2n = a3n − an , v 3n a2n = a5n − a3n , and v 5n a∞ = a∞ − a5n . The fact that B’s
and D’s shares are equal implies that

vn v 5n
· (1 − v 2n ) =
i i
which is equivalent to (v 2n )2 + v 2n − 1 = 0, implying that

−1 ± 5
v 2n = .
2
Since v is positive, only the + sign is admissible:

2n −1 + 5
v = = 0.618033988...
2
so
v n = 0.7861513777... .
Then the shares of A, B, C, D will be in the ratio

1 − v n : v n − v 3n : v 3n − v 5n : 1 − v 5n

i.e.
0.2138486221 : 0.3002831059 : 0.1855851657 : 0.6997168937
8. (a) Find the present value of an annuity which pays 4,000 at the beginning of
each 3-month period for 12 years, assuming an effective rate of 2% interest per
4-month period.
(b) Suppose that the owner of the annuity wishes to pay now so that payments
under his annuity will continue for an additional 10 years. How much should
he pay?
(c) How much should he pay now to extend the annuity from the present 12 years
to a perpetuity?
(It is intended that you solve this problem “from first principles”, not by substitution
into formulæ in [5, Chapter 4].)
Solution:
Information for Students in MATH 329 2009 01 3061

(a) If i be the effective interest rate per 4-month period, the effective rate per 3-
3
month period will be j = (1 + i) 4 − 1. Accordingly the value of the desired
annuity is

1 − (1 + j)−48
4000ä48 j = 4000(1 + j) ·
j
3 1 − (1 + i)−36
= 4000(1 + i) 4 · 3
(1 + i) 4 − 1
1 − (1.02)−36
= 4000 3
1 − (1.02)− 4
= 138, 317.4894.

(b) Repeating the calculations above for 48 + 40 = 88 payments, we obtain

1 − (1 + j)−88
4000ä88 j = 4000(1 + j) ·
j
3 1 − (1 + i)−66
= 4000(1 + i) 4 · 3
(1 + i) 4 − 1
1 − (1.02)−66
= 4000 3
1 − (1.02)− 4
= 197, 897.4338,

so the additional payments will cost 197, 897.4338−138, 317.4894 = 59, 579.9444
today.
(c) The cost of the perpetuity-due today would be
1
4000ä88 j = 4000(1 + j) ·
j
3 1
= 4000(1 + i) 4 · 3
(1 + i) 4 − 1
4000
= 3
1 − (1.02)− 4
= 271, 329.4837,

so the additional payments will cost

271, 329.4837 − 138, 317.4894 = 133, 011.9943

today.
Information for Students in MATH 329 2009 01 3062

9. (No time diagram is needed for the solution to this problem.) In Problem 9 of
Assignment 2 you were asked to apply the Bisection Method to determine the
solution to an interest problem to 3 decimal places. The equation in question was
(109):
300(1 + i)4 + 200(1 + i)3 + 300(1 + i)2 = 1000 .
and the solution given began with the values of

f (x) = 3x4 + 2x3 + 3x2 − 10 .

at x = 0 (f (0) = −10), x = 2 (f (2) = 66 > 0), and x = −2 (f (−2) = 34 > 0),


and we were interested in the solution between 0 and 2 — a solution that is unique
because f 0 is positive in this interval. Apply Linear Interpolation 4 times in an
attempt to determine the solution we seek. (You are not expected to know the
general theory of error estimation.) The intention is that you apply linear inter-
polation unintelligently, using it to determine a point where you find the function
value and thereby confine the zero to a smaller subinterval: the point that you find
will replace the midpoint in the bisection method. In some situations, as in the
present one, the procedure may not be better than the bisection method. Indeed,
in the present example, it could take many more applications than the bisection
method to obtain the accuracy you obtained with that method.
Solution: We take x1 = 0, x2 = 2. Then
−10
x3 = 0 + (2 − 0) ·
−10 − 66
= 0.2631578947
f (x3 ) = −9.741407754
−9.741407754
x4 = 0.2631578947 + (2 − 0.2631578947) ·
−9.741407754 − 66
= 0.4865401588
f (x4 ) = −8.891376200
−8.891376200
x5 = 0.4865401588 + (2 − 0.4865401588) ·
−8.891376200 − 66
= 0.6662236083
f (x5 ) = −7.486007579
−7.486007579
x6 = 0.6662236083 + (2 − 0.6662236083) ·
−7.486007579 − 66
= 0.8020951913
f (x6 ) = −5.796139773
−5.796139773
x7 = 0.8020951913 + (2 − 0.8020951913) ·
−5.796139773 − 66
Information for Students in MATH 329 2009 01 3063

= 0.8988026707
f (x7 ) = −4.166425912
−4.166425912
x8 = 0.8988026707 + (2 − 0.8988026707) ·
−4.166425912 − 66
= 0.9641908820
f (x8 ) = −2.825434839
−2.825434839
x9 = 0.9641908820 + (2 − 0.9641908820) ·
−2.825434839 − 66
= 1.006713115
f (x9 ) = −1.837664218
−1.837664218
x10 = 1.006713115 + (2 − 1.006713115) ·
−1.837664218 − 66
= 1.033620406
f (x10 ) = −1.162055435
−1.162055435
x11 = 1.033620406 + (2 − 1.033620406) ·
−1.162055435 − 66
= 1.050340958
f (x11 ) = −0.721587971
−0.721587971
x12 = 1.050340958 + (2 − 1.050340958) ·
−0.721587971 − 66
= 1.060611435
f (x12 ) = −0.442976533
−0.442976533
x13 = 1.060611435 + (2 − 1.060611435) ·
−0.442976533 − 66
= 1.066874356
f (x13 ) = −0.270019571
−0.270019571
x14 = 1.066874356 + (2 − 1.066874356) ·
−0.270019571 − 66
= 1.070676410
f (x14 ) = −0.163879567
−0.163879567
x15 = 1.070676410 + (2 − 1.070676410) ·
−0.163879567 − 66
= 1.072978227
f (x15 ) = −0.099198908
−0.099198908
x16 = 1.072978227 + (2 − 1.072978227) ·
−0.099198908 − 66
= 1.074369462
Information for Students in MATH 329 2009 01 3064

f (x16 ) = −0.059950550
−0.059950550
x17 = 1.074369462 + (2 − 1.074369462) ·
−0.059950550 − 66
= 1.075209488
f (x17 ) = −0.036195811
−0.036195811
x18 = 1.075209488 + (2 − 1.075209488) ·
−0.036195811 − 66
= 1.075716385
f (x18 ) = −0.021840825
−0.021840825
x19 = 1.075716385 + (2 − 1.075716385) ·
−0.021840825 − 66
= 1.076022149
f (x19 ) = −0.013174269
−0.013174269
x20 = 1.076022149 + (2 − 1.076022149) ·
−0.013174269 − 66
= 1.076206548
f (x20 ) = −0.007944937
−0.007944937
x21 = 1.076206548 + (2 − 1.076206548) ·
−0.007944937 − 66
= 1.076317739
f (x21 ) = −0.004790698
−0.004790698
x22 = 1.076317739 + (2 − 1.076317739) ·
−0.004790698 − 66
= 1.076384781
f (x22 ) = −0.002888504
−0.002888504
x23 = 1.076384781 + (2 − 1.076384781) ·
−0.002888504 − 66
= 1.076425201
f (x23 ) = −0.001741533
−0.001741533
x24 = 1.076425201 + (2 − 1.076425201) ·
−0.001741533 − 66
= 1.076449571
f (x24 ) = −0.001049952
−0.001049952
x25 = 1.076449571 + (2 − 1.076449571) ·
−0.001049952 − 66
= 1.076464263
f (x25 ) = −0.000632996
Information for Students in MATH 329 2009 01 3065

−0.000632996
x26 = 1.076464263 + (2 − 1.076464263) ·
−0.000632996 − 66
= 1.076473120
f (x26 ) = −0.000381633
−0.000381633
x27 = 1.076473120 + (2 − 1.076473120) ·
−0.000381633 − 66
= 1.076478460
f (x27 ) = −0.000230079
−0.000230079
x28 = 1.076478460 + (2 − 1.076478460) ·
−0.000230079 − 66
= 1.076481679
f (x28 ) = −0.000138723
−0.0001387233
x29 = 1.076481679 + (2 − 1.076481679) ·
−0.0001387233 − 66
= 1.076483620
f (x29 ) = −0.000083635
−0.000083635
x30 = 1.076483620 + (2 − 1.076483620) ·
−0.000083635 − 66
= 1.076484790
f (x30 ) = −0.000050430
−0.000050430
x31 = 1.076484790 + (2 − 1.076484790) ·
−0.000050430 − 66
= 1.076485496
f (x31 ) = −0.000030391
The reason that the method is not efficient here is that the graph of the function is
far from linear in the interval under consideration.38

B.2.4 Fourth 2003/2004 Problem Assignment, with Solutions


Distribution Date: Mounted on the Web on Wednesday, March 31st, 2004
Solutions were due by Wednesday, March 17th, 2004
Corrected as of April 29th, 2004.
(Solutions presented subject to further correction of errors and omissions.)
1. (a) Find, to the nearest unit, the accumulated value 19 years after the first payment
is made of an annuity on which there are 7 payments of 3000 each made at 1 12 -
year intervals. The nominal rate of interest convertible semiannually is 6%.
38
Issues of this type are beyond MATH 329, and are not covered adequately in the current textbook;
if you are interested in Numerical Anaylsis, consider taking MATH 317.
Information for Students in MATH 329 2009 01 3066

(b) Find, to the nearest unit, the present value of a 20-year annuity-due which pays
200 at the beginning of each half-year for the first 8 years, increasing to 250
per half-year thereafter. The effective annual rate of interest is 6%.
Solution:
(a) We will interpret the payments as being made under an annuity-immediate
with time-intervals of 1 12 years. The clock starts ticking (i.e. t = 0) 1 21 years
before the first payment; the last payment is made at time t = 7 × 1.5 = 10.5.
The evaluation is to be made at time t = 19 − 1.5 = 17.5, i.e., 7 years after the
last payment; this is 14
3
intervals of length 1 12 years; it is simpler to view this
as 14 intervals of length 12 year, for each of which the effective interest rate is
3%. The effective interest rate — call it j — per 1 12 years is j = (1.03)3 − 1 .
The accumulated value is, therefore,
3000 ¡ ¢
3000(1.03)14 · s7 = · (1.03)14 · (1 + j)7 − 1
j
3000 14
¡ 21
¢
= · (1.03) · (1.03) − 1
(1.03)3 − 1
= 42100.12386

which is 42,100 to the nearest unit.


1 1
(b) With 1 + i = (1.06) 2 , v = (1.06)− 2 ,

Present Value = 250ä40 − 50ä16


µ ¶ µ ¶
1 − v 39 1 − v 15
= (250 − 50) + 250 − 50
i i
250 (1 − v 39 ) − 50 (1 − v 15 )
= 200 +
i
200 − 250v 39 + 50v 15
= 200 +
i
= 5342.993032.

To the nearest unit the present value is 5343.


2. (a) The present value of a perpetuity-immediate paying 1 at the end of every 5
years is 1.637975. Find i and d.
(b) The present value of a perpetuity-due paying 1 at the beginning of every 5 years
is 1.637975. Find d and i.
Solution:
Information for Students in MATH 329 2009 01 3067

(a) A payment of 1 at the end of every 5th year for 5 years is equivalent to a
payment of s−1
5
at the end of every year. The equation of value is

1
1.637975 = s−1 ·
5 i
1
⇒ 1.637975 =
(1 + i)5 − 1
⇒ (1 + i)5 = 1.610500
⇒ i = 10.0000%,
0.1 1
⇒ d= = .
1.1 11
(b) A payment of 1 at the beginning of every 5th year is equivalent to a payment
of ä−1
5
at the beginning of every year. The equation of value is

1
1.637975 = s̈−1 ·
5 d
1
⇒ 1.637975 =
1 − (1 + i)−5
⇒ (1 + i)−5 = 0.389490
⇒ i = 20.7538%,
⇒ d = 15.1703%.

3. Determine the present value, at a nominal interest rate of 6% compounded quarterly,


of the following payments made under an annuity: 120 at the end of the 3rd year,
110 at the end of the 4th year, decreasing by 10 each year until nothing is paid.
Solution: The payments decrease until a payment of 10 at the end of the 14th year.
We can think of a decreasing annuity-immediate of value 10 (Da)14 starting with a
payment of 140 at the end of the 1st year, and then make corrections. The effective
annual interest rate i is given by
µ ¶4
0.06
1+i= 1+ = (1.015)4 .
4

We need only subtract the present values of the payments due at the end of the
first and second years.

Present Value = 10(Da)14 − 140v − 130v 2


¡ ¢
10 14 − a14
= − 140(1.015)−4 − 130(1.015)−8
i
Information for Students in MATH 329 2009 01 3068

³ ´
1−(1.015)−56
10 14 − (1.015)4 −1
= − 140(1.015)−4 − 130(1.015)−8
(1.015)4 −1
= 532.1438213

4. Find the present value, at an effective annual interest rate of 5.75%, of a perpetuity-
immediate under which a payment of 100 is made at the end of the 1st year, 300 at
the end of the 2nd year, increasing until a payment of 2500 is made, which level is
maintained for exactly a total of 10 payments of 2500 (including the first of them
in the count of 10), after which the payments fall by 400 each year until they reach
a level of 100, which is maintained in perpetuity. (Note: You are expected to show
explicitly how you decompose the payments; it is not sufficient to simply show a
few numbers and a sum.)
Solution: The payments reach the level of 2500 at the end of year 13, and continue
at that level until the end of year 22, after which they fall by 400 annually until
they reach 100 at the end of year 28.
Since the steady state is a constant perpetuity, we can begin with a perpetuity-
100
immediate of 100 per year, whose present value is . The remaining non-zero
0.0575
portions of payments are finite in number: there are additional amounts of 200 at
the end of year 2, 400 at the end of year 3, . . . , 2400 at the ends of years 13 through
22, 2000 at the end of year 23, . . . , and a final amount of 400 at the end of year 27.
The value at time t = 22 of the remaining amounts for years 23 through 27 is
400 · (Da)5 , so the value at time t = 0 is (1.0575)−22 · 400 · (Da)6 . The present value
of the remainders of the payments at the ends of years 2, 3, ..., 12 is (1.0575)−1 ·
200 · (Ia)11 . Thus the sum

100
+ (1.0575)−22 · 400 · (Da)5 + (1.0575)−1 · 200 · (Ia)11
0.0575
covers all the payments except for a level annuity of 2400 payable at the ends of
years ##13. . . 22, whose present value is
¡ ¢
2400 a22 − a12 .

Thus the present value of the entire scheme of payments is


100
+ (1.0575)−22 · 400 · (Da)5 + (1.0575)−1 · 200 · (Ia)11
0.0575 ¡ ¢
+2400 a22 − a12
100 5 − a5 ä − 11v 11
= + (1.0575)−22 · 400 · + (1.0575)−1 · 200 · 11
0.0575 0.0575 0.0575
UPDATED TO April 14, 2009
Information for Students in MATH 329 2009 01 3069

¡ ¢
+2400 a22 − a12
−5
100 5 − 1−(1.0575)
= + (1.0575)−22 · 400 · 0.0575
0.0575 0.0575
1.0575−(1.0575)−10
−1 0.0575
− 11(1.0575)−11
+(1.0575) · 200 ·
0.0575
(1.0575)−12 − (1.0575)−22
+2400 ·
0.0575
= 1739.130435 + 1543.013920 + 8225.826240 + 9138.807264 = 20646.77786 .
5. Find, to the nearest unit, the present value of a 25-year annuity-due which pays
100 immediately, 104 at the end of the 1st year, 108.16 at the end of the 2nd year,
where each subsequent payment is obtained from its predecessor by multiplying by
a factor of 1.04. The annual effective rate of interest is 8.%.
Solution:
24
X
Present Value = 100v n (1.04)n
n=0
¡ 1.04 ¢25
1 − 1.08
= 100 ·
1 − 1.04
1.08
= 1648.998444
or 1649 to the nearest unit.
6. (a) A loan of 15,000 is being repaid with payments of 1,500 at the end of each year
for 20 years. If each payment is immediately reinvested at 6% effective, find
the effective annual rate of interest earned over the 20-year period.
(b) A loan of 15,000 is being repaid with payments of 1,500 at the end of each year
for 10 years. Determine the yield rate to the investor.
Solution:
(a) Let the effective yield rate be i. The payments do not become available until the
maturity date, after 20 years. Until that time they are locked into a payment-
scheme that accumulates to value 1500s20 6% . We are asked for the interest rate
that was earned. There are thus just two transactions: the loan at time 0,
in the amount of 15,000, and the repayment at time 20, in the amount given
above. The equation of value at time t = 0 is
1500s20 6% (1 + i)−20 = 15000
10(0.06)
⇔ (1 + i)−20 =
(1.06)20 − 1
⇔ i = 6.7293555%.

UPDATED TO April 14, 2009


Information for Students in MATH 329 2009 01 3070

(b) We have to determine i such that 1500 · a10 i = 15000. This may appear to be
a difficult problem. But remember that

a10 = v + v 2 + v 3 + . . . + v 10

and that v ≤ 1. That means that the sum cannot be more than 10; in order
for it to equal 10, each of the summands must equal 1, so v = 1. 1 + i = 1, and
i = 0.

B.2.5 Fifth 2003/2004 Problem Assignment, with Solutions


Distribution Date: Mounted on the Web on Monday, April 7th, 2004.
Assignment was mounted on the Web on Wednesday, March 17th, 2004,
hard copy of the assignment was distributed on Friday, March 19th, 2004.
Solutions were to be submitted by Friday, April 2nd, 2004
(SUBJECT TO CORRECTION OF TYPO’S AND OTHER ERRORS)

1. A loan is being repaid with instalments of 1000 at the end of each year for 15 years,
followed by payments of 2000 at the end of each year for 10 years. Interest is at an
effective rate of 4% for the first 10 years, and an effective rate 6% for the next 15
years.
(a) Showing all your work, find the numeric value of the amount of interest paid
in the 4th instalment without making use of a schedule.
(b) Showing all your work, find the amount of principal repaid in the 20th instal-
ment, without making use of a schedule.
(c) Then use the information you have computed to compile the lines of a schedule
corresponding to the payments at the ends of years 20, 21, . . . , 25.
(d) Now solve (a), (b), (c) again, this time assuming that all payments are at the
beginnings of the years: the interest rates remain precisely the same.
Solution:
(a) By the prospective method, the unpaid balance of the loan at time t = 0 is
¡ ¢
1000 · a10 4% + (1.04)−10 2000 · a15 6% − 1000 · a5 6%
1 − (1.04)−10
= 1000 ·
0.04
1 − (1.06)−15 1 − (1.06)−5
+2000 · (1.04)−10 · − 1000 · (1.04)−10 ·
0.06 0.06
= 18387.66857 .
Information for Students in MATH 329 2009 01 3071

By the retrospective method, the unpaid balance just after the 3rd payment of
1000 is, therefore,
(1.04)3 − 1
18387.66857(1.04)3 − 1000 · s3 4% = 18387.66857(1.04)3 − 1000 ·
0.04
= 17562.02642 .

The interest component of the 4th instalment is, therefore,

0.04(17562.02642) = 702.48 .

(b) By the prospective method, the unpaid balance just after the 19th instalment
is
1 − (1.06)−6
2000 · a6 6% = 2000 · = 9834.648653 .
0.06
The interest component of the 20th instalment is, therefore,

0.06(9834.648653) = 590.0789192 ,

so the component for reduction of principal is

2000 − 590.0789192 = 1409.921081 .


Payment Payment Interest Principal Outstanding
Number amount paid repaid loan balance
19 2000 ... ... 9834.65
20 2000 590.08 1409.92 8424.73
(c) 21 2000 505.48 1494.52 6930.21
22 2000 415.81 1584.19 5346.02
23 2000 320.76 1679.23 3666.78
24 2000 220.01 1779.99 1886.79
25 2000 113.21 1886.79 0
(d) Since the only interest rate affecting the last payments has not changed, there
will be no changes at all in (b) or (c).
By the prospective method, the unpaid balance of the loan at time t = 0, just
before the first payment, is
¡ ¢
1000 · ä10 4% + (1.04)−10 2000 · ä15 6% − 1000 · ä5 6%
1 − (1.04)−10 −10 1 − (1.06)
−15
= 1000(1.04) · + 2000(1.06) · (1.04) ·
0.04 0.06
−5
1 − (1.06)
−1000(1.06) · (1.04)−10 ·
0.06
= 19328.71077 .
Information for Students in MATH 329 2009 01 3072

By the retrospective method, the unpaid balance just after the 3rd payment of
1000 is, therefore,
19328.71077(1.04)2 − 1000 · s3 4%
(1.04)3 − 1
= 19328.71077(1.04)2 − 1000 ·
0.04
= 17784.33357 .
The interest component of the 4th instalment is, therefore,
0.04(17784.33357) = 711.37 .
2. (This problem is modelled on [5, Exercise 23, p. 198].)
On a loan of 30,000, the borrower has agreed to pay interest at 7% effective at the
end of each year until the loan is repaid. The borrower has decided to deposit a
fixed amount at the beginning of each year into a sinking fund earning 4% effective.
At the end of 11 years the sinking fund is exactly sufficient to pay off exactly two-
thirds of the loan. He plans to continue accumulating the sinking fund until a year
when a deposit of not more than this fixed amount will bring the fund balance up
to 30,000 and the loan can be immediately repaid.
(a) Calculate the total amount the borrower has to pay out each year (at the
beginning, and at the end), except possibly in the year when the loan is repaid.
(b) Complete the following table to show how the sinking fund attains the target
value of 30,000, and the net amount of the loan after payments ##10, 11, . . .
until the loan is paid off.
Payment Interest Sinking Interest earned Amount in Net amount
Number paid fund deposit on sinking fund sinking fund of loan
10
11
12
...
Solution:
(a) The borrower is paying a constant amount each year of 7% of 30,000, or 2,100 to
cover the interest costs of servicing the loan; denote by X the constant amount
the borrower spends each year; thus the amount she contributes to the sinking
fund is X − 2100. An equation of value at time t = 11 is (X − 2100) · s̈11 4% =
20000, so
20000 × 0.04
X = 2100 + = 3525.943064 .
((1.04)11 − 1) 1.04
Thus the level contribution to the sinking fund at the beginning of each year
except the last is 3525.94 − 2100.00 = 1425.94.
Information for Students in MATH 329 2009 01 3073

(b) The intention is that the column labelled “Amount in sinking fund” shows the
amount just after the payment with the given number.
Payment Interest Sinking Interest earned Amount in Net amount
Number paid fund deposit on sinking fund sinking fund of loan
10 2,100.00 1,425.94 603.62 17,120.03 12,879.97
11 2,100.00 1,425.94 684.80 19,230.77 10,769.23
12 2,100.00 1,425.94 769.23 21,425.64 8,574.36
13 2,100.00 1,425.94 857.03 23,708.61 6,291.39
14 2,100.00 1,425.94 948.34 26,082.89 3,917.11
15 2,100.00 1,425.94 1,043.32 28,552.15 1,447.85
16 2,100.00 305.76 1,142.09 30,000.00 0.00
3. A borrows 12,000 for 10 years, and agrees to make semiannual payments of 1,000,
plus a final payment. The lender receives 12% convertible semiannually on the
investment each year for the first 5 years and 10% convertible semiannually for the
second 5 years. The balance of each payment is invested in a sinking fund earning
8% convertible semiannually.
(a) Find the amount by which the sinking fund is short of repaying the loan at the
end of the 10 years.
(b) Complete the following table to show how the sinking fund attains its maximum
value, and the net amount of the loan after payments.
Payment Interest Sinking Interest earned Amount in Net amount
Number paid fund deposit on sinking fund sinking fund of loan
09
10
11
12
Solution:
(a) The interest payments for the first 10 half-years are 6% of 12,000, i.e. 720 per
half-year; and, for the second 10 half-years, 600 per half-year. This leaves 280
at the end of each of the first 10 half-years, and 400 at the end of each of the
second 10 half-years to accumulate in the sinking fund, which earns 4% effective
every half year. The accumulated balance in the sinking fund at maturity will
be
1 ¡ ¡ ¢ ¡ ¢¢
120s10 4% + 280s20 4% = 120 (1.04)10 − 1 + 280 (1.04)20 − 1
0.04
¡ ¢
= 25 120(1.04)10 + 280(1.04)20 − 400
= 9778.594855
implying that the shortfall to repay the loan will be 12, 000−9778.59 = 2221.41.
Information for Students in MATH 329 2009 01 3074

(b) Just after the 8th payment the balance in the fund is
280 ((1.04)8 − 1)
280 · s8 4% = = 2579.98 .
0.04
Interest earned by the 9th payments is 103.20. We can now begin to fill in the
schedule:
Payment Interest Sinking Interest earned Amount in Net amount
Number paid fund deposit on sinking fund sinking fund of loan
09 720.00 280.00 103.20 2963.18 9036.82
10 720.00 280.00 118.53 3361.71 8638.29
11 600.00 400.00 134.47 3896.18 8103.82
12 600.00 400.00 155.85 4452.03 7547.97
4. (a) A borrower takes out a loan of 3000 for 10 years at 8% convertible semiannually.
The borrower replaces one-third of the principal in a sinking fund earning 5%
convertible semiannually, and the other two-thirds in a sinking fund earning
7% convertible semiannually. Find the total semiannual payment.
(b) Rework (a) if the borrower each year puts one-third of the total sinking fund
deposit into the 5% sinking fund and the other two-thirds into the 7% sinking
fund.
Solution:
(a) The semiannual contribution to the sinking funds is
1000 2000
+
s20 2.5% s20 3.5%
and the semiannual interest payment is 4% of 3, 000, or 120. Hence the total
semiannual payment is
1000 2000 25 70
+ + 120 = + + 120
s20 2.5% s20 3.5% (1.025) − 1 (1.035)20 − 1
20

= 229.8692824
(b) Let the total sinking fund deposit be D. Then the equation of value at maturity
is
D 2D
· s20 2.5% + · s20 3.5% = 3000 ,
3 3
implying that
9000
D =
s20 2.5% + 2s20 3.5%
9000
= (1.025)20 −1 20 −1

0.025
+ 2 · (1.035)
0.035
= 109.6170427 ,
Information for Students in MATH 329 2009 01 3075

so the total semi-annual payment is 109.6170427+120=229.6170427.


5. A payment of 800 is made at the end of each month for 10 years to repay a loan
of 30,000. The borrower replaces the capital by means of a sinking fund earning a
nominal annual rate of 6% compounded monthly.
(a) Find the effective annual rate i paid to the lender on the loan.
(b) Suppose that the lender had elected to amortize the loan by equal monthly
payments, at the same rate as he is now paying to the lender. What would be
the amount of those equal payments?
Solution:
(a) The monthly contribution to the sinking fund is
30000 600
= = 183.0615058 .
s120 0.5% (1.005)120 − 1
Hence the monthly interest payment is 800 − 183.0615058 = 616.9384942, i.e.,
2.056461647% of the principal of 30,000. The effective annual rate is, therefore,

(1.02056461647)12 − 1 = 27.6691838%.

(b) The level monthly payment necessary to amortize a loan of 30000 over 120
payments at an effective rate of 2.056461647% is
30000 (30000)(0.02056461647)
= = 675.6703969 .
a120 2.056461647% 1 − (1.02056461647)−120
(The amount is lower than the borrower is now paying because he is holding
funds in his sinking fund and earning less there than he his paying the lender
for the use of the capital.)

B.2.6 2003/2004 Class Test, Version 1


Instructions

• The time available for writing this test is about 45 minutes.


• This test booklet consists of this cover, Pages 3076 through 3076 containing ques-
tions together worth 60 marks; and Page 3076, which is blank.
• Show all your work. All solutions are to be written in the space provided on the
page where the question is printed. When that space is exhausted, you may write
on the facing page, on the blank page, or on the back cover of the booklet, but you
must indicate any continuation clearly on the page where the question is printed!
(Please inform the instructor if you find that your booklet is defective.)
Information for Students in MATH 329 2009 01 3076

• All your writing — even rough work — must be handed in.


• Calculators. While you are permitted to use a calculator to perform arithmetic
and/or exponential calculations, you must not use the calculator to calculate such
actuarial functions as ani , sni , (Ia)ni , (Is)ni , (Da)ni , (Ds)ni , etc. without first
stating a formula for the value of the function in terms of exponentials and/or
polynomials involving n and the interest rate. You must not use your calculator in
any programmed calculations. If your calculator has memories, you are expected
to have cleared them before the test.
• In your solutions to problems on this test you are expected to show all your work .
You are expected to simplify algebraic and numerical answers as much as you can.
• Your neighbours may be writing a version of this test which is different from yours.

1. (a) [5 MARKS] Suppose that the nominal annual rate of interest, compounded 8
times per year, is 5%. Showing all your work, determine the equivalent effective
annual rate of discount.
(b) [5 MARKS] Suppose that the nominal annual rate of discount, compounded 3
times per year, is 7%. Showing all your work, determine the equivalent annual
rate of interest.
(c) [5 MARKS] State the nominal annual interest rate, compounded instanta-
neously, which is equivalent to an effective annual interest rate of 4%.
1
(d) [5 MARKS] Suppose that the effective interest rate for year is 3%. Determine
4
the equivalent nominal interest rate, compounded every 2 years.
2. [15 MARKS] The accumulated value just after the last payment under a 12-year
annuity of 1000 per year, paying interest at the rate of 5% per annum effective,
is to be used to purchase a perpetuity at an interest rate of 6%, first payment to
be made 1 year after the last payment under the annuity. Showing all your work,
determine the size of the payments under the perpetuity.
3. [25 MARKS] A loan of 5000 is to be repaid by annual payments of 250 to commence
at the end of the 6th year, and to continue thereafter for as long as necessary. Find
the time and amount of the final payment if the final payment is to be larger than
the regular payments. Assume i = 4%.

continuation page for problem number

You must refer to this continuation page on the page where the problem is printed!
Information for Students in MATH 329 2009 01 3077

B.2.7 2003/2004 Class Test, Version 2


Instructions
• The time available for writing this test is about 45 minutes.
• This test booklet consists of this cover, Pages 3077 through 3078 containing ques-
tions together worth 60 marks; and Page 3078, which is blank.
• Show all your work. All solutions are to be written in the space provided on the
page where the question is printed. When that space is exhausted, you may write
on the facing page, on the blank page, or on the back cover of the booklet, but you
must indicate any continuation clearly on the page where the question is printed!
(Please inform the instructor if you find that your booklet is defective.)
• All your writing — even rough work — must be handed in.
• Calculators. While you are permitted to use a calculator to perform arithmetic
and/or exponential calculations, you must not use the calculator to calculate such
actuarial functions as ani , sni , (Ia)ni , (Is)ni , (Da)ni , (Ds)ni , etc. without first
stating a formula for the value of the function in terms of exponentials and/or
polynomials involving n and the interest rate. You must not use your calculator in
any programmed calculations. If your calculator has memories, you are expected
to have cleared them before the test.
• In your solutions to problems on this test you are expected to show all your work .
You are expected to simplify algebraic and numerical answers as much as you can.
• Your neighbours may be writing a version of this test which is different from yours.

1. [25 MARKS] A loan of 1000 is to be repaid by annual payments of 100 to commence


at the end of the 5th year, and to continue thereafter for as long as necessary. Find
the time and amount of the final payment if the final payment is to be NO larger
than the regular payments. Assume i = 4.5%.
1
2. (a) [5 MARKS] Suppose that the effective interest rate for year is 0.02. Determine
5
the equivalent nominal interest rate, compounded every 3 years.
(b) [5 MARKS] Suppose that the nominal annual rate of interest, compounded 9
times per year, is 6%. Showing all your work, determine the equivalent effective
annual rate of discount.
(c) [5 MARKS] Suppose that the nominal annual rate of discount, compounded 6
times per year, is 5%. Showing all your work, determine the equivalent annual
rate of interest.
(d) [5 MARKS] State the nominal annual interest rate, compounded instanta-
neously, which is equivalent to an effective annual interest rate of 8%.
Information for Students in MATH 329 2009 01 3078

3. [15 MARKS] The accumulated value just after the last payment under a 12-year
annuity of 1000 per year, paying interest at the rate of 5% per annum effective,
is to be used to purchase a perpetuity of 500 per annum forever, first payment to
be made 1 year after the last payment under the annuity. Showing all your work,
determine the effective interest rate of the perpetuity, assuming it comes into effect
just after the last payment under the annuity.

B.2.8 2003/2004 Class Test, Version 3


Instructions

• The time available for writing this test is about 45 minutes.


• This test booklet consists of this cover, Pages 3078 through 3079 containing ques-
tions together worth 60 marks; and Page 3079, which is blank.
• Show all your work. All solutions are to be written in the space provided on the
page where the question is printed. When that space is exhausted, you may write
on the facing page, on the blank page, or on the back cover of the booklet, but you
must indicate any continuation clearly on the page where the question is printed!
(Please inform the instructor if you find that your booklet is defective.)
• All your writing — even rough work — must be handed in.
• Calculators. While you are permitted to use a calculator to perform arithmetic
and/or exponential calculations, you must not use the calculator to calculate such
actuarial functions as ani , sni , (Ia)ni , (Is)ni , (Da)ni , (Ds)ni , etc. without first
stating a formula for the value of the function in terms of exponentials and/or
polynomials involving n and the interest rate. You must not use your calculator in
any programmed calculations. If your calculator has memories, you are expected
to have cleared them before the test.
• In your solutions to problems on this test you are expected to show all your work .
You are expected to simplify algebraic and numerical answers as much as you can.
• Your neighbours may be writing a version of this test which is different from yours.

1. [15 MARKS] The accumulated value just after the last payment under a 9-year
annuity of 2000 per year, paying interest at the rate of 8% per annum effective,
is to be used to purchase a perpetuity at an interest rate of 4%, first payment to
be made 1 year after the last payment under the annuity. Showing all your work,
determine the size of the payments under the perpetuity.
Information for Students in MATH 329 2009 01 3079

2. [25 MARKS] A loan of 1000 is to be repaid by annual payments of 200 to commence


at the end of the 4th year, and to continue thereafter for as long as necessary. Find
the time and amount of the final payment if the final payment is to be larger than
the regular payments. Assume i = 5%.
3. (a) [5 MARKS] State the nominal annual interest rate, compounded instanta-
neously, which is equivalent to an effective annual interest rate of 6%.
1
(b) [5 MARKS] Suppose that the effective interest rate for year is 0.015. Deter-
6
mine the equivalent nominal interest rate, compounded every 4 years.
(c) [5 MARKS] Suppose that the nominal annual rate of interest, compounded 3
times per year, is 8%. Showing all your work, determine the equivalent effective
annual rate of discount.
(d) [5 MARKS] Suppose that the nominal annual rate of discount, compounded 12
times per year, is 6%. Showing all your work, determine the equivalent annual
rate of interest.

continuation page for problem number

You must refer to this continuation page on the page where the problem is printed!

B.2.9 2003/2004 Class Test, Version 4


Instructions

• The time available for writing this test is about 45 minutes.


• This test booklet consists of this cover, Pages 3080 through 3080 containing ques-
tions together worth 60 marks; and Page 3080, which is blank.
• Show all your work. All solutions are to be written in the space provided on the
page where the question is printed. When that space is exhausted, you may write
on the facing page, on the blank page, or on the back cover of the booklet, but you
must indicate any continuation clearly on the page where the question is printed!
(Please inform the instructor if you find that your booklet is defective.)
• All your writing — even rough work — must be handed in.
• Calculators. While you are permitted to use a calculator to perform arithmetic
and/or exponential calculations, you must not use the calculator to calculate such
actuarial functions as ani , sni , (Ia)ni , (Is)ni , (Da)ni , (Ds)ni , etc. without first
Information for Students in MATH 329 2009 01 3080

stating a formula for the value of the function in terms of exponentials and/or
polynomials involving n and the interest rate. You must not use your calculator in
any programmed calculations. If your calculator has memories, you are expected
to have cleared them before the test.
• In your solutions to problems on this test you are expected to show all your work .
You are expected to simplify algebraic and numerical answers as much as you can.
• Your neighbours may be writing a version of this test which is different from yours.

1. (a) [5 MARKS] Suppose that the nominal annual rate of discount, compounded
6 times per year, is 0.5%. Showing all your work, determine the equivalent
annual rate of interest.
(b) [5 MARKS] State the nominal annual interest rate, compounded instanta-
neously, which is equivalent to an effective annual interest rate of 10%.
1
(c) [5 MARKS] Suppose that the effective interest rate for year is 2%. Determine
5
the equivalent nominal interest rate, compounded every 6 years.
(d) [5 MARKS] Suppose that the nominal annual rate of interest, compounded 7
times per year, is 2%. Showing all your work, determine the equivalent effective
annual rate of discount.
2. [25 MARKS] A loan of 1000 is to be repaid by annual payments of 200 to commence
at the end of the 4th year, and to continue thereafter for as long as necessary. Find
the time and amount of the final payment if the final payment is to be NO larger
than the regular payments. Assume i = 5%.
3. [15 MARKS] The accumulated value just after the last payment under a 10-year
annuity of 1000 per year, paying interest at the rate of 6% per annum effective,
is to be used to purchase a perpetuity of 800 per annum forever, first payment to
be made 1 year after the last payment under the annuity. Showing all your work,
determine the effective interest rate of the perpetuity, assuming it comes into effect
just after the last payment under the annuity.

continuation page for problem number

You must refer to this continuation page on the page where the problem is printed!
Information for Students in MATH 329 2009 01 3081

B.2.10 Solutions to Problems on the 2003/2004 Class Tests


The first four problems listed are concerned with equivalent rates of interest and discount
(each in 4 parts); the next four concern annuities and perpetuities; and the last four are
concerned with unknown type and final balloon or drop payments.
1. (a) [5 MARKS] [VERSION 1 #1(a)] Suppose that the nominal annual rate of
interest, compounded 8 times per year, is 5%. Showing all your work, determine
the equivalent effective annual rate of discount.
Solution: We are given that i(8) = 0.05, and asked to determine d.
µ ¶8
i(8)
1+ = 1+i
8
and (1 − d)(1 + i) = 1
µ ¶−8
i(8)
⇒d = 1− 1+ = 0.0486225508 = 4.86%
8
(b) [5 MARKS] [VERSION 1 #1(b)] Suppose that the nominal annual rate of dis-
count, compounded 3 times per year, is 7%. Showing all your work, determine
the equivalent annual rate of interest.
Solution: We are given that d(3) = 0.07, and asked to determine i.
µ ¶3
d(3)
1− = 1−d
3
and (1 − d)(1 + i) = 1
µ ¶−3
d(3)
⇒i = 1− − 1 = 0.073398300 = 7.34%
3
(c) [5 MARKS] [VERSION 1 #1(c)] State the nominal annual interest rate, com-
pounded instantaneously, which is equivalent to an effective annual interest rate
of 4%.
Solution: The rate we seek is the solution i to the equation
µ ¶n
i
lim 1 + = 1.04
n→∞ n
which is equivalent to ei = 1.04. Solving by taking logarithms, we obtain
i = ln(1.04) = 0.039220713151 = 3.92%.
1
(d) [5 MARKS] [VERSION 1 #1(d)] Suppose that the effective interest rate for
4
year is 3%. Determine the equivalent nominal interest rate, compounded every
2 years.
Information for Students in MATH 329 2009 01 3082

i(4) 1
Solution: We are given the value of = 3%, and asked to determine i( 2 ) .
4
The equation we have to solve is
µ ¶4 Ã ! 12
i( 2 )
1
i(4)
1+ =1+i= 1+ 1 .
4 2

This equation implies that


õ ¶ !
(4) 8
1 i
i( 2 ) =
1
1+ − 1 = 0.1333850405 = 13.3%.
2 4

2. (a) [5 MARKS] [VERSION 2 #2(b)] Suppose that the nominal annual rate of
interest, compounded 9 times per year, is 6%. Showing all your work, determine
the equivalent effective annual rate of discount.
Solution: We are given that i(9) = 0.06, and asked to determine d.
µ ¶9
i(9)
1+ = 1+i
9
and (1 − d)(1 + i) = 1
µ ¶−9
i(9)
⇒d = 1− 1+ = 0.0580479306 = 5.80%
9
(b) [5 MARKS] [VERSION 2 #2(c)] Suppose that the nominal annual rate of dis-
count, compounded 6 times per year, is 5%. Showing all your work, determine
the equivalent annual rate of interest.
Solution: We are given that d(6) = 0.05, and asked to determine i.
µ ¶6
d(6)
1− = 1−d
6
and (1 − d)(1 + i) = 1
µ ¶−6
d(6)
⇒i = 1− − 1 = 0.051491358 = 5.15%
6
(c) [5 MARKS] [VERSION 2 #2(d)] State the nominal annual interest rate, com-
pounded instantaneously, which is equivalent to an effective annual interest rate
of 8%.
Solution: The rate we seek is the solution to the equation
µ ¶n
i
lim 1 + = 1.08
n→∞ n
Information for Students in MATH 329 2009 01 3083

which is equivalent to ei = 1.08. Solving by taking logarithms, we obtain


i = ln(1.08) = 0.07696104114 = 7.70%.
(d) [5 MARKS] [VERSION 2 #2(a)] Suppose that the effective interest rate for
1
year is 0.02. Determine the equivalent nominal interest rate, compounded
5
every 3 years.
i(5) 1
Solution: We are given the value of = 2%, and asked to determine i( 3 ) .
5
The equation we have to solve is
µ ¶5 Ã ! 13
i( 3 )
1
(5)
i
1+ =1+i= 1+ 1 .
5 3

This equation implies that


õ ¶ !
(5) 15
1 i
i( 3 ) =
1
1+ − 1 = 0.1152894460 = 11.5%
3 5

3. (a) [5 MARKS] [VERSION 3 #3(c)] Suppose that the nominal annual rate of
interest, compounded 3 times per year, is 8%. Showing all your work, determine
the equivalent effective annual rate of discount.
Solution: We are given that i(3) = 0.08, and asked to determine d.
µ ¶3
i(3)
1+ = 1+i
3
and (1 − d)(1 + i) = 1
µ ¶−3
i(3)
⇒d = 1− 1+ = 0.0759156521 = 7.59%
3

(b) [5 MARKS] [VERSION 3 #3(d)] Suppose that the nominal annual rate of dis-
count, compounded 12 times per year, is 6%. Showing all your work, determine
the equivalent annual rate of interest.
Solution: We are given that d(12) = 0.06, and asked to determine i.
µ ¶12
d(12)
1− = 1−d
12
and (1 − d)(1 + i) = 1
µ ¶−12
d(12)
⇒i = 1− − 1 = 0.061996367 = 6.20%
12
Information for Students in MATH 329 2009 01 3084

(c) [5 MARKS] [VERSION 3 #3(a)] State the nominal annual interest rate, com-
pounded instantaneously, which is equivalent to an effective annual interest rate
of 6%.
Solution: The rate we seek is the solution to the equation
µ ¶n
i
lim 1 + = 1.06
n→∞ n

which is equivalent to ei = 1.06. Solving by taking logarithms, we obtain


i = ln(1.06) = 0.05826890812 = 5.83%.
(d) [5 MARKS] [VERSION 3 #3(b)] Suppose that the effective interest rate for
1
year is 0.015. Determine the equivalent nominal interest rate, compounded
6
every 4 years.
i(6) 1
Solution: We are given the value of = 1.5%, and asked to determine i( 4 ) .
6
The equation we have to solve is
µ ¶6 Ã ! 14
i( 4 )
1
i(6)
1+ =1+i= 1+ 1 .
6 4

This equation implies that


õ ¶ !
(6) 24
1 i
i( 4 ) =
1
1+ − 1 = .1073757030 = 10.7%
4 6

4. (a) [5 MARKS] [VERSION 4 #1(d)] Suppose that the nominal annual rate of
interest, compounded 7 times per year, is 2%. Showing all your work, determine
the equivalent effective annual rate of discount.
Solution: We are given that i(7) = 0.02, and asked to determine d.
µ ¶7
i(7)
1+ = 1+i
7
and (1 − d)(1 + i) = 1
µ ¶−7
i(7)
⇒d = 1− 1+ = 0.197733746 = 1.98%
7

(b) [5 MARKS] [VERSION 4 #1(a)] Suppose that the nominal annual rate of dis-
count, compounded 6 times per year, is 0.5%. Showing all your work, determine
the equivalent annual rate of interest.
Information for Students in MATH 329 2009 01 3085

Solution: We are given that d(6) = 0.005, and asked to determine i.


µ ¶6
d(6)
1− = 1−d
6
and (1 − d)(1 + i) = 1
µ ¶−6
d(6)
⇒i = 1− − 1 = 0.005014616 = 0.5014616%
6

(c) [5 MARKS] [VERSION 4 #1(b)] State the nominal annual interest rate, com-
pounded instantaneously, which is equivalent to an effective annual interest rate
of 10%.
Solution: The rate we seek is the solution to the equation
µ ¶n
i
lim 1 + = 1.10
n→∞ n

which is equivalent to ei = 1.10. Solving by taking logarithms, we obtain


i = ln(1.10) = 0.09531017980 = 9.53%.
1
(d) [5 MARKS] [VERSION 4 #1(c)] Suppose that the effective interest rate for
5
year is 2%. Determine the equivalent nominal interest rate, compounded every
6 years.
i(5) 1
Solution: We are given the value of = 2%, and asked to determine i( 6 ) .
5
The equation we have to solve is
µ ¶5 Ã ! 16
i( 6 )
1
i(5)
1+ =1+i= 1+ 1 .
5 6

This equation implies that


õ ¶ !
(5) 30
1 i
i( 6 ) =
1
1+ − 1 = 0.1352269307 = 13.5%.
6 5

5. [15 MARKS] [VERSION 1 #2] The accumulated value just after the last payment
under a 12-year annuity of 1000 per year, paying interest at the rate of 5% per
annum effective, is to be used to purchase a perpetuity at an interest rate of 6%,
first payment to be made 1 year after the last payment under the annuity. Showing
all your work, determine the size of the payments under the perpetuity.
Information for Students in MATH 329 2009 01 3086

Solution: Let X be the level payment under the perpetuity. The equation of value
just after the last annuity payment is

1000 · s12 5% = X · a∞ 6%

implying that
s12 5%
X = 1000 ·
a∞ 6%
6 ¡ ¢
= 1000 · · (1.05)12 − 1 = 955.03.
5

6. [15 MARKS] [VERSION 2 #3] The accumulated value just after the last payment
under a 12-year annuity of 1000 per year, paying interest at the rate of 5% per
annum effective, is to be used to purchase a perpetuity of 500 per annum forever,
first payment to be made 1 year after the last payment under the annuity. Showing
all your work, determine the effective interest rate of the perpetuity, assuming it
comes into effect just after the last payment under the annuity.
Solution: Let i be the interest rate of the perpetuity. The equation of value just
after the last annuity payment is
500
1000 · s12 5% = 500 · a∞ i =
i
implying that
500 1
i = ·
1000 s12 5%
500 0.05
= ·
1000 ((1.05)12 − 1)
= 3.14%.

7. [15 MARKS] [VERSION 3 #1] The accumulated value just after the last payment
under a 9-year annuity of 2000 per year, paying interest at the rate of 8% per
annum effective, is to be used to purchase a perpetuity at an interest rate of 4%,
first payment to be made 1 year after the last payment under the annuity. Showing
all your work, determine the size of the payments under the perpetuity.
Solution: Let X be the level payment under the perpetuity. The equation of value
just after the last annuity payment is

2000 · s9 8% = X · a∞ 4%
Information for Students in MATH 329 2009 01 3087

implying that
s9 8%
X = 2000 ·
a∞ 4%
4 ¡ ¢
= 2000 · · (1.08)9 − 1 = 999.00
8
8. [15 MARKS] [VERSION 4 #3] The accumulated value just after the last payment
under a 10-year annuity of 1000 per year, paying interest at the rate of 6% per
annum effective, is to be used to purchase a perpetuity of 800 per annum forever,
first payment to be made 1 year after the last payment under the annuity. Showing
all your work, determine the effective interest rate of the perpetuity, assuming it
comes into effect just after the last payment under the annuity.
Solution: Let i be the interest rate of the perpetuity. The equation of value just
after the last annuity payment is
800
1000 · s10 6% = 800 · a∞ i =
i
implying that
800 1
i = ·
1000 s10 6%
800 0.06
= ·
1000 ((1.06)10 − 1)
= 6.07%.
9. [25 MARKS] [VERSION 1 #3] A loan of 5000 is to be repaid by annual payments
of 250 to commence at the end of the 6th year, and to continue thereafter for as long
as necessary. Find the time and amount of the final payment if the final payment
is to be larger than the regular payments. Assume i = 4%.
Solution: (cf. [5, Exercise 32, p. 91]) Let the time of the last — balloon — payment
be n, and let the amount of the last payment be X. Then n is the largest integer
solution to the inequality
1 − (1.04)−(n−5)
5000 ≥ 250(1.04)−5 an−5 = 250(1.04)−5 ·
0.04
5000 × 0.04 × (1.04)5
⇔ (1.04)−(n−5) ≥ 1 −
¡ 250 ¢
⇔ −(n − 5) ln 1.04 ≥ ln 1 − 20 × 0.04 × (1.04)5
ln (1 − 0.8(1.04)5 )
⇔ −(n − 5) ≥
ln 1.04
ln (1 − 0.8(1.04)5 )
⇔ n≤5− = 97.39832188.
ln 1.04
Information for Students in MATH 329 2009 01 3088

Thus we conclude that the balloon payment is made at time t = 97. The equation
of value at time t = 22 is

5000(1.04)97 = 250s92 + (X − 250)

implying that
250 ¡ ¢
X = 250 + 5000(1.04)97 − (1.04)92 − 1 = 346.8818 .
0.04

10. [25 MARKS] [VERSION 2 #1] A loan of 1000 is to be repaid by annual payments
of 100 to commence at the end of the 5th year, and to continue thereafter for as long
as necessary. Find the time and amount of the final payment if the final payment
is to be NO larger than the regular payments. Assume i = 4.5%.
Solution: Let the time of the last — drop — payment be n, and let the amount of
the last payment be X. Then n is the smallest integer solution to the inequality

1 − (1.045)−(n−4)
1000 ≤ 100(1.045)−4 an−4 = 100(1.045)−4 ·
0.045
4
1000 × 0.045 × (1.045)
⇔ (1.045)−(n−4) ≤ 1 −
¡ 100 ¢
⇔ −(n − 4) ln 1.045 ≤ ln 1 − 10 × 0.045 × (1.045)4
ln (1 − 0.45(1.045)4 )
⇔ −(n − 4) ≤
ln 1.045
ln (1 − 0.45(1.045)4 )
⇔ n≥4− = 21.47594530.
ln 1.045
Thus we conclude that the drop payment is made at time t = 22. The equation of
value at time t = 22 is
1000(1.045)22 = 100s̈17 + X
implying that
100 × 1.045 ¡ ¢
X = 1000(1.045)22 − (1.045)17 − 1 = 48.143638 .
0.045

11. [25 MARKS] [VERSION 3 #2] A loan of 1000 is to be repaid by annual payments
of 200 to commence at the end of the 4th year, and to continue thereafter for as long
as necessary. Find the time and amount of the final payment if the final payment
is to be larger than the regular payments. Assume i = 5%.
Solution: (cf. [5, Exercise 32, p. 91]) Let the time of the last — balloon — payment
be n, and let the amount of the last payment be X. Then n is the largest integer
Information for Students in MATH 329 2009 01 3089

solution to the inequality

−3 −3 1 − (1.05)−(n−3)
1000 ≥ 200(1.05) an−3 = 200(1.05) ·
0.05
3
1000 × 0.05 × (1.05)
⇔ (1.05)−(n−3) ≥ 1 −
µ 200 ¶
1000 × 0.05 × (1.05)3
⇔ −(n − 3) ln 1.05 ≥ ln 1 −
200
ln (1 − 0.25(1.05)3 )
⇔ −(n − 3) ≥
ln 1.05
ln (1 − 0.25(1.05)3 )
⇔ n≤3− = 10.00252595
ln 1.05
Thus we conclude that the balloon payment is made at time t = 10. The equation
of value at time t = 10 is
1000(1.05)10 = 200s7 + (X − 200)
implying that
200 ¡ ¢
X = 200 + 1000(1.05)10 − (1.05)7 − 1 = 200.492935 .
0.05
12. [25 MARKS] [VERSION 4 #2] A loan of 1000 is to be repaid by annual payments
of 200 to commence at the end of the 4th year, and to continue thereafter for as long
as necessary. Find the time and amount of the final payment if the final payment
is to be NO larger than the regular payments. Assume i = 5%.
Solution: Let the time of the last — drop — payment be n, and let the amount of
the last payment be X. Then n is the smallest integer solution to the inequality
1 − (1.05)−(n−3)
1000 ≤ 200(1.05)−3 an−3 = 200(1.05)−3 ·
0.05
1000 × 0.05 × (1.05)3
⇔ (1.05)−(n−3) ≤ 1 −
¡ 200 ¢
⇔ −(n − 3) ln 1.05 ≤ ln 1 − 5 × 0.05 × (1.05)3
ln (1 − 0.25(1.05)3 )
⇔ −(n − 3) ≤
ln 1.05
ln (1 − 0.25(1.05)3 )
⇔ n≥3− = 10.00252595.
ln 1.05
Thus we conclude that the drop payment is made at time t = 11. The equation of
value at time t = 11 is
1000(1.05)11 = 200s̈7 + X
Information for Students in MATH 329 2009 01 3090

implying that
200 × 1.05 ¡ ¢
X = 1000(1.05)11 − (1.05)7 − 1 = 0.517581.
0.05

B.2.11 Final Examination, 2003/2004


1. In each of the following problems you are expected to show all your work.
(a) [3 MARKS] If v = 0.97, determine the value of d(4) .
1
(b) [3 MARKS] If i(12) = 6%, determine the value of i( 2 ) .
(c) [3 MARKS] Showing all your work, determine the nominal interest rate, com-
pounded semi-annually, under which a sum of money will triple in 12 years.
(d) [3 MARKS] Showing all your work, determine the rate of interest, convertible
continuously, that is equivalent to a nominal interest rate of 8% per annum,
convertible monthly.
2. In each of the following problems, give a formula in terms of i alone; then evaluate
the formula and determine the numerical value.
(a) [5 MARKS] Determine the present value of a perpetuity-due of 100 payable
every three months, at an effective annual interest rate of 6%.
(b) [5 MARKS] The present value of a perpetuity-immediate paying 1000 at the
125, 000
end of every 3 years is .Determine the effective annual interest rate.
91
3. In each of the following problems, give a formula in terms of i alone; then evaluate
the formula and determine its numerical value.
(a) [7 MARKS] At a nominal annual interest rate of 8% compounded quarterly,
determine the value, 2 years after the last payment, of a decreasing annuity
paying 5,000 at the end of the first half-year, 4,500 at the end of the 2nd half-
year, and continuing to decrease at 500 per half-year until the final payment of
500.
(b) [8 MARKS] Three years before the first payment, determine the value of an
annuity that pays 4,000 the first year, 3,900 the second year, with payments
continuing to decrease by 100 until it pays 2,000 per year, after which it pays
2,000 forever. The interest rate is 5% effective per year until the payment of
3,000, after which the interest rate becomes 4% effective forever.
4. One of the following equations is always true, and one is true only when i = 0.
1 1
I. = +i
an i sn i
1 1 1
II. = +
sn i an i i
Information for Students in MATH 329 2009 01 3091

Table 3: Several Useful Formulas that you were not expected to memorize

än i −nv n
(Ia)n i = i
n−an i
s̈n i −n (Da)n i = i
(Is)n i = i
n(1+i)n −sn i
s
n+1 i
−(n+1) (Ds)n i = i
(Is)n i = i

1 1 1
III. = +
an i sn i i
1 1
IV. = +i
sn i an i
(a) Explain which is always true, and prove it
i. [4 MARKS] algebraically; and
ii. [4 MARKS] by a verbal argument, referring to a sinking fund.
(b) [4 MARKS] Prove algebraically that one of the other equations is true for i = 0.
5. The purchase of a new condominium is partially financed by a mortgage of 120,000
payable to the vendor; the mortgage is amortized over 25 years, with a level payment
at the end of each half-month, at a nominal annual rate of 6.6% compounded every
half-month.
(a) [3 MARKS] Determine the half-monthly payments under this mortgage.
(b) [2 MARKS] Divide the 1st payment into principal and interest.
(c) [3 MARKS] Determine the outstanding principal immediately after the 50th
payment.
(d) [4 MARKS] Divide the 52nd payment into principal and interest.
(e) [3 MARKS] The amortization by half-monthly payments was designed to ac-
commodate the purchase, whose salary was being deposited automatically to
his bank account every half-month. The purchase changes his profession, after
2 years, and now would prefer to make a single payment once every half-year.
Determine the amount of that payment if the interest rates are unchanged, but
if the mortgage is now amortized to be paid off 5 years earlier than previously.
Information for Students in MATH 329 2009 01 3092

6. (a) [8 MARKS] Find the price of the following bond, which is purchased to yield
6% convertible semi-annually: the bond has face value of 10,000, matures in 15
years at a maturity value of 11,500, and has a nominal coupon rate of 9% per
annum, compounded semi-annually; the investor is replacing the principal by
means of a sinking fund earning 7% convertible semi-annually.
(b) [7 MARKS] Suppose that the bond is callable when t = 13 at a premium of
1, 000 above the maturity value. Explain what price the investor should pay if
he is no longer plans to deposit any of the interest in a sinking fund.
7. A loan is being repaid with 15 annual payments of 1,000 each. At the time of the
5th payment the borrower is permitted to pay an extra 2000, and then to repay the
balance over 5 years with a revised annual payment.
(a) [5 MARKS] If the effective annual rate of interest is 6%, find the amount of the
revised annual payment.
(b) [8 MARKS] Complete an amortization table for the full 10 years of the loan,
with the following columns:
Payment Payment Interest Principal Outstanding
number amount paid repaid loan balance
0
1
... ... ... ... ...
10
8. [8 MARKS] It was n years ago when James deposited 10,000 in a bank paying
2.4% interest compounded monthly. If he had, instead, placed his deposit in a
syndicate paying interest by cheque annually at the rate of 5% per annum, and he
had invested only this interest with the bank, how much more interest would he
have earned altogether? Show all your reasoning, and express your answer in terms
of n.

B.2.12 Supplemental/Deferred Examination, 2003/2004


1. In each of the following problems you are expected to show all your work.
(a) [3 MARKS] If v = 0.95, determine the value of d(3) .
(b) [3 MARKS] Showing all your work, determine the nominal interest rate, com-
pounded quarterly, under which a sum of money will double in 10 years.
(c) [3 MARKS] Showing all your work, determine the rate of discount, convertible
continuously, that is equivalent to a nominal discount rate of 8% per annum,
convertible semi-annually.
(d) [3 MARKS] If i( 2 ) = 1 , determine the value of i(12) .
1

30
Information for Students in MATH 329 2009 01 3093

Table 4: Several Useful Formulas that you were not expected to memorize

än i −nv n
(Ia)n i = i
n−an i
s̈n i −n (Da)n i = i
(Is)n i = i
n(1+i)n −sn i
s
n+1 i
−(n+1) (Ds)n i = i
(Is)n i = i

2. You must show all your work in solving the following problems:
(a) [5 MARKS] Determine the present value of a perpetuity-immediate of 1000
payable every three months, at an effective annual interest rate of 8%.
(b) [5 MARKS] The present value of a perpetuity-immediate paying 100 at the end
129, 600
of every 4 years is . Determine the effective annual interest rate.
1, 105
3. Show detailed work in your solutions to each of these problems.
(a) [7 MARKS] At a nominal annual interest rate of 6% compounded quarterly,
determine the value — 5 years after the last payment — of a decreasing annuity
paying 6,000 at the end of the first half-year, 5,500 at the end of the 2nd half-
year, and continuing to decrease at 500 per half-year until the final payment of
500.
(b) [8 MARKS] Three years before the first payment, determine the present value of
an annuity that pays 6,000 the first year, 5,900 the second year, with payments
continuing to decrease by 100 until it pays 4,000 per year, after which it pays
4,000 forever. The interest rate is 8% effective per year until the first payment
of 4,000, after which the interest rate becomes 5% effective forever.
4. The purchase of a new condominium is partially financed by a mortgage of 120,000
payable to the vendor; the mortgage is amortized over 35 years, with a level payment
at the end of each half-month, at a nominal annual rate of 9.6% compounded every
half-month.
(a) [3 MARKS] Determine the half-monthly payments under this mortgage.
(b) [2 MARKS] Divide the 1st payment into principal and interest.
Information for Students in MATH 329 2009 01 3094

(c) [3 MARKS] Determine the outstanding principal immediately after the 60th
payment.
(d) [4 MARKS] Divide the 62nd payment into principal and interest.
(e) [3 MARKS] The amortization by half-monthly payments was designed to ac-
commodate the purchaser, whose salary was being deposited automatically to
his bank account every half-month. The purchaser changes his profession 4
years after the mortgage is executed, and now wishes to make a single payment
once every half-year. Determine the amount of that payment if the interest
rates are unchanged, but if the mortgage is now amortized to be paid off after
25 years.
5. One of the following equations is always true, and one is true only when i = 0.
1 1
I. = +i
sn i an i
1 1 1
II. = +
sn i an i i
1 1
III. = +i
an i sn i
1 1 1
IV. = +
an i sn i i
(a) Explain which is always true, and prove it
i. [4 MARKS] algebraically; and
ii. [4 MARKS] by a verbal argument, referring to a sinking fund. (A detailed
explanation is expected.)
(b) [4 MARKS] Prove algebraically that one of the other equations is true for i = 0.
6. (a) [8 MARKS] Find the price of the following bond, which is purchased at a pre-
mium to yield 5% convertible semi-annually: the bond has face value of 10,000,
matures in 12 years at a maturity value of 11,500, and has a nominal coupon
rate of 8% per annum, compounded semi-annually; the investor is replacing the
premium by means of a sinking fund earning 4% convertible semi-annually.
(b) [7 MARKS] Suppose that the bond is callable at the end of 9, 10, or 11 years at
a premium of 1, 000 above the maturity value. Explain what price the investor
should pay if she is no longer plans to deposit any of the interest in a sinking
fund.
7. [8 MARKS] It was n years ago when James deposited 10,000 in a bank paying
1.8% interest compounded monthly. If he had, instead, placed his deposit in a
syndicate paying interest by cheque annually at the rate of 6% per annum, and he
had invested only this interest with the bank, how much more interest would he
Information for Students in MATH 329 2009 01 3095

have earned altogether? Show all your reasoning, and express your answer in terms
of n.
8. A loan is being repaid with 16 annual payments of 1,000 each. At the time of the
4th payment the borrower requests permission, and is permitted to pay an extra
3000, and then to repay the balance over 8 years with a revised annual payment.
(a) [5 MARKS] If the effective annual rate of interest is 6%, find the amount of the
revised annual payment.
(b) [8 MARKS] Complete an amortization table for the last 8 payments, with the
following columns:
Payment Payment Interest Principal Outstanding
number amount paid repaid loan balance
5
... ... ... ... ...
12

B.3 2004/2005
B.3.1 First Problem Assignment, with Solutions
Mounted on the Web on Thursday, February 10th, 2005
(Caveat lector! 39 There could be some undetected misprints or errors.)
Full solutions were to be submitted by Monday, January 31st, 2005

Students were advised that “These problems are to be solved with full solutions, modelled
either on the solutions to problems in the textbook, or in the notes on the Web for this
or previous years. The essence is that the reader should be able to reconstruct every
step of the proof from what you have written: getting the right answer is never enough.
You are not being graded for elegance, but simply for the proof being logical, without
serious gaps.”
1. (a) Show that the function 225 − (t − 10)2 cannot be used as an amount function
for t ≥ 10.
(b) For the interval 0 ≤ t ≤ 10, determine the accumulation function a(t) that
corresponds to A(t) = 225 − (t − 10)2 .
(c) Determine, for the above functions, I3 , i6 , d7 .
(d) Suppose that A(0) is invested at time t = 0 and accumulates to A(10) at time
t = 10. What is the equivalent effective annual rate of compound interest,
compounded every year, which would yield the same accumulation after 10
years.
39
Let the reader beware!
Information for Students in MATH 329 2009 01 3096

(e) Suppose that A(0) is invested at time t = 0 and accumulates to A(10) at time
t = 10. What is the equivalent annual rate of simple interest which would yield
the same accumulation after 10 years.
(f) Suppose that A(0) is invested at time t = 0 and accumulates to A(10) at time
t = 10. What is the equivalent annual rate of simple discount.
(g) Suppose that A(0) is invested at time t = 0 and accumulates to A(10) at time
t = 10. What is the equivalent annual rate of compound discount.
(h) Verify that the rates i and d of compound interest and discount have the prop-
erty that (1 + i)(1 − d) = 1.
Solution:
(a) By condition [5, 2, p. 2], an accumulation function must be increasing (or at
least non-decreasing); the same property must hold for an amount function,
since it is a positive multiple of its corresponding accumulation function. But
A(t) = 225 − (t − 10)2 has the property that
d
A(t) = 2(10 − t)
dt
which is increasing only when 20 − t ≥ 0, i.e., when t ≤ 10.
A(t) 125 + 20t − t2 (5 + t)(25 − t)
(b) a(t) = = = .
A(0) 125 125
(c)
I3 = A(3) − A(2) = 15
A(6) − A(5) 9
i6 = a(6) − a(5) = =
A(5) 200
I7 A(7) − A(6) 7
d7 = = =
A(7) A(7) 216

(d) Let i be the equivalent annual rate of compound interest. Since A(10) = 225,
A(0) = 125,
µ ¶1
10 225 10
125(1 + i) = 225 ⇒ 1 + i =
125

10
⇒ i = 1.8 − 1 = 6.0540482%.
(e) Let i be the equivalent annual rate of simple interest. Then
225
125(1 + 10i) = 225 ⇒ 1 + 10i =
125
⇒ i = 8%.
Information for Students in MATH 329 2009 01 3097

(f) Let d be the equivalent annual rate of simple discount. Then


2
125 = (1 − 10d)225 ⇒ d = = 4.4444%.
45
(g) Let d be the equivalent annual rate of simple discount. Then
r
5
125 = 225(1 − d)10 ⇒ d = 1 −
10
= 5.70845551%.
9
(h) (1 + 0.060540482)(1 − 0.0570845551) = 1 .
2. Determine the accumulated value of 100 at the end of 2 years if
(a) the nominal annual rate of interest is 8% convertible quarterly
(b) the nominal annual rate of discount is 8% convertible once every 6 years
(c) interest is compounded instantaneously at the rate of 8%
(d) an annual effective compound discount rate of 8% is applied
(e) an annual simple discount rate of 8% is applied
(f) interest is earned during the first year at a nominal annual rate of 8% convertible
quarterly, and during the second year at a nominal annual rate of discount of
8% convertible quarterly.
Solution:
(a) A nominal annual interest rate of 8% convertible quarterly is equivalent to a
3-month rate of 84 % = 2%. The accumulated value of 100 after 21 = 8 quarter
4
years is
100(1.02)8 = 117.17 .
(b) We are given that
d( 6 ) = 8% .
1

2 1
The accumulated value of 100 after 6
= 3
6-year periods
2
100 (1 − (6 × 0.08))− 6 = 124.36 .

(c) Interest compounded instantaneously at the rate of 8% yields an accumulation


factor of µ ¶m
0.08
lim 1 + = e0.08 .
m→∞ m
After 2 years 100 accumulates to

100e2×0.08 = 117.35 .
Information for Students in MATH 329 2009 01 3098

(d) At an annual effective compound discount rate of 8%, 100 accumulates after 2
years to
100(1 − 0.08)−2 = 118.15 .
(e) At an annual simple discount rate of 8%, 100 accumulates after 2 years to
100
= 119.05 .
1 − 2(0.08)

(f) The accumulated value of 100 after the first year will be 100(1.02)4 . During
the second year this amount will accumulate to a final amount of
µ ¶−4 µ ¶4
4 0.08 1.02
100(1.02) 1 − = 100 = 117.36 .
4 0.98

3. Today is New Year’s Day. In return for payments of 1500 at the end of January,
February, and March, and of 3000 at the end of May, July, and September, an
investor agrees to pay now the total value of the 6 payments, and to either make
or receive an additional payment at the end of December. Find the amount of
that additional payment if it is known that the nominal annual interest rate is 6%,
compounded monthly. (First set up an equation of value.)
Solution: Let us denote the final payment by the investor by X. The effective
6
monthly interest rate is 12 % = 12 %. We will take as the comparison date the end of
December (although any date chosen would yield the same information). The value
on 31 December of the payments paid to the investor is
¡ ¢
1500 (1.005)11 + (1.005)10 + (1.005)9
¡ ¢
+3000 (1.005)7 + (1.005)5 + (1.005)3 = 13, 957.74 .
The value on 31 December (just after she has paid the last payment) of the payments
paid out by the investor is

(3(1500) + 3(3000))(1.005)12 + X = 14332.65 + X .

Equating the two values yields X = −374.91. Thus the investor is entitled to
receive a final payment of 374.91 at the end of December. (The fact that the final
payment would be received by the investor, rather than paid by her could have been
reasoned without calculation; we didn’t need the mathematical calculation to tell
us that from the sign of the answer.)
4. Analogously to the “rule of 72”, you are asked to develop a rule of n to approximate
how long it takes for money to increase to 1 12 times its initial value. (That is, to
Information for Students in MATH 329 2009 01 3099

determine a 2-digit integer N = 10n1 + n0 (where n0 , n1 are decimal digits), for


which (0.01) · Ni is a good approximation to the number of years required.) Your
approximation should be best around 8%.
Solution: We have to approximate a solution to the equation

(1 + i)n = 1.5,

which is equivalent to
ln 1.5 i
n= · .
i ln(1 + i)
We find that
i
i (ln 1.5) ·
ln(1 + i)
7.0% 0.419
8.0% 0.421
9.0% 0.423
so we take N = 42.
5. (a) Find the smallest nominal rate of interest convertible monthly at which the
accumulated value of 15,000 at the end of 3 years is at least 24,000.
(b) Find the smallest nominal rate of discount convertible semi-annually at which
the accumulated value of 15,000 at the end of 3 years is at least 24,000.
Solution:
(a) Let i denote the nominal interest rate sought. Then
µ ¶36
i
15000 1 + ≥ 24000
12
µ ¶36
i
⇔ 1+ ≥ 1.6000
12
i 1
⇔ 1+ ≥ (1.6) 36 = 1.013141254
12
so i =≥ 15.7695048%. The smallest interest rate is 15.8%.
(b) This could be considered a trick question; but the fact is that the word largest
was not intended. I “solve” it both as written, and with the intended wording.
Let d denote the nominal discount rate.
As written, “largest” Then
µ ¶−6
d
15000 1 − ≥ 24000
2
Information for Students in MATH 329 2009 01 3100

µ ¶6
5d
⇔ ≤1−
82
r
d 6 5
⇔ 1− ≤ = .9246555971
2 8
implying that d ≥ 15.068880%. Thus any discount rate d ≥ 15.1% has the
desired property. There is no largest rate, since, as the rate d(2) approaches
100% from below, the accumulated value approaches ∞. We haven’t at-
tached a meaning to discount rates above 100%. (As for a discount rate of
100% — that does make sense, but would not permit the accumulation of
funds: if we discount a sum X of money back from time 1 to time 0 at an
effective discount rate of 100%, we obtain a present value of 0.)
Corrected to “smallest”
µ ¶−6
d
15000 1 − ≤ 24000
2
µ ¶6
d 5
⇔ 1− ≤
2 8
r
d 6 5
⇔ 1− ≤ = .9246555971
2 8
implying that d ≥ 15.068880%. Thus the smallest discount rate is 15.1%.
6. Let x be a positive real number, 0 ≤ x < 1. Prove that
1
≥1+x
1−x
for all such x. Conclude that the accumulated value of 100 after 10 years at an
interest rate of x% is always less than or equal to the accumulated value of 100
after 10 years at a discount rate of x%, with equality holding only when x = 0.
Solution: Since x2 is a square, it cannot be negative, and can be 0 only when x = 0;
hence 1 − x2 ≤ 1, with equality holding precisely when x = 0. But this inequality
is equivalent to
1 ≥ (1 − x)(1 + x)
or to
1
≥1+x
1−x
since the inequality is preserved when we divide both sides by the positive number
1 − x; again, equality holds when x = 0.
Information for Students in MATH 329 2009 01 3101

¡ ¢
x 10
The accumulated value of 100 at the interest rate of x% is 100 1 + 100 . We
x
apply the preceding argument 10 times, after replacing x by 100 : this cannot exceed
¡ ¢
x −10
100 1 − 100 , which is the accumulated value of 100 at the discount rate of x%.
x
Equality holds when 100 = 0, i.e., when x = 0.

B.3.2 Second Problem Assignment, with Solutions


Mounted on the Web on March 3rd, 2005
Full solutions were to be submitted by February 14th, 2005.
(Subject to Correction)
These problems were to be solved with full solutions, modelled either on the solutions
to problems in the textbook, or in the notes on the Web for this or previous years. The
essence is that the reader should be able to reconstruct every step of the proof from what
you have written: getting the right answer is never enough. You are not being graded
for elegance, but simply for the proof being logical, without serious gaps.
1. (a) At a certain rate of compound interest an investment of 1000 will grow to 1500
at the end of 12 years. Determine its value at the end of 5 years.
(b) At a certain rate of compound interest an investment of 1000 will grow to 1500
at the end of 12 years. Determine precisely when its value is exactly 1200.
(c) A debt of 7000 is due at the end of 5 years. If 2000 is paid at the end of 1 year,
what single payment should be made at the end of the 2nd year to liquidate the
debt, assuming interest at the rate of 6.5% per year, compounded quarterly.
(d) George agrees to buy his brother’s car for 7000. He makes a down payment of
4000, and agrees to pay two equal payments, one at the end of 6 months, and
the other at the end of a year. If interest is being charged at 5% per annum
effective, how large should each of the equal payments be?
(e) A bill for 1500 is purchased for 1000 15 months before it is due. Determine the
nominal rate of discount, compounded monthly, which the purchaser is paying.
(f) Bills for 1500 are regularly purchased for 1000 15 months before they are due.
The purchaser knows that, among 10 such bills, he will be unable to collect
anything on one of them, will have to pay his lawyers 500 each to effect collection
on 2 others, and will collect the others without any impediment. Lumping all of
these together, what is the effective annual interest rate earned by the purchaser
on his investment, if it is assumed that the lawyers’ accounts are due at the
same time as the bills?
Solution:
(a) Let I be the effective annual rate of compound interest. Then
1000(1 + i)12 = 1500 (114)
Information for Students in MATH 329 2009 01 3102

implies that
µ ¶ 125
5 1500
(1 + i) = = 1.184053587
1000
so the investment is worth 1184.05 after 5 years.
(b) Let t be the time in years when the investment is worth exactly 1200. Then

1000(1 + i)t = 1200 ,

where i is the effective annual rate of compound interest. By equation (114),

ln 1.5
ln(1 + i) = .
12
Hence
ln 1.2 12 × ln 1.2
t= = = 5.395923442 :
ln(1 + i) ln 1.5
the investment has the desired value after 5.40 years.
(c) Let x denote the payment that must be made at the end of the 2nd year. The
equation of value at that time is
µ ¶4 µ ¶−12
0.065 0.065
x + 2000 1 + = 7000 1 +
4 4

implying that
µ ¶4 µ ¶−12
0.065 0.065
x = −2000 1 + + 7000 1 + = 3635.67
4 4

should be paid at the end of the second year to liquidate the debt.
(d) Let x denote the amount of each of the payments that should be made at the
ends of 6 months and 1 year. Then the equation of value at time 0 is
³ 1
´
x (1.05)− 2 + (1.05)−1 = 7000 − 4000 = 3000 ,

implying that
3000
x= 1 = 1555.79 .
(1.05)− 2 + (1.05)−1
(e) Let d be the effective monthly rate of discount. Then

1000 = (1 − d)15 1500 ,


Information for Students in MATH 329 2009 01 3103

implying that
µ ¶ 151
1000
d=1− = 2.66689392% .
1500
This is the effective monthly rate. The nominal annual rate of discount, com-
pounded monthly, is, therefore 12d = 32%.
(f) Of 10 bills which mature in 15 months, the total return that will enure to the
purchaser at that time is

7(1500) + 2(1500 − 500) + 0(1500) = 12500 .

If we denote the effective annual interest rate by i, then the discounted value
15
of 12500 at time t = 0 will be 12500(1 + i)− 12 . The equation of value at time
t = 0 is, therefore,
15
12500(1 + i)− 12 = 10(1000) = 10000 ,
5
implying that (1 + i) 4 = 1.25, so i = 1.250.8 − 1 = 19.5440625%.
2. A government offers savings bonds in multiples of $1000, which mature in 10 years
at $2000, but pay no interest until they are redeemed.
(a) Assuming interest compounded semi-annually, what nominal annual rate of
interest does the bond holder earn?
(b) Suppose that the bonds earn interest at the nominal rate of i − 0.01 for the first
5 years, compounded semi-annually; and that they earn interest at the nominal
rate of i + 0.01 compounded semi-annually for the last 5 years. Determine i.
(c) The government has contracted with a bank to market the bonds, at a cost
of $40 per $1000 bond. What interest rate, compounded semi-annually, is the
government paying for the net proceeds it receives for each $1000 bond?
Solution:
(a) Let i be the nominal interest rate, compounded semi-annually, earned by the
purchaser. The equation of value at time 10 is then
µ ¶20
i
1000 1 + = 2000
2

implying that ³ 1 ´
i = 2 2 20 − 1 = 7.0529848%.
Information for Students in MATH 329 2009 01 3104

(b) The equation of value is now


µ ¶10 µ ¶10
i−1 i+1
1000 1 + 1+ = 2000
2 2

implying that
µ ¶µ ¶
i − 0.01 i + 0.01 1
1+ 1+ = 2 10
2 2
µ ¶2
i (0.01)2 1
⇒ 1+ − = 2 10
2 4
µ ¶1
i 1 (0.01)2 2
⇒1+ = 2 +10
2 4
µ ¶1
1 (0.01)2 2
⇒ i = 2 2 10 + −2
4
= 7.0553996%

(c) Let i be the nominal interest rate, compounded semi-annually. From the gov-
ernment’s point of view the equation of value at maturity is
µ ¶20
i
960 1 + = 2000
2

implying that õ !
¶ 201
2000
i=2 −1 = 7.4760322%
960
3. The cash price of a new automobile is 18,000 plus 15.025% tax. The purchaser
is prepared to finance the car and taxes at 18% convertible semi-monthly, and to
make payments of 230 at the end of every half-month for 3.5 years, with the first
payment to be made one half-month after delivery. The dealer requires a down
payment upon delivery, both to make up the gap in the financing, and from which
to pay his immediate costs (commission to the salesperson, preparation costs, sales
taxes).
(a) Determine the value of this down payment.
(b) Determine the value of the down payment if the purchaser decides, instead of
semi-monthly payments, to make a payment of 460 at the end of every month,
first payment a month after delivery, last payment to be made 3.5 years after
delivery. The interest rate and compounding period do not change.
Information for Students in MATH 329 2009 01 3105

(c) Determine the value of the down payment if the original conditions are changed
so that the first payment of 230 is made 1.5 months after delivery, but the same
number of payments of 230 are made as originally planned. The interest rate
and compounding period do not change.
Solution:
(a) The present value of the future payments is
µ ¶
1 − (1.0075)−84
230 · a(3.5)(24) 0.18 = 230
24 0.0075
= 14295.41185

The excess of the purchase price and taxes over the present value of the future
payments is, therefore,

(1.0015025)(18000) − 14295.41185 = 6409.08815 .

Thus the down payment will be 6,409.09.


(b) We can still interpret the payments as constituting an annuity-immediate. But
the interval of payments has paid, while the compounding interval has not. So
we need to determine the interest rate that corresponds to one month. This is

(1.0075)2 − 1 = 0.01505625 .

The present value of the future payments is


1 − (1.01505625)−42
460a(3.5)(12) 0.01505625 = 460 ·
0.01505625
= 14242.00433

The excess of the purchase price and taxes over the present value of the future
payments is, therefore,

(1.0015025)(18000) − 14242.00433 = 6462.49567 .

Thus the down payment will be 6,462.50.


(c) Since the repayment schedule is delayed by 2 payments, which are added at the
end, the present value of the future payments is
µ ¶
1 − (1.0075)−86 1 − (1.0075)−2
230a(3.5)(24)+2 0.18 − 230a2 0.18 = 230 −
24 24 0.0075 0.0075
= 230(63.20976257 − 1.977722907)
= 14083.36912
Information for Students in MATH 329 2009 01 3106

The excess of the purchase price and taxes over the present value of the future
payments is now

(1.0015025)(18000) − 14083.36912 = 6621.13088 .

Thus the down payment will now be 6,621.13.


4. An employee aged exactly 40 decides to accumulate a fund for retirement at age 65
by depositing 200 at the beginning of each month for 25 years. When she reaches
age 65, she plans to withdraw a fixed amount at the beginning of each year for 15
years. Assuming that all payments are made, determine the amount of the annual
payments that she will be able to withdraw:
(a) if the interest rate is always taken to be (a nominal annual rate of) 6% per
annum, compounded monthly.
(b) if the interest rate is taken to be 6% per annum compounded monthly during
the time when the fund is being built up, and 8% per annum effective when
she reaches age 65.
(c) if the interest rate is taken to be 4% per annum compounded monthly for the
coming 5 years, then 6% per annum compounded monthly for the next 20 years
while the fund is being built up, then 8% per annum effective when the fund is
paying out annual payments.
Solution:
(a) The effective interest rate per month is 6%
12
= 12 %. The number of contributions
will be 25 × 12 = 300. The value of the fund at maturity, when the employee
reaches age 65, will be
à !
(1.005)300 − 1
200s̈25×12 1 % = 200 ¡ 1 ¢
2 (0.005) 1.005
= 139291.7864

The withdrawals from the fund will be once a year. If we wish to interpret them
as payments under an annuity-due, we need to determine the rate of interest
that corresponds to the period between the payments, i.e. to a 1-year period;
that is, we need to determine i, knowing i(12) . This gives us an annual rate of
(1.005)12 − 1 = 6.1677812%. If the annual withdrawal from the fund, at the
beginning of each year, be denoted by X, then the equation of value just before
the first withdrawal is

X · ä15 6.1677812% = 139291.7864


Information for Students in MATH 329 2009 01 3107

which we can solve to determine


139291.7864
X = 1−(1.061677812)−15
0.061677812( 1.061677812
1
)
= 15393.80469
so the withdrawals will be 15393.80, at the beginning of each of 15 years.
(b) Proceeding as in the previous case, we replace the interest rate of 6.1677812%
by 8%. Now we solve
X · ä15 8% = 139291.7864
to determine
139291.7864
X = 1−(1.08)−15
0.08( 1.08
1
)
= 15067.95928
so the withdrawals will be 15,067.96, at the beginning of each of 15 years.
(c) We need to recompute the value at the time of maturity of the fund of the
contributions. The last 20 years of payments accumulate at maturity to
à !
(1.005)240 − 1
200s̈20×12 1 % = 200 ¡ 1 ¢
2 (0.005) 1.005
= 92870.21994
The first five years of payments accumulate to
200 · s̈60 4
%
12

at the end of 5 years; at maturity these payments have accumulated to


à !
¡ ¢ ¡ ¢ 60
(1.003333333) − 1
(1.005)20×12 200s̈60 4 % = (1.005)20×12 200 ¡ 1
¢
12 (0.003333333) 1.00333333333
= 44, 038.93886
The sum of the two components of the fund is
44, 038.93886 + 92, 870.21994 = 136, 909.1588
so the annual withdrawal will be
136909.1588 136909.1588(0.08)(1.08)−1
=
ä15 8% 1 − (1.08)−15
= 14810.21733
or 14,810.22.
Information for Students in MATH 329 2009 01 3108

5. In his will, a benefactor of McGill contributes a large sum of money to establish


a fund, whose proceeds are to be used to provide annual bursaries of 5000 to 6
actuarial students. The principal of the fund is to remain constant, and the bursaries
are funded by the interest earned, forever.
(a) If the effective annual interest rate is assumed to be 6% forever, determine the
lump sum that the benefactor’s estate needs to contribute to McGill a year
before the first payment of bursaries.
(b) If the effective annual interest rate is assumed to be 6% forever, determine the
lump sum that the benefactor’s estate needs to contribute to McGill if the first
bursaries are to be issued immediately.
(c) Suppose that each student receives not a bursary of 5000, but an annual
annuity-due of 1250 for 4 years. Under these changed conditions, determine
the lump sum payment needed now if the first bursaries are to be awarded
immediately or 1 year hence respectively. (The number of awards will remain
the same — each year 6 students are awarded a 4-year sequence of bursary
payments of 1250, the first payment to be made immediately.)
Solution:
(a) The lump sum payment must equal the present value of the perpetuity-immediate
of annual payments of 30,000, i.e.
30000
6(5000)a∞ 6% = = 500, 000.
0.06
(b) The lump sum payment must equal the present value of the perpetuity-due of
annual payments of 30,000, i.e.
30000
6(5000)ä∞ 6% = 1 = 530, 000.
0.06 × 1.06

Alternatively, this may be viewed as the cost of funding the bursaries of the
preceding question, to which must be added the cost of the bursaries that must
be paid out immediately in the amount of 6 × 5, 000.
(c) We must replace each payment of 5000 by

1250 · ä5 = (1250)(1.06) · 1 − 1.06−4 0.06 = 4591.264936.

i. The lump sum payment when the bursaries are awarded beginning one year
hence is
4591.264936
× 500, 000 = 459126.4936
5000
or 459,126.49.
Information for Students in MATH 329 2009 01 3109

ii. The lump sum payment when the bursaries are awarded beginning imme-
diately is
4591.264936
× 530, 000 = 486674.0832
5000
or 486,674.08.
6. Let m and n be positive integers. Consider an annuity-immediate which pays 1 at
the end of every period for mn periods. Explain in words why each of the following
formulæ represents the value of this annuity m + 1 years before the first payment
is made. (If you have doubts about the truth of this claim, you may wish to verify
algebraically that the claim is correct before you attempt to explain it in words.)
(a) v m · amn
(b) am(n+1) − am
(c) v m · am(n+1) − v m(n+2) · sm
Solution:
(a) One period before the first payment, the value of an annuity of 1 per period
payable for mn periods is amn . At a time m periods earlier the value must be
1
discounted by a factor of 1+i per year, or v m for m years.
(b) am(n+1) is the value of an annuity-immediate of 1 payable for mn + n periods.
At a time m periods ago, this represents the value then of the annuity under
present consideration, augmented by an annuity of m payments of 1, the last
to be made now. If we subtract the value of these m payments as of m periods
ago, we are left with the value of the mn payments under consideration.
(c) Consider an annuity that consists of all the payments under present consider-
ation, extended by an additional m payments. Such a scheme is worth am(n+1)
now, and v m · am(n+1) m periods ago. The m payments added at the end
are worth sm at the time of the last of them, which is mn + m periods from
now. We may discount that back to a time m periods ago by multiplying by
v mn+m+m = v m(n+2) .

B.3.3 Third Problem Assignment, with Solutions


Mounted on the Web on March 7th, 2005
Full solutions were to be submitted by March 7th, 2005.
(Caveat lector! There could be misprints or other errors!)

These problems were to be solved with full solutions, modelled either on the solutions
to problems in the textbook, or in the notes on the Web for this or previous years. The
essence is that the reader should be able to reconstruct every step of the proof from what
Information for Students in MATH 329 2009 01 3110

you have written: getting the right answer is never enough. You are not being graded
for elegance, but simply for the proof being logical, without serious gaps.
1. McGill plans to create a scholarship fund that will eternally pay 50 students a
monthly stipend of $200 at the beginning of months September through April, plus
an amount of $300 on the following May 1st.
(a) If interest is assumed to be at a nominal annual rate of 6% per annum, com-
pounded monthly, determine the amount that is needed in this fund on Septem-
ber 1st just before the fund begins making payments.
(b) Determine the amount that will be in the fund just after the December schol-
arship payments in the first year, and on September 1st of the following year,
just before the September payments.
(c) Suppose that at the beginning of September, 8 years after the fund is estab-
lished, it is decided to increase the capital in the fund because the interest rate
has changed to 4% per annum compounded monthly. Determine how much
additional capital needs to be added to the fund.
(d) Suppose that 2 years after the interest rate is changed to 4%, it changes again,
this time to 8%. This time it is decided to leave the capital unchanged, but
to increase the payments to students by a lump sum of $M to each student,
payable on December 1st, together with the regular December payment under
the scholarship. Determine the amount of that lump sum payment.
Solution:
(a) Since monthly payments are not completely regular, we cannot interpret this
fund as a monthly perpetuity with regular payments. We could still develop
its properties from first principles, but it is easier to interpret it as an annual
perpetuity-due. The payments made to each student in an academic year are
worth
µ ¶
−8 1 − (1.005)−8
200ä8 0.5% + 300(1.005) = 200(1.005) + 300(1.005)−8
0.005
= 1860.680368 .
Alternatively, we can think of an annuity-due of 9 payments of 200 together
with an additional payment of 100 alongside the last payment:
µ ¶
−8 1 − (1.005)−9
200ä9 0.5% + 100(1.005) = 200(1.005) + 100(1.005)−8
0.005
= 1860.680368 .
The effective annual interest rate is
(1.005)12 − 1 = 6.1677812% .
Information for Students in MATH 329 2009 01 3111

It follows that the amount in the fund at the beginning of September, just
before the September payments, must be
(1.005)12
50(1860.680368)ä∞ (1.005)12 −1 = 50(1860.680368) = 1, 601, 421.16 .
(1.005)12 − 1
(b) We could evaluate the fund at the beginning of each month, determining the
growth in principal, and then subtracting the payments that would need to
be made on that day. Instead, we will shift all payments to the dates under
examination. On December 1st, just after the December payments, the fund
will be worth
1, 601, 421.16(1.005)3 − 50(200)s4 0.005
µ ¶
3 (1.005)4 − 1
= 1, 601, 421.16(1.005) − 50(200)
0.005
= 1, 585, 261.78
On the following September 1st, just before the first payments of the new
academic year, the value of the fund must be
1, 585, 261.78(1.005)9 − 50(200)s4 0.005 (1.005)5 − 50(300)(1.005)4
µ ¶
9 (1.005)4 − 1
= 1, 585, 261.78(1.005) − 50(200) (1.005)5 − 50(300)(1.005)4
0.005
= 1, 601, 421.16
This is no surprise — we constructed this fund so that the amount just before
the September 1st payments would be constant.
(c) The amount in the fund on September 1st, just before payments are made,
has been constant. Because of the revised interest rate, the annual costs per
participant will be, as of the time of the September payment,
200ä8 1 % + 300(1.0033333333)−8
3
µ ¶
1 − (1.0033333333)−8
= 200(1.0033333333) + 300(1.0033333333)−8
0.00333333333
= 1873.636976 .
The effective annual interest rate is now
(1.0033333333)12 − 1 = 4.0741539% .
The amount of the fund will, therefore, need to be
1.040741539
50 × 1873.636976 × = 2, 393, 100.36 .
0.040741539
Information for Students in MATH 329 2009 01 3112

The additional capital required is, therefore,

2, 393, 100.36 − 1, 601, 421.16 = 791, 679.20 .

(d) Since this is a perpetuity, the amount in the account is periodic. Unless the
interest rate or other conditions change, the amount is always the same on 1
September, and it does not matter how many years have passed. We repeat the
calculations of the preceding part, this time for a nominal rate of 8%. Because
of the revised interest rate, the annual costs per participant will be, as of the
time of the September payment,

200ä8 2 % + 300(1.0066666667)−8
3
µ ¶
1 − (1.0066666667)−8
= 200(1.0066666667) + 300(1.0066666667)−8
0.0066666667
= 1847.870679 .

The effective annual interest rate is now

(1.0066666667)12 − 1 = 8.2999511% .

The amount of the fund to support the previous obligations will, therefore,
need to be
1.082999511
50 × 1847.870679 × = 1, 205, 575.19 .
0.082999511
The capital no longer required for the previously defined purposes is, therefore,

2, 393, 100.36 − 1, 205, 575.19 = 1, 187, 525.17

on September 1st, prior to the payments due on that date. On December 1st
this excess is worth

1, 187, 525.17(1.0066666667)3 = 1, 211, 434.36

in total, or 24,228.68728 for each of the 50 participants at any time. This can
purchase a perpetuity-due of
24228.68728 24228.68728 × 0.082999511
M= = = 1856.851435 .
ä∞ 8.2999511% 1.082999511
(The problem was somewhat ambiguous: some students thought the intention
was that there would be a single lump sum payment in just one December. If
that had been the intention, the value would have been 24,228.69 determined
above.)
Information for Students in MATH 329 2009 01 3113

2. A loan of 10,000 is to be repaid by regular, half-yearly payments of 1,000, the first


to be made at the end of the 3rd year. The loan will be paid off by a final payment
which must be at least 1,000.
(a) If the interest rate is to be 8% per annum, compounded semi-annually, deter-
mine the amount of the final payment, and when it is made.
(b) Suppose that the final payment is now not more than 1,000. and the interest
rate remains 8% per annum compounded semi-annually. Determine the amount
of the final payment, and when it is made.
(c) Suppose that, after signing his original commitment, the borrower decides that
he would like to make the first payment 6 months from the date of borrowing,
and that all the payments under this loan should be exactly equal. If the interest
rate is 6% per annum, compounded semi-annually, determine that payment
level that would be closest to 1000 per half-year, and determine exactly when
the loan will be paid off.
Solution:
(a) Let n be the number of the last payment (counting in half-years from the date
of the loan). Then n will be the largest integer such that

10000(1.04)n ≥ 1000sn−5 4%

since the first 5 opportunities for payments are missed. The inequality is equiv-
alent to
10000(1.04)5 sn−5
≥ = an−5
1000 (1.04)n−5
and to
(1.04)−(n−5) ≥ 1 − 0.4(1.04)5
which implies that
− ln (1 − 0.04(1.04)5 )
n−5≤ = 17.00170887
− ln(1.04)
so the last payment is made at n = 22. The excess of the amount of the
payment over 1000 will be

10000(1.04)22 − 1000s17 = 1.67552

rounding to 1.68.
(b) This time the final payment will be #23; the amount of the payment will be
the accumulation of the residue remaining unpaid after the payment at time
n = 22, i.e., (1.04)1.67552 = 1.74254 rounding to 1.74.
Information for Students in MATH 329 2009 01 3114

(c) If the last payment is n half-years after the date of borrowing, the payments
will be in the amount
10000
.
an
The object is to minimize ¯ ¯
¯ 10000 ¯
¯ − 1000 ¯
¯ an ¯
or to minimize ¯ ¯
¯ 10 ¯
¯ − 1¯
¯ an ¯
Since an is an increasing function of n, its reciprocal is decreasing. We need to
find the value of n such that

an 3% ≤ 10 < an+1 3%

i.e., the largest n such that an 3% ≤ 10, i.e., such that v n ≥ 0.7, i.e., such that
− ln(0.7)
n≤ = 12.06662371. So the only candidates are n = 12 and n = 13,
ln(1.03)
and the corresponding semiannual payments are
10000
= 1004.620855
a12

and
10000
= 940.2954396
a13
Taking the payment closest to 1000, we find that the last payment will be at
the end of 6 years, and the amount of the regular payments will be 1004.62.
3. (An acknowledgement of the source of this problem will be contained in the solu-
tions, when published.) Katherine, 25 years old, deposits 10,000 at the beginning
of every 4-year period into an RSSP account. The account pays compound interest
annually, at the effective annual rate i. The accumulated amount in the account at
the end of 40 years is X, which is 6 times the accumulated amount in the account
at the end of 20 years. Determine X.
Solution: This problem was modelled on Problem 8 of Course 2, May, 2003, of the
Society of Actuaries.
There are several ways of attacking a problem like this. One method is to determine
from i the interest rate that would apply to the period of payments, i.e. 4 years;
another is to determine the annual payment in advance that would correspond to
a payment of 10,000 in advance every 4 years.
Information for Students in MATH 329 2009 01 3115

The problem on which this is modelled was posed as a multiple-choice problem,


with 5 possible numerical answers. None of the answers was expressed in terms of
i; so, in fact, there was no need for the question to even mention i. You may judge
for yourself whether that inclusion made the problem harder or easier.
Denote by Y the annual payment in advance that is equivalent to a payment every
4-years in advance of 10,000. Then

Y · ä4 i = 10000 ,

10000
so Y = . The values of the account at the ends of 20 years and 40 years are,
ä4 i
respectively

(1 + i)40 − 1
X = Y · s̈40 i = Y · (1 + i) ·
i
(1 + i)20 − 1
and Y · s̈20 i = Y · (1 + i) ·
i
From the hypothesis that these amounts are in the ratio of 6 : 1, we conclude that
(1+i)40 −1
Y · (1 + i) ·
6= i
(1+i)20 −1
= (1 + i)20 + 1
Y · (1 + i) · i

which implies that (1 + i)20 = 5. We were not asked to compute the interest rate,
and don’t actually require it. (If you did require it, you could solve the preceding
1
equation to show that 1 + i = 5 20 = 1.083798, so i = 8.38%.) Then
(1+i)40 −1
(1 + i) · i
X = 10000 · 4
(1 + i) · 1−v
i
40
(1 + i) − 1 10000(25 − 1) 240000 240000
= 10000 · 4
= 1 = =
1−v −
1−5 5 1 − 0.72478 0.275220336
= 872, 028.58

4. For each of the following sequences of payments as of the time stated,


• express the value using standard symbols, as simple as possible;
• give a formula for the value (in terms of i, v, d, etc.);
• evaluate using a calculator or computer — not with tables.
(a) a perpetuity-immediate paying 1 per year at effective annual interest rate 4.25%,
evaluated 1 year before the first payment;
Information for Students in MATH 329 2009 01 3116

(b) a perpetuity paying 1 per year at effective annual interest rate 4.25%, evaluated
2 years before the first payment;
(c) a perpetuity-due paying 1 per half-year at nominal interest rate 5%, com-
pounded semi-annually, evaluated just before the first payment;
(d) a 20-year increasing annuity paying 1 the first year, 2 the second year, 3 the
3rd year, etc., at an interest rate of 7% per year effective, evaluated just after
the last payment;
(e) a 20-year increasing annuity paying 1 the first year, 2 the second year, 3 the
3rd year, etc., at an interest rate of 7% per year effective, evaluated at the time
of the 2nd payment;
(f) a 10-year decreasing annuity paying 1 less each quarter-year, evaluated just after
the last payment, where the nominal interest rate is a 8% annual, compounded
quarterly;
Solution:
1 1
(a) a∞ 4.25% = = = 23.52941176 .
i 0.00425
v 1
(b) v · a∞ 4.25% = = = 22.57017915 .
i 1.0425(0.0425)
1 1
(c) ä∞ 0.025 = = = 41.
iv 0.025(1.025)−1
21
(1.07) −1
s − 21 − 21
(d) (Is)20 0.07 = 21 0.07 = 0.07
= 340.9310971 .
0.07 0.07
(e) v 18 · (Is)20 0.07 = (1.07)−18 · (340.9310971) = 100.8692096 .
40
n(1 + i)n − sn 40(1.02)40 − (1.02)
0.02
−1
(f) (Ds)40 0.02 = = = 1395.980168.
i 0.02
5. X and Y have sold their home for 400,000, and wish to purchase an annuity-
immediate so that they can spread the proceeds over the next 10 years. The first
payment will be one month from the date of purchase of the annuity.
(a) Determine the level monthly payments they will receive if the effective annual
interest rate is 6%.
(b) X and Y live frugally, and don’t think they need as much income now as they
will as time passes. Accordingly they plan to receive monthly payments which
will gradually increase. If the first payment is 3,000, and the payments increase
each month by the same dollar amount K, determine K. The effective annual
interest rate remains 6%. What is the final monthly payment?
(c) Suppose that the payments remain constant in any year. The first year’s
monthly payments are all 3000, the next year’s 3000+L, the next years’ 3000 +
Information for Students in MATH 329 2009 01 3117

2L, 3000 + 3L, . . . , 3000 + 9L. Determine L if the nominal annual interest rate
is 6%, compounded monthly.
Solution:
1
(a) The effective monthly interest rate is (1.06) 12 −1 = 0.4867551%, which we shall
denote by j below. The payments will be
1
400000 400000((1.06) 12 − 1)
= −120
a120 (1.06) 121 −1 1 − (1.06) 12

1
400000((1.06) 12 − 1)
= = 4408.961439
1 − (1.06)−10
(b) We solve for K the equation of value
400000 = K · (Ia)120 j + (3000 − K) · a120 j
à !
ä120 j − 120(1 + j)−120
⇔ 400000 = K + (3000 − K) · a120j
j
à !
ä120 j − 120(1 + j)−120
= K − a120j + 3000 · a120j
j

Hence
¡ ¢
j 400000 − 3000 · a120 j
K =
ä120 j − 120(1 + j)−120 − j · a120j
¡ ¢
j 400000 − 3000 · a120 j
=
a120 j − 120(1 + j)−120
= 26.23459805
so the final payment will be
3, 000.00 + 119(26.23459805) = 6, 121.92 .
(c) Consider payments of L at the end of every month for a year. Discounted back
to the beginning of the year, these payments are worth
µ ¶
1 − 1.005−12
L · a12 0.005 = L = L(11.61893206) .
0.005
The value of all of the payments as of the date of purchase of the annuity is,
therefore,
3000 · a1200.005 + L · a12 0.005 · (Ia)9 (1.005)12 −1 = 400000 .
Information for Students in MATH 329 2009 01 3118

Now
ä9 (1.005)12 −1 − 9(1.005)−9(12)
(Ia)9 (1.005)12 −1 =
(1.005)12 − 1
(1.005)12 · a9 (1.005)12 −1 − 9(1.005)−9(12)
=
(1.005)12 − 1
a9 (1.005)12 −1 − 9(1.005)−10(12)
=
1 − (1.005)−12
1−(1.005)−9(12)
(1.005)12 −1
− 9(1.005)−10(12)
=
1 − (1.005)−12
= 31.08041107 .

Solving for L, we obtain


400000 − 3000 · a120 1 %
L = 2

11.61893206(31.08041107)
¡ ¢
= 0.002769153521 400000 − 3000 · a120 0.005
µ ¶
1 − (1.005)−120
= 0.002769153521 400000 − 3000 ·
0.005
= 359.3797471.

The monthly payments in the first year will be 3,000; those in the last year will
be
3, 000 + 9(359.3797471) = 6, 234.42 .

B.3.4 Fourth Problem Assignment, with Solutions


Mounted on the Web on March 31st, 2005
Full solutions were to be submitted by March 21st, 2005.

These problems were to be solved with full solutions, modelled either on the solutions
to problems in the textbook, or in the notes on the Web for this or previous years. The
essence is that the reader should be able to reconstruct every step of the proof from what
you have written: getting the right answer is never enough. You are not being graded
for elegance, but simply for the proof being logical, without serious gaps.
1. A loan of 20,000 is to be repaid by 15 annual payments beginning one year after
the loan was received, payments which increase by a factor of 1.03, each over the
preceding. If the effective annual interest rate is 7%, determine
Information for Students in MATH 329 2009 01 3119

(a) the amount of the first payment


(b) the amount of the loan which is outstanding immediately after the 10th pay-
ment, determined using the Retrospective Method
(c) the amount of the loan which is outstanding immediately after the 10th pay-
ment, determined using the Prospective Method
Solution:
(a) Denote the amount of the first payment by X. The value at time 0 of all future
payments is

(1.07)−1 (1.03)0 X + . . . + (1.07)−r (1.03)r−1 X + . . . + (1.07)−15 (1.03)14 X


à µ ¶ µ ¶2 µ ¶14 !
X 1.03 1.03 1.03
= 1+ + + ... +
1.07 1.07 1.07 1.07
¡ 1.03 ¢15
X 1 − 1.07
= · ¡ ¢
1.07 1 − 1.03 1
1.07
à µ ¶15 !
X 1.03
= 1− = 10.8830229X .
0.04 1.07

Equating to 20,000, we find that X = 1837.72472 is the first payment.


(b) By the Retrospective Method, the amount outstanding after the 10th payment
is
¡ ¢
10, 000(1.07)10 − X (1.03)9 + (1.03)8 (1.07) + . . . + (1.03)0 (1.07)9
à ¡ 103 ¢10 !
1 −
= (1.07)10 (20, 000) 1 − 107
¡ 103 ¢15
1 − 107
= 10, 709.669 .

(c) By the Prospective Method, the amount outstanding after the 10th payment is

X X X
(1.03)10 + (1.03)11 2
+ . . . + (1.03)14
1.07 (1.07) (1.07)5
à µ ¶ µ ¶4 !
2
(1.03)10 103 103 103
= ·X · 1+ + + ... +
1.07 107 107 107
= 10, 709.669 .

2. Mary always dreamed of owning a Maserati, and now had the chance. Her employer
was buying a new Ferrari, and was willing to part with his used Maserati for a mere
Information for Students in MATH 329 2009 01 3120

20,000. Mary agreed to repay the loan by equal monthly payments over 3 years at
4% interest, compounded monthly, first payment one month after the sale. Two
and one-half years after the sale Mary’s employer declared bankruptcy, and Mary’s
future payments became one of the assets to be administered by the trustee.
(a) Determine Mary’s regular monthly payment.
(b) The trustee in bankruptcy needs to know the present value of Mary’s future
payments. Determine this value just after the payment made on the date of
bankruptcy if
i. the interest rate remains a nominal 4% compounded monthly;
ii. the interest rate is now 6% compounded monthly.
(c) Complete an amortization table beginning on the date of bankruptcy, when the
rate is 4% compounded monthly, under the headings
Payment Payment Interest Principal Outstanding
Number Amount paid repaid loan balance
30
31
(d) Suppose that Mary’s future payments, discounted at 6%, have been assigned to
one of the employer’s creditors. You have computed the present value of these
payments. Now set up an amortization table to show how his outstanding debt
in this amount will be amortized at 6%.
Solution:
4%
(a) The effective interest rate per month is 12
= 13 %. Let X be the regular monthly
payment. Then

X· a36 1 % = 20000
3
µ ¶−36
20000 1 20000
⇒ X= 1 1− 1+ = ³ ¡ 300 ¢36 ´ = 590.4797061
300
300 300 1 − 301

so the monthly payment is 590.48.


(b) i. By the prospective method, the unpaid balance is
a6 1 %
X · a6 1 % = 20, 000 · 3
3 a36 1 %
3
¡ 300 ¢6
1 − 301
= 20, 000 · ¡ ¢36 = 3501.909296
1 − 300301

so the unpaid balance is 3,501.91.


Information for Students in MATH 329 2009 01 3121

ii. By the prospective method, the unpaid balance is


a6 1 %
X · a6 1 % = 20, 000 · 2
3 a36 1 %
3
¡ ¢6
1 − 200 201 2
= 20, 000 · ¡ 300 ¢36 · = 3481.695351
1 − 301 3

so the unpaid balance is 3,481.70.


(c) The unpaid balance before the 30th payment was

3, 501.91 + 590.48 = 4, 092.39 .


4, 092.39
One month earlier, this was worth = 4, 078.79, so interest earned
1.00333333
in the 30th period was the difference,

4, 092.39 − 4, 078.79 = 13.60 .

Thus the principal reduction in the 30th payment was

590.48 − 13.60 = 576.88 .

Payment Payment Interest Principal Outstanding


Number Amount paid repaid loan balance
30 590.48 13.60 576.88 3,501.91
31 590.48 11.67 578.81 2,923.10
32 590.48 9.74 580.74 2,342.36
33 590.48 7.81 582.67 1,759.69
34 590.48 5.87 584.61 1,175.08
35 590.48 3.92 586.56 585.52
36 590.48 1.96 588.52 0.00
(d) The new interest rate takes effect after the 30th payment.
Payment Payment Interest Principal Outstanding
Number Amount paid repaid loan balance
30 590.48 13.60 576.88 3,481.70
31 590.48 17.41 573.07 2,908.63
32 590.48 14.54 575.94 2,332.69
33 590.48 11.66 578.81 1,753.87
34 590.48 8.77 581.71 1,172.16
35 590.48 5.86 584.62 587.54
36 590.48 2.94 587.54 0.00
Information for Students in MATH 329 2009 01 3122

3. On January 1st, 2005, John receives a loan of 50,000 from his brother, and agrees to
repay it by semi-annual payments: 2,000 every July 1st, and 2,500 every January
1st; the payments begin on July 1st, 2005, and continue until one last, possibly
irregular payment which will be either less than 2,000 if it is on July 1st, or under
2,500 if it is on January 1st. The effective interest rate is 6% per annum.
(a) Determine the outstanding principal just after the payment on January 1st,
2015.
(b) Complete an amortization schedule beginning with January 1st, 2020, under
the following headings
Date Payment Interest Principal Outstanding
Amount paid repaid loan balance
January 1, 2020
Solution:
(a) Just after a January 1st payment of 2,500, that payment and the payment of
2,000 from the preceding July 1st are together worth

2, 500 + 2, 000 1.06 = 4, 559.26028 .

By the retrospective method, the outstanding principal just after the payment
of January 1st, 2015, is
³ √ ´
50, 000(1.06)10 − 2, 500 + 2, 000 1.06 · s10 6%
(1.06)10 − 1
= 50, 000(1.06)10 − (4559.126028) · = 29, 449.47952
0.06
so the outstanding balance is 29,449.48.
(b) We need to begin the table with January 1st, 2020. It is convenient to repeat
the preceding computation: the outstanding balance as of January 1st, 2019, is

(1.06)14 − 1
50, 000(1.06)14 − (4, 559.126028) · = 17, 234.86372 .
0.06
1
or 17,234.86. The effective interest rate per half year is 1.06 2 − 1 = 2.9563%.
Information for Students in MATH 329 2009 01 3123

Date Payment Interest Principal Outstanding


Amount paid repaid loan balance
January 1, 2019 2,500.00 17,234.86
July 1, 2019 2,000.00 509.51 1,490.49 15,744.37
January 1, 2020 2,500.00 465.45 2,034.55 13,709.82
July 1, 2020 2,000.00 405.30 1,594.70 12115.12
January 1, 2021 2,500.00 358.16 2,141.84 9,973.28
July 1, 2021 2,000.00 294.84 1,705.16 8,268.12
January 1, 2022 2,500.00 244.43 2,255.57 6,012.55
July 1, 2022 2,000.00 177.75 1,822.26 4,190.29
January 1, 2023 2,500.00 123.88 2,376.12 1,814.17
July 1, 2023 1,867.80 53.63 1,814.17 0.00
4. A 30-year loan is amortized by level payments at the end of every month, at a
nominal interest rate of 16.8%, compounded monthly. The amount of interest paid
in the 120th payment is 486.72. Determine
(a) the amount of interest in the 300th payment;
(b) the original amount of the loan;
(c) the amount due at the end of 30 years if the lender defaults on40 the last 6
payments.
Solution:
(a) Suppose that the original amount of the loan was X. The level payments are
X
then , which I will denote by Y . By the prospective method, the amount
a360 1.4%
owing just after the rth payment is Y · a360−r 1.4% , so the interest component
of the r + 1st payment is
¡ ¢
(0.014)Y · a360−r 1.4% = Y 1 − (1.014)r−360 .

The data tell us that


¡ ¢
Y 1 − (1.014)119−360 = 486.72

so Y = 504.4061464: the monthly payments are 504.41. It follows that the


interest component of the 300th payment is
¡ ¢
Y 1 − (1.014)299−360 = 285.8812253

or 288.40.
40
=fails to pay
Information for Students in MATH 329 2009 01 3124

(b) The original amount of the loan was


504.41 ¡ ¢
504.4061464 · a360 1.4% = 1 − (1.014)−360 = 35, 787.47073
0.014
or 35,787.47.
(c) At the due date of the last payment, the value of the last 6 payments is
504.4061464 · s6 1.4% = 3, 134.360302 or 3,134.36.
5. A loan of 100,000 is to be maintained by a semi-annual interest payment, with the
principal being repaid in a single payment at the end of 25 years. The borrower’s
total cost is 5,000 at the end of each 6-month period, being made up of the interest
and a contribution to a sinking fund maintained by a 3rd party earning 4% effective:
the proceeds will be paid to the lender upon maturity. Find
(a) the effective semi-annual interest rate received by the lender of the loan.
(b) the effective interest rate paid out by the borrower. (For this purpose it will
suffice to determine an equation for the interest rate, and to use the tables in
your book to locate the rate between two tabulated values; you are not expected
to interpolate.)
Solution:
(a) The sinking fund earns interest at an effective annual rate of 4%; the equivalent
semi-annual rate is √
1.04 − 1 = 1.9803903%.
The semi-annual payments into the sinking fund will each be in the amount of

100000 100000( 1.04 − 1)
= = 1188.826454
s50 √1.04−1 (1.04)25 − 1

or 1,188.83. This leaves a residue of 5,000-1,188.83=3,811.17. The interest rate


per half year is, therefore
3, 811.17
= 3.811% .
100, 000
(b) Let i be the effective semi-annual interest rate being paid by the borrower.
Then
5, 000 · a50 i = 100, 000
so a50 i = 20. We see from the tables that

a50 4.5% = 19.7620


a50 4% = 21.4822
Information for Students in MATH 329 2009 01 3125

(i is approximately 4.427%.)
NOTE TO THE GRADER: STUDENTS WERE NOT EXPECTED TO IN-
TERPOLATE OR ITERATE TO OBTAIN A GOOD APPROXIMATION.
6. Pierre borrows 18,000 for 8 years, and agrees to make level annual payments. The
lender receives 6% on the investment for each of the first 5 years, and 8% for the
last 3 years. Throughout the 8 years, the balance of each payment is invested in a
sinking fund earning 7%.
(a) Determine the amount of the lender’s total annual payment.
(b) Complete the following table.

Period Total Interest Interest Earned Contribution to Balance in


Payment to Lender in Sinking Fund Sinking Fund Sinking Fund
0
Solution:
(a) Let Pierre’s annual payment be X. The interest component in this payment
will be
.06 × 18, 000 = 1, 080 during the first 5 years
.08 × 18, 000 = 1, 440 during the last 3 years
Hence the annual contributions in arrears to the sinking fund will be
X-1,080 during the first 5 years
X-1,440 during the last 3 years
This information enables us to infer an equation that determines X:

(X − 1, 080)s8 7% − 360 · s3 7% = 18, 000

from which it follows that


18, 000 + 360 · s3 7%
X = 1, 080 +
s8 7%
1, 260 + 360 ((1.07)3 − 1)
= 1, 080 +
(1.07)8 − 1
= 2, 947.2254

(b) Growth of the Sinking Fund:


Information for Students in MATH 329 2009 01 3126

Period Total Interest Interest Earned Contribution to Balance in


Payment to Lender in Sinking Fund Sinking Fund Sinking Fund
0 0.00
1 2,947.23 1,080.00 0.00 1,867.23 1,867.23
2 2,947.23 1,080.00 130.71 1,867.23 3,865.17
3 2,947.23 1,080.00 270.56 1,867.23 6,002.96
4 2,947.23 1,080.00 420.21 1,867.23 8,290.40
5 2,947.23 1,080.00 580.33 1,867.23 10,737.96
6 2,947.23 1,440.00 751.66 1,507.23 12,996.85
7 2,947.23 1,440.00 909.78 1,507.23 15,413.86
8 2,947.23 1,440.00 1,078.96 1,507.23 18,000.06
Is it surprising that the balance is 18,000? Of course not — the repayment
scheme was designed so that Pierre would accumulate in the fund just the
amount needed to repay the loan after 8 years.

B.3.5 Fifth Problem Assignment, with Solutions


Mounted on the Web on April 12th, 2005
Full solutions were to be submitted by April 4th, 2005.

1. (a) A 25,000 7% bond matures on June 30th, 2020. Interest is payable semiannually
on June 30th and December 31st. Determine the price to be paid for the bond
on June 30th, 2005, in order to earn the investor a yield of 6%. (Remember the
convention for bonds — interest is normally interpreted as a nominal (annual)
rate, compounded semi-annually, even when not explicitly stated.)
(b) Repeat problem 1a if the purchase date is December 31st, 2012.
(c) A 10,000 bond has coupon rate 8% payable semiannually, and is redeemable
after a certain number of years at 11,250. The bond is purchased to yield 7%
convertible semiannually. If the present value of the redemption value of the
bond is 4,927 at the given yield rate, determine the purchase price.
(d) A 1,000 bond matures after a m years at par, and has a coupon rate of 10%
convertible semiannually. It is purchased at a price to yield 7% convertible
semi-annually. If the term of the bond is 2m years, the price of the bond will
increase by 107. Determine the price of the m-year bond to the nearest dollar.
Solution:
1
(a) F = C = 25, 000, r = 2
× 7% = 3.5%, i = 3%, n = 2 × 15 = 30. By the Basic
Formula

P = (25, 000)(0.035) · a15 3% + (25, 000)(1.03)−30


Information for Students in MATH 329 2009 01 3127

µ µ ¶ ¶
1 − (1.03)−30 −30
= 25, 000 (0.035) + (1.03)
0.03
= 27, 450.06

(b) F = C = 25, 000, r = 3.5%, i = 3%, n = 15.

P = (25, 000)(0.035) · a15 3% + (25, 000)(1.03)−15


µ µ ¶ ¶
1 − (1.03)−15 −15
= 25, 000 (0.035) + (1.03)
0.03
= 26, 492.24

(c) F = 10, 000, C = 11, 250, r = 4%, i = 3.5%, K = 4927. This last value of K
implies that K = Cv n = 11, 250(1.025)−n , so (1.035)n = 11,250
4,927
, where n is the
number of half-years remaining before maturity. By the Basic Formula,

P = F r · an 0.035 + K
µ ¶
1 − (1.035)−n
= 400 + 4, 927
0.035
à 4,927
!
1 − 11,250
= 400 + 4, 927 = 11, 350.37
0.035

(d) For the m-year bond, F = C = 1, 000, n = 2m, r = 5%, i = 3.5%. For the
2m-year bond, n = 4m. Denote the price of the m-year bond by P . Then,
using the Premium/Discount formula, we have

P = 1, 000 + (50 − 35) · a2m 3.5% (115)


P + 107 = 1, 000 + (50 − 35) · a4m 3.5% (116)

Subtracting yields ¡ ¢
107 = 15 a4m 3.5% − a2m 3.5%
which implies that

1.035−2m − (1.035)−4m
0.2496666 =
0.035
yielding, approximately,

(1.035)−2m = 0.5182574184 or 0.4817425816


Information for Students in MATH 329 2009 01 3128

so, by (115),
µ ¶
1 − 0.5182574184
P = 1, 000 + 15 = 1206.46
0.035
or µ ¶
1 − 0.4817425816
P = 1, 000 + 15 = 1, 222.11;
0.035
to the nearest dollar there are two answers, 1,206 and 1,222.
2. An investor is considering the purchase of two 1,000 face value bonds which are
redeemable at the end of the same number of years, and are both to be bought to
yield 7.5% convertible semiannually. One bond costs 907, and pays coupons at the
rate of 6% per year, convertible semiannually. The second bond pays coupons at
7% per half-year.
(a) Determine the price he should pay for the second bond so that it is an equivalent
investment to the first.
(b) Determine the price the investor should pay for the second bond if the first
bond costs 960.45, and pays a premium of 100 upon redemption, while the
second pays a premium of 150 upon redemption.
Solution: The earlier references in the problem to interest rates always stated that
the rate was “convertible semiannually”. But, in this case, those words were not
mentioned. It appears that the rate of 7% stated is intended to be an effective
semi-annual rate, equivalently that the nominal annual rate is 14%. However, that
would be a much different rate than the others given in the problem. Because of
this ambiguity, I will solve the problem with both interpretations.
Assuming 3.5% effective per half year: (a) For both bonds we know that F =
C = 1000, i = 0.0375. By the Premium/Discount Formula applied to the
first bond we obtain

907 = 1000 + (30 − 37.5)an0.0375 ,

implying that an 0.0375 = 12.4. Substitution of this value into the same
formula applied to the second bond yields a price for that bond of

1000 + (F r − Ci)an 0.0375 = 1000 + (35 − 37.5)an 0.0375


= 1000 − 31 = 961.

(b) In this case the Premium/Discount Formula applied to the first bond gives

960.45 = 1100 + (30 − 41.25)an 0.0375 ,


Information for Students in MATH 329 2009 01 3129

which implies that an 0.0375 = 12.404444. Substitution of this value into the
same formula applied to the second bond yields a price for that bond of

1150 + (F r − Ci)an 0.0375 = 1150 + (35 − 43.125)an 0.035


= 1150 − 8.125(12.404444) = 1049.21

Assuming 7% effective per half year: (a) For both bonds we know that F =
C = 1000, i = 0.0375. By the Premium/Discount Formula applied to the
first bond we obtain

907 = 1000 + (30 − 37.5)an0.0375 ,

implying that an 0.0375 = 12.4. Substitution of this value into the same
formula applied to the second bond yields a price for that bond of

1000 + (F r − Ci)an 0.0375 = 1000 + (70 − 37.5)an 0.0375


= 1000 + 403 = 1403.

(b) In this case the Premium/Discount Formula applied to the first bond gives

960.45 = 1100 + (30 − 41.25)an 0.0375 ,

which implies that an 0.0375 = 12.404444. Substitution of this value into the
same formula applied to the second bond yields a price for that bond of

1150 + (F r − Ci)an 0.0375 = 1150 + (70 − 43.125)an 0.035


= 1150 + 26.875(12.404444) = 1483.37

3. A loan of 96,000 is being repaid by a monthly payment of interest at a nominal


rate of 6.0% compounded monthly, together with a monthly payment into a sinking
fund which earns interest at the rate of 4.0% per annum, also compounded monthly.
The total monthly payment is constant, at 1,000, with the exception of the very
last payment, which could be less than 1,000, in order to bring the fund up to its
target level of 96,000; at that time the entire sinking fund will be paid over to the
lender and the loan will have been repaid. Determine when the final deposit into
the sinking fund will be made, and its amount.
Solution: The monthly interest payment to the lender will be
1
(6%) × 96, 000 = 480,
12
Information for Students in MATH 329 2009 01 3130

so the monthly contribution to the sinking fund will be 1, 000 − 480 = 520. Regular
contributions of 520 will be made for as long as
520 · sn 1 % ≤ 96, 000
µ 3 ¶
(1.00333333)n − 1
⇔ 520 ≤ 96, 000
0.00333333
⇔ (1.0033333)n ≤ 1.615385
⇔ n ≤ 144.1116636
i.e., until n = 144. Immediately after that payment the balance in the sinking fund
will be 520 · s144 1 % = 95, 906.43598. But, by the time for the next payment, this
3
will have grown to
1.0033333 × 95, 906.44 = 96, 226.12
so, the amount needed to bring the balance up to the target is −226.12: the lender
will receive a refund! (Note that this refund is from the sinking fund only — he
still has to pay interest of 480 to the lender.)
4. A 5000 par value 18-year bond has 7% semiannual coupons, and is callable at the
end of the 12th through the 17th years at par.
(a) Find the price to yield 6% convertible semiannually.
(b) Find the price to yield 8% convertible semiannually.
(c) Find the price to yield 8% convertible semiannually, if the bond pays a premium
of 250 if it is called.
(d) Find the price to yield 6% convertible semiannually, if the bond pays a premium
of 250 if it is called at the end of years ##12, 13, 14, and a premium of 150
if it is called at the end of years 15 or 16. (It may still be called at the end of
year 17 without premium, and otherwise will mature at the end of year 18.)
Solution:
(a) F = C = 5, 000, n = 36, r = 3.5%, i = 3%. Let m be the coupon number
at whose date the bond is called or matures. Then, by the Premium/Discount
Formula
P = 5000 + (175 − 150)am 3% , (n = 24, 26, 28, 30, 32, 34, 36).
Without knowing which will be the date of call — if any — we take the worst
possible date in order to minimize the price; since am 3% is an increasing function
of m, and is multiplied by a positive number, 25, we minimize by making m as
small as possible, i.e., 2 × 12 = 24:
P = 5000 + (175 − 150)a24 3% = 5000.00 + 25 × 16.9355 = 5, 423.39 .
Information for Students in MATH 329 2009 01 3131

(b) When the semi-annual yield rate is 4%, the multiplier is negative, 175 − 200 =
−25, and we must choose the largest value of m, i.e., m = 36, for a price of

P = 5000 + (175 − 200)a36 4% = 5000.00 − 25 × 18.9083 = 4, 527.29 .

(c) If the bond should be called at the time of any of coupons ##24, . . . , 34,
C = 5, 250 and
P = 5250 + (175 − 210)am 4%
which is minimized when m = 34: P = 4, 605.61. We must compare this with
the price if not called,

5000 + (175 − 200)a36 4% = 4527.293 ,

so the price will be the latter, 4,527.29.


(d) We will have to determine the minimum of

5250 + (175 − 157.50)am 3% if m = 24, 26, 28 (117)


5150 + (175 − 154.50)am 3% if m = 30, 32 (118)
5000 + (175 − 150)am 4% if m = 34, 36 (119)

i.e., of

5, 250 + (175 − 157.51)a24 3% = 5, 250 + 17.50 × 16.9355 = 5546.37


5, 150 + (175 − 154.50)a30 3% = 5, 150 + 20.50 × 19.6004 = 5551.81
5, 000 + (175 − 150)a34 3% = 5, 000 + 25 × 21.1318 = 5528.30

so the price should be 5528.30.


5. (a) Construct a bond amortization schedule for a 4 year bond of face amount 5000,
redeemable at 5250 with semiannual coupons, if the coupon rate is 6% and the
yield rate is 7% — both converted semiannually. Use the format
Time Coupon Interest Principal Book
Value Adjustment Value
0
..
.
(b) Construct a bond amortization schedule for a 4 year bond of face amount 5000,
redeemable at 5250 with semiannual coupons, if the coupon rate is 7% and the
yield rate is 6% — both converted semiannually.
Solution:
Information for Students in MATH 329 2009 01 3132

(a) By the Basic Formula the purchase price of the bond will be

5, 250(1.035)−8 + 150a83.5% = 3, 986.910670 + 150(6.8740) = 5, 018.01

Time Coupon Interest Principal Book


Value Adjustment Value
0 5,018.01
1 150.00 175.63 -25.63 5,043.64 .
2 150.00 176.53 -26.53 5,070.17
3 150.00 177.46 -27.46 5,097.63
4 150.00 178.42 -28.42 5,126.05
5 150.00 179.41 -29.41 5,155.46
6 150.00 180.44 -30.44 5,185.90
7 150.00 181.51 -31.51 5,217.41
8 150.00 182.61 -32.61 5,250.02
(b) By the Basic Formula the purchase price of the bond will now be

5, 250(1.03)−8 + 175a83% = 4, 144.398480 + 175(7.019692190) = 5, 372.84 .

Time Coupon Interest Principal Book


Value Adjustment Value
0 5,372.84
1 175.00 161.19 13.81 5,359.03
2 175.00 160.77 14.23 5,344.80
3 175.00 160.34 14.66 5,330.14
4 175.00 159.90 15.10 5,315.04
5 175.00 159.45 15.55 5,299.49
6 175.00 158.98 16.02 5,283.47
7 175.00 158.50 16.50 5,266.97
8 175.00 158.01 16.99 5,249.98
6. An 8-year bond of face value 10,000 with semiannual coupons, redeemable at par,
is purchased at a premium to yield 6% convertible semiannually.
(a) If the book value (just after the payment of the coupon) six months before the
redemption date is 10024.27, find the total amount of premium or discount in
the original purchase price.
(b) Determine the nominal annual coupon rate of the bond, compounded semian-
nually.
(c) Give the amortization table for the last 2 years.
Solution:
Information for Students in MATH 329 2009 01 3133

(a) The book value just after the pænultimate41 coupon is


10, 024.27 = 10, 000v + F r · a10.03
10000 + F r
= 10, 000v + F r · v =
1.03
so
F r = 1.03 × 10, 024.27 − 10000 = 325 .
Knowing the amount of each coupon we can now evaluate the purchase price
of the bond to have been
10, 000(1.03)−16 + 325a160.03 = 6, 231.67 + 4, 082.36
= 10, 314.03 .
The bond was purchased at a premium of 10, 314.03 − 10, 000 = 314.03.
325
(b) The rate per period was 10,000 = 3.25%; hence the nominal rate compounded
semi-annually, is 2 × 3.25% = 6.5%.
(c) For convenience we could compile this table backwards, beginning with Time=16.
We were given that B15 = 10, 024.27. Hence the Principal Adjustment con-
tained in the 16th coupon is
10, 024.27 − 10, 000 = 24.27.
The book value at Time=15 will be
¡ ¢
(10, 000)(1.03)−2 + 325 (1.03)−1 + (1.03)−2 = 10, 047.84 .
The book value at Time=14 will be
(10, 000)(1.03)−3 + 325(1.03−1 + (1.03)−2 + (1.03)−3 ) = 10, 070.72 .
etc. Alternatively, we could observe by the Basic Formula that the book value
just after the payment of the 12th coupon is
10, 000(1.03)−4 + 325a4 0.03 = 10, 092.93
and so we may construct the table:
Time Coupon Interest Principal Book
Value Adjustment Value
12 325.00 ... ... 10,092.93
13 325.00 302.79 22.21 10,070.72
14 325.00 302.12 22.88 10,047.84
15 325.00 301.44 23.56 10,024.28
16 325.00 300.73 24.27 10,000.01
41
preceding the last, i.e., 2nd last
Information for Students in MATH 329 2009 01 3134

B.3.6 Solutions to Problems on the Class Test


Distribution Date: Mounted on the Web on Saturday, March 12th, 2005
Distributed in hard copy on Wednesday, March 16th, 2005
(Subject to correction)

There were 4 versions of this test. The final grades of all were scaled upwards slightly,
by slightly different factors, in an attempt to equalize the difficulty of the versions.

Problems on rates of interest and discount


1. Problem 1 on Version 1 Showing your work in detail, determine each of the
following; the rates you determine should be accurate to 4 decimal places, or as a
percentage accurate to 2 decimal places:
(a) [4 MARKS] the nominal annual interest rate, compounded monthly, corre-
sponding to an effective annual interest rate of i = 6%
(b) [4 MARKS] the effective annual interest rate corresponding to a nominal dis-
count rate, compounded quarterly, of d = 3.6%
(c) [4 MARKS] the effective monthly interest rate corresponding to a force of in-
terest of δ = 0.06.
Solution:
1
(a) The effective interest rate per month is (1.06) 12 − 1 = 0.4867551%; the nom-
inal annual interest rate, compounded monthly, i(12) , is 12 times this rate,
5.8410612%.
³ ´4
d(4) 1
(b) 1−d = 1 − 4 = (1−0.009)4 = 0.9644830906. 1+i = 1−d = (0.9644830906)−1 =
1.036824813, so the equivalent effective annual interest rate is 3.6824813%.
(c) Since δ = 0.05 = ln(1 + i), 1 + i = e0.06 .
³ 0.06 ´
i(12) = 12 e 12 − 1 = 12 × 0.5012521% = 6.0150252%

i(12)
The effective monthly interest rate is = 0.5012521.
12
2. Problem 2 on Version 2 Showing your work in detail, determine each of the
following; the rates you determine should be accurate to 4 decimal places, or as a
percentage accurate to 2 decimal places:
(a) [4 MARKS] the effective annual discount rate corresponding to a nominal in-
terest rate, compounded quarterly, of i = 2.4%
(b) [4 MARKS] the nominal annual interest rate, compounded quarterly, equivalent
to an effective semi-annual discount rate of d = 4%
Information for Students in MATH 329 2009 01 3135

(c) [4 MARKS] the effective semi-annual interest rate corresponding to a force of


interest of δ = 0.04.
Solution:
(a) The effective interest rate per quarter is 14 × 2.4% = 0.006. If i and d be the
effective annual interest and discount rates, then
1 1
1−d= = ,
1+i (1.006)4
so
1
d=1− = 2.36442751%
(1.006)4
(b) The annual discount factor corresponding to an effective semi-annual discount
rate of 4% is (1 − 0.04)2 ; the annual accumulation factor will, therefore, be
(0.96)−2 . The accumulation factor corresponding to a quarter of a year will be
−2 −2
(0.96) 4 , and so the equivalent effective quarterly interest rate is (0.96) 4 − 1;
the nominal annual interest rate, compounded quarterly, is 4 times this, i.e.
³ −2
´
4 (0.96) 4 − 1 = 8.2482904%.

(c) If i be the equivalent effective annual interest rate, then ln(1 + i) = 0.04, so
1 + i = e0.04 . The effective semi-annual interest rate corresponding is

1 + i − 1 = e0.02 − 1 = 2.0201340% .

3. Problem 3 on Version 3 Showing your work in detail, determine each of the


following; the rates you determine should be accurate to 4 decimal places, or as a
percentage accurate to 2 decimal places:
(a) [4 MARKS] the effective annual interest rate, corresponding to an nominal
annual interest rate of i = 6% compounded every 4 months.
(b) [4 MARKS] the effective monthly interest rate corresponding to a force of in-
terest of δ = 0.12.
(c) [4 MARKS] the effective annual interest rate corresponding to d(4) = 3.2%
Solution:
(a) A nominal interest rate of 6% compounded every 4 months is equivalent to an
effective interest rate of 2% for a 4-month period. The accumulation factor for
1 year will be (1.02)3 , and the equivalent annual interest rate will, therefore,
be (1.02)3 − 1 = 6.1208%.
Information for Students in MATH 329 2009 01 3136

(b) If i be the equivalent annual interest rate, ln(1 + i) = 0.12, so 1 + i = e0.12 .


1
The accumulation factor for 1 month will be (1 + i) 12 = e0.01 , so the effective
interest rate for 1 month will be e0.01 − 1 = 1.0050167%.
(c) Let i and d denote the effective annual interest and discount rates. Then
1 1 1
1+i= =³ ´4 = = 1.032650385
1−d d(4) (1 − 0.008)4
1− 4

so the effective annual interest rate is 3.2650385%.


4. Problem 4 on Version 4 Showing your work in detail, determine each of the
following; the rates you determine should be accurate to 4 decimal places, or as a
percentage accurate to 2 decimal places:
(a) [4 MARKS] the nominal annual interest rate, compounded quarterly, equivalent
to an effective semi-annual discount rate of d = 4%
(b) [4 MARKS] the effective semi-annual interest rate corresponding to a force of
interest of δ = 0.04.
(c) [4 MARKS] the effective annual discount rate corresponding to a nominal in-
terest rate, compounded quarterly, of i = 2.5%
Solution:
(a) The discount factor for a year is (1 − 0.04)2 = (0.96)2 . If i is the effective
annual interest rate, then 1 + i = (0.96)−2 , and the accumulation factor for
1 2
one quarter of a year is (1 + i) 4 = (0.96)− 4 . The effective interest rate for one
2
quarter of a year is, therefore, (0.96)− 4 − 1, and the nominal annual interest
rate, compounded quarterly, is
³ ´
− 24
4 (0.96) − 1 = 4 × 2.0620726% = 8.2482904%.

(b) If i be the equivalent effective annual interest rate, then ln(1 + i) = 0.04, so
1
1 + i = e0.04 , and (1 + i) 2 = e0.02 is the accumulation factor for half a year. The
effective semi-annual interest rate is, therefore
e0.02 − 1 = 2.0201340%.
(c) The accumulation factor for a year is (1 + 0.025
4
)4 = 1.025235353, so the effective
annual interest rate is 2.5235353%. If d is the effective annual discount rate,
then µ ¶−4
0.025
1+d= 1+ = 0.9753857951
4
so d = 2.46142049%.
Information for Students in MATH 329 2009 01 3137

Problems on the values of annuities and perpetuities with constant payments

1. Problem 2 on Version 1 For each of the following sequences of payments, deter-


mine, as of the given time, and for the given interest or discount rate, the value,
showing all of your work. Before determining the numeric value you are expected
to express the value using standard symbols.
(a) [6 MARKS] the value now of 20 payments of 1 at the end of every year starting
1 year from now, at an interest rate of 4%
(b) [6 MARKS] the value 1 year ago of 10 payments of 1 at the end of every half-
year, the first to be paid 5 years from now, at a nominal interest rate of 8%
compounded semi-annually
(c) [6 MARKS] 300 payments of 1 at the end of every month, as of the date of the
100th payment, which has just been made; the interest rate is 12% compounded
monthly
Solution:
(a) This question was ambiguous. One reading was that the payments would start
one year from now, in which case the value is
1 − (1.04)−20
a20 4% = = 13.59032635 .
0.04
Another reading was that the first payment would be at the end of the year
that starts one year from now, in which case the preceding value should be
1
multiplied by v = , yielding
1.04
1 − (1.04)−20
v · a20 4% = = 13.06762149 .
(0.04)(1.04)

(b) The effective half-yearly interest rate is 21 ·8% = 4%. The value of the payments
4.5 years from now is a10 4% . The value 1 year ago — i.e. 5.5 years prior to the
value just given, is

11 −11 1 − (1.04)−10
v · a10 4% = (1.04) · = 5.268683237 .
0.04
1
(c) The effective monthly interest rate is 12 · 12% = 1%. The value of the payments
just after the last of them is s300 1% . Precisely 200 months earlier the value is

(1.01)100 − (1.01)−200
v −200 · s300 1% = = 256.8127449 .
0.01
Information for Students in MATH 329 2009 01 3138

2. Problem 3 on Version 2 For each of the following sequences of payments, deter-


mine, as of the given time, and for the given interest or discount rate, the value,
showing all of your work. Before determining the numeric value you are expected
to express the value using standard symbols.
(a) [6 MARKS] the value of 25 payments of 1 at the end of every year, the last one
having just been made, at an interest rate of 6%
(b) [6 MARKS] the value 4 months from now of 30 payments of 1 at the end of
every 4 months, the first to be paid 2 years from now, at a nominal interest
rate of 9% compounded 3 times a year
(c) [6 MARKS] 120 payments of 1 at the end of every 2 months, as of the date of the
90th payment, which has just been made; the interest rate is 6% compounded
every 2 months.
Solution:
(1.06)25 − 1
(a) s25 6% = = 54.86451200 .
0.06
(b) The effective interest rate per 4 months is 13 (9%) = 3%. The first payment is
due 24 months from now; if we wish to interpret the payments as an annuity-
immediate, then the value 20 months from now will be a30 3% . The value 4
months from now, i.e., 16 months, or 34 of a year before “the clock starts” will
be

4 1 − (1.03)−30
−4
v a30 3% = (1.03) ·
0.03
(1.03)−4 − (1.03)−34
= = 17.41473827 .
0.03
2
(c) The effective interest rate for a 2-month period is 12 · 6% = 1%. At the time of
the 120th payment the value of all the payments is s120 1% . 30 payments prior
to that date, the value is

(1.01)120 − 1
v 30 s120 1% = (1.01)−30 · = 170.6709757 .
0.01
3. Problem 4 on Version 3 For each of the following sequences of payments, deter-
mine, as of the given time, and for the given interest or discount rate, the value,
showing all of your work. Before determining the numeric value you are expected
to express the value using standard symbols.
(a) [6 MARKS] the value one half-year ago of 12 payments of 1 at the end of every
half-year, the first to be paid 4 years from now, at a nominal interest rate of
6% compounded semi-annually
Information for Students in MATH 329 2009 01 3139

(b) [6 MARKS] 180 payments of 1 at the end of every month, as of the date of the
100th payment, which has just been made; the interest rate is 18% compounded
monthly
(c) [6 MARKS] the value now of 16 payments of 1 at the end of every year starting
1 year from now, at an interest rate of 4%
Solution:
(a) The effective semi-annual interest rate is 21 (6%) = 3%. Three and one-half
years from now the payments will be worth a12 3% . One half-year ago, they
were worth
v 8 − v 20 (1.03)−8 − (1.03)−20
v 8 · a12 3% = = = 7.857782670 .
i 0.03
1
(b) The effective monthly interest rate is 12 (18%) = 1.5%. As of the last payment
the payments are worth s180 1.5% ; 80 payments earlier the value is

80 (1.015)100 − (1.015)−80
v · s180 1.5% = = 275.2103668 .
0.015
(c) This problem is ambiguous. Does the word “starting” refer to the payments or
to the years? If it refers to the payments, the value is
1 − (1.04)−16
a16 4% = = 11.65229561 .
0.04
If the word refers to the years, then one has an annuity-immediate which has
been deferred one year, and the value is
1 − (1.04)−16
v · a16 4% = = 11.20413039 .
(0.04)(1.04)
4. Problem 1 on Version 4 For each of the following sequences of payments, deter-
mine, as of the given time, and for the given interest or discount rate, the value,
showing all of your work. Before determining the numeric value you are expected
to express the value using standard symbols.
(a) [6 MARKS] 90 payments of 1 at the end of every 2 months, as of the date of the
18th payment, which has just been made; the interest rate is 9% compounded
every 2 months.
(b) [6 MARKS] the value of 20 payments of 1 at the end of every year, the last one
having just been made, at an interest rate of 5%
(c) [6 MARKS] the value 4 months from now of 45 payments of 1 at the end of
every 4 months, the first to be paid 3 years from now, at a nominal interest
rate of 6% compounded 3 times a year
Information for Students in MATH 329 2009 01 3140

Solution:
2
(a) The effective interest rate for a 2-month period is 12 (9%) = 1.5%. Two months
before the first payment the value of all the payments is a90 1.5% . Immediately
after the 18th payment the value of all the payments is

(1.015)18 − (1.015)−72
(1.015)18 · a90 1.5% = = 64.33404251 .
0.015
20
(b) s20 5% = (1.05)
0.05
−1
= 33.06595410 .
4
(c) The effective interest rate per 4-month period is 12 (6%) = 2%. Two and two-
thirds years from now the value of the payments is a45 2% . Hence one-third year
from now the value will be
(1.02)−7 − (1.02)−52
(1.02)−7 · a45 2% = = 25.67295884 .
0.02

Problems on increasing and decreasing annuities and perpetuities


1. Problem 3(a) on Version 1 [6 MARKS] An annuity at interest rate i consists of
payments of 10 now, 12 at the end of 1 year, 14 at the end of 2 years, increasing by
a constant amount until the last payment in the amount of 40, is to be evaluated
as of 1 year ago. Express its value in terms of symbols (Ia)n , (Is)n , an , sn , i, v,
but do not evaluate.
Solution: The total increase of 40 − 10 = 30 will be spread over 40−10
2
= 15 years:
the last payment will be 15 years from now. The value 1 year ago was

8 · a16 + 2(Ia)16

2. Problem 4(a) on Version 2 [6 MARKS] An annuity at interest rate i consists of


payments of 100 now, 95 at the end of 1 year, 90 at the end of 2 years, decreasing
by a constant amount until the last payment in the amount of 25, is to be evaluated
as of the time of the payment of 95. Express its value then in terms of symbols
(Ia)n , (Is)n , an , sn , i, v, but do not evaluate.
Solution: The total decrease of the annuity is 75, at 5 per year; so the last payment
will be 75
5
= 15 years from now. The value is
¡ ¢
100(1 + i) + 95 1 + a14 − 5(Ia)14

3. Problem 1(a) on Version 3 [6 MARKS] An annuity at interest rate i consists of


payments of 10 now, 13 at the end of 1 year, 16 at the end of 2 years, increasing
by a constant amount until the last payment in the amount of 40, and is to be
Information for Students in MATH 329 2009 01 3141

evaluated as of 2 years ago. Express its value in terms of symbols (Ia)n , (Is)n , an ,
sn , i, v, but do not evaluate.
Solution: The increase of 40 − 10 = 30 will be spread over 303
= 10 years. The value
of the annuity today is 10ä11 + 3(Ia)10 . As of 2 years ago the value was

10v · a11 + 3v 2 · (Ia)10

4. Problem 2(a) on Version 4 [6 MARKS] An annuity at interest rate i consists of


payments of 118 now, 112 at the end of 1 year, 106 at the end of 2 years, decreasing
until the last payment in the amount of 34, the totality to be evaluated as of the
time of the payment of 112. Express its value in terms of symbols (Ia)n , (Is)n , an ,
sn , i, v, but do not evaluate.
Solution: The total decrease of 118 − 34 = 84 is to be spread over 84 6
= 14 years.
The value of the payments at time 1 is

118(1 + i) + 112 + 112ä14 − 6(Ia)13

Problems on combinations of annuities and perpetuities


1. Problem 3(b) on Version 1 [12 MARKS] The accumulated value just after the
last payment under a 15-year annuity of 1000 per year, paying interest at the rate
of 8% per annum effective, is to be used to purchase a perpetuity, first payment to
be made 2 years after the last payment under the annuity. Showing all your work,
determine the size of the payments under the perpetuity, assuming that the interest
rate from the time of the last payment under the 15-year annuity is 5%.
Solution: Let the payment under the perpetuity be X. Then the equation of value
at time 15 is
X X X
1000 · s15 8% = − =
0.05 1.05 (0.05)(1.05)
since the first payment of the perpetuity-immediate “whose clock begins to tick at
time 15” is not made. Solving the equation yields X = 1425.49.
2. Problem 4(b) on Version 2 [12 MARKS] The accumulated value just after the
last payment under a 12-year annuity of 1000 per year, paying interest at the rate of
6% per annum effective, is to be used to purchase a 10-year annuity at an interest
rate of 7%, first payment to be made 4 years after the last payment under the
annuity. Showing all your work, determine the size of the payments under the 10-
year annuity. Assume that the 7% rate begins from the time of the last payment
under the 12-year annuity.
Solution: Let X denote the payment under the 10-year annuity. The value of the
payments under the 12-year annuity at the time of the last of them is 1000 · s12 6% =
Information for Students in MATH 329 2009 01 3142

(1.07)−3 a10 7% . The value


¡ of the payments
¢ under the deferred 10-year annuity, as
of that same date, is X a13 7% − a3 7% . Equating these amounts and solving, we
obtain
1000 · s12 6% 100(1.07)3 s12 6%
X= = = 2942.43
a13 7% − a3 7% a10 7%
3. Problem 1(b) on Version 3 [12 MARKS] The accumulated value just after the
last payment under a 15-year annuity of 1000 per year, paying interest at the rate
of 9% per annum effective, is to be used to purchase a perpetuity at an interest
rate of 8%, first payment to be made at the same time as the last payment under
the annuity. Showing all your work, determine the size of the payments under the
perpetuity.
Solution: Denote the amount of the payments under the perpetuity by X. The
equation of value at time 15 is
1.08
1000 · s15 9% = X · ä∞ 8% = X ·
0.08
0.08×1000×((1.09)15 −1)
implying that X = 1.08×0.09
= 2174.88
4. Problem 2(b) on Version 4 [12 MARKS] The accumulated value just after the
last payment under a 14-year annuity of 1000 per year, paying interest at the rate of
10% per annum effective, is to be used to purchase a 12-year annuity-immediate at
an interest rate of 7%, first payment to be made 1 year after the last payment under
the 14-year annuity. Showing all your work, determine the size of the payments
under the 12-year annuity. Assume that the 7% rate applies from the time of the
last payment under the 10% annuity.
Solution: If we denote the amount of the payments under the 12-year annuity by
X, the equation of value at time 14 is
1000 · s14 10% = X · a12 7%
implying that X = 3522.11.

Problems on drop and balloon payments


1. Problem 4 on Version 1 [18 MARKS] A loan of 6000 is to be repaid by annual
payments of 350 to commence at the end of the 1st year, and to continue thereafter
for as long as necessary. Find the time and amount of the final payment if the final
payment is to be larger than the regular payments. Assume i = 4%.
Solution: The balloon payment will take place at the time — call it n — which is
the largest solution of the inequality
350 · an 4% < 6000
Information for Students in MATH 329 2009 01 3143

i.e., of the inequalities

1 − (1.04)−n 6000
<
0.04 350
110
(1.04)−n >
350
ln 350 − ln 110
n < = 29.51126321 .
ln 1.04
It follows that the last payment is at the end of the 29th year. The amount of the
payment is
µ ¶
29 (1.04)28 − 1
6000(1.04) − 350(1.04) = 523.70851
0.04

2. Problem 1 on Version 2 [18 MARKS] A loan of 7000 is to be repaid by annual


payments of 450 to commence immediately, and to continue at the beginning of
each year for as long as necessary. Find the time and amount of the final payment
if the final payment is to be no larger than the regular payments. Assume i = 6%.
Solution: The drop payment will take place at the time — call it n — which is the
smallest solution of the inequality

450 · än+1 6% ≥ 7000

i.e., of the inequalities

1 − (1.06)−(n+1) 7000
(1.06) · ≥
0.06) 450
7000 6 19
(1.06)−(n+1) ≤ 1 − · =
450 106 159
19
ln
n + 1 ≥ − 159 = 36.45967106
ln 1.06
implying that n = 36. The amount of the final drop payment will be
1.06 ¡ ¢
7000(1.06)36 − 450s̈36 6% = 7000(1.06)36 − (450) · (1.06)36 − 1
0.06
= 210.11059

so the final payment is in the amount of 210.11.


3. Problem 2 on Version 3 [18 MARKS] A loan of 8000 is to be repaid by annual
payments of 500 to commence at the end of the 1st year, and to continue thereafter
Information for Students in MATH 329 2009 01 3144

for as long as necessary. Find the time and amount of the final payment if the final
payment is to be smaller than the regular payments. Assume i = 5%.
Solution: Let the final payment be made at time n, which will be the smallest
integer such that
500 · an 5% > 8000
⇔ 1 − (1.05)−n > 16(0.05) = 0.8
ln 0.2
⇔ n>− = 32.98693373 ,
ln 1.05
implying that n = 33. If the present value of the deficiency of the last payment
from 500 be denoted by X, then
X = 500 · an 5% − 8000
µ ¶
1 − (1.05)−33
= 500 − 16 = 1.27460500
0.05
so the final payment is
500.00 − (1.27460500)(1.05)33 = 493.6229109
or 493.62.
4. Problem 3 on Version 4 [18 MARKS] A loan of 9000 is to be repaid by annual
payments of 800 to commence immediately, and to continue at the beginning of
each year for as long as necessary. Find the time and amount of the final payment
if the final payment is to be no smaller than the regular payments. Assume i = 8%.
Solution: Let the final payment be at time n, i.e., be the (n + 1)st payment. Then
n will be the largest integer such that
800 · än+1 8% ≤ 9000
90
⇔ än+1 8% ≤
8
90 0.08
⇔ 1 − 1.08−(n+1) ≤ ·
8 1.08
ln 6
⇔ n+1≤ = 23.28138292
ln 1.08
from which it follows that n = 22. The excess of the final payment over 800 will be
(1.08)23 − 1
9000(1.08)22 − 800 · s23 = 9000(1.08)22 − 800 ·
0.08
= 214.22726 ,
The final payment will, therefore, be 800 + 214.22726 = 1014.23.
Information for Students in MATH 329 2009 01 3145

B.3.7 Final Examination 2005/2005

McGILL UNIVERSITY — FACULTY OF SCIENCE


— FINAL EXAMINATION

MATH 329 2005 01 — THEORY OF INTEREST


Instructions
1. Fill in the above clearly.
2. Do not tear pages from this book; all your writing — even rough work — must be
handed in.
3. Calculators. While you are permitted to use a calculator to perform arithmetic
and/or exponential calculations, you must not use the calculator to calculate such
actuarial functions as ani , sni , (Ia)ni , (Is)ni , (Da)ni , (Ds)ni , etc. without first
stating a formula for the value of the function in terms of exponentials and/or
polynomials involving n and the interest rate. You must not use your calculator in
any programmed calculations. If your calculator has memories, you are expected
to have cleared them before the examination.
4. This examination booklet consists of this cover, Pages 1 through 7 containing ques-
tions; and Pages 8 and 9, which are blank. For all problems you are expected to
show all your work, and to simplify algebraic and numerical answers as much as
you can. All solutions are to be written in the space provided on the page where
the question is printed. When that space is exhausted, you may write on the facing
page. Any solution may be continued on the last pages, or the back cover of the
booklet, but you must indicate any continuation clearly at the bottom of the page
where the question is printed! You may do rough work anywhere in the booklet.
5. You are advised to spend the first few minutes scanning the problems. (Please
inform the invigilator if you find that your booklet is defective.)
6. Several useful formulas are printed on page 3147. You should not assume that any
of these formulas is/are required in the solution of any of the problems on this
examination.
1. Showing your work in detail, determine each of the following; the rates you de-
termine should be accurate to 4 decimal places, or as a percentage accurate to 2
decimal places.
Information for Students in MATH 329 2009 01 3146

(a) [2 MARKS] The nominal annual interest rate, compounded quarterly, corre-
sponding to an effective annual interest rate of 8%.
(b) [2 MARKS] The effective annual interest rate corresponding to a nominal dis-
count rate, compounded monthly, of 6%.
(c) [3 MARKS] The effective semi-annual interest rate corresponding to a force of
interest of δ = 0.04.
(d) [3 MARKS] d( 2 ) , corresponding to a nominal annual interest rate, compounded
1

every 2 months, of 12%.


2. Showing all your work, determine for each of the following sequences of payments,
the value as of the given time and for the given interest or discount rate.
(a) [3 MARKS] The value as of one year ago of 30 payments of 1 at the end of
every half-year, the first to be paid 18 months from now, at a nominal interest
rate of 9% compounded semi-annually.
(b) [3 MARKS] The value now of 24 payments of 1 at the end of every 3 months,
the first to be paid one year from now, at an effective annual interest rate of
9%.
(c) [4 MARKS] The value exactly 5 years from now of a perpetuity consisting of an
unending sequence of payments of 1 at intervals of one year, the first to be paid
in 6 months from now, at a nominal annual interest rate of 4% compounded
every 4 months.
3. [5 MARKS] Give a formula in terms of i alone, for the value of the following,
showing all your work:. An annuity at an effective semiannual interest rate of i, of
100 now, 97 six months from now, 94 one year from now, the payments decreasing
by a constant amount until a final payment of 34.
4. The purchase of a new condominium is partially financed by a mortgage of 120,000
payable to the vendor; the mortgage is amortized over 25 years, with a level payment
at the end of each month, at a nominal annual rate of 6% compounded monthly.
(a) [2 MARKS] Determine the level monthly payments under this mortgage.
(b) [4 MARKS] Divide the 120th payment into principal and interest.
(c) [9 MARKS] When the payments from the mortgagor start to arrive, the vendor
deposits them into an account earning 4% per annum compounded monthly.
He will continue to do so until the account contains exactly 120,000, with the
last deposit being possibly smaller than the others. Determine the time and
amount of that last deposit.
5. The Smiths have sold their house for 500,000. They wish to purchase a 20-year
annuity-certain with the money, but are concerned that living costs will be rising,
and should be planned for by having the amount of the payment in any month
Information for Students in MATH 329 2009 01 3147

Table 5: Several Useful Formulæ that you were not expected to memorize
You may wish to make use of some of the following formulæ, but remember that your
final answers for this problem must be expressed in terms of i alone.

än i − nv n s̈n i − n
(Ia)n i = i (Is)n i = i

s − (n+1) ä∞ i
n+1 i
(Is)n i = i (Ia)∞ i = i

n − an i n(1+i)n − sn i
(Da)n i = i (Ds)n i = i

1.005 times the amount of the payment for the preceding month. The first payment
is to be 1 month after the sale. The interest rate is 4% per annum, compounded
monthly.
(a) [4 MARKS] Showing all your work, determine the amount of the first payment.
(b) [2 MARKS] Determine the amount of the last payment, to be made 20 years
from now.
(c) [4 MARKS] Suppose that, with the same first payment, the annuity payments
will increase by a constant amount and not by a constant factor . Showing all
your work, determine the amount of the last payment. (You may wish to use
the formulæ in Table 3 on page 3147 of this examination.)
6. X sold Y his home for 500,000. In addition to a down payment of 100,000 cash, Y
mortgaged the property to X for 400,000. The mortgage provides for level quarterly
payments over 25 years, at a nominal rate of 4% compounded every 3 months.
(a) [5 MARKS] Construct an amortization table showing the first 6 payments,
under the following headings:
Payment Payment Interest Principal Outstanding
Number Amount Paid Repaid Loan Balance
0
1 ... ... ... ...
... ... ... ... ...
6 ... ... ... ...
Information for Students in MATH 329 2009 01 3148

Table 6: Several Useful Formulæ that you were not expected to memorize
You may wish to make use of some of the following formulæ, but remember that your
final answers for this problem must be expressed in terms of i alone.

än i − nv n s̈n i − n
(Ia)n i = i (Is)n i = i

s − (n+1) ä∞ i
n+1 i
(Is)n i = i (Ia)∞ i = i

n − an i n(1+i)n − sn i
(Da)n i = i (Ds)n i = i

(b) [5 MARKS] After 10 years — just after receiving the 40th payment — X sells
the (remaining payments of the) mortgage to Z at a price to yield 6% convertible
quarterly. Determine the price P that Z pays, and construct the first 3 and
last 3 lines of an amortization table for the 60 payments showing, under the
following headings, how Z is recovering her investment, under the following
headings:
Payment Payment Interest Principal Outstanding
Number Amount at new rate Repaid Loan Balance
0 P
1 ... ... ... ...
... ... ... ... ...
3 ... ... ... ...
58. . . ... ... ... ...
59. . . ... ... ... ...
60. . . ... ... ... ...
7. Consider a 10,000 par-value 10-year bond with semi-annual coupons paying interest
at 5% compounded semiannually. Suppose that the bond can be redeemed for 11,000
at the time of either of the 16th or 17th coupons, at 10,500 at the time of the 18th
or 19th coupons, or can be held until maturity, at the time of the 20th coupon,
where it matures without premium.
(a) [5 MARKS] What price should an investor pay in order to be certain of a yield
rate of 6% compounded semiannually?
Information for Students in MATH 329 2009 01 3149

(b) [5 MARKS] What price should an investor pay in order to be certain of a yield
rate of 4% compounded semiannually?

B.3.8 Supplemental/Deferred Examination 2004/2005


Instructions

1. Fill in the above clearly.


2. Do not tear pages from this book; all your writing — even rough work — must be
handed in.
3. Calculators. While you are permitted to use a calculator to perform arithmetic
and/or exponential calculations, you must not use the calculator to calculate such
actuarial functions as ani , sni , (Ia)ni , (Is)ni , (Da)ni , (Ds)ni , etc. without first
stating a formula for the value of the function in terms of exponentials and/or
polynomials involving n and the interest rate. You must not use your calculator in
any programmed calculations. If your calculator has memories, you are expected
to have cleared them before the examination.
4. This examination booklet consists of this cover, Pages 1 through 7 containing ques-
tions; and Pages 8 and 9, which are blank. For all problems you are expected to
show all your work, and to simplify algebraic and numerical answers as much as
you can. All solutions are to be written in the space provided on the page where
the question is printed. When that space is exhausted, you may write on the facing
page. Any solution may be continued on the last pages, or the back cover of the
booklet, but you must indicate any continuation clearly at the bottom of the page
where the question is printed! You may do rough work anywhere in the booklet.
5. You are advised to spend the first few minutes scanning the problems. (Please
inform the invigilator if you find that your booklet is defective.)
6. Several useful formulas are printed on page 3150. You should not assume that any
of these formulas is/are required in the solution of any of the problems on this
examination.
1. Showing your work in detail, determine each of the following; the rates you de-
termine should be accurate to 4 decimal places, or as a percentage accurate to 2
decimal places.
(a) [2 MARKS] The effective annual discount rate corresponding to a nominal
interest rate, compounded monthly, of 9%.
(b) [2 MARKS] The nominal annual interest rate, compounded quarterly, corre-
sponding to an effective annual interest rate of 6%.
Information for Students in MATH 329 2009 01 3150

(c) [3 MARKS] d( 3 ) , corresponding to a nominal annual interest rate, compounded


1

every 2 months, of 6%.


(d) [3 MARKS] The effective quarterly interest rate corresponding to a force of
interest of δ = 0.06.
2. Showing all your work, determine for each of the following sequences of payments,
the value as of the given time and for the given interest or discount rate.
(a) [3 MARKS] The value now of 36 payments, each of 1, at the end of every 4
months, the first to be paid one year from now, at an effective annual interest
rate of 9%.
(b) [3 MARKS] The value as of 6 months ago of 30 payments of 1 at the end of
every half-year, the first to be paid 18 months from now, at a nominal interest
rate of 9% compounded semi-annually.
(c) [4 MARKS] The value exactly 7 years from now of a perpetuity consisting of an
unending sequence of payments of 1 at intervals of one year, the first to be paid
in 12 months from now, at a nominal annual interest rate of 12% compounded
every 4 months.
3. [5 MARKS] Give a formula for the value of the following, showing all your work:.
An annuity at an effective semiannual interest rate of i, of 100 now, 103 six months
from now, 106 one year from now, the payments increasing by a constant amount
until a final payment of 340. Express your formula in terms of any of the functions
i, v, ani , sni , (Ia)ni , (Is)ni , (Da)ni , (Ds)ni .

Table 7: Several Useful Formulæ that you were not expected to memorize
If you wish, you may make use of the following formulæ.

än i − nv n s̈n i − n
(Ia)n i = i (Is)n i = i

s − (n+1) ä∞ i
n+1 i
(Is)n i = i (Ia)∞ i = i

n − an i n(1+i)n − sn i
(Da)n i = i (Ds)n i = i
Information for Students in MATH 329 2009 01 3151

4. The purchase of a new home is partially financed by a mortgage of 150,000 payable


to the vendor; the mortgage is amortized over 25 years, with a level payment at the
end of each month, at a nominal annual rate of 9% compounded monthly.
(a) [2 MARKS] Determine the level monthly payments under this mortgage.
(b) [4 MARKS] Divide the 100th payment into principal and interest.
(c) [9 MARKS] When the payments from the mortgagor arrive, the vendor deposits
them into an account earning 6% per annum compounded monthly. He will
continue to do so until the account contains exactly 150,000, with the last
deposit being at least as large as the others. Determine the time and amount
of that last deposit.
5. The Trudeaus have sold their house for 500,000. They wish to purchase a 24-year
annuity-certain with the money, but are concerned that living costs will be rising,
and should be planned for by having the amount of the payment in any month
1.005 times the amount of the payment for the preceding month. The first payment
is to be 1 month after the sale. The interest rate is 6% per annum, compounded
monthly.
(a) [4 MARKS] Showing all your work, determine the amount of the first payment.
(b) [2 MARKS] Determine the amount of the last payment, to be made 24 years
from now.
(c) [4 MARKS] Suppose that, with the same first payment, the annuity payments
will increase by a constant amount and not by a constant factor . Showing all
your work, determine the amount of the last payment. (You may wish to use
the formulæ in Table 3 on page 3150 of this examination.)
6. X sold Y his home for 500,000. In addition to a down payment of 100,000 cash, Y
mortgaged the property to X for 400,000. The mortgage provides for level quarterly
payments over 20 years, at a nominal rate of 4% compounded every 3 months.
(a) [5 MARKS] Construct an amortization table showing the first 6 payments,
under the following headings:
Payment Payment Interest Principal Outstanding
Number Amount Paid Repaid Loan Balance
0
1 ... ... ... ...
... ... ... ... ...
6 ... ... ... ...
(b) [5 MARKS] After 5 years — just after receiving the 20th payment — X sells the
(remaining payments of the) mortgage to Z at a price to yield 6% convertible
quarterly. Determine the price P that Z pays, and construct the first 3 and last
Information for Students in MATH 329 2009 01 3152

3 lines of an amortization table for the remaining 60 payments showing, under


the following headings, how Z is recovering her investment, under the following
headings:
Payment Payment Interest Principal Outstanding
Number Amount at new rate Repaid Loan Balance
0 P
1 ... ... ... ...
... ... ... ... ...
3 ... ... ... ...
58. . . ... ... ... ...
59. . . ... ... ... ...
60. . . ... ... ... ...
7. Consider a 10,000 par-value 20-year bond with semi-annual coupons paying interest
at 5% compounded semiannually. Suppose that the bond can be redeemed for 11,000
at the time of any of coupons ##14, 15, 16, 17, 18, 19, 20; at 10,500 at the time of
any of coupons ##22, 24, 26, 28, 30, 32, 34, 36, 38; or can be held until maturity,
where it matures without premium.
(a) [4 MARKS] What price should an investor pay in order to be certain of a yield
rate of 6% compounded semiannually?
(b) [6 MARKS] What price should an investor pay in order to be certain of a yield
rate of 4% compounded semiannually?

S-ar putea să vă placă și